Download as pdf or txt
Download as pdf or txt
You are on page 1of 255

,940005

NEWSENIOR .
MATHEMATICS
'Three Unit Course for Years 11&12
... ':'.' :- .. ' . ,.~' ..:-.".:.~::;:.>" .
The text New Senior Mathematics, Three Lfnlt Cf}urse provides an', 8 "~<;'"
extension and more in-depth discussion of sol1:1e of the topicscqntain~d ~, \\
in the Two Unit Course and also covers furtHer, iffiport. a, rit't.o. PiC,S. ; ,(_~', '-:)'
\\
Exercises within each chapter are presente{j on'a gradecj ba,tiil'¥.;-)/Ilith ') ,
many worked problems and solutions to ex~rcises., SumrrrarieS' of / '
important terms and concepts are included at to,e back' of~ ~.
along with sample examination papers with answers::P~~ isr:n.ad ,,_-:::r-
for the use of a calculator. , .... "-,,. ...

,
j' .
-
VrDALE

, ,
:.j

c~ t5 ~-.:A-n-\ AN Lv lAl q1/C~ D{(

ttc;~
~hh YfAt1t\ liM ?J 20/11
~~\.\0 (~\t\4\\ '"" pI\4:~
\2. 0, {\J 'J... o~
bL'D,
~S(Pr , C~ ttTfiSf ~1-\f'(\-\'L
~\.,~~
¥~h, 2fJ{Jl~

I
t
j
j j
j
j
j
j

NEW SENIOR j
j
j

MATHEMATICS j
j
j

Three Unit Course for Years 11 &12 j


j
j
J. B. Fitzpatrick j
j
j
j
j
j
j
j
j
j
j
j
j
, :
! ;
\ " j
j
j
j
j
1-/('; l} crnalll} j
j
j
j
j
j
j
j
j
Rigby Heinemann
a division of Reed International Books Australia Pty Ltd
22 Salmon Street, Port Melboume, Victoria 3207
Offices in Sydney, Brisbane and Adelaide. Associated companies,
branches and representatives throughout the worW.
©Bemard Fitzpatrick 1984
First published 1984
Reprinted 1986, 1987. 1988, 1990, 1991, 1992, 1993, 1994
All rights reserved. No part of this publication may be reproduced, stored
in a retrieval system or transmitted in any fonn or by any means whatsoever
without the prior permission of the copyright owner. Apply in writing to
the publishers.
Technical drawings by James Mackie (technimac)
Cover design by Judith Groenendijk
Typeset in Hong Kong by Setrite Typesetters
Printed by Chong Moh Offset Printing. Singapore
National Library of Australia
cataloguing-in-publication data:
Fitzpatrick, 1. B. (John Bernard)
New senior mathematics: three unit course for
years 11 and 12.
For senior secondary school students.
Includes index.
ISBN 0 85859 353 X.
l.Mathematics. I.Title.
510

The author has taught senior mathematics for many years at


Xavier College, Melbourne, and has written a number of
mathematics text books for the years 11 and 12 syllabuses in
Victoria.
-~------------------~-~

CONTENTS

Introduction vi
20 Plane Geometry 1
/21 . Trigonometry 22
22 Trigonometric Equations 41
/ 23 Miscellaneo~us 57
/24 Calculus 68
25 Applications of Calculus to the Physical WorId 95
- 26 Inverse Trigonometric Functions 131
'/-27 Polynomials 157
28 Permutations and Combinations 180
29 Binomial Theorem and Binomial Probability 200
Test Papers 220
Summary 230
Answers 235
Index 250

v
INTRODUCTION

New Senior Mathematics, Three Unit Course Years 11 and 12 has


been written to meet the requirements of the syllabus for the three-
unit course for_ years 11 and 12 as approved by the New South
Wales Board of Senior School Studies. It covers all aspects of the
prescribed syllabus for the two year course.
This book is a sequel to the two-unit book and must be studied in
conjunction with it by students who take the three-unit course. The
two-unit book contains Chapters 1 to 19 and the three-unit book
contains Chapters 20 to 29.
The author wishes to express his thanks to Mr John Sattler of
St lves High School for reading the manuscript and offering many
helpful suggestions.
J. B. FITZPATRICK

vi

- - - ... ~-----
CHAPTER 20

Plane
Geometry
20.1 The circle _ _ _ _ _ _ _ _ _ _ _ _ __

A circle is a set of all points in a plane that are the same


distance from a fixed point in the plane.
The fixed point, 0, is the centre of the circle.

Circumference

o
A semicircle is a half-circle bounded by a diameter and half
the circumference of the circle.

o
Semicircle
A chord is a straight line segment joining two points on the
circle.
AB is a chord.
8
A chord that passes through the centre is a diameter.

Chord
A secant is a straight line that cuts the circle in two points.
AB is a secant.

Secant
2 NEW SENIOR MATHS: THREE UNIT COURSE

An arc is a part of the circumference of a circle.


Major arc
A YB is the minor arc.
AXB is the major arc.

Minor arc

A sector is part of the plane bounded by two radii and an'


arc.
A YBO is a minor sector.
AXBO is a major sector.

Minor sector

A segment is part of the plane bounded by a chord and an segment


arc.
APBA is a minor segment.
AXBA is a major segment. A

Concentric circles are circles that have the same centre but
different radii.

Concentric circles

LAOB is the angle-that the arc AB subtends at the centre


of the circle .. Note that A and B are the endpoints of the
arc.
Note: The verb 'subtend' means 'to extend under' or 'to
be opposite to'.

Angle at the centre


-_ .....- --------------------

20/PLANE GEOMETRY 3

In a given circle (or in equal circles) equal arcs subtend D


equal angles at the centre and conversely equal angles at
the centre stand on equal atcs.

p
L APB is the angle that the arc AB sub tends at the
circumference of the circle for different positions of P on
the major arc APB.

Angles at the circumference

p
Angles APB are angles in the same segment for different
positions of the point P. Notice that this diagram is the
same as the one above it.

Angles in the same segment


Theorem:

Equal angles at the centre of a circle stand on equal chords.

Given: LAOB = LDOC


To prove: AB = DC
Proof: In l::,. A OB and l::,. DOC
AO CO (equal radii)
BO = DO (equal radii)
L AOB L COD (given)
:. l::,.A OB == l::,.DOC (SAS)
AB = DC
Conversely: Equal chords subtend equal angles at the
centre.
The perpendicular from the centre of a circle to a chord
bisects the chord
i.e. if OM .1 AB, then AM = MB.
The line from the centre of a circle to the mid-point of a
chord is perpendicular to the chord
i.e. if AM = MB, then OM .1 AB.
These follow as a consequence of the properties of th(
isosceles triangle OAB (Chapter 4)
4 NEW SENIOR MATHS: THREE UNIT COURSE

D
Equal chords in a circle (or in equal circles) are equidistant
from the centre
i.e. if AB CD, then OM = ON
Chords in a circle which are equidistant from the centre are
equal c
i.e. if OM ON, then AB = CD.
Theorem:

The angle that an arc of a circle subtends at the centre is twice the angle it
subtends at the circumference.

Given: AB is an arc of a circle, centre 0, and P is any point on the circle but not on
the arc AB.
To prove: LAOB 2 L APB.
Construction: Draw the ray POQ.
Proof: In the isosceles 6APO, PO is produced to Q.
x = a + a (Exterior angle is equal to the sum of the two
2a remote interior angles)
In the isosceles 6BPO, PO is produced to Q.
y = b + b (Exterior angle is equal to the sum of the two
= 2b remote interior angles)
x +y 2a + 2b
= 2(a + b)
:. LAOB = 2 LAPB

EXAMPLE 1
20/PLANE GEOMETRY 5

Theorem:

n angle in a semicircle is right an

A I<::-------.::_--r----...JJB
Given: Semicircle APB. 180 0

To prove: LAPB 90°.


Proof: L A DB 2 L APB (Angle at the centre is twice the angle at the circumference)
But L ADB 180° (A straight angle)
LAPB 90°
Theorem:

Angles in the same segment are equal.


or
Angles sub tended at the circumference by the same arc are equal.
p

""
- --- /'
./
/

Given: LAPB and LAQB are any two angles in the major segment APQB or are any
two angles subtended at the circumference by the minor arc AB.
To prove: LAPB = LAQB
Proof' LADB = 2 LAPB (Angle at centre is double the angle at the circumference)
LADB = 2 LAQB (Same reason)
:. LAPB LAQB

EXAMPLE 2
A

c
6 NEW SENIOR MATHS: THREE UNIT COURSE

8
EXAMPLE 3
Given: AC is a diameter,
LABD = 62°, LACB 54°, find x, y, z.
L ABC 90° (Angle in a semicircle is a right angle)
:. x + 62 = 90
x = 28
L A CD L ABD (Angles in same segment are equal)
y = 62
z = 180 - (90+ 54)
36 (Sum of angles of .6.ABC = 180°)

EXAMPLE 4
A, Band C are three points on the circumference of A
a circle such that chord AB = chord AC. P is any
point on the circle such that A and P are on opposite
sides of BC. Prove that LAPB LAPC.
Proof:
LABC LACB r:
AB = AC)
LABC = LAPC (angles in the same segment)
L ACB = L APB (angles in the same segment)
:. LAPB LAPC r:
LABC LACB)

EXERCISES 20(a)
1. In each of the following, 0 is the centre of the circle. Use a protractor to measure the
angles marked. What do you conclude in each case?

(a) (b) (c) (d)

In each of the following, find the value of the pronumerals. Give reasons for your
answers. 0 is the centre of the circle.

2. 3. 4.
20/PLANE GEOMETRY 7

5. 6. 7.

8. 9. 10.

11. 12. 13.

14. 15. 16.

17. 18. 19.

20. Two chords AB and CD of a circle interect at a point E inside the circle. Prove that
triangles AEC and DEB are equiangular.
21. Two circles intersect at A and B. AX is a diameter of one circle and A Y is a diameter
of the other. Prove that the points X, Band Yare collinear, i.e. are on the same
straight line. (Construction: Join the common chord AB.)
8 NEW SENIOR MATHS: THREE UNIT COURSE

22. ABC is a triangle inscribed in a circle,


centre 0, and AD is drawn per-
pendicular to BC. State the reasons (i),
(ii), (iii), (iv) in the steps to prove that
LBAE = LDAC
Proof: L ABE 90° (i)
In DABE and 6ADC
LABE = LADC (ii)
LAEB = LACD (iii)
:. LBAE = LDAC (iv)
23. AB is a chord of a circle, centre O. The bisector of angle OAB meets the circle at D.
Prove that OD II A B . [
24. A, B, C, Dare 4 points in order on a circle. Chords AC and BD intersect at E. If
AD II BC, prove that triangles AED and BEC are isosceles.
25. ABC is an isosceles triangle inscribed in a circle, centre O. AD is a diameter. State the
reasons (i), (ii), (iii), (iv), (v) in the steps to prove that L BED = 90°.
Proof:
A
LABC = LACB (i)
LACB LADB (ij)
LADB = LABC
LEDB + LEBD = LABD (iii)
But LABD 90° (iv)
LEDB + LEBD 90°
LBED = 90° (v)
D

26. A chord BC and the diameter from A meet


at a point P such that OC = CPo If L CPO
aO, prove that LAOB = 3ao. This is a
method of trisecting an angle by
construction.

27. AB and A C are equal chords of a circle. AD and BE are parallel chords through A
and B respectively. Prove that AJS is parallel to CD.

(~ ABCD is a parallelogram.
',I Prove that 6ABE is isosceles. D¥-----'~-I

29. Two chords AB and CD intersect


at point P outside the circle.
Prove that 6APD III 6CPB.

----------------------------~~~~~~--
---------------------~===.

20/PLANE GEOMETRY 9

30. AB and DC are equal chords of a circle. Prove


that
(a) l:::.CDE and l:::.BAE are congruent,
(b) l:::.EDA is isosceles, D
(c) l:::.ADC and l:::.DAB are congruent,
(d) AC and BD are equal chords.

Cn. AB is a chord of a circle, centre O. The circle with AO as diameter cuts AB at C.


',j Prove that C is the mid-point of AB.
32. In a triangle APB, the side AB remains fixed and the vertex P moves in a plane, what
is the locus of P if angle APB remains the same size? .
33. In a triangle APB, angle APB is a right angle. What is the locus of P in a plane?
/j4) A, B, C and D are four points in order on a circle, centre O. If L BAC 28 0 and
' / L OBA = 53 calculate the angles OAC,
0
, ACB and ADC.
,r\
('3S. Two perpendicular chords AB and CD of a circle, centre 0, intersect at E. Prove that
'-"
LAOD + LBOC = 180°.

20.2 Cyclic quadrilaterals


A cyclic quadrilateral is a quadrilateral whose four vertices lie on the circumference of a
given circle.
Theorem:

(a) The opposite angles of a cyclic quadrilateral are supplementary.


(b) If a side of a cyclic quadrilateral is produced, the exterior angle so
formed is equal to the interior opposite angle.
D

Given: ABCD is a cyclic quadrilateral.


(a) To prove: b + d 180
a + c = 180 E
Construction: Join centre 0 to A and C.
Proof: x = 2d (Angle at centre is double the angle at the circumference)
y = 2b (Same reason)
:. x + y 2(b + d)
But x + y 360
:. b + d = 180
Similarly, by joining centre 0 to Band D we can prove that
a + c 180
(b) To prove: L CBE = LADC
Proof: L CBE + L CBA 180 0 (straight angle)
LADC + LCBA 180 0 (opposite angles supplementary)
:. LCBE LADC
,
10 NEW SENIOR MATHS: THREE UNIT COURSE

EXAMPLE 5

Theorem: (Converse of the previous theorem)

If the opposite angles of a quadrilateral are supplementary, the


quadrilateral is cyclic

Given: ABCD is a quadrilateral with ~ __...;:::,~......,c


LDAB + LDCB = 180 0
To prove: ABCD is cyclic
Construction: Suppose the circle through B, A and D
does not pass through C; let it cut DC
(produced if necessary) at E. Join EB.

Proof: LDAB + LDEB = 180 0 (opposite angles supplementary)


LDAB + LDCB 180° (given)
:. LDEB = LDCB
But this is impossible, since an exterior angle of a triangle cannot be equal
to an interior opposite angle.
:. E must coincide with C
Hence, the circle through D, A and B must pass through D.
i.e. ABCD is cyclic.
This converse provides us with the test for four concyclic points:

Four points A, B, C and D in order are concyclic if the opposite angles of


the quadrilateral ABCD are supplementary.

EXAMPLE 6
Given: LDAC 45°, LBAC 35°, LABC 85° find x and y.
L DAB = 80° (45 0 + 35°)
L DCB = 100° (opposite angles
supplementary)
LA CB = 60° (sum of angles of
6ABC 180°)
:. LDCA = 40° (LDCB - LACB)
:. x 40
y = 95 (opposite angles Band Dare
supplementary)
20/PLANE GEOMETRY 11

EXAMPLE 7
Two circles intersect at A and B. PAQ and XBYare straight lines through A and B cutting
the circles at P, X and Q, Y respectively. Prove that PX is parallel to Q Y . _ _~
--A
R.--J~---=::1K---q

Construction: Join AB; produce AP to R


Proof: Quadrilaterals PABX and QABYare
cyclic.
LABX = LRPX (exterior angle equals interior opposite)
L ABX = L .ltQY (exterior angle equals the interior opposite angle)
:. LRPX = LAQY
PX II QY (corresponding angles equal)

EXAMPLE 8
ABCD is a cyclic quadrilateral and AB is produced to E. The bisector of angle ADC cuts
the given circle around ABCD at F. Prove that FB (produced if necessary) bisects the
angle EBC.
Proof:
L FBC = L CDF (angles in same segment) E
L EBF = L ADF (exteri'or angle of cyclic
quadrilateral FBAD equals
interior opposite angle)
But L ADF = L CDF (given)
LFBC = LEBF

EXERCISES 20(b)
Find the value of the pronumerals in each of the following. (0 is the centre of the circle)

1. 2. 3.

4. 5. 6.

-~~---~ -
.... ... - - - - - - -
12 NEW SENIOR MA THS: THREE UNIT COURSE

7. 8. 9.

10. 11.

13.

16. State the reasons (i), (ii), (iii), (iv) in the steps of the
following proof.
Given: AB = AC
To prove: BC II ED
Proof: LABC LACB (i) A
LABC + LEBC = 180 0 (ii)
LEBC + LEDC = 180 0 (iii)
LABC = LEDC
LACB = LADE
BC II ED (iv)
17. ABC is an isosceles triangle with AB = AC. P and Q are points on AB and AC
respectively and PQ is parallel to BC. Prove that the quadrilateral PBCQ is cyclic.
18. Two circles intersect at A and B. PAQ and XBYare parallel straight lines through A
and B cutting the circles at P, Q and X, Y respectively. Join AB and prove
LPXY = LPQY. 0.--__

c
19. ABCD is a cyclic quadrilateral with AB = AD. If
L ABD = xo, prove that L DCB = 2x o.
20/PLANE GEOMETRY 13

20. State the reasons in the steps of the following proof.


Given: A cyclic quadrilateral ABCD with
ABIIDC.
To prove: L ADC = L BCD
Construction: Produce AB to E
Proof: L CBE = L BCD (i)
L CBE = L ADC (ii)
LADC = LBCD
21. Prove that a parallelogram which is cyclic must be a rectangle.
22. ABC is an'isosceles triangle with AB = AC. A straight line through C perpendicular
to AC meets the bisector of L BAC at D. Prove that the points A, B, D, Care
~I I concyclic.
",V @. AB and AC are two chords of a circle on opposite sides of centre O. P and Q are the
mid-points of AB and AC respectively. Prove that A, P, 0 and Q are concyclic.
24. ABCD is a cyclic quadrilateral in which AD = DC = CB. Prove that
.\~'l\ (a) LADC =. LDCB (b) DC II AB
~,...t ..!\.

v·~25. C is any point on a circle diameter AB. P and Q are points on the minor arcs AC and
BC. Prove that L APC + L CQB = 3 right angles.
26. ABCD is a cyclic quadrilateral with AB II DC. Prove that LDAB = LABC.
27. A, B, C, D are four points in order on a circle. If AB is a diameter and LADC =
LBCD, prove that (a) DC II AB (b) LDBA = L CAB
-\ ,i:~. to
,~
A, B, C are three points on a circle and .6-ABC is acute-angled. AD is drawn parallel
BC and CD is drawn parallel to BA. Prove that D cannot lie on the circle. (D is the
point of intersection of the two parallels).
29. ABCD is a rectangle. The line through C perpendicular to AC meets AB and AD
produced at P and Q. Prove that P, B, D, Q are concyclic.

20.3 Tangents to a circle _ _ _ _ _ _ _ _ _ __

Fig. 20-2
Figure 20-1 shows a series of parallel secants AB. Observe that, as they move further away
from the centre of the circle, the points A and B draw closer together until finally the two
points A and B take up the same position. Let us call this point P. The secant in this
position is called a tangent to the circle and P is called the point of contact. \
14 NEW SENIOR MATHS: THREE UNIT COURSE

Definition:
A tangent to a circle is a straight line that touches the circle in one point
only.

Figure 20-2 shows that, arguing from symmetry, a tangent and the radius at the contact
point are at right angles.

AxioFm~:__________________________________________________~

A tangent to a circle is perpendicular to the radius drawn at the point of


contact.

Theorem:

The tangents to a circle from an external point are equal in length.

Given: PQ and PR are tangents to the circle centre O.


To prove: PQ = PR
Construction: Join OQ, OP, OR
Proof: In6POQ and 6POR
OQ OR (equal radii)
OP = OP (common)
LOQR LORP (right angles)
l::,.POQ =l::,.POR (RHS)
PQ PR
Also L OPQ L OPR
and L POQ L POR

EXAMPLE 9
From a point P 20 cm from the centre of a circle of radius 12 cm, a tangent is drawn.
Calculate the length of the tangent.
Op2 = OQ2 + PQ2 (Pythagoras' Theorem)
400 = 144 + PQ2
PQ2 = 256
PQ = --./256 .
= 16
The length of the tangent is 16 cm
20/PLANE GEOMETRY 15

Theorem:

If two circles touch, the line joining their centres passes through their
point of contact.
x x

y y
Fig. 20-3 Fig. 20-4

Given: A and B are the centres of two circles which touch at P (externally in fig. 20-3,
internally in fig. 20-4).
To prove: A, Band P are collinear.
Proof: Since XY is a tangent to circle, centre A L XPA a rt. angle.
Since XY is a tangent to circle, centre B L XPB = a rt. angle.
:. PA and PB are both perpendicular to XY.
:. A, Band P must be in a straight line.

20.4 Alternate segment _ _ _ _ _ _ _ _ _ __


XY is a tangent to the circle at P. The chord
PQ divides the circle into two segments
(sections) PQR and PQS.
The segment PQR is said to be alternate to
LQPY
The segment PQS is said to be alternate to
LQPX
x y
Theorem:

The angles between a tangent and a chord through the point of contact
are equal to the angles in the alternate segments.

x y

Given: XY is a tangent at P to the circle, centre 0, and PQ is a chord through P.


To prove: (a) L QPY = any angle in alternate segment PTQ
(b) L QPX = any angle in alternate segment PSQ
16 NEW SENIOR MATHS: THREE UNIT COURSE

Construction: Draw the diameter PR and join RQ


Proof: (a) L QPY + L QPR = 90° (tangent ..1 to radius)
L PRQ + L QPR = 90° (L PQR = 90°, angle in a semi-circle)
LQPY = LPRQ
L QPY = any angle in segment PTQ.
(b) L QPX + L QPY = 180° (straight angle)
. L PSQ + L PRQ = 180° (PSQR is cyclic)
But L QPY = L PRQ (proved in (a»
LQPX = LPSQ
L QPX = any angle in segment PSQ

EXERCISES 20(c)
Find the value of the pronumerals in each of the following. In each case 0 is the centre of
the circle.
1. 2. 3.

4. 5. 6.

7. 8. 9.

13.

14. Use the fact that two tangents, drawn to


a circle from an external point, are equal
in length to find the perimeter of
LABC. B 3cm c
20/PLANE GEOMETRY 17

15. In the right-angled triangle ABC, BC (A


24 cm, DB = 3 cm, AD = x cm. Use the (
Theorem of Pythagoras to form an (
eq~~tion in x and then solve it. \

\, ~L
Hj..J" ['S U-~t-=----------..;;::o..c
\\\'-" - F
\
r~\ ;
\J~. Calculate (a) the distance of a chord of length 24 cm from the centre of a circle of
',.' radius 13 cm (b) the length of the tangents drawn from an external point to the

1~~~7'e:::::i:s :: :::i:~:~'and 8 cm touch each other externally. Calculate the length of


'-l' the common tangent.
-
.. '1. If the radii of two intersecting circles are 17 em and 10 cm and the length of the
common chord is 16 cm, calculate (a) the length of the line joining the centres of the
/; circles (b) the length of the common tangent.
If the radii of two intersecting circles are 51 cm and 74 cm and the length of the
common chord is 48 cm, calculate (a) the length of the line joining their centres
(b) the length of the common tangent. C

B
1120J ABCD is a cyclic quadrilateral and FAE D
V is a tangent at A. Angle DAE 50° .
- Calculate the size of (a) L BAF
(b) L BCD given also that BD II FE.
F A E

21. From a point P outside a circle two straight lines are drawn. The first, PAB, cuts the
circle at A and B. The second, PTX, is a tangent to the circle at T. A T and BT are
joined. Prove that L BTX = L APT + LA TP.
22. Two points A and B are taken on a circle and C is the other end of the diameter through
A. If AE is the perpendicular from A on to the tangent at B, prove that AB bisects
angle CAE.
23. Two circles intersect at A and B. The tangent to the first circle at A cuts the second
circle at C and the tangent to the second circle at A cuts the first circle at D. Prove that
t:::,ABC and 6.DBA are similar. Q

24. The sides of the quadrilateral PQRS are


tangents to the circle. Calculate the size p
of each angle of the quadrilateral
ABCD.
25. PQ is a tangent at Q and PS II QR. R
Prove that L SPQ = L QSR.

Q P
26. Two circles intersect at A -and B. The tangent to the second circle at A cuts the first
circle at C and the tangent to the first circle at B cuts the second Circle at D. Prove that
AD is parallel to BC.
18 NEW SENIOR MATHS: THREE UNIT COURSE

27. Two circles touch internally at P. The lines PAB and peD cut the smaller circle at A
and C and the larger circle at Band D. Prove that, by drawing the common tangent at
P, AC is parallel to BD. Q

28. Two circles touch internally at A. The


tangent at P on the smaller circle cuts the R
larger circle at Q and R. Prove that AP
bisects L RA Q.
p
29. The diagram shows 3 circles each of A
radius 2 cm. Calculate the height of the
highest point P above AB.

B
30. Two circles, centres a and P, touch externally at A. The direct common tangent
touches the circles at X and Y respectively. The common tangent at A meets the direct
common tangent at Z. Prove that (a) ZX = ZY (b) L OZP = 90°
31. Two circles intersect in X and Y. The tangent at X to the first circle cuts the second
circle at A and A Yproduced cuts the first circle at B. Prove that XB is parallel to the
tangent at A to the second circle.
32. ABC is a triangle inscribed in a circle. The tangent at C meets AB produced at P and
the bisector of LACB meets AB at Q. Prove that PC = PQ.
33. PA and PB are two tangents to a circle and X is the mid-point of the minor arc AB.
Prove that (a) XA bisects L PAB (b) XB bisects L PBA.
34. AB and CD are parallel chords of a circle. The tangent at B meets CD produced at E.
Prove that 6ABC and 6DBE are similar.
35. Two circles intersect at A and B. A line through A cuts the first circle at P and the
second circle at Q. From an external point T a tangent TP is drawn and TQ produced
meets the second circle again at R. Prove that the points P, T, R, Bare concyclic.
36. Ten equal circular discs can just be placed tightly within a frame in the form of an
i
equilateral triangle. Nine of them touch the frame; the tenth, in the centre, touches six
of the other discs. If the internal sides of the frame are each of length measure R,
express the radius of each circular disc in terms of R, and show that the outside
perimeter of the figure which remains when the frame is removed is
g (3 - .,)3) 1rR

37. Six equal circular discs are placed so that their centres lie on the circumference of a
given circle and each disc touches its two neighbours. If the radius of the given circle
is r, find:
(a) the radius of a seventh disc which will touch each of the six,
(b) the length of the outside perimeter of the given figure.
38. Inside a circle K of radius length measure R, three circular discs A, Band C each of
radius r are placed so that each touches the other two and K. Express R in terms of r.
In the space between K, A and B, another circular disc D is placed which just
touches K, A and B. If its radius is s, show that
(6 + .,)3)s = (2 + .,)3)r
20/PLANE GEOMETRY 19

39. A playground is composed of three equal circular portions, each of which touches the
other two externally, and the area between them. If the circles have radius r, find:
(a) the length measure of the perimeter of the playground and
(b) its total area measure.
If a boy stands in the centre of the playground, what is the greatest distance another
boy in the ground can be from him?
40. Three circular discs whose radii are 10 cm, 20 cm and 30 cm respectively touch each
other. Calculate:
(a) the length of the perimeter
(b) the area enclosed between the discs.
41. Three circular discs, each of radius length a units, touch each other. Calculate in
terms of a
(a) the area enclosed between the discs
(b) the radius of the largest disc which can fit into the space between the three discs.

EXERCISES 20(d)
(These questions are an extension to the work in Chapter 4).
1. ABCD is a parallelogram and a is the mid-point of AB. If DC 2AD, prove that
LDaC = 90°.
2. P, Rand T are points on AB and Q, S are points on AC in the triangle ABC. If
AP = PQ QR = RS = ST prove that L CST 5 L AQP.
3. ABC is an equilateral triangle. K is a point on BC such that L CAK = 3 L KAB. A
line KL is drawn perpendicular to BC to meet AB at L. Prove that AL = LK.
4. ABCDE is a regular pentagon. BD cuts CE at P. Prove that BP = BA.
5. ABC is a triangle. AP is the perpendicular from A to the bisector of L ABC. If PQ is
drawn parallel to BC to cut AB at Q, prove that AQ QB = PQ.
6. AD is an altitude of f',ABC. If BD = x cm, AD = 2x cm and DC = 4x cm, prove
that L BAC = 90°.
7. In the figure, prove that
x = 540 - (a + b + c)

8. The altitudes BD and CE of f',ABC meet at H. If HB = HC, prove that AB = A C.


9. ABCD is a parallelogram. BP and DQ are two parallel lines cutting AC at P and Q
respectively. Prove that BPDQ is a parallelogram.
10. a is a point inside an equilateral f',ABC; OAP is an equilateral triangle such that a
and P are on opposite sides of AB. Prove that BP = ac.
11. ABC is an isosceles triangle with AB = AC. The bisector of L ABC meets AC at D.
P is a point on AC produced so that L ABP = LADB. Prove that BC = CPo
12. ABC is an isosceles triangle with AB = AC; BA is produced to E. The bisector of
L ACB meets AB at D. If L ACD XO show that L CDA 3xo and L CAE = 4x O.
20 NEW SENIOR MATHS: THREE UNIT COURSE

13. AB = BC = CD; BCFE is a parallelo-


gram with BE = 2BC. Prove that ED 1-
AF.

14. ABCD is a quadrilateral with LDAB =


90° = L DCB. The bisectors of L ADC A
and LABC meet the diagonal AC at E
and F respectively. Prove that DE and
BF are parallel.

A 8
15. ABCD is a parallelogram. P is the mid point of AB; CP and DA are produced to meet
at Q; DP and CB are produced to meet at R. Prove that QR = CD.
16. ABC is an acute-angled triangle. Squares BAHK and CAXYare constructed on AB
and AC respectively and outside the triangle. The diagonals HB and XC meet when
produced at P. Prove that L BPC + L BA C 90° .
17. The diagonals AC and BD of the square A BCD intersect at O. The bisector of L BAC
meets OB at X and BC at Y. Through C, a line CZ is drawn parallel to the diagonal
DB to meet A Y produced at Z. Prove that (a) 6CYZ is isosceles (b) OX !CY

18. ABCD is a trapezium with AB II DC. The diagonals intersect at O. The line through 0
parallel to AB cuts AD and BC at P and Q respectively. Prove that PO QO.

19. ABC is an equilateral triangle. BC is produced to E so that CE = !BC. If CE x


units, prove that AE = --.fix.
20. ABCD is a parallelogram. Through A and C lines are drawn parallel to BD and
through Band D lines are drawn parallel to AC. These four lines form a
parallelogram PQRS. Prove that
(a) A, B, C, D are the mid-points of the sides of the parallelogram PQRS.
(b) each side of ABCD is parallel to one of the diagonals of PQRS.
21. ABC is an isosceles triangle with AB = AC; BC is produced to D so that CD CA;
CD is the produced to E so that DE = DA; EA is then produced to any point F. If
LAEB x o , show that LFAB = 5xo.
22. Squares ABXYand ACPQ are drawn outwardly on the sides AB and AC respectively
of 6ABC right-angled at A. Perpendiculars to BC at Band C meet XY and PQ,
produced if necessary, at M and N respectively. Prove that
(a) 6ABC, 6BXM and 6CPN are congruent
(b) BCNM is a square.
23. ABCD is a parallelogram. ABXY and ADPQ are squares drawn outwards on AB and
AD respectively. Prove that YQ = AC.
20/PLANE GEOMETRY 21

24. ABC is an equilateral triangle. Points X, Yand Z are on BC, CA and AB respectively
such that XYZ is an equilateral triangle. Prove that X, Yand Z are the mid-points of
the sides.
25. ABCD is a quadrilateral. A line AF parallel to BC meets BD at F. A line BE parallel
to AD meets AC at E. Prove that EF is parallel to CD.
26. ABCD is a parallelogram. A line through A cuts DB, CB and DC produced at E, F
and G respectively. Prove that (a) 6ADE 1116FBE (b) 6ABF III 6 GCF.
CHAPTER 21

Trigonometry

21.1 Applied trigonometry ---------II!I


This section is an extension of the work done in Chapter 8. c

EXAMPLE 1
In the figure, AC = 15 em, AB = 7 em,
DB 10 cm and L DBA = a. Find the
perpen'dicular distance CE of C from DA in
terms of a and evaluate it when a 25°.
Applying the cosine rule to 6.DBA,
DA 2 102 + 72 - 2. 10. 7 cos a
= 149 - 140 cos a D
DA .,,)149 - 140 cos a (1)
Applying the sine rule to 6.DBA.
10 DA
sin a
. A
sm 10 sin a
= --=:---- (2)

In the right-angled 6. CAE,


CE 15 sin A
150 sin a
from (1) and (2)
.,,)149 - 140 cos a
150 sin 25°
when a = 25°
.,,)149 - 140 cos 25°
= 13'48 (using a calculator)
Distance is 13'48 cm

EXAMPLE 2
From a point P due south of a vertical tower, the angle of elevation of the top of the tower
is 20° and from a point Q due east of the tower it is 35°. If the distance from P to Q is
40 metres, find the height of the tower.
21/TRIGONOMETRY 23

8
We have three right-angled triangles, namely
t:,PAB and t:,QAB in vertical planes and
t:,PAQ on the horizontal plane.
Let the height of the tower be x m.

In t:,PAB,
tan 20° = ~ or tan 700
. = PA
x
PA
x PA = x tan 70°
:. PA = tan 200
x
= 2·747x
0·3639
= 2·747x

Int:,QAB,
QA
tan 35° = ~ or tan 55°
QA x
x QA x tan 55°
:. QA
tan 35° p AI-I-------='-=-~Q
x 1·428x
0'7002
= 1·428x "428x
a----.;.~.:::..;.;..-__,. Q
Now apply the Theorem of Pythagoras to t:,PAQ
(2'747x)2 + (l'428x)2 = 40 2
7'546x 2 + 2'039x 2 = 1600
9'585x 2 = 1600
{1600
x =-J 9-585
= 12·92 P
The height of the tower is approximately 12·92 m.
Note how right angles are drawn on a horizontal plane in a 3-dimensional situation.

EXERCISES 21 (a)
1. P, A, Band C are four points in a plane such that the angles BPA and CPA are
obtuse' and on opposite sides of PA. PA 8 cm, BP = 10 cm, PC 12 cm,
AB = 14 cm and A C = 18 cm. Calculate the length of BC and the area of the triangle
ABC.
2. P, A, B, C are four points, in order, on a straight road that runs up a hillside and
makes a constant angle of 10° with the horizontal. A flagpole whose height is h m
stands at P. From A and B, the top of the flagpole has elevations of 30° and 5°
respectively above the horizontal. If AB is 100 m long, what is the height of the
flagpole?
If BC is also 100 m long, what is the elevation of the top of the flagpole from C?
24 NEW SENIOR MA THS: THREE UNIT COURSE

3. From a point, P, a man observes that the angle1:lf elevation of the top of a cliff, A, is
40°. After walking 100 m towards A along a straight road, inclined upwards at an
angle of 15° to the horizontal, the angle of elevation of A is observed to be 50°. Find
the vertical height of A above P.
4. A ship, sailing in a direction 065°T, observes two lighthouses in a line due north.
After travelling 4 km one of the lighthouses bears 285°T and the other 315°T. Show
that the distance between the lighthouses is
4[sin 70° _ sin 40
sin 45°
0

sin 75°
J
5. A and B are the feet of two vertical towers each 30 m high. B lies due north of A.
From a point, P, due east of A and in the same horizontal plane, the angles of
elevation of the tops of towers A and Bare 45 ° and 36 ° 52 I respectively. Calculate the
distance
(a) from P to the foot of each tower
(b) between the two towers.
6. From a point P an observer finds that the angle of elevation of the top of a vertical
tower is a 0. After walking x metres horizontally towards the foot of the tower, the
observer finds that the angle of elevation is {30. If the height of the tower is h metres,
show that
x h(cot aO
7. A flagpole 5 m high stands on top of a vertical tower. From a point on the ground, the
angles of elevation of the top and bottom are 68° and 62° respectively. Show that the I'
height of the tower, h m, is given by
h = 5 tan 62°
tan 68° tan 62°
8. Three points P, Q and R lie in a horizontal plane. Angles RPQ and RQP are a and {3
respectively. If PQ is x units in length, show that the perpendicular distance y from R
to PQ is given by , D
x tan a tan
y = tan a + tan
9. An open rectangular tank a units deep c
and b units wide holds water and is tilted
so that the base BC makes an angle (j A
with the horizontal. When BC is
returned to the horizontal, show that the
· a 2 cot (j .
dept h 0 f t he wa ter IS umts.
8
10. A conical vessel, vertex at 0, semi-
Cross-section of tank
vertical angle a, is suspended from a
point A on the rim of the base; G is a
point on the axis OC of the cone, such
2
that OG = 3" ~C.
If the vessel rests with G vertically
below A, show that the angle {3 which
AO makes with the vertical is given by
2 tan a
tan {3 = 1 + 3 tan 2 a o
211TRIGONOMETRY 25

11. ABC is a triangle in which AC = 7 cm. A circle, centre B and radius BC, cuts AB
internally at D. AD = 5 cm, DC = 4 cm. Calculate the length of BC.
12. A and B are two towers, B being 4 km due east of A. The true bearings of a flagpole,
C, from A and Bare (x0 east of north and exO west of north respectively. The true
bearings of a second flagpole, D, from A and B are (ex + {l)0 east of north and
(ex - mowest of north respectively. Draw a sketch to indicate the positions of A, B,
C and D. Assuming that A, B, C and D are on level ground, and that ex = 25, (l 10,
find the distance between C and D.
13. A, B, C and D are four points, equally spaced on the circumference of a horizontal
circle of radius length 3 cm. P is a point above the circle such that PA = PD = 4 cm
and PB = PC = 6 cm. Calculate the size of angle APC.
14. The angle of elevation of the top of a building from a point P due east of it is 40° and
from a point Q due south of P the angle of elevation is 30°. If the distance from P to
Q is 20 metres, find the height of the building.
15. From A to B, two railway stations, the line runs south-west. Two spires, 3 km apart
are both N-W of A. From B, the bearings of the spires are N 7~0 E and N 37~0 E.
Show that AB = 6 sin 52!0 cos 7!0. Find the average speed of the train if it takes
4 minutes to travel from A to B.
16. Two buildings of equal height are 40 m apart and at a point on the horizontal line
joining their feet the angles of elevation of the tops of the buildings are 47° and 28°.
Show that the height h of the buildings is given by
h 40 tan 47° tan 28°
tan 47° + tan 28°
17. An observer's eye is 2 m above the ground. A vertical pole, fixed in the ground,
subtends an angle of 45° at the observer's eye and the angle of depression of the
foot of the flagpole from the eye is 15°. Show that the height of the pole is
2(1 + tan 30° tan 7.5°) m.
18. From a point A, two points Band C are in line in a direction 049°T. From a point D
100 m from A in a direction 139°T, B is in a direction 352°T and C is in a direction
022°T. Calculate the distance between Band C.
19. From a point A, the angle of elevation of the top of a tower due north of it is 20° .
From B, due east of the tower, the angle of elevation is 18°. A and Bare 100 m apart.
Show that the height h of the tower is given by
100
h = .J(tan2720 + tan 2 700) C

20. In the figure, AB == ~5 cm, BC == 27 cm,


A C := 40 cm. Calculate the length of
AD.

A D

21. ABCD is a cyclic quadrilateral in which AB 5 cm, BC = 6 cm, CD 7 cm,


13
AD = 8 cm. Show that cos L ADC = 43
26 NEW SENIOR MATHS: THREE UNIT COURSE

22. A and B represent successive positions ~r-______________-,B


of an aeroplane flying horizontally at an
altitude of 2000 metres. From a point P
the angles of elevation of A and Bare 2000m

L~---I--1
15° and 25° respectively. If LDPC =
60°, calculate the distance AB.

P
23. The elevation of a hill at a point P due east of it is 42 ° and at a point Q due south of it,
the elevation is 22°. If the distance from P to Q is 400 metres, find the height of the
hill.
24. 0, A and Bare 3 points in that order in a straight line. The bearings of A and B from
o are 020oT. From a point P, 4 km from 0 in a direction NW, the bearings of A and
Bare l12°T and 064°T respectively. Calculate the distance from A to B.
25. From a point P an observer finds that the angle of elevation of the top of a vertical
tower is ex 0. After walking x m horizontally towards the foot of the tower, the
observer finds that the angle of elevation is {30. If the height of the tower is h m, prove
th~ A
h = x sin exO sin {30
sin ({3 - ex)O

26. Using the sine rule, show that ~


y
= ..fj and also

that area 6ABC = ..fj


area 6ADC 2
B x C y D
27. The side of a hill may be regarded as a plane making an angle of 20° with the
horizontal. .
(a) From a point A at the base of the hill, I walk 100 m up the hill along the line of
steepest slope to the point B. Find the vertical height of B above A.
(b) A tower 40 m tall is constructed at B. Find the angle subtended by the tower at my
eyes, which are 1·5 m above the ground, when I stand at A.
(c) A straight road is to be constructed on the hill-side with a slope of 1 m 10 i.e. 1 m
vertical rise for every 10 m along the road. Find the angle this road must make
with the line of steepest slope on the hillside.

21.2 Addition theorems _ _ _ _ _ _ _ _ _ __


y
In this section, we shall derive formulae
for finding trigonometric functions of sums
and differences of angles, Le. formulae for
cos (0 ± 1», sin (8 ± 1», tan (8 ± 1» in P(cos 0, sin 0)
terms of trigonometric functions of the
angles 8 and 1>. We obtain a formula for AX.
cos (8 - 1» first and deduce formulae for
the others from it. Fig. 21-1
211TRIGONOMETRY 27

Starting from A, we mark off, on a unit circle, LA OP = 0 and LA OQ = <1>(0 > <I> for
convenience) as shown in fig. 21-1. The coordinates of P and Q are thus (cos 0, sin 0) and
(cos <1>, sin <1» respectively, and L POQ = 0 - <1>.
Applying the distance formula, which you have met in the study of coordinate
geometry, we get
PQ2 = (cos 0 cos <1>)2 + (sin 0 - sin <1>)2
= 2 2(cos 0 cos <I> + sin 0 sin <1» (i)
Applying the cosine rule to l:c,POQ, we get
PQ2 = 1 + 1 - 2 cos L POQ
=2 2cos(O <1» (ii)
Equating (i) and (ii), we get
2 - 2 cos (0 - <1» = 2 - 2(cos 0 cos <I> + sin 0 sin <1»
Le., I cos (0 - <1» = cos 0 cos <I> + sin 0 sin <I> I (1)

This formula is true for all values of 0 and <1>.


If. in place of <1>. we write -<1>, remembering that cos (-<I» cos <I> and sin (-<I» = -sin <1>,
we get
cos (0 + <1» = cos 0 cos (-<1» + sin 0 sin (-<1»

i.e., I cos (0 + <1» = cos 0 cos <I> - sin 0 sin <I> (2)

In place of 0, put 90 0 0 in (1)


cos [90 0 - (0 + <1»] = cos (90 0 - 0) cos <I> + sin (90 0
- 0) sin <I>

i.e., [ sin (0 + <1» = sin 0 cos <I> + cos 0 sin <I> (3)

In place of <1>, write -<I> in (3).


sin (0 - <1» = sin 0 cos <-<1» + cos 0 sin <-<1»
i.e., I sin (0 - <1» = sin 0 cos <I> - cos 0 sin <I> (4)

tan (0 + <1» = sin (0 + <1»


cos (0 + <1»
sin 0 cos <I> + cos 0 sin <I>
cos 0 cos <I> sin 0 sin <I>
Divide numerator and denominator by cos 0 cos <I> o. "*
Then tan (0 + <1» = tan 0 + tan <I> (5)
1 tan 0 tan <I>

~in (0 - <1»
tan (0 - <1» =
cos (0 - <1»
sin 0 cos <I> - cos 0 sin <I>
cos 0 cos <I> + sin 0 sin <I>
Divide numerator and denominator by cos 0 cos <I> o. "*
Then tan (0 - <1» = tan 0 - tan <I> (6)
1 + tan 0 tan <I>
28 NEW SENIOR MATHS: THREE UNIT COURSE

EXAMPLE 3
(a) sin (3x + 2y) sin 3x cos 2y + cos 3x sin 2y
(b) cos (2a + (3) = cos 2a cos {3 - sin 2a sin {3
tan A + tan 45 0 1 + tan A
(c) tan (A + 45°)
1 - tan A tan 45 0
(d) sin (2a + (3) cos {3 - cos (2a + (3) sin {3 sin [(2a + (3) {3]
sin 2a
(e) cos (2P 3a) cos 29 + sin (20 - 3a) sin 29 cos [(29 3a) - 29]
= cos <-3 a)
= cos 3a

21.3 Double angle formulae _ _ _ _ _ _ _ __


In place of r./>, write 9 in (2).
Then cos (0 + 9) = cos 0 cos 0 - sin 0 sin 0

i.e., cos 2 0 - sin 2 0


= 2 cos 2 0 - 1. because sin 2 0 cos 2 0 (7)
= 1 2 sin 2 0, because cos 2 0 = sin 2 9

In place of r./>. write 0 in (3).


Then sin (0 + 0) = sin 0 cos 0 + cos 0 sin 0

i.e., 2 sin 0 cos 0 (8)

In place of r./>, write 0 in (5).


Then tan (0 + 0) = tan 0 + tan 0
I-tanOtanO

2 tan 0
Le., tan 20 = 1 lan 2 0' tan 0 *' ± (9)

Denoting tan ~ by I. (9) can be written as

2t
tan 0 =1_ 12 ' 1 *' ± 1

sin 0 and cos 0 may also be express~d in terms of tan ~

The derivation of these will be left as an exercise. Are they true for all values of 07
,\

211TRIGONOMETR Y 29

EXAMPLE 4
cos 30 sin 30
Prove that - ,-a-
Sin v
+ --a
cos v
= 2 cot 20

L.R,S, = cos 30 cos 0 + sin 30 sin 0


.. cos
=c_o-;s-"(3-;-0----::-:-o-0~)
~ sin 20
2 cos 20
sin 20
= 2 cot 20

EXAMPLE 5
Prove that ~~-'=-'-~--C-_ _

L.H.S. ~
ex - + cos ex
sin ex (2 cos ex + 1)
cos ex (2 cos ex + 1)
sin
= cos
ex, '
ex
If 2 cos ex + 1 0 '*
= tan ex

EXAMPLE 6

If tan A = - ~,90 0 < A < 180 0 and cos B = 153' 0° < B < 90° write down the value of:
(a) sin (A - B) (b) cos 2A (c) tan (A + B)
y y
12
3

-4 o x o 5 x
3 'B 12
sin A
5 Sin = 13
4 5
cos A -5 cos B =0
3 tan B 12
tan A
4
(a) sin (A - B) = sin A cos B - cos A sin B
3 5 4 12
= 5'0 + S'O
63
65
tanA + tanB
(b) cos 2A = cos 2 A - sin 2 A (c) tan (A + B) = 1 _ tan A tan B
16 9 +
= 25 - 25
7 ::::
33
= 25 56
30 NEW SENIOR MATHS: THREE UNIT COURSE

EXERCISES 21 (b)
Expand: (1. to 6.)
1. sin (A + 2B) 2. sin (2x - y)
3. cos (2x - 3y) 4. cos (20 + 60°),
5. tan (8 + a) 6. tan (A - 135°)
Simplify: (7. to 16.)
7. sin A cos (A B) + cos A sin (A - B)
8. cos (8 + a) cos (8 - a) + sin (8 + a) sin (8 - a)
9. sin 2A cos A - cos 2A sin A
10. cos 60° cos 30° - sin 60° sin 30°

11. tan 0 - tan 20°


-,----~----c:-

tan 2a + tan a
12. 1 tan 2a tan a

13. sin (2A + B) cos (A + B) - cos (2A + B) sin (A + B)


14. cos (38 + a) cos (28 + a) + sin (30 + a) sin (20 + a)

15. -0-----:----
tan x
tan x
16 tan (A + B) + tan C
• 1 - tan (A + B) tan C
17. Write down the expansion of sin (a + (3). In place of (3, write -(3, and hence show that
sin (a (3) sin a cos (3 cos a sin (3.
18. Write down the expansion of cos (A + B), and, from it, deduce the expansion of
cos (A B).

19. Write down the expansion of cos (0 </J). In place of 0, write (90° - O),'and hence
deduce the expansion of sin (0 + </J).

20. If 0 and </J are angles between 0° and 90°, 'and sin 8 = ~ and tan </J
7
24' find
without the use of tables, the value of each of the following:
(8) sin (0 - </J)
(b) cos (8 + </J)
(c) tan (0 - </J)

21. If tan A 4 and tan B ~, and A and B are acute angles, prove, without using
tables, that A - B = 45°.

22. If tan a = j and cos (3 = !i, where 0 < (3 < a < 90", evaluate, without using tables,
(8) sin 2a
(b) tan 2a
(c) cos (a - (3)
211TRIGONOMETRY 31

23. Using the expansion of


(a) sin (A + B), prove that sin 750 = ~ -J2 !
(b) tan (A + B), prove that tan 75° = 2 + -J3
24. Find, in simplest surd form, the value of
(a) cos 75°
(b) tan 15°
(c) cos 15°

25. If sin 0 = ~, 90° < 0 < 180°, evaluate, in surd form,


(a) sin 20
(b) cos 20
(c) tan 20
In which quadrant does 20 lie?
26. Express
(a) sin 30 in terms of sin 0
(b) cos 30 in terms of cos 0

27. Use the expansion of sin (A + B) to evaluate sin 195°.


Simplify: (28. to 35.)
sin 2A
28. (a) 1 + cos 2A

(b) i sin 20 tan 0

29. (a) cos 2 20 - sin 2 20


(b) cos 2 30° - sin 2 30°
30. (a) sin 4x cos 4x
(b) 1 + cos (180° + 20)
31. (a) sin x cos x cos 2x
(b) 2 sin 2x cos 2x

32. (a) (sin 0 + cos 0)2


(b) (sin A - cos A)2

33 . () 2 tan 0 h /J 22lo
a 1 _ tan 2 0 w en v ~

(b) sin 2 50° + sin 2 40°


x)

35. (a) 2 cos 2 3x - 1


1-(1- 0
(b) 1 + t 2 where t = tan 2
Prove the following identities: (36. to 56.)
36 sin 20 cos 0 cos 20 sin 0 _ t 0
. cos 20 cos 0 + sin 20 sin 0 - an
1

32 NEW SENIOR MATHS: THREE UNIT COURSE

37. sin A + c~s A tan B = tan (A + B)


cos A - sm A tan B
38. sin (0 + <X) sin (0 - <X) = sin 2 0 - sin 2 ex
sin (A - B)
39.
tan sin (A + B)
2
40. tan (45° + A) + tan (45° - A) = --=--:
cos

41. sin 3A

42. sin 20 + cos 0 + sin 0


cos = cos 0 - sin 0
43. cos 4x 8 cos 4 X - 8 cos 2 X +
44 • 1 -. cos x = tan ~
smx 2

45.
cos 0
.
+ sin 0 + cos 0
0
sin 0
. 0
= 2
sec
20
cos cos + sm
x
sin x + + tan 2
46.
1 - tan~
2
47. sin (A +B + sin A cos B cos C + sin B cos C cos A + sin C cos A cos B -
C)
sin A sin B sin C
What is the resulting identity if C is replaced by (90° B)?
48. cos (A + B + C) = cos A
cos B cos C - cos A sin B sin C - cos B sin C sin A -
cos C sin A sin B
What is the resulting identity if B is replaced by (90° C)?
sin A + sin (90° A) + 1 A
cot -
49. sin A - sin (90° A) + 1 2

50. si?3 0
sm 0
+
+
3
cos 0
cos 0
= ! sin 20
tan 2 0 - tan 2 ex
51. tan (0 + ex) tan (0 <X)
1 tan 2 0 tan 2 ex

52. cot (x + ) = cot x cot 1


y cot x + coty

2 cos 2o - - cos 0
53. 0
2 cos 2+ + cos 0 + cos 2
54. tan (0 + 45°) tan (0 45°) -1

55 tan 20 - tan 0 = tan2 0


• tan 20 + cot 0
211TRIGONOMETRY 33

56 1 tan 0 tan 20 = 4 cos2 0 - 3


• 1 + tan 0 tan 20
57. If A, Band C are the measures of the angles of a triangle, prove that
cos A cos B sin A sin B + cos C == 0

58. If tan A == ~. express in terms of p and q


(a) q sin A cos A + P sin 2 A
(b) p sin 2A + q cos 2A

59. Find tan x in terms of tan 0 if tan 0 = cos


cos
i: + x~
- x

60. Given sin 18° i(v'S - I), find cos 36° in surd form.

61. Using the double angle formula for tan O. find tan 22!° in simplest surd form.
62. If tan ex k tan (3, show that
(k - 1) sin (ex + (3) == (k + 1) sin (ex - (3)
21.4 Products as sums or differences _ _ _ __
In section 21.2 we established the addition theorems:
sin (0 + <1» = sin 0 cos <I> + cos 0 sin <I> (1)
sin (0 <1» = sin 0 cos <I> cos 0 sin <I> (2)
cos (0 + <1» = cos 0 cos <I> sin 0 sin <I> (3)
cos (0 - <1» cos 0 cos <I> + sin 0 sin <I> (4)
Add (1) and (2):
2 sin 0 cos <I> == sin (0 + <1» + sin (0 - <1» (a)
= sin (sum) + sin (difference)
Subtract (2) from (1):
2 cos 0 sin <I> sin (0 + <1» sin CO - <1» (b)
== sin (sum) - sin (difference)
Add (3) and (4):
2 cos 0 cos <I> = cosJO + <1» + cos (0 - <1» (c)
= cos (sum) + cos (difference)
Subtract (3) from (4):
2 sin 0 sin <I> == cos (0 - <1» cos (0 + <1» (d)
cos (difference) cos (sum)

21.5 S,lIms or differences as products _ _ _ __


I
Let U 0 + <I> and V 0 - <1>. and so .
o == U+V d) U~V
- 2 - an 'P = - 2 -
Substituting in (a), (b), (c) and (d), we get
sin U + sin V 2 sin U + V cos U - V (e)
2 2
'U - sm
' V. = 2 cos U+V.U V
sm - 2 - sm (f)

cos U + cos V = 2 cos U + V cos U - V (g)


34 NEW SENIOR MATHS: THREE UNIT COURSE

2 ~.
cos V cos U
(h)
U + V. U - V }
cos U - cos V - "m - 2 - sm - 2 -
Formulae (e), (f), (g), (h) can be remembered as:

(e) sin + sin = 2 sin (~sum) cos (~difference)


(f) sin - sin = 2 cos (~sum) sin (~ difference)
(g) cos + cos = 2 cos (~ sum) cos ( ~ difference)

(h) cos cos -2 sin (~sum) sin (~difference)


where the word 'difference' means 'first - second'.
EXAMPLE 7
Express the following as sums or differences:
(a) 2 cos 5x sin x (b) 2 sin 4A sin A (c) cos 30 cos 50
(a) 2 cos 5x sin x = sin (5x + x) - sin (5x - x)
= sin 6x - sin 4x
(b) 2 sin 4A sin A cos (4A A) - cos (4A + A)
= cos 3A cos 5A

(c) cos 30 cos 58 = 2:2 cos 38 cos 50

:21 [cos (38 + 58) + cos (30 50)]

2:1 [cos 80 + cos ( 20)]

= :21 [cos 80 + cos 20]

EXAMPLE 8
Convert the following sums or differences into products:
(a) sin 6x - sin 4x (b) cos 3A cos 5A (c) cos 80 + cos 20
(a) . 6x - sm
sm . 4x = 2 cos (6X +2 4X) sm
. (6X 2 4X)

= 2 cos 5x sin X-

(b) cos 3A - cos 5A = - 2 sm. (3A + 2 5A) sm


. ( ---:---
3A

= -2 sin 4A sin (-A)


= 2 sin 4A sin A
(c) cos 80 + cos 20 = 2 cos ( 80 + 20) cos (80 2 20)
2 cos 50 cos 3 0 .
What conclusion can be drawn from examples 7 and 81
211TRIGONOMETRY 35

EXAMPLE 9

tan 30

sin 0 + sin 30 + sin 50 (sin 50 + sin 0) + sin 30


cos (cos 50 + cos fJ) + cos 30
2 sin 3fJ cos 20 + sin 3fJ
- 2 cos 3fJ cos 2fJ + cos 3fJ
_ sin 3fJ (2 cos 2fJ + 1)
- cos 3fJ (2 cos 2fJ + 1)
= sin 33fJfJ if 2 cos 2fJ + 1
cos
'* 0
= tan 30
EXERCISES 21 (c)
Express the following as sums or differences: (1. to 20.)
1. 2 sin 40 cos 20 2. 2 cos 3A cos 5A
3. cos 4A sin 2A 4. sin (fJ + a) cos (0 a)
5. 2 cos (45° + A) sin (45° A) 6. cos (2fJ + a) cos (fJ - 2a)
7. 2 cos (3a + 2{3) sin (a - {3) 8. 2 sin fJ sin 2fJ

9. sin 4fJ sin (-2fJ) 10. sm


. 3x
2
+ y cos 3x-
----;;;:--"-

11. 2 sin 75° cos 15° 12. sin (A - B) sin (A + B)


13. cos 75° cos 45° 14. 2 sin (fJ + ¢) cos (0 ¢)

. A+B A-B
15. sm cos - 2 - 16. sin 100° sin 130°

17. 2 sin 30 cos 0 18. cos (0 + 2¢) sin (20 + ¢)


19. 2 sin 2A cos 4A 20. 2 cos 3A sin 7A
Express the following as products: (21. to 40.)
21. sin 3x - sin x 22. sin (x + a) - sin x
23. cos (x + h) - cos x 24. sin (fJ + a) + sin (fJ - a)

25. cos(fJ ~ a) + cos(fJ ;a) 26. cos (A + B + C) - cos (A - B + C)

27. cos (2x + y) + cos (x + 2y) 28. sin (90° + sin (90° B)
A)
29. sin (90 0
- A) + cos 3A •
30. cosx + cos (x + 120°) + cos (x + 240°)
31. cos (2A B) - cos (A + 2B) 32. cos (A + B + C) + cos (A B C)
33. sin 2x + sin 3x 34. sin (2A + 2B) - sin (2A - 2B)
35. sin 165° - sin 105° 36. sin fJ - cos ¢
37. cos 75 0
cos 451:> 38. sin 50° + cos 20°
39. sin (A B) - sin A 40. cos 80° sin 50°
36 NEW SENIOR MATHS: THREE UNIT COURSE

Prove the following: (41. to 62.)

41 sin 50 - sin 30 tan 0 42. sin x + sin = tan x + y


· cos 50 + cos 30 cos x + cos y 2
44 sin 2A sin 2B tan (A - B)
43 •cos
.
x - cos 3x = -t
. 3
2
an x
smx sm x . sin 2A + sin 2B tan (A + B)
45 cos (0 + a) - cos (0 - a) = -tan a
· sin (0 + a) + sin (0 - a)

46 sin x + sin (x + y) + sin (x + 2y) - t ( )


· cos x + cos (x + y) + cos (x + 2y) - an x +y
47. sin 50 + sin 30 - 2 sin 20 cos 0 = 2 sin 20 cos 30
48. sin 2x + sin 4x + sin 6x = 4 cos x cos 2x sin 3x

49. sin 2 50 sin 2 30 sin 80 sin 20 50. sin 35° - sin 25° .J3 sin 5°
51. cos 35° + cos 45° + cos 75° + cos 85° = 2 cos 5° cos 20°

53. sin 25° sin 35° - sin 20° sin 10°

54. sin 10° + cos 40° = sin 70° 55.


cos + cos
56 · sin 3x - sin 3x -co
- t2
x 57. cos(A + B) cos (A -B) = cos 2 A sin2 B
cos x - cos x
sin 0 + sin 70 59 sin (0 + ¢) - sin ( 0 ¢ ) = -cot 0
. 30 + sm
58. sm . 50 = 2 cos 20 - 1 . cos (0 + ¢) - cos (0 - ¢)

60 cos 75° + cos 15° = .J3


• sin 75° sin 15°

61. sin A + sin (A + B) + sin (A + 2B) = tan (A + B)


cos A + cos (A + B) + cos (A + 2B)
62 sin(n + 1)0 + 2 sin nO + sin (n - 1)0 cot fi
• cos(n -1)0 cos(n + 1)0 2

21.6 Half-angle formulae _ _ _ _ _ _ _ _ __


In 21.3. we established the double-angle formulae:
sin 20 = 2 sin 0 cos 0
cos 20 = cos 2 0 - sin 2 0
2 tan 0
tan 20
1 - tan 2 0
Just as 0 is the half-angle of 20, so too ~ is the half-angle of 0 and so:
. 0 = 2'
sm sm -0 cos-0
2 2
cos 0 . 2 -0
cos 2 -0 - sm
2 2
211TRIGONOMETRY 37

tan 0
1 - tan 2
o
2
o
If we denote tan 2 by t, then tan 0
2t

' 'Iar1y
SIml 'Ll
sm u = 2'sm 20 cos 20
t 1 t
= 2 . .Jl+ti· ..Ji+i2
= - -2
2t
1 + t
and cos 0 = cos 2 2
o- 0
sin 2 2
1
= 1 + t2 -

1 - t2
1 + t2 2t
o tan 0
Summary: If tan 2 t, then 1 - t2
, O-~
Sin - 1 + t2
1 - [2
cos 0 = 1 + t 2

EXERCISES 21 (d)
If tan ~= t, express each of the following in terms oft: (1. to 14.)

1. sin () + cos 0 2. sin 0- cos 0 3. 3 sin 0 + 4 cos 0


4. 2 cos 0 sin 0 5. cot () 6. cot () tan 0
1, Ll 0
8 . 1 -2smutan2

o
tanO-tan-
o 10. 2
9. 1 + tan 0 tan 2
cot 2 + tan 0

o
11. cot 2 - 2 cot 0
1 + sin 0 + cos 0
12. 1 + sin 0 - cos 0

. 0 ,0
sm + sm
2
13. --~--··-~O
14. 1 . cos 0
sm 0
1 + cos 0 + 'cos 2
o
15. If tan 2 t, solve for t the equation tan 0 = c

16. If tan ~ = t, solve for t the equation 12 tan 0 5 and 180


0
< () < 270
0
38 NEW SENIOR MATHS: THREE UNIT COURSE

21.7 Use of radian measure __________


EXAMPLE 10

If cos x - ~ and i ~ x ~ 11", find the value of:


(a) sin x (b) sin 2x
From the diagram,
() ' .J7
a smx = 4

(b) sin 2x = 2 sin xcos x


.J7 3
=2'4'-4
-3.J7
8

EXAMPLE 11

(a ) S'Impl'f
IY (')' 4
I sm (11" 4 - x ) (")
x ) cos (11" 11" cos (511" - sm
II cos 3" , 3"11",
sm (511"

(b) Express sin (i - 0) + sin (i - as a product. <1»

(a) (i) sin (r - x) cos (r - x) = ~ sin (r - x) cos (r - x)

~ sin (i - 2X) double angle formula


=

= ~ cos 2x
11" 11" . 11". 11"
(ii) cos - cos-
3 6
sm-sm-
3 6
= cos (11"-3 +-611")

(b) . • (11"
sm 2 0) + sin (i - c/> ) = cos 0 + cos c/>
O+c/> 0-
= 2 cos -2- cos

EXERCISES 21 (e)
Simplify: (1. to 8.)

l~ 1 (i - Q:)
+ tan2 2. 1 - cos + 2 (11" 0)

3, sin 0cos (i - 0) + cos 0sin (i - 0) 4. 2 cos (5 - 1 2


11"

5.1 - sin ocos (i - 0) 6. sin 0) cos c/> - (11" . cos (11" - 0) sin c/>
211TRIGONOMETRY 39

tan 0 tan ~
7. - - - - -
11"
6
1 + tan '6 tan 0
8. cos 2 (i
9. If tan 0 ~ and 11" < 0 < 3 11", find the value of
2
(a) sin 0 (b) cos 0 (c) cos 20
11" 5
10. If 2 < x < 11" and cos x = -6' find the value of
(a) sin x (b) sin 2x (c) tan 2x
311"
11. If cosec 0: = - 817 and 11" <0: < 2' find the value of
(a) cot ex (b) tan 20:
• 3 11" 7 11"
12. If sm 0 = 5' 2 < 0 < 11" and tan ¢ = 24' 0 < ¢ < 2' find the value of
(a) sin (0 - ¢) (b) cos (0 ¢) (c) tan (0 ¢)

13. If tan x = 4"5 and tan y =


1
9" and 0 < y < x <
11"
2' prove that x - y
11"
4"

Prove each of the following: (14. to 20.)

14. 2 cos (i + x) cos (i - x) = cos 2x

15. tan (0 + i) tan (0 - i) == -1

16. (sec 2 0- 1) tan (~ - 0) = tan 0


17 t ( + 311") = tan x - I
.anx 4 tan x +

1 - sin (~ - 2X)
18. . 2
sm x
= tan x

19. tan (i + A ) + tan ( i A) = 2 sec 2A


20. sin (0 + ~") sin (0 - ~) = sin 0 4
1 2

21. Simplify:
2 tan 0 711" 211"
(a) 1 _ tan! 0 when 0 "'8 (b) 2 cos 2 3x - 1 when x = 9""

22. Express sin (~ - x) + sin (~ - y) as a product.


23. If 0 ..;; 0 ..;;
11"
2' prove that tan 0 =
11 -+
1
cos 20
cos 20'

Hence show that the exact value of tan i is .J2 1


40 NEW SENIOR MA THS: THREE UNIT COURSE

24. If 4 tan (0: - (:1) 3 tan 0:, prove that tan


f.I
fJ = 7 +sincos
20:
2
0:

25. If sec 0 - tan 0 = j, show that sin 0 = I~


26. If tan 0 t, express sin 20 and cos 20 in terms of t. Find the values of t for which
(k + I) sin 20 + (k - 1) cos 20 = k + 1

27. Use the factors of x 3 y3 to show that cos 6 0 - sin 6 0 = (I - ~ sin 2 20) cos 20

[2 m2 1r
28. If cos 0 f2 + m2 and 0 < 0 < 2 express tan 0 and sin 20 in terms of / and m.
29. If A, Band C are in arithmetic sequence, prove that
sin A + sin C = 2 sin B cos (B - A)

30. Show that 4 sin 0 sin (0 - f) sin (0 1r


23 ) = sin 30
31. If tan 0: = k tan (:1. show that (k - I) sin (0: + (:1) = (k + I) sin (0: (:1)
CHAPTER 22
Trigonometric
Equations
A statement of equality involving trigonometric functions of a number, or an angle is an
identity, if it is satisfied by all values of the variable in the domain in which the functions
are defined e.g. sin 2 e + cos 2 e = 1 is an identity, because it is true for all e.
If a statement of equality (or inequality) is true only for certain numbers in the domain,
then the statement is a conditional equation (or inequation). An identity is an
unconditional equation.
e.g. . e = 2:1.IS true onIy
sm

when e = :!.! or 57r


6
restricting the domain from Oto 27r; it is true only when the angle e = 30 0 or 150 0 (Fig. 22-1).
There are many other values if we extend the domain; we shall be concerned with
solution sets for the most part between 0 and 27r or 0 0 and 360 0 •
y

y = sinO
I
'2
I
------------------y ="2
1

I
I

'lr 2'lr 0
"2

-1
Fig. 22-1
. e > 2:1.IS true only when 6"7r < e < 57r
sm

sin e < 1 when 0 < i or when 5 7r < e 0;;;


0;;; () 27r
6
There is no general method of solving trigonometric equations and inequations but
there are certain standard procedures and types, which can be best illustrated by
examples.
42 NEW SENIOR MATHS: THREE UNIT COURSE

Type 1

The simplest type of equation involves an equation of a single variable e.g. sin 0 = ~ as
7f
solved above. In place of 0 we may have a more complicated variable such as 20 or 2x
6
etc.

EXAMPLE 1
Solve the equation 3 sin 20 = 1'5, 0 ,.;; 0 ,.;; 7f
3 sin 20 1'5
:. sin 20 = 0·5
. 7f . 57f
= sm 6' sm
and so, 20 =~ 57f
6' 6
i.e. o
You can check this result approximately by drawing the graph of y 3 sin 20 and
drawing the horizontal line with equation y = 1'5. It intersects the graph of y = 3 sin 20
at two points whose O-values are 1; and ~; in the domain 0 ,.;; 0 ,.;; 7f. There are, of course,
many other values of 0 outside this interval.

EXAMPLE 2
"';3
Solve the equation cos (2X 2,0 ,.;; x ,.;; 27f

cos 0 ~3 where 0 2x - i
7f 11 7f 7f 11 7f
Hence, 0= 6' ~,27f + 6' 27f + ~ (i)

7f 7f 11 7f 13 7f 23 7f
:.2x
6 6'~'~'~
Add i to both sides.
7f 77f
Hence, 2x = 3' 27f, T' 47f
7f 77f
X = 6' 7f, 6' 27f

Note that in (i) it was necessary to go round the circle twice because the last step
involved division by 2 to give x between 0 and 27f.

EXAMPLE 3

Find the values of x for which cos 2x ,.;; J2' 0 ,.;; x ,.;; 27f

It is advisa.ble to solve the equation cos 2x = J2 and then with the aid of a sketch of the
graph of y cos 2x, we can solve the inequation.
~~~~~~~~--~y------------------~-----

221TRIGONOMETRIC EQUATIONS 43

Put 1
cos 2x "" -J2
. 7f 77f 7f 77f
.. 2x = 4' 4' 27f + 4' 27f + 4
7f 77f 97f 157f
= 4'4'4'4
7f 77f 97f 157f
X 8'8'8'8

Fig. 22-2

From the graph of y = cos 2x and the line y = J2' (Fig. 22-2), we can see that
1 7f 77f 97f 157f
cos 2x ~ when 8 ~ x ~ 8 or 8 ~ x ~ 8

Type 2 Quadratic equations

EXAMPLE 4
Solve tan 2 0 + tan 0 2 = 0, 0° < 0 < 360°. The given equation is quadratic in tan 0.
Factorizing, we get:
(tan 0 I)(tan 0 + 2) 0
i.e. tan 0 1
o = 45°, 225°
or tan 0 2
o = 116 0 34' ,296 0 34'
EXAMPLE 5
Solve the equation cos 2 x = 2 cos x, -7f ~ X ~ 7f
cos 2 X = 2 cos X
cos 2 X -
2 cos x 0
cos x (cos x - 2) 0
cosx = 0
7f 7f
x=
or cosx 2 for which there is no solution
since -I ~ cos x ~ 1 for all
values of x
44 NEW SENIOR MA THS: THREE UNIT COURSE

EXERCISES 22(a)
Solve between 0 and 211" inclusive unless otherwise stated: (1. to 38.)
- ()
1 . sm = ..)3
2 2. tan x = -1

3. cos x = -0-5 4. ..)3 tan () = 1

5. cos x = 0-4, 0° ~ x ~ 360° 6. sin 2() = -~


7. cosec () = -2 8. cot 2x = ..)3
9. 2 tan 2x - 1 0, 0° < x < 360° 10. sec 2() = ..)2
11. tan2 x - I =0 12• ..)2 sin 2() + 1=0
13. 4 tan 2() + 3 = 0, 0° < () < 360° 14. 2 cos 2x + 1 0, -11" ~ X ~ 11"

15 • ..)2 sin 2x - 1 0, -11" < X < 11" 16. sin (() i) J2


+

17. tan (() - i) = -..)3 18. cos (2X + f) 0'5 =

19. 2 sin (2X - i) = 1 20. tan (2() - i)+ 1 = 0

21. 2 cos (2X - i}= ..)3 22. cos (2x + 60°) = 0'7242,0° < x < 360°

23. sin 2() = -cos 7 11" 24. 5 sin (2x - 70°) + 4 = 0,0° < x < 360°
4
25. sin 2 x sin x = 0 26. cos z () - 2 cos () + 1 =0
27 . ..)3 tan 2 x + tan x = 0 28. 4 sin 2 ()

29. sin x = cos x 30. sin z x sin x cos x 0


31. 2 cos 2 () - 3 cos () - 2 0 32. 2 sin z () + cos () = 1
33. 2 sin 2 () - 3 cos () 2 34. (2 cos x + l)(sin x-I) = 0
35. 2 tan 2 x + tan x 15,0° < x < 360° 36. 5 cos 2 () + 2 sin () = 2,0° < () < 360°
37.3 tan3 () - 3 tan 2 () - tan () + 1 = 0 38. cos 3 () - 2cos z () + cos () = 0
Find the values of x for which: (39.· to 50.)

39. sin x ..,. ~ 40. cos x < 2


I

..)3
41. sin x > 0 42. cos x > 2
1 .
43. sin x < 1 44. sin 2x ..,. 2' 0 < x < 11"

45. cos x > 0 46. tan x > 1


47 . ..)2 cos x > -1 48. cos 2x ~ 0
49. 2 sin 2x ~ ..)3 50. 2 cos 2x > -1
221TRIGONOMETRIC EQUATIONS 45

Type 3
Equations involving the use of the addition theorems and double angle formulae.

EXAMPLE 6
Solve sin 2x = 3 cos x, 0 ~ x ~ 21r
In place of sin 2x, write 2 sin x cos x.
Hence 2 sin x cos x - 3 cos x = 0
cos x (2 sin x 3) = 0
cos x = 0
1r 31r
x 2'2
or 2 sin x 3
sin x = 1-5
There are no values of x for which sin x = 1-5 since -1 ~ sin x ~ 1.

EXAMPLE 7
Solve the equation
cos 2x cos a sin 2x sin a = -0-5, 0 ~ x ~ 21r, where,a = i
The left-hand side of this equation is equivalent to cos (2X + i).
Hence cos (2X + i) = -0-5
1r 21r 41r 21r 41r
2x + 6 = 3' 3' 21r + 3' 21r +
Subtract i from both sides_
1r 71r 51r 191r
Hence
2x 2'6'2'6
1r 71r 51r 191r
X = 4' IT' 4'
EXERCISES 22(b)
Solve the following equations
1. cos 28 :::: cos 8, 0 ~ () ~ 21r
2. 2 cos 2() 4 cos 8 3, 0 ~ () ~ 21r
3. 3 tan 2() = 2 tan 8, 0 ~ () ~ 21r
4. tan () + 2 cot 8 = 3, 0° ~ () ~ 360°
5. 5 sin x = 2 sec x, 0° ~ x ~ 360°

6. cos 2x cos i-sin 2x sin i = 0-5, 0 ~ x ~ 21r

- 2x cos 3
7• sm 1r + cos 2x sm
- 3
1r =
-./3
2' -1r ~ X ~ 1r

8. sin () = cos 2(), 0 ~ () ~ 21r


46 NEW SENIOR MATHS: THREE UNIT COURSE

9. tan 0= sin 20, 0 ~ 0 ~ 271"


o
10. cos 2 0 = 2. cos 2 2' 0 0
~ 0 ~ 360 0

11'. sin 3x cos x - cos 3x sin x = ~3, 0 ~ x ~ 271"


12. tan 20 = 2 tan 0, 0 ~ 0 ~ 271"

Type 4 Transformations:
a sin x + b cos x = r sin (x + ex) etc.
Techniques for changing an algebraic or trigonometric expression from one form to
another are common and useful in the study of mathematics. In Chapter 12 we changed
polynomial and rational expressions from one form to another by the use of 'un-
determined coefficients and we transformed quadratic expressions into the 'completing of
the square' form, thereby enabling us to find the maximum or minimum values of
quadratic functions.
So too we can express a sin x ± b cos x in the form r sin (x ± ex) and a cos x ± b sin x in
the form r cos (x =F ex), where a, b, r and ex are positive real numbers.

EXAMPLE 8
Express a sin x + b cos x in the form r sin (x + ex)
Let a sin x + b cos x == r sin (x + ex)
== r(sin x cos ex + cos x sin ex)
== r cos ex sin x + r sin ex cos x
This is an identity if the coefficients of sin x and cos x in each form are the same Le.
r cos ex a (j)
and r sin ex = b (ij)
Squaring (i) and (ii) and adding, we get
r 2 (cos 2 ex + sin 2 ex) = a 2 + b 2
i.e. ,2
= a2 + b 2
r = .J (a 2 + b 2 ), taking the positive
square root only.
Also from (i) and (ij) cos ex = a - an d sm
' ex = b
r r
If a and b are positive constants, then cos ex and sin ex are positive and hence ex is in the
first quadrant such that tan ex = '2..
a
Hence:

(i) a sin x + b cos x = r sin (x + ex)


In the same way, we can show that
(ii) a sin x b cos x r sin (x - ex)
(iii) a cos x + b sin x = r cos (x - ex)
(iv) a cos x - b sin x = r cos (x + ex), where in each case
r = .Ja2 + b 2 and ex is in the first quadrant such that tan ex = a'2.
22/TRIGONOMETRIC EQUATIONS 47

EXAMPLE 9
Express:
(a) ",J3 sin x - cos x in the form, sin (x - Cl)
(b) 3 cos x - 4 sin x in the form, cos (x + Cl)
(a) ",J3 sin x - cos x , sin (x - Cl)
!E ,(sin x cos Cl cos x sin Cl)
!E , cos Cl • sin x - , sin Cl. cos x
Equating coefficients of sin x and cos x, we get
, cos Cl = ",J3 (0
and , sin Cl 1 (ii)
Squaring 0) and (ii) and adding, we get
r 2 (cos 2 Cl + sin 2 Cl) =4
i.e. r2 4
r = 2, taking the positive root only.
Also from (i) and (ii) cos Cl = ~3 and sin Cl =~
Since cos Cl and sin Cl are both positive, Cl is in the first quadrant such that
1 • 'If
tan Cl == ",J3 I.e. Cl =6
Hence, ",J3 sin x- cos x= 2 sin (x - i}
(b) r = ",J3 2 + 42 = 5; tan Cl = 43" and so Cl 53" 8'
Hence 3 cos x 4 sin x = 5 cos (x + 53" 8')
These results have two important applications:
(a) to find the maximum and minimum values of a sin x ± b cos x etc. and thus assist in
sketching the graphs of f(x) = a sin x ± b cos x.
a sin x + b cos x r sin (x + Cl) and since the greatest and least values of sin (x + Cl)
are 1 and -1 respectively, the greatest and least values of a sin x. + b cos x are, and -,
respectively.
EXAMPLE 10
Sketch the graph of y = ",J3 sin x - cos x, 0 " x " 2'If
",J3 sin x - cos x = i)
2 sin (x -

Its greatest value is 2, when sin (x - i) =


11" 'If
x
6 2
• 211"
I.e. x = T

Its least value is -2, when sin ( x - i)


'If 3 'If
x 6= '2
• 5 'If
l.e.x = T
48 NEW SENIOR MATHS: THREE UNIT COURSE

y
2

cos x
1

2'11" 211" X
3
-1

-2 Fig. 22-3

The graph crosses the X-axis when sin (x - i) = <>

7r
i.e. when X 6= 0, 7r

7r 77r
X = 6'"6
We could also sketch the graph by considering the transformations on the sin curve
.7r
caused by the constants 2 and 6.
(b) to solve equations of the form a sin x ± b cos x c etc., where a, b and c are
constants.

EXAMPLE 11
Solve the equations:
(i) ..)3 sin x - cos x = 1,0 ;E; x ;E; 27r (ii) 8 cos x + 6 sin x = -3, 0° ;E; x ;E; 360°

(i) ..)3 sin x - cos x = 2 sin (x i) and so


2 sin (x - i) I =

sin (x ~ i}= O· 5
7r 7r 57r
X - 6 = 6'"6
7r
X = 3' 7r
(ii) r= ..)8 2 + 62 = 10
tan a =~ = 0·75 and so a = 36° 52'
8
Hence the equation
8 cos x + 6 sin x = -3
becomes 10 cos (x - 36° 52') = -3
cos (x - 36° 52') = -0·3
x-36° 52' = 107° 28',252° 32'
x = 144° 20' ,289° 24'
22/TRIGONOMETRIC EQUATIONS 49

Alternative method
We can express sin x and cos x in terms of tan ~. for all values of x except x 1r, 31r,

5 1r"" ' x. df'df


SInce tan 2 IS not e me or x
1r31r51r
= 2' T·'·"
. 2t d 1 {2
sm x = 1 + t 2 an cos x

(i) .J3 sin x - cos x =1


2.J3t 1 t2 x
Le. 1 where t = tan 2
+
2.J3( = 2
1
t = .J3
x 1r X
"2
-
6
for 0 ~ 2~ 1r
1r
-
X for 0 ~ x ~ 21r
3
x = 1r does not appear in the solution for reasons stated above. Thus we must always
test.
.J3 sin 1r - cos 1r 0 + 1 = 1
Thus x = 1r is a solution.
8(1 - (2) 12t
(ii)
1 + (2 +
5t 2 - 12(
t = 12 ± .J 144 + 220
10
tan ~ 3'108 or -0'708
2
~ = 72° 10' or 144° 42' for 0° ~ ~ ~ 180°
:. x = 144° 20', 289° 24' for 0° ~ x ~ 360°

EXERCISES 22(c)
1. Express each of the following in the form r sin (x + a):
(i) sin x + cos x (ii) 3 sin x + .,)3 cos x
(iii) 5 sin x + 12 cos x (iv) 2 sin x + cos x
2. Express each of the following in the form r sin (x a):
(i) sin x - .,)3 cos x (ii) 2 sin x - 3 cos x
(iii) 2 sin x cos x (iv) 3 sin x - 3 cos x
3. Express each of the following in the form r cos (x - a):
(0 cos x + sin x (ii) 24 cos x + 7 sin x
(iii) 2 cos x + 2.,)3 sin x (iv) 3 cos x + 2 sin x
4. Express each of the following in the form r cos (x + a):
(i) cos x - sin x (ii) .,)3 cos x - sin x
(iii) 8 cos x - sin x (iv) 5 cos x - 3 sin x

7 ..··
50 NEW SENIOR MATHS: THREE UNIT COURSE

5. Find (a) the maximum, and (b) the minimum values of the following expressions and
the smallest positive values of x for which they have these values:
(i) sin x - "';3 cos x (ii) cos x + sin x
(iii) 2"';3 cos x - 2 sin x (iv) 5 sin x + 12 cos x

Solve (6. to 15.)


6. sin x + cos x l , 0 ~ x ~ 211' 7. cos x + "';3 sin x = 2, 0 ~ x ~ 211'
8. 3 cos x + 2 sin x = "';13, 0° ~ x ~ 360° 9. 3 sin x "';3 cos x ,;" "';3, 0 ~ x ~ 211'
10. 6 sin x + 8 cos x = -5, 0° ~ x ~ 360° 11. 4 cos x + 3 sin x = -1, 0° ~ x ~ 360°

12. cos x - "13 sin x = 2, 0 ~ x ~ 211' 13. cos x - sin x

14. 3 sin x + 4 cos x = -2, -180° ~ x ~ 180°15. "';2 sin x - cos x = 1·5, 0° ~ x ~ 360°
16. Sketch the graph of f(x) = "';3 cos x - sin x, 0 ~ x ~ 211'.
Use your sketch to find the values of x for which (a) f(x) (b) f(x) > 1
17. Sketch the graph of f(x) = cos x + sin x, -11' ~ X ~ 11'.
Use your sketch to find the values of x for which (a) f(x) -1 (b) f(x) ,.. -1

Type 5
Using formulae whic? convert sums and differences to products.

EXAMPLE 12
Find all values of x in the domain 0 ~ x ~ 11' for which (a) sin 4x - sin 2x = 0
(b) cos 3x cos 2x + cos x
=0
(a) sin 4x - sin 2x =Q
i.e. 2 cos 3x sin x =0
:. cos 3x =0
11' 311'511' 711'
3x
2'2'2'2""
11' 11' 511'
x
6'2'6
or sin x o
x = 0, 11'
(b) Since cos 3x + cos x 2 cos 2x cos x
then cos 3x cos 2x + cos x 0 becomes
2 cos 2x cos x - cos 2x 0
:. cos 2x 0
. 11' 311' 511'
.. 2x = 2' 2' 2' ...
11' 311'
X = 4' 4
or 2cosx-l=0
1
cos x = 2:
11'
x=]
22/TRIGONOMETRIC EQUATIONS 51

EXERCISES 22(d)
Express the left-hand side of each of the following equations as a product and hence solve
the equations:
1. sin 30 + sin 0 = 0, 0 " 0 " 27r 2. cos 30 - cos 0 ::= 0, 0 " 0 " 27r

3. cos 4x + cos 2x = 0, 0 " x " 7r 4. sin x- sin (347r - x)= 0,0" X" 27r
5. cos x - cos (i - x) 0, 0 " x ,,27r 6. sin 3x + sin x - sin 4x = 0, 0 " x " 1\

7. cos 5x + cosx cos 3x ::= 0, -7r "X" 7r8. cos 30 + cos 20 + cos 0 0,0" 0 " 7r

General solution of equations


Equations of the type: sin e = sin ex

EXAMPLE 13

Find all values of 8 for which sin 0 = ~


1
sin 0 "2
"sm
,. .7
0 = sm 6' rsm, (7r - 6
7r)'(2 7r) .'"
,sm 7r + 6

Consider a coordinate diagram (fig, 22-4).


LXOP =! Q p
6
LXOQ = 7r 6
The ray OP defines an infinite number of
angles in the first quadrant. If we rotate OP x
about the origin as centre (either clockwise
or anti-clockwise), then once during each
revolution it lies along the original ray OP, Fig, 22-4
Each rotation changes the angle turned through by 27r, so that we find that OP is the
. terminal ray defining angles:
7r2 7r + 6'
6' 7r 4· 7r + 6' ' loc
7r ", f or antl-c kwlse
' rotation,
, an d

"27r + ~, -47r + i, -67r + i, ' ,. for clockwise rotation,


We write this totality of angles as
7r
n 7r + 6 where n = 0, ± 2, ± 4, ' .. (1)
52 NEW SENIOR MA THS: THREE UNIT COURSE

or n.180° + 30° in degree measure


Similarly, the terminal ray OQ defines an infinite number of angles
11" - :!! 311" - :!! 511" - :!! ... for anticlockwise rotation, and
6' 6' 6'
11" 11" 11"
-11" 6' -311" - 6' -511" - 6' ... for clockwise rotation.
11"
i.e. n 11" - 6 where n = ± 1, ± 3, ± 5, ••• (2)
or n.180° - 30° in degree measure

Statements (1) and (2) may be replaced by:


11"
8 == n 11" + (- 1)" 6
or 8 = n. 180 + (- 1)" 30
0 0
in degree measure
Note that ( 1)n is 1 when n is zero or even and is -1 when n is odd.
Thus the general solution to the equation sin 8 = sin ~ may be written as:

8 n1l" + (-l)n ~ in radian measure


n.180° + (-l)n ~o in degree measure

The preceding pattern for the general solution can be readily observed by considering
the 8 values at the points of intersection of the curves y = sin 8 and y =~ (Fig. 22-5)
Y
y == sin 8

-3 ()

Fig. 22-5

Observe from symmetry that the line y = ~ intersects the sin curve at values of 8, ~ units
to the right of n 11" when n 0, ± 2, ± 4, ••• and ~ units to the left of n 11" when n = ± 1,
± 3, .... Both of these solutions are contained in the statement
11"
8 n1l" + (-1)" 6
11"
8 {n1l" + ~ n zero or even
or
n1l" - 6' n odd

EXAMPLE 14

Find all angles 8for which sin (8 + i) = -...[2


1

sin (8 + i)
22/TRIGONOMETRIC EQUATIONS 53

i.e. . (()
sm + 411") = . 511"
sm 4
11" 511"
() + 4 = n 11" + (- l)n 4
511" 11"
() = n1l" + (-I)n 4 - 4
(n + 1) 11", n zero or even

= { (n - ~)11"' nodd
Equations of the type: cos e= cos a

EXAMPLE 15

Find all angles () for which cos () = ~


1
cos () = 2"

i.e. cos () = cos f' cos (211" - f} cos (211" + f}· .. p


In the diagram, OP defines the angles Y
11" 11" 11"
3' 211" + 3' 411" + 3' ...
and OQ defines the angles
11" 11" 11"
-3,211" - 3,411" - 3'··· x
The totality of angles may be represented by
11"
() = 2n 11" ± 3 Fig. 22-6
or () = n -. 360 ± 60 in degree measure
0 0
Q
Thus the general solution to the equation cos () = cos a may be written as:

() = 2n 11" ± a in radian measure


n. 360 0 ± a O in degree measure

The preceding pattern for the general solution can also be observed by considering the ()
values at the points of intersection of the curves y = cos () and y = ~ (fig. 22-7).
Y

/J

Fig. 22-7
Observe from the symmetry that the line y = ~ intersects the cos curve at values of (), f
units to both the left and the right of 2n 11", where n = 0, ± 1, ± 2, .... That is,
11"
() = 2n1l" ± 3
------~-------- ---------------------- ---

54 NEW SENIOR MATHS: THREE UNIT COURSE

EXAMPLE 16
Find all values of x for which 2 cos (3x + 30°) + .J3 = 0
cos (3x + 30°) =
.J3
-"2
= cos 150 0
:. 3x + 30 = n. 360 0 ±
0
150 0
= n. 360 0 + 150° or n. 360° - 150°
3x = n. 360° + 120 0 or n . 360° 180 0
x = n. 1200 + 40 or n. 120 - 60 0
0 0

EXAMPLE 17
Find all angles (J for which sin 2(J = cos (J
sin 2(J = cos (J
Le. 2 sin (J cos (J = cos (J
2 sin (J cos (J cos (J = 0
cos (J (2 sin (J - 1) = 0
. 11"
.. cos(J=O=cos
Z
(J = 2n 11" ± ~ in radian measure
or (J = n. 360
± 90° in degree measure 0

or 2sin9-1=0
'll 1.11"
smv=Z=sm
6
(J n1l" i
+ (-I)n in radian measure
or 9= n. 180° + ( -1)n 30° in degree measure
Equations of the type: tan e= tan Q'

EXAMPLE 18
Give the general solution of the equation tan 9 = 1
tan 9 = 1
i.e.

y
In the diagram the ray OP defines the p
angles
11" 11" 11"
4,211" + 4,411" + 4""
and OQ defines the angles
11" 11" x
11" + 4 ' 311" + 4'" .
This totality of angles may be represented by:
9 = n1l" + i Q
Fig. 22-8

or 9 n. 180 0 + 45° in degree measure


22/TRIGONOMETRIC EQUATIONS SS

Thus, the general solution to the equation tan 0 tan a may be written as:

(J = n 11" + a in radian measure


n . 180 + a in degree measure
0 O

The above pattern for the general solution can also be observed by considering the 0
values at the points of intersection of the curves y tan 0 and y = 1 (fig. 22-9).
Observe, from the symmetry, that the line y = 1 intersects the tan curve at values of 0, r
units to the right of n1l", n = 0, ± 1, ±2, ....
Y

I
I
I
_.L.!.=.!.
I
(J

Fig. 22-9
EXAMPLE 19
Find all values of 0 for which tan x = 3 cot x.
3
=
tan x 3 cot x
x
tan 2 x = 3
tan x = -J3 or tan x = --J3
i.e. tan x = tan "311" or tan x = tan
211"
3
11" 211"
X = n 11" + "3 or x = n 11" + 3
or x = n. 180 + 60° or x n. 180 +
0 0
120 0

EXAMPLE 20
Give the general solution to the equation -J3 cos x sin x =
-J3 cos x - sin x = r cos (x + a~ where r = 2 and tan a = ~, a i
= 2 cos (x + i)
:. 2 cos(x + i) = 1
cos (x + i) = ~ = cos;
11" 11"
X +6 = 2n1l" ± "3
+ !! or 2n 11" - !!
x = 2n 11"
6 2
or x = n. 360 + 30 or n .360
0 0 0
- 90 0
56 NEW SENIOR MATHS: THREE UNIT COURSE

EXERCISES 22(e}
Find general solutions to the following equations
1. sin x = 1 2. cos x = 0
3. tan x = -1 4. VJ cosec x = 2
x
S. sec = -2 6. cot x = VJ
7. 2 sin (() - i) + o ()
8. cos 2" == 1
9. 2 sin 2 x = 1 10. 4 cos 2 x - I = 0
11. cos 2 X - 2 cos x + 1 =0 12. sin2 x == sin x
13. cos 2() = sin () 14. sin2 x = 1 - cos x
IS. cos 2() = 2 + cos () 16. tan 2x == cot x
17. cos 2x == cos x 18. 2 sin x sec x
19. tan 2 x = tan x 20. 2 cos (2X j) = VJ
21. ..f2 sin 2() + i = 0 22. tan (() - j) = -VJ
23. cos 2x cos i-sin 2x sin i = 0'5 24. tan () = sin 2()
25. tan () == 3 cot () 26. VJ sin x cos x = 1
27. sin x + cos x = 1 28. cos x VJ sin x = 2
29. 4 cos x + 3 sin x = -1 30. cos x - sin x = 1
31. sin 38 + sin () = 0 32. sin 4x - sin 2x 0
33. cos 4x + cos 2x = 0 34. cos 3(J = cos 8
35. 2 cos 2() = 4 cos () - 3 36. 3 tan 2(J 2 tan 8
37. tan (2() - r) + 1 = 0 38. 2 cos (2X - = VJ j)
39. cos () = sin () - 1 2
40. 2 sin x + cos x = 1
41. cos x VJ sin x = 42. 4 + sin 8 6 cos 2 ()
43. sin x = cos x 2
44. 1 + 2 cos X 5 sin x
45. tan 3 () - tan () = 0 46. tan x = sin x
47. sec 2x = 'cosec 2x 48. sin 28 = tan ()
49. sin 2x + sin 4x = sin 3x SO. sin 3x sin 2x
CHAPTER 23

M iscella neous
23.1 Inequations _ _ _ _ _ _ _ _ _ _ _ _ __

(This is an extension of the work on inequations in Chapter 3).

Listed below are some of the main properties of inequalities, with particular emphasis on
operations which reverse the inequality sign.
1. The nature of an inequality is not altered by adding or subtracting the same number
from both sides.
2. If both sides of an inequality are multiplied by a negative number then the sign is
reversed, e.g. 3 > 2 but -6 < -4. Multiplication by a positive number does not alter
the nature of the inequality.
3. Taking reciprocals reverses the nature of an inequality when both sides have the same
sign but not if the signs are different, e.g.
5 < 7 but ~ > ~
3 > -4 and ~ > -~
4. Squaring
Since this process produces a positive number on both sides, the direction of the
inequalitya 2 S b 2 will depend upon which of lal and Ibl is the larger.
5. Extraction of a Square Root
This process is only defined provided both sides of the inequality are positive. The
nature of the inequality is preserved, i.e. if a > b then ..Ja > ..J b provided a is positive
and b is positive or zero.

EXAMPLE 1 y

For what values of x is (a) X2 > 4 (b) X2 < 4. y X2


-- Y = 4
00 ~>4
Ixl > 2
This means that the magnitude of x must exceed 2
and hence either x > 2 or x < -2.
(b) X2 <4 -2 o 2 X
Ixl < 2 Fig. 23~1
58 NEW SENIOR MATHS: THREE UNIT COURSE

This means that the magnitude of x must


be between 0 (inclusive) and 2.
Hence x > -2 and x < 2, i.e. -2 < x < 2. This example illustrates the general result that
if x 2 > a 2 , then x < -a or x > a
and if X2 < a 2 , then -a < x < a, where a > O. (See Fig 23-1)

EXAMPLE 2
Find the values of x for which X2 x - 2 ;;;.. 0
Method 1
X2 X 2;;;.. 0
(x 2) (x + 1) ;;;.. 0
:. x - 2 ;;;.. 0 and x + 1 ;;;.. 0 or x - 2 ~ 0 and x + 1 ~ 0
x ;;;.. 2 or x ~ -1
Method 2
Sketch the graph off(x) (x 2)(x + 1).
Now (x - 2)(x + 1) ;;;.. 0 corresponds to
the sections of the graph on or above the
X-axis and for these sections x ~ -lor x ;;;.. 2. X
This method has the advantage of pro-
viding a visual picture and also of giving an
easy method for higher degree inequations.
(Fig 23-2)
Fig. 23-2

EXAMPLE 3
y
For what values of x is
(2 - x)(2x - l)(x + 3) ~ O?
From the graph shown in Fig. 23-3 the
in equation is seen to be satisfied for x in the
interval -3 ~ x ~ ~ or for x ;;;.. 2. ....---\-------I-+---~--...X
...

EXAMPLE 4
For what values of x is x + 21 ;;;.. O? Fig. 23-3
x-
We cannot simply multiply both sides by (x 2) as in an equation and claim x + 1 ;;;.. O. Why?
Method 1
Multiply both sides by (x - 2)2 since this is positive.
(x + 1) 2
Thus (x _ 2) (x 2);;;.. 0
so that (x + I)(x - 2) ;;;.. 0
This is the same in equation as was solved in Example 2 so we appeal to that solution.
However, there is one important difference the value x = 2 which was in the solution
set of Example 2 is not permissible in the present case since x + 21 is undefined for x = 2.
x-
Thus x ~ -lor x > 2 is the required solution.
23/MISCELLANEOUS 59

Method 2
Sketch the graph of y = x+1
x-
x =1= 2. The inequation is true for all x for which the graph
is on or above the X-axis. The point of intersection with the X-axis can be obtained by
· t he equation
soIvmg . x+1
- - 2 = 0.
x-
Method 3
If x - 2 > 0 i.e. if x > 2, then, multiplying both sides of the inequation by x- 2, we get
x+I~O
i.e. x ~ -I
i.e. x ~ -I and also x > 2 implies that x >2
If x - 2 < 0 i.e. if x < 2, then, multiplying both sides of the inequation by x 2, we get
x + 1 ~ 0
i.e. x ~ -1
Le. x ~ -I and also x < 2 implies x ~ -1
x ~ -lor x > 2 as before.

EXAMPLE 5
For what values of x is 2x + 3 > I?
x-
If x 4 > 0 i.e. if x > 4, then multiplying both sides of the inequation by x 4 gives
. 2x+3>x-4
i.e. x > -7
i.e. x > -7 and also x > 4 implies x > 4
If x - 4 < 0, i.e. if x < 4, then multiplying both sides by x - 4 gives
2x+3<x-4
i.e. x < -7
i.e. x < ..., and also x < 4 implies x < -7
x < -7 or x >4

EXERCISES 23(a)
Solve the following inequations:
1. X2 - 2x - 15 ~ 0 2. x(x 1) ~ 6
3. 4X2 - 12x + 10 > 0 4. X2 + 4x + 13 < 0
5. -3x2 + lOx + 8 ~ 0 6. 2X2 + 5x + 2 ~ 0
7. (x - I)(x + 3)(x - 2) < 0 8. (2 + x)(x 5)(x + I) > 0
9. X2(X 1) ~ 0 10. ~~ 3 >0
x-2
11. I > 6 12. x + >-2
x
13. 2x
4x - 3
+ I ~ 3 ·~I!I <3
I 16. 2 >-1
15. 2x _ 1 ~ 2 -x
1 > 1 18.
60 NEW SENIOR MA THS: THREE UNIT COURSE

7 _
19 1 ..; -1
• (x - l)(x - 3) 20. (3 x}(x + 3) > 1

21. 3xl + 5x + 1 >


1
4 22 2x 4 x +2
. x + > +
2x - 3 4x - 4
23. 4x _ 5 + 2 < 0 24. 1 < 2x + 3 < 3
1 5 26. 22x - 5(2x) +4 .r;;; 0
25. 5xl - 2x - 7 < 13
27. 2lx - 2(2x) .r;;; -1

Find values of x for which the following inequations are simultaneously satisfied:
x+4 x-6
28. - - 6 < 0 and - - 4 > 1
x - x-
29. Xl 5x +4 .r;;; 0 and 6 - x Xl >0
1
30. x + Ixl > 0 and Xl - X - 2 < 0

23.2 Angle between two intersecting


lines _________________ _

(This is an extension of the work on co-ordinate geometry in Chapter 9).


y

x
Fig. 23-4
In co-ordinate geometry of the straight line, we associate gradients with angles.
Let PQ and PR have gradients m 1 and m l respectively and make angles 01 and 0l with
tlie X-axis.
Then tan 01 m 1 and tan 0l m l • With the aid of tan tables or a calculator we can find
01 and Oz.
If Ci is the acute angle between the two lines, then Ci == 01 - 0l'
Alternatively, Ci == 01 - Ol
tan Ci == tan (0 1 - 0l)
tan 01 - tan Ol
1 + tan 01 tan Ol
= m 1 - ml
1 + m1m l
Hence angle Ci is found using the formula:

If the lines are parallel, Ci = 0 and so tan Ci =0


o when m l - m l = 0 i.e. m l = m l
23/MISCELLANEOUS 61

If the lines are perpendicular, a = 90° and tan 90° is not defined.
t~ m 2 is not defined when 1 + m]m2 = 0 i.e. m]m2 = -1
mjm Z
These results agree with what we have seen before in relation to parallel and perpendicular
lines.

EXAMPLE 6
Find the acute angle between the lines x + 2y = 5 and x - 3y = -3
The gradient of x + 2y = 5 is - ~
1
tan 01 = -2 y

OJ = 180° 26° 34'


153 0 26'

The gradient of x 3y = -3 is }
1 o
\ x
tan O2 = :3
02 = 18° 26' Fig. 23-5
a = 01 - 02
135 0
0
The acute angle is 45
Alternatively tan a = m1 m2 - ---"_,.-'-
1 + mjm2 -
:. a = 135 0 as above.
and so the acute angle is 45°

23.3 Division of an interval in a given


ratio
(This is an extension of the work on co-ordinate geometry in Chapter 9).

If A and B are two points in the plane whose co-ordinates are (XI' YI) and (x2, yJ
respectively, then the co-ordinates of the point that divides AB internally in the ratio m : n
are:

8(x2. Y2)
,-1::1
n '1
I",
_________ J::..

:;;:. X2 - a D
: I
______________ .J1-1::1

a - XI C

Fig. 23-6
62 NEW SENIOR MATHS: THREE UNIT COURSE

Proof:
Let P (a, b) be the point in the interval joining A (XI' Yl) and B (X2' Y2) that divides AB
internally in the ratio m : n
AP m
PB = n
The triangles APC and PBD are similar. Hence
AC AP CP AP
PD = PB and DB PB
i.e. = m and b - m
x2 - a
n Y2 - n
i.e. na - nX I mX2 - ma and nb - nYI = mY2 - mb
i.e. a mX2 + nXI b = mY2 + nYI
m+n' m+n
Note that m, which corresponds to the interval AP, is multiplied by the co-ordinates of
B while n, which corresponds to the interval PB, is multiplied by the co-ordinates of A.

EXAMPLE 7
Find the co-ordinates of the point which divides the interval joining the points <-6, 2) and
(4, 7) in the ratio 3 : 2
AtP,
3x4+2x 6=0 2 8(4,7)
X = 3 + 2
y=3X7+2X2=5
+
A~
(-6.2)
The co-ordinates of Pare (0, 5).
If the point P divides the interval AB externally in the ratio m : n, then its co-ordinates
are:

mX2 - nX I mY2
( m - n ' m

Yj m..... _ - - - - - -:.=:._ Pia, bl


""., ...B(xz, yz) ,;!. . . '- :
,/ -------- .... 0
" I
, " II

or' "I - --- - ~i:'-2~-7-- -- --- --~ C.. x

Note that for external division, the point P is outside the interval AB, beyond Band
PB = m
AP n as m".mternaI d"IVlsJOn.
".

Proof:
The triangles APC and PBD are similar" Hence
AC AP CP AP
BD = PB and DP PB
23/MISCELLANEOUS 63

a m and bb YI m, from which


i.e. =
a x2 n Y2 n
a mX2 nX1 and b = mY2 - nYI
m-n m-n
m-,-,--x2~+~(--.:..;.n~)x.:..!l and b = m·~Y,,-2-,+~{--.:..;.n.£.:.)Y:...J1
m+(-n) m+(-n)
This is the same form as for internal division with n replaced by -no
P divides AB externally in the ratio 3 : 2 can be denoted by 3 : -2 and the same formula
can be applied for both internal and external division.

EXAMPLE 8
Find the coordinates of the point which divides the interval joining (3,. -2) and n, 4)
externally in the ratio 5 : 3.
x-eo-ordinate = 5 x -5 - 3 x 3 = -17
5 - 3
y-co-ordinate 5 x 4 - 3 x -2 = 13

EXERCISES 23(b)
Find the acute angle between the pair of straight lines (1. to 6.):
1. with gradients 4 and ~
2. with gradients -3 and 2
3. with gradients ~ and - ~
4. with equations 3x - 4y = 3 and x 2y = 11
5. with equations y = 4x + 5 and 6x + 3y = 7
6. with equations x. - 2y = 6 and y 3x - 1
7. The points (0,2), (2, 3), (4, 0) and (2, -1) are the vertices of a quadrilateral. Find the
tangent of the acute angle between its diagonals.
8. Prove that the points (4, 0), (6, 1), (4, 3), (3, 2) are the vertices of a cyclic
quadrilateral, by proving that its opposite angles are supplementary.
9. Find the co-ordinates of the point that divides the interval from n, 6) to (4, -3) in the
ratio (a) 1 : 2 (b) 2 : 1 (e) 3 : 1
10. Find the co-ordinates of the point that divides txternallY the interval joining the
points
(a) f5, 6) and (-2, 3), in the ratio 3 : 2
(b) (-5, 6) and (1, 0) in the ratio 3 : 1
11. Find the co-ordinates of the points of trisection of the interval joining (I, 2) and (7, 8)
12. A (-2, -4), B (4, 8) and C (7, -5) are the vertices of a triangle. Find the
co-ordinates of:
(a) D, the mid-point of AB
(b) G, the point on CD that divides CD in the ratio 2 : 1
(e) E, the mid-point of BC
(d) the point on AE that divides AE in the ratio 2 : 1
(e) Compare answers to (b) and (d)
64 NEW SENIOR MA THS: THREE UNIT COURSE

13. A, Band C are the points (Xl' Yl)' (xz' Yz), (X3' Y3)' D is the mid-point of BC and G
divides AD in the ratio 2 : 1. Find the co-ordinates of G and show that the three
medians of l:,.ABC are concurrent. (A median is a line drawn from a vertex of a
triangle to the mid-point of the opposite side).
14. Show that the point P (-2,0) lies on the line joining A (7, -3) and B <-5, 1) and find the
ratio in which it divides the line AB.
15. The·vertices of a triangle are the points P (5,6), Q (4,3) and R (2, -3). Find the co-
ordinates of the centroid i.e. the point where the medians meet.

23.4 Mathematical induction _ _ _ _ _ _ _ __


The term induction involves the deriving of a general statement or rule from one or more
particular cases. A few special cases may make a statement true. Does it follow, then, that
it is true in all cases? This is the reverse of deduction in which we proceed from the general
to the particular.
1. It is true that 60 is divisible by 1,2,3,4,5,6. This establishes a pattern. Does it follow
that 60 is divisible by all positive integers? (No)
2. P
1 + 3 22
1+ 3 + 5 32
+ 3 + 5 + 7 = 42
The sum of the first two odd numbers is a perfect square; so too is the sum of the
first three odd numbers, the first four odd numbers. Does it follow that the sum of the
first n odd numbers is a perfect square? The pattern seems to indicate that it is. We
shall prove shortly that it is.
3. It is true that, when n = 1,2, 3,4, 5,6, 7, n 2 + n + 11 is a prime number. Does it
follow that it is prime for all values of n? The pattern seems to indicate that this is
possibly so. If n = 10, n 2 + n + 11 = 121, which is not a prime number. Test whether
n 2 + n + 41 is a prime number for all values of n. (No)
Proof by Induction
The method of proof by induction is applied to propositions containing the natural
number n. Proof involves three steps:
Step 1. Verify that the statement is true for a special case, usually n = 1.
Step 2. Assume that the statement is true for some integer, say, n = k and then prove
that it must be true for n = k + 1.
Step 3. Hence, if it is true for n = 1, then it must be true for n = 2 and then, being true
for n = 2, it must be true for n = 3 and so on. Hence it is true for all natural
numbers.

EXAMPLE 9
Prove, by induction, that the sum of the first n odd numbers is n 2, that is, that
1 + 3 + 5 + 7 + ... + (2n - 1) = n 2
Proof
Step 1. Put n = 1 and since 1 12, the statement is true.
Step 2. Assume that it is true for n k. Then
23/MISCELLANEOUS 65

Sk== 1 + 3 + 5 + 7 + ... + (2k - 1) == k 2


and so when n = k + 1, adding 2k + 1 to both sides we get
Sk+l 1 + 3 + 5 + 7 + ... + (2k 1) + (2k + 1) == k 2 + 2k + 1
i.e. 1 + 3 + 5 + 7 + ... + (2k + 1) == (k + 1)2
Sk+1 has the same form as Sk with k + 1 replacing k in all places.

Hence, if the statement is true for n == k, then it is also true when n == k + 1


because the sum of the first (k + 1) terms is the perfect square ( k + 1)2.
Step 3. The statement is true when n = 1 and so it is true for n 2 and so on. Hence, it
is true for all n, a natural number.

EXAMPLE 10
Prove, by induction, that 4n - I is divisible by 3 for all n ~ I.
Proof
Step 1. When n == 1, 4 - I 3 which is divisible by 3.
Step 2. Assume that when n = k,4k - 1 is divisible by 3 i.e. 4k - 1 3m where m is an
integer.
When n k + I, 4k + 1 - I 4k+l - 4 + 3
4(4k - 1) + 3
= 4 x 3m + 3
= 12m + 3
= 3(4m + I) which is divisible by 3
Step 3. It is true when n == I and so it is true when n 2 and so on. Hence it is true for
all n, a positive integer.

EXAMPLE 11
Prove, by induction, that the sum of n terms of a geometric sequence whose first term is a
. a(r n - I)
and common ratio r is 1
r -
To prove:
Sn = a + ar + ar 2 + + arn - l = a(rn
-'--~
I)
r 1
Step 1. For n I, SI = a. It is true.
Step 2. Assume it is true for n = k.
a(rk - I)
Sk = + ar + ar2 + .... + ark - 1
a
r - 1
and so when n == k + I, adding ark to both sides, we get,
1
Sk+l a + ar + ar2 + ... + ark- 1 + ark a(;k_- ) + ark
1
ark - a + ark+ 1 - ark
r-
a(rk+ 1 - I)
= r - I
Hence if it is true for n == k, it is also true for n = k + 1.
Step 3. It is true for n = 1 and hence is true for n = 2 and so on. Hence it is true for
all n.
66 NEW SENIOR MATHS: THREE UNIT COURSE

EXAMPLE 12
Prove, by induction, that
. 1
. 1. 2 + 2.3 + 3.4 + ... + n(n + 1) = 3n (n + l)(n + 2)
for all positive integers.
. . 1
Let Sn = 1.2 + 2.3 + 3.4 + ... + n(n + 1) = 3n (n + l)(n + 2)

Step1. Whenn == I,SI 1.2 = ~.1.2.3 = 2. It is true for n == 1.

Step 2. Assume it is true for n = k.


Then Sk == 1.2 + 2.3 + 3.4 + + k(k + 1) ~k (k + l){k + 2)
and so when n == k + 1,
Sk+1 == 1.2 + 2.3 + 3.4 + + k(k + 1) + (k + l)(k + 2)

= ~k (k + l)(k + 2) + (k + l)(k + 2)

~[k(k + l)(k + 2) + 3(k + l)(k + 2)]

= ~ (k + l)(k + 2)(k + 3)
Thus if it is true for n k, then it is true for n k + 1.
Step 3. It is true for n = 1 and hence it is true for n = 2 and so on. Hence it is true for
all n.

EXAMPLE 13
Prove, by induction, that 3 n > 1 + 2n i.e. 3n - 2n 1 > 0 for n > 1.
Proof:
Step 1. Put n = 2 and since 3 2 - 2.2 - 1 > 0, the statement is true for n 2.
Step 2. Assume that it is true for n == k i.e. assume that 3k - 2k - 1 > 0
When n = k + 1,
3k +1 - 2(k + 1) 1 = 3. 3k - 2k - 3
= 3. 3k - 6k - 3 + 4k
== 3(3 k - 2k - 1) + 4k
> 0 since 3k - 2k 1 > 0 and 4k > 0
Hence, if the statement is true for n k, then it is also true for n = k + 1.
Step 3. The statement is true for n = 2 and so it is true for n 3 and so on. Hence it is
true for all n > 1
EXERCISES 23(c)
Prove each of the following by induction where n is a positive integer.
n
1. 1 + 2 + 3 + 4 + ... + n = 2 (n + 1)
./

2. 1 + 2 + 4 + 8 + ... + 2 n - 1 = 2n - 1
3. n 2 - 2n + 5 > 0,
1 - rn
4. 1 + r + r2 + ... + r n - I =
----------------------~.,

23/MISCELLANEOUS 67

5. 5n + 3 is divisible by 4 6. 311 ~ 1 + 2n, n ~ 1.


. n
7. 2 + 5 + 8 + ... + (3n - 1) =2 (3n + 1)
1 1 1 1 n
8·1.2+2.3+3.4+"·+n(n+l) n+
3n - 1
.9. 1 + 3 + 32 + 33 + .. , + 3,,-1 = -2-
1 1 1 1 n
10. 2.3 + 3.4 + 4.5 + ... + (n + l)(n + 2) 2(n + 2)

11. t
k=1
k2 = n(n + 1)(2n
6
+ I)

n 1 n
12. ~ (=2-;-k--:l-;:-)(=2k-;---":+-=I) = 2n + 1

13. E" (4r -


r=1
6) = 2n(n - 2)

14. The sum of n terms of an arithmetic sequence whose first term is a and common
difference d is ~[2a + (n - l)dJ
15. (l + x)n ~ 1 + fIX, x >0 16. 7" - 1 is divisible by 3.
17. t
k=1
k(k + 1) = n(n + Ij(n + 2) 18. n(n + 1) is an even number.

19. 2 + 4 + 6 + ... 2n n(n + 1)


20. n2 - 11 n + 30 ~ 0 for n ~ I.
21. 2" > n 2 for n > 4.
22. n(n + l)(n + 2) is divisible by 3.
P 22 n2 n(n + 1)
23. D + 3.5 + ... + (2n 1)(2n + 1) 2(2n + 1)
n
24. E (2r
r=1
1)3 = n2(2n2 1)

25. xn 1 is divisible by x - I .
26. ! (xn) = fIX,,-1 for all n ~ 1.
27. 3 3n + 2,,+2 is divisible by 5.
28. The sum of the cubes of three consecutive integers is divisible by 3.
29. }3 + 2 3 + 3 3 + ... + n3
n42 (n + 1)2

30. (x + y)n > xn + yn

31. t k=2
log (k .~ I) = log n
32. n 2 - 3n + 2 ~ 0 for n ~ 1.
33. 12" > 7 n + 5" for n ~ 2.
34.. The cube of the sum of three consecutive integers is divisible by 3.
35. The sum of the angles of a polygon of n sides is (2n - 4) right angles, n ~ 3.
36. The greatest number of regions that n straight" lines can divide a circle is
~ (n2 + n + 2), n ~ I.
CHAPTER 24

Calculus

24.1 Rational algebraic functions _ _ _ _ _ __


(This section is an extension of curve sketching in Chapter IS).

EXAMPLE 1

Sketch the curve whose equation is y =


By inspection:
(a) The function is not defined when x ± 2. As x ....... ± 2, Iy I . . . . 00. The lines x = ± 2
are vertical asymptotes.
(b) The curve passes through the origin.
(c) The equation of the curve is unaltered if we put -x in place of x. The graph is thus
symmetrical about the Y-axis. Hence y defines an even function.

By division, y = 1 + 4

As x ....... 00, y ....... 1 from above


As x ....... -00, y ...... 1 from above
} so y = 1 is a horhontal asymptote.
dy _ -8x
dx - (x 2 - 4)2
y

~! 1 -2
1
----------,-----
0
i~
-----;----- - y---
=1
x

Fig. 24-1.
-.-... ~~~-----=--~---------------

24/CALCULUS 69

Since, when x =1= ± 2, the denominator is always positive, the sign Of;t is the sign of -8x.

Thus ;t > 0 for all x < 0, except x = -2, and :; < 0 for all x > 0, except x = 2.

The curve has a maximum turning point at the origin. The range is the set of all real
numbers except those in the interval (0, IJ

EXAMPLE 2

Sketch the curve whose equation is y = ~1


x +
Since x 2 + 1 =1= 0 for any values of x, the denominator is never zero and so y is defined for
all x. There are no vertical asymptotes. The curve crosses the X-axis at x 0 and the Y-axis
at the origin.
The function is odd. Why? The curve has point symmetry about the origin. Dividing
numerator and denominator by x 2 , we get
1
y=
x
1 +-
X2
As x co y 0+ }
As x
-+
-+ -r:" y
-+
-+ 0- y = 0 is a horizontal asymptote.
dy 1- X2
dx (x 2 + 1)2 = owhen x ±1
> 0 when -1 < x <
< 0 when x < -lor when x >
y

(1 , ~)

Fig. 24-2

The curve has a minimum turning point at (-1, i) and a maximum turning point at
(1, &)
The range is the closed interval
EXAMPLE 3

Sketch the curve with equation y = X2 ~ 4

Since X2 + 4 =1= 0 for any values of x, the denominator is never zero and so y is defined for
all x. There are no vertical asymptotes.
The curve does not cross the X-axis and crosses 'the Y-axis at y = i.
70 NEW SENIOR MATHS: THREE UNIT COURSE

The function is even. Why? The curve is symmetrical about the Y-axis. Dividing
numerator and denominator by X2 we get

As x -- 00, y -- 0+ }
As x __ - 00, y __ 0+ Y 0 is a horizontal asymptote.
dy -2x
dx (X2 + 4)2 = 0 when x = 0
> 0 when x < 0
< 0 when x > 0
The curve has a maximum turning point at (0, i).
The range is 0 < Y '" i. y

1
4

Fig. 24-3
EXERCISES 24(a)
Sketch the graph of each of the following locating any stationary points and asymptotes.
State the range in each case.
x + 1 X2 + 3
1. Y = x(x
X2 + 2x 2. Y - 3)
x x + 1
3. Y 4. Y = (x - 1)2
x 1 x
5. Y = X29 6. Y = 1 + X2 + 1
x2 X2
7. Y = (x - l)(x - 2) 8. Y X2 - 4
x x + 3
9. Y X2 - 9 10. Y X2 - 9
4 . x + 3
2
11. Y = X2 + 1 12. Y = x(x + 3)
. 1 X2 2x - 3
X2
13. Y = + 14. Y = =-:::--=----:-
2x2 + 2x + 1
2x 2 + 2x + 3
15. Y
2x2 - 2x + 5
16. Y = 4x2 + X1
x2 - be
17. Show that the curve Y = 2x _ b c' b > c, has stationary points at x = band
at x = c.
Sketch the curve when b 2 'and c -3
24/CALCULUS 71

24.2 Parametric equations __________


It is a common practice in mathematics to express two related variables, say x and y, in
terms Of a third variable, say t or 0, so that
x = J(t), y g(t)
or x = J(O), y = g(O)
These equations are· called parametric equations and tor 0 is called the parameter.
In your study of the trigonometric functions, the functions cosine and sine are defined
as the x and y co-ordinates respectively of a point on the unit circle X2 + y2 I. Thus the
unit circle can be represented by the parametric equations:
x = cos 0, y sin 0
where 0 is the parameter.
EXAMPLE 4
Find the cartesian equation of the curve whose parametric equations are
x = I 4- t, Y = t 2
Transposing the first equation we get:
t x - I and so since Y
y t2, it follows that
y (x - 1)2, the graph of which is a parabola
There are no restrictions placed on the
values of t and hence x can assume all values.
However, if say t ;;.. 0, then since x = I + (,
x ;;.. 1 and the graph is only part of the
parabola. Note that since y t2, Y ;;.. O.
Why? o x
Fig. 24-4
EXAMPLE 5
Find the cartesian equation of the curve whose parametric equations are x = sin 8, y = cos 20
y = cos 20 Y
= 1 - 2 sin 2 0
= 1 2X2

Since -1 ~ sin 0 ~ 1 for all 0, it follows that


-1 ~ x ~ 1 and so the parametric equations
represent only part of the parabola. The car- x
tesian equation is y = 1 - 2x2 , -1 ~ x ~ 1.
EXERCISES 24(b)
Fig. 24-5
Find the cartesian equation of the curves whose parametric equations are:
1. x = = t + 2,
21, Y 2. x = t, Y = (2
I
3. x = t, Y = t 4. x = 2 cos 0, y 2 sin 0, 0 ~ 0 ~ 21r
5. x = 2 cos 0, y = 2 sin 0, 0 ~ 0 ~ 1r 6. x = ( + 3, y t2 5, ( ;;.. 0
7. x 2u 2, y = 3u + 1, 1 ~ u ~ 3 8. x = v3, y = 1 v 2 , -1 ~ v ~ I
9. x t + 2, y = (2 - 1 10~ x = cos 2t, Y = cos t, 0 ~ t ~ 21r
11. x = 2t2, Y = 4t 12. x =
2 cos 0, y = .J3 sin 0, 0 ~ 0 ~ 21r
2t 1
13. x 2 cos t. Y = sin t, 0 ~ 0 ~ 1r 14. x = 1 + t 2 ' y =
72 NEW SENIOR MATHS: THREE UNIT COURSE

24.3 Parametric equation of the


parabola _ _ _ _ _ _ _ _ _ _ _ _ _ __

The parabola X Z = 4ay can be represented by the parametric equations:

I x=2at,y=al z
l

The point (2al, a( 2) on the parabola is a variable point depending on the value of I and is
frequently referred to as the point I.
We have seen earlier how the Differential Calculus can be used to find the gradient of
the tangent and normal to a curve at any point on the curve and hence the equation of the
tangent and normaL We shall apply those principles to the parabola, with particular
reference to the parametric equations of the parabola. We shall also consider some simple
locus problems.

Fig. 24-6
Since x 2al and y al z,
dx dy
dt = 2a and dl 2al
dy dy dt
dx = dl' dx
1
2at' 2a
I
= gradient of tangent at any point I
Hence the parameter I denotes tan () where () is the angle the tangent makes with the
X-axis.
1
or y = 4a X2
dy 1
dx 2a'
x
1
= 2a' 2al when x = 2al
= I as before.
Equation of tangent at P is
y - Yt = m(x- xJ
i.e. Y - al 2 I(X 2at)

Y = Ix - al 2
24/CALCULUS 73

The gradient of the normal is t, (t *' 0) and so the equation of the normal is
Y YI = m(x - XI)
1
i.e. Y at 2 = --(x - 2at)
t

X + ty = aP + 2at

Alternatively
1
y = 4a x2
dy 1
dx = 2a' x
1-
= 2a XI at any point (XI> YI)
Equation of tangent is
Y - YI m(x - Xl)

= 2a
Xl (X - x)
I

2ay 2aYI = XXI - Xf


= XXI 4aYI since ~ = 4aYI

1 XXI = 2a(y + YI) I-

Compare this equation of the tangent with the cartesian equation of the parabola. What
do you observe?
EXAMPLE 6
Find the equation of the tangent and normal to the parabola represented by the equations
X t, Y = 1 t 2 at the point on the parabola where.t 2.
When t = 2, X 2 and Y = -3
dx dy
dt = 1 and dt - 21
dy dy dt
dx = dt' dx = 21 = -4 when 1 =2
Equati.on of tangent is y + 3 = -4(x - 2) i.e. 4x + y 5
.~ 1·
Equation of normal is y + 3 = 4 (x. 2) i.e. X - 4y = 14
EXAMPLE 7
Tangents drawn from two variable points P and Q on the parabola X2 = 4ay intersect at
right angles at a point T. Find the cartesian equation of the locus of (a) T, (b) the mid-
point, R, of PQ.
(a) Let the coordinates of P and Q be (2at, at 2 ) and (2as, as 2 ) respectively.
Since the gradients of the tangents at P and Q are t and s respectively and these two
tangents are at right angles, the product of their gradients is -1, i.e. Is -1 and so
1
s = -(

Hence the coordinates of Q may be written as (_ 2a, ~).


- I t~
74 NEW SENIOR MATHS: THREE UNIT COURSE

X
Fig. 24-7

Equation of tangent PT is y Ix - a/ 2 (i)


EquationoftangentQTisy = sx - as 2 (ii)
At the point of intersection, T, of these tangents, (i) and (ii) are simultaneously
satisfied and so
tx at 2 = sx - as 2
(t - s)x a(f2 - S2)
Divide both sides by t - s, since t ;# s. Why?
Then x T = a(t + s)
= a~ - T) because s t
(iii)

The x co-ordinate of T is a (t - T). t ;# 0

Substituting x = a (t Dinto (i) we get

Yr = a(t T)t al 2
a (iv)
The y co-ordinate of T is -a.
Equations (iii) and (iv) are the parametric equations of T. For all values of t ;# O. the
point T lies on the line with equation y = -a. This line is the directrix and is the
cartesian equation of the locus of T.

(b) Since R is the mid-point of PQ. its co-ordinates are given by:
x = ~ (2at + 2as) = a(t + s) = a(t - T)
y = ~ (at2 + as 2 ) = ~ (t2 + A)
These equations are the parametric equations of the locus of R.
To find the cartesian equation of the locus of R, it is necessary to eliminate t.

X2 a
2
G TJ
= a 2 (t2 2 + A)
= a 2 (t2 + A) - 2a 2
a2. 2y _ 2a2
a
X2 = 2a(y - a)
24/CALCULUS 75

This is the cartesian equation of the locus of R.


Since it is of the form (x - h)2 4a(y - k), the locus is a parabola with vertex (0, a),
focallength~, focus (0, 3;). directrix the line x = ~.
The diagram shows one position of R. You should now sketch the locus.

EXAMPLE 8
Prove that the tangents at the extremities of a focal chord intersect at right angles on the
directrix.

Y= Fig. 24-8

Let the co-ordinates of P and Q be (2al, at2 ) and (2as, as 2 ) respectively of the parabola
X2 = 4ay
A focal chord is any chord which passes through the focus S (0, a).
Gradient of PS = gradient of QS
a/ 2 - a as 2 a
2at
12 - 1 S2 -
--
t s
ts 2 - (t 2 - l)s t =
(ts + l)(s - t) = 0
°
s = tor
t
But s "* t and hence s = t
rJJ:.-~
dx - 2a
·
:. grad lent f
0 tangent at
P = 2at
2a = t
Gradient of tangent at Q = s
= --t1
Since the product of their gradients is -1, the tangents are at right angles.
Equation of tangent at P is y = tx - al 2
When y -a, x = a(t2 - 1)
t
Equation of tangent at Q is y = sx - as 2
x a
t- j2
76 NEW SENIOR MATHS: THREE UNIT COURSE

2
When y -a, x a(t t- 1)

The tangents meet at the variable point R (a(t2 t 1), -a ) which is on the directrix

because y = -a.

EXAMPLE 9
The normals to the parabola X2 4ay at points P and Q intersect at R. If the chord PQ
varies in such a way that, for all positions of P and Q, the chord PQ when produced
passes through the fixed point C (0, -2a), show that R lies on the parabola.
y

C 10, -2al Fig. 24-9

Let the co-ordinates of P and Q be (2at, at 2) and (2as, as 2) respectively.


Gradient of PC = gradient of QC
at 2 + 2a as 2 + 2a
2at
t2 + 2 S2 + 2
t S
ts 2 - (12 +
2)s + 2t = 0
(ts - 2)(s t) = 0
2
s = tor t

But s '*t, because P and Q are distinct points.


Hence the relationship between sand t is
2
s=- (1)
t
Equation of normal at P:
x + ty = at' + 2at (2)

Equation of normal at Q:
x + sy = as 3 + 2as (3)
At the point of intersection, R, of these normals, (2) and (3) are first degree
simultaneous equations in x and y.
Solving these equations, we get the co-ordinates of R.
x R = -ast(s + t)
2a(t 2 + 2) . 2
=- t SInce s = t
24/CALCULUS 77

Substituting this value of x into (2) gives


--,---::-+----" + a(t2 + 2)

_ a(t2 + 2)2
- t2

X1 = --'--;;-+-'-

= 4aYR
Since the square on the x-eo-ordinate of R is equal to 4a times the y-coordinate of R, the
point R lies on the parabola.

EXAMPLE 10
(a) Find the cartesian equation of the parabola x 4t, Y ""2t 2 •
(b) The normal at any point P (4t, ""2t2) on the parabola cuts the Y-axis at T. Find the
cartesian equation of the locus of the mid-point Q of PT.
X2 x2
(a) y = -2[2 = -2'16 = -g

(b) : = - ~ = - t = gradient of tangent at P.

:. Gradient of normal at P = ~
Equation of normal at P:
1
y + 212 = -(x - 4t) y
t
At T, x = 0 and so y = -4 - 2t 2 •
Co-ordinates of Tare (0, -4 - 2/2)
Mid-point Q of PT is (2/, -2 - 212)
i.e. xQ = 2t
YQ = -2 - 2/2
X2 X
2 - 2. 4 since I = i
-2
2 (y + 2) is the equation of locus of Q. Fig. 24-10

It is a parabola with vertex (0, -2), focal length ~ unit, focus (0, -~), directrix the line
Y

EXAMPLE 11
Find the equation of the chord of contact of
the tangents to the parabola X2 4ay from
an external point P (x o, Yo)
x
Let the tangents from the external point P (x o, Yo)
touch the parabola at Q and R.
The chord QR is called the chord of contact. Fig. 24-11
78 NEW SENIOR MATHS: THREE UNIT COURSE

If Q is the point (Xl> YI)' the tangent PQ has equation


XXI = 2a(y + Yt)
P is on this tangent and so its co-ordinates (xo. Yo) satisfy it.
:. XOXI = 2a(yo + YI) (1)

If R is the point (x 2 • Y2)' the tangent PR has equation


xx! 2a(y + Y2)
P is also on this tangent and so its co-ordinates (x o• Yo) satisfy it.
:. XOXl = 2a(yo + Y2) (2)
From (1) and (2) we conclude that (XI. YI) and (X2' Y2) lie on the line
. I XXo = 2a(y + Yo) I
Hence this is the equation of the chord of contact QR.
Note: To find its equation it is necessary to know only the equation of the parabola and
the co-ordinates of P.

EXERCISES 24(c)
Write down the cartesian equation of the tangent and normal to the following parabolas
at the points indicated. (1. to 6.)
1. X = 2t, Y = t 2 at the point where t 1. I
I

t2 I
2. X t, Y = 2" + 3 at the point where t = 2.
3. X = t + 2, Y = t 2 at the point where t 1.

)( 4. x = cos 2t, Y = cos t at the point :where t = i.


5. x t + 3, Y t2 - 5 at the point where t 1.
t2
4+ I at the vertex.

7. Find the equations of the parabolas with common focus (0, 2) and directrices given by
Y = -2 and Y = 4 respectively. Find the co-ordinates of the points of intersection of
the parabolas.
Show that, at the points where the parabolas meet, the tangents are perpendicular.
8. The chord PQ of the parabola X2 = 4y sub tends a right angle at the origin O. If the
co-ordinates of P and Q are (2t, t 2 ) and (2s, S2) respectively:
(a) write down the gradients of PO and QO,
'< (b) show that ts -4,
(c) express the coordinates of Q in terms of t,
(d) write down the co-ordinates of the mid-point M of PQ,
(e) show that the cartesian equation of the locus of M is Xl = 2(y - 4).
9. The tangent to the parabola Xl == 4ay at the variable point P (2at, at 2) cuts the X-axis
at Q and the Y-axis at R.
(a) Write down the equation of the tangent at P.
(b) Find the coordinates of Q and R.
y-. (c) Find the cartesian equation of the locus of M, the mid-point of QR.
24/CALCULUS 79

10. P is a variable point on the parabola x = 21, Y = 12 with focus S and vertex A. Q is the
mid-point of SP and R the mid-point of AQ. Find the cartesian equation of the locus
of R. Show that the locus is a parabola and find its vertex and focus.
11. P is a variable point on the parabola X2 = -4y. The tangent from P cuts the parabola
X2 = 4y at Q and R. Show that 3x2 = 4y is the equation of the locus of the mid-point
of the chord RQ.
12. P (4/,2/2) and Q (S/, S/2) are two variable points on the parabola X2 = Sy. Tangents
X from P and Q intersect at T. Find the cartesian equation of the locus of T.
13. The normal at a point P (4/, 2(2) on the parabola x 2 = Sy intersects the Y-axis at Q.
Find, in terms of I, the co-ordinates of Q and find the equation of the locus of the
mid-point of PQ.
14. P and Q are the end points of a focal chord of the parabola X2 = 4ay with focus S. If
the co-ordinates of P and Q are (2ap, ap2) and (2aq, aq2) respectively,
(a) Write down the gradients of PS and QS.
(b) Show that pq = -1.
(c) Find the coordinates of the mid-point R of PQ in terms of p.
(d) Show that the equation of the locus of R is X2 = 2a(y ., a),.
(A focal chord is a chord passing through the focus)
15. Find the co-ordinates of three points on the parabola X2 = 4y such that the normals
through these three points pass through the point <-12, 15).
16. Let P be a variable point on the parabola X2 = 4ay whose focus is at S. Find the locus
of the mid-point of PS. Show that it is a parabola and write down the co-ordinates of
its vertex and focus.
17. Through the vertex A of a parabola,! chords AP and A Q are drawn at right angles to
one another. Show that for all positions of P, PQ cuts the axis of the parabola in a
fixed point K.
18. If two tangents to the parabola x = 2al, y = al 2 meet at riglit angles at Q and the
normals at their points of contact meet at R, prove that QR is parallel to the axis of
the parabola.
19. The normals to the parabolax2 = 4ay at the point P (2al, a( 2) and Q (2as, as 2) meet
at R. Find the co-ordinates of R in terms of I and s. If sl = -2, find the cartesian
equation of the locus of R.

Q
20. In the diagram AP is parallel to the axis
of the parabola and S is the. focus. Prove
that angle APQ = angle SPR. (Hint:
Remember that angles are associated
with gradients) A parabolic mirror has
the property that a ray of light parallel
to the axis passes through the focus after
reflection from the mirror. Likewise, if
S is a source of light, rays of light which
strike the mirror will be reflected
parallel to the axis. Fig. 24-11
80 NEW SENIOR MATHS: THREE UNIT COURSE

21. P (2at, at 2) is any point on the parabola X2 = 4ay. The normal to the parabola at P
meets the Y-axis at G and the X-axis at H. Q (u, v) is the fourth vertex of the rectangle
GOHQ. Show that u = vt.

22. The normal at any point P (2at, at 2 ) on the parabola X2 = 4ay cuts the Y-axis at Q
and is produced to R so that PQ QR.
(a) Express the coordinates of R in terms of t.
(b) Find the cartesian equation of the locus of R.

23. P is a variable point (2at, at 2 ) on the parabola X2 = 4ay and S is the focus. The line
joining P and S is produced to Q so that PS = SQ.
(a) Write down the co-ordinates of Q in terms of t.
(b) Find the cartesian equation of the locus of Q.

24. P (2at, at 2 ) and Q (2as, as 2 ) are two points on the parabola X2 = 4ay. The chord PQ
subtends a right angle at the origin O. If R is the fourth vertex of the rectangle POQR,
show that
(a) st = -4
2
(b) R is specified parametrically by x 2a(t 4), y = a(t4 t; 16)
t
(c) the cartesian equation of the locus of R is X2 = 4a(y 8a)
Describe the locus.

25. Show that only one normal can pass through the focus of the parabola X2 = 4ay and
find from which point on the parabola it originates.

26. How many normals pass through (0, ka), a point on the axis of the parabola
X2 = 4ay. for k > 2? For k = 3, find where the normal meets the parabola again.

27. Find the equation of the chord of contact of the tangents to the parabolas:
(a) X2 = 8y from the point (3, -2)
(b) X2 = Y from the point (-1, -5)
{~) x 2 = -2y from the point (5, 6)

28. From what external point are the tangents to the parabola X2 4y to be drawn so that
3x 2y = 4 is the equation of the chord of contact?

29. A chord of contact to the parabola X2 4y has equation 2y - 3x + 2 = O. From


what external point are the tangents drawn?

30; The chord of contact of the tangents to the parabola X2 = 4ay from the point
P (x o, Yo) passes through the point Q (0, 2a). Show that the locus of the mid-point of
PQ is the X-axis.

31. Show that the chord of contact of tangents from the point (a, -a) to the parabola
X2 = 4ay has length 5a.

32. P and Q are points on the parabola X2 = 4ay. Tangents TP and TQ are drawn from
, an external point T and these tangents cut the X-axis at A and B. Show that the line
joining the focus to the mid-point of AB is perpendicular to the chord of contact PQ.
~~~.,~.~~~-------------------------~~--

24/CALCULUS 81

1 _ _ __
24.4 Fundamental limit: lim si~ (j
0-0
Consider a sector OAB of a circle of unit
radius (Fig 24-12), The tangent at A meets
c
OB at C_ Let the radian measure of angle
AOB be 8,

Area of 6AOB < area of sector AOB < area of 60AC


i.e,
~ ~
sin e < 8 < tan 8 (0 < 8 < ~ ~)
sin e < e < tan 8
0 A
1 11k' ,
~ > a > t---;)' ta mg reclproca s
1
sm v v an v Fig, 24-12
1 > sin 8 > cos v,
il
mu I'tipI'ymg by sm
' vil

As e -- 0, cos e -- '
1 an d smce sin e I'les b id
etween tam m t he I'Imlt,
an cos v, we 0 b"
il ,

I iim sinee = 1.
6-0
I
Note that in this theory, 0 is given in radian measure, Hence all deductions which use
this fundamental limit must consider 0 E R and not 8° .
We can show, by considering a small value of 8, say 0'1, and by the use of a calculator
that sin 0 "" 1 when 0"" O.

When e=
0,1 . sin 0,1 = sin 0,1 c "'" 0,09983 0-9983
'0,1 0·1 0,1
When 8 = 0,01 sin 0-01 = sin 0·01 c "'" 0,0099998 = 1
'0,01 0-01 0-01
In effect, this means that for small values of 0, sin 0 "'" 0 and cos 0 = 1 and, in the limit,
asO--O
sin 0 = 0 and cos 0 = 1
EXAMPLE 12

Find (a) Ii sin 4x (b) lim tan 0


x.!n 2x 0-0

- tan 0
()
a , sin
I1m 4x I' sin 4x
- - " , = 1m - - x
2 (b) I1m = 1m sin 0 + cos 0
I'
x_o 2x x-o 4x . 0-0 0-0

. sin 0 , sin 0 + I'1m cos 0


= 11m
0-0
x 2 where 4x =0 I1m
0-0 0-0
1 x 2 1 + I
= 2

EXERCISES 24(d)
Evaluate the following limits
1 r sin 2h , sin x
• hl~ -h- 2• IIm--
x-o 3x
- 2 0
- sin-5x
I1m2x
3• x-o - 4 r sm
• Q~ -0-
~_ ~ ___ ......... /I

82 NEW SENIOR MATHS: THREE UNIT COURSE

1 - cos 2x ' sin (7f - x)


5. x_o
lim -----::--- 6• I1m --'-----'-
X2 x-o x
1 - cos 0 8 r tan 2h
7. lim
8-0
02 'h~3h
, sin 2x 10. lim sin 3x
9• I1m - , -
x-o sm x x-o 2x .

24.5 Derivative of sin x and cos x _ _ _ _ _ __


Letf(x) = sin x and sof(x + h) = sin (x + h)
By definition,
f'(x) = li~ f(x + h) - f(x)
h-O h
r sin (x + h) - sin x
hlEA h
, 2 cos (x +h !h) sin !h by expressmg
= I1m 'h t ed'ff
l erence
h-O of two'smes as a product
= ' cos (x + 2-1)
I1m I'1m -sinIh-
h . h-O !h
h-O "2
= cos x.1
== cos x
The derivative of cos x can be deduced from the derivative of sin x by using the fact that
cos x= sin (~ - x) and the chain rule,
Let y = cos x
= sin (~ - x)
'h
== sm 7f - x
u were u == 2
dy dy du
dx :::: du' dx
=cosu.-1
== -cos(~ - x)
sin x

24.6 Some techniques of integration _ _ _ _ __


(a) Change of variable (use of substitution)
This method may be regarded as the converse of the method of differentiating a
composite function, The aim of the substitution is to transform a situation to one which

involves a standard result, e.g. un du f --1


un + 1 + c
n + 1

Let y :::: ff(U) du, where u g(x)

so that dy :::: f(u)


du
24/CALCULUS 83

dy dy du
But dx = duo dx
du
= feu) 'dx

i.e. y = !f(U).~~dX

I.e. !f(u).:dX =!f(u).dU

This form of the rule is convenient when, by means of a substitution u = g(x), the
expression whose primitive is sought may be expressed as the product of ~~ and a function
of U.
du
For example, if f(x) = 3X2(X3 - 1)4 and we put u Xl - 1 so that dx = 3x2 thenf(x)
may be written as
f(x) = (x 3 1)4 (3X2)
=u 4 du
- where u = x3
dx
If f(x) = x vii + x 2 , we observe that 2x is the derivative of 1 + X2 and hence, if we
make the substitution u = 1 + X2 and write x as ~ (2x), then
1 du du
f(x) = 2UI12 dx where u 1 + X2 and dx = 2x

If f(x) = (x;. : ixp' we observe that 2x .--r 2 is the derivative of x 2 + 2x and hence if

we make the substitution u X2 + 2x and write x + 1 as ~(2X + 2), then


1 du
f(x) = "2 U-3 dx
Iff(x) = x and if we make the substitution u = 1 x so that x = 1 u and
du
- = -I ' then
dx
f(x) = - (1 - U)U 1/2 ( 1)
- (U 1l2 - U l/2 )(_1)

= -feu) ~~
This is a little different from the previous examples. However, in all cases, the
substitution will be given.

EXA-MPLE 13
Find

0) !3X 2(X 3 - 1)4 dx using the substitution ~ = x3 - 1.

(ii) ! x .,JJ+X2 dx using the substitution u = 1 + x 2•


84 NEW SENIOR MATHS: THREE UNIT COURSE

(iii) J(X; : L~ i dx using the substitution u = X2 + 2x.

(iv) J
x .JI - x dx using the substitution u = 1 - x.

(v) J~ dt using the substitution u = 1 + t.

(vi) J(3X - 5)4 dx using the substitution u = 3x - 5·.

(i) J 3x 2(X 3 - 1)4 dx

=
J du
u 4 -dx where u
dx
= x3 - 1 and -
du
dx
= 3x2.

= f
1
u 4 du

= -u 5 + C
5
1
= S(x 3 -1)5 +c

(ii) J .J + x 1 x 2 dx

=J~(2x) .J + 1 x 2 dx

=f i ~~dX
du
UI12
where u = + X2 and dx
- = 2x

= ~JU1/2 du
1
= -U 3/2 + C
3
1
= 3" (1 + x2)3/2 + C

( iii) J ++(X2
x 1 d
2x)3 x

=f ~(2X + 2)(x:2 + 2xt3 dx

=:2If ~ u-3 dx dx where u = x2 + 2x and ~


dx = 2x + 2

= ~fU-3 du
1
--u-2 + C
4
1 + C
4(X2 + 2x)2
24/CALCULUS 85

(iv) fX~dX
= f (l - U)U I12 : dx where U 1
du
X and - =
dx
1.

- f(U 1I2 - U312 )( -1) dx

- f(U '12 - U312 ) du


= - 23-U 312 + -U
2 S12
5
+C
2 2
5" (1 - X)512 - 3" (1 - X)312 +C

(v) fk dt

f -JUu - 1 du
. dt dt where u = 1
du
+ t and dt = 1

j(U 1l2 u- '12 )(I) dt

j(U 1l2 - U- 1I2 ) du


2 .
3"U
3/2
- 2u l12 +C
~ (1 + tp/2 2(1 + t)1I2 +C
3

(vi) j (3x - 5)4 dx

= f~ (3)(3x - 5)4 dx

= !fU
3
du dx where u
dx
4 = 3x - 5 and du
dx
= 3

= ~fU4 du
= ~U5 + C
15
= /5 (3x 5)5 +C

Note: you should check your answers in each case by differentiating your answers.
,

86 NEW SENIOR MATHS: THREE UNIT COURSE

EXERCISES 24(e)
Find (1. to 20.)

1. f 2x(x2 1)4 dx, using the substitution u = Xl - 1

2. f 3X2(X3 + 4)3 dx, using the substitution u = x 3 + 4

3. f X2 .J.X3+l dx, using the substitution u = x 3 + 1

4. J (21 + 1)3 dt, using the substitution u = 21 + 1

5. J .J ()X_ 4) dx, using the substitution u = x 2 - 4

6. j(2X + l){x2 + X + 2)5 dx, using the substitution u = X2 + X + 2

7. f d:(2x 3)3' using the substitution u = 2x +3

8. J (3 - 2X)6 dx, using the substitution u =3- 2x

3x + 1
9•
J d . h b' .
(3x 2 + 2x· + 5)2 x, usmg t e su stItubon u 3x 2 + 2x +5

10.J (x 2 - 2x)(x 3 - 3x2 + 1)4 dx, using the substitution u x3 3x 2 + 1

n. J 3X2(X3 + 1)4 dx, using the substitution u = x3 + 1

12. J.J (1 t_ t 2 ) dt, using the substitution u 1 - f2

13. J(3x - 5)213 dx, using the substitution u = 3x - 5

14. J2t dt, using the substitution u =1 t2

15. Jx .Ja 2 - X2 dx, using the substitution u = a2 - X2

I~. Jz ~ Z2 + 1 dz, using the substitution u = Z2 +1


17. Jy dy, using the substitution u y +

'18. J~
(x IP dx, using the substitution u x-I

19. J.J(2x
x
1) dx, using the substitution u = 2x
24/CALCULUS 87

20. JX 2'V'(1 + x 3) dx, using the substitution u = 1 + x3


21. If: = x 'V' X2 - 4, and y
(Use the substitution u X2 -
2 when x = ..[5, find y in terms of x.
4).

22. Iff' (x) 'V'()~ 1) for all x, findf(x) given thatf(O) 2.


(Use the substitution u = X2 + 1)
'dx 1-1 ..
23. If dl = 'V'(l2 _ 2t + 4) and x = 10 when I = 0, fmd x m terms of I.
(Use the substitution u = t 2 - 2t + 4).

,,24. Given ~~ = (1 ~ r)4' and that r(O) = 0, find r in terms of O.


(Use the substitution u = 1 r).

25. Find the equation of the curve whose gradient at any point x > ~ is 'V'2x - 1 and

which co~tains the point (~, 9). (Use the substitution u == 2x 1).

26. Given that ~~ = 2(4 ~ x)2 and that x = 0 when t 0 find x in terms of I.
(Use the substitution u 4 - x).

F;valuation of definite integrals using substitution


A most important consequence of change of variable by use of substitution in definite
integrals is that the terminals (or limits) of integration must also change in a manner
defined by the substitution.

EXAMPLE 14

r
Evaluate:

(i) 2x'V' X2 - 1 dx using the substitution u == X2 -

(ii) r: x ~ dx using the substitution u == 4 - x

(iii) L X2(X3 + 1)4 dx using the substitution u = x 3 + 1.

du
(i) Put u = X2 1 and so dx 2x
Terminals: When x = 1, u = X2 - 1 0
When x 2, u 3
.. 3 du
Hence
~
r 2x .JX2=-I dx
1
2
Jo .JU dx dx
= J0
3 u 1l2 du

= 2 3
[ 3-u !2
J3 == -32 . 3
0
3/2 = 2..[3
.
88 NEW SENIOR MATHS: THREE UNIT COURSE

(ii) Put u = 4 - x where x ~ 4. Thus ~~ -1 and x ~= (4 u)-JU


Terminals: When x -5, u = 4 - x =9
x 3, u = 1

Hence ,r: x -"/4 - x dx =


1
J9 (u 4)v'U ~~ dx
4U 1l2 ) du

8-U 312
3
Jl
9

=[(~ D-(~.3S - ~.33) J


412
15

(iii) Put u = x3 + 1 and so du 3x 2


Terminals: When x = 0, u = 1
When x = 1, u =2
Hence If I
-
3 0
3X2(X3 + 1)4 dx = -1 j~2 u 4 du
3~1
- dx
dx

= -1
3
J2 I
u 4 du

= /5[ usT
= /5 [32 - 1]
31
15
EXERCISES 24(f)
Evaluate: (1. to 20.)

r I
1. , x -,,/.-xzdi using the substitution u = I - Xl
~ 0

'v;' 2. r
.' -I
2 x x dx using the substitution u = 2 - x

'>< 3. J o
2 V 22x
(x + 1)
dx using the substitution u =~ + 1
4•
J
~
312
1I2 -,,/ (2x

3
2 - 2x d . h b' .
_ X2) x usmg t e su stltutIon u = 2x - X2

5. J x -,,/(x
2 . 1) dx using the substitution u = x-I
1/2

6. J -112
-,,/ (l X
- X
2) dx using the substitutionu = 1 - X2
24/CALCULUS 89

? fa v' -/3 (1 : X2) dx using the substitution u

8. f 3
4
(2x 3)(X2 - 3x + 2)2 dx using the substitution u X2 - 3x + 2

9. J02 (x 2 ~ 2)2 dx using the substitution u X2 + 2

10. J 4
312
v'21 + 1 dt using the substitution u = 21 +1
~ 1

11. J v' (1 1+ t) dt using the substitution u =


0 1 +t

12. r13xl
oJ 0
(X 3 - 1)4 dx using the substitution u x3

13. f_: xv'a 2 - xldx using the substitution u a 2 - Xl

-/2
14.
f 0 z~ Z2 + 1 dz using the substitution u = Z2 +1

15. r X v'Xl -
., 2
3 4 dx using the substitution u = Xl 4

16. l5 v'2x - 1 dx using the substitution u = 2x - 1

17. r y ...J.Y+I dy using the substitution u = y + 1


., 0
3
.
4
18. J X v'16 - X2 dx using the substitution u = 16
~

0 X2

19. J h
~ 2

dx using the substitution u = 4 - X2


1

20. r:X<3X - 1)4 dx using the substitution u = 3x - 1

21. Find the area bounded by the curve y = x~, the X-axis and the lines x 0
and x ='I. (Use the substitution u == 1 X2).
x
22. Find the area of the region bounded by the curve y = -;==;:==0-. the X-axis and the
lines X = .J2 and x = ...)5.
23~ Sketch the curve defined by y2 = x2(l - x) and fino the area of the loop.

(2~ If f(x) = X2(X - 3) calculate the area between the X-axis, the lines x = 4 and'x = 6
'-/ and the curve y = v'f(x).

~ Calculate the area of the region bounded by the curve y = v' (X 2X+ 4)' the X-axis and
. the lines x = 0 and x = 2.J3. (Use the substitution u = X2 + 4).
90 NEW SENIOR MATHS: THREE UNIT COURSE

(b) Integration of sin2 x and cos 2x


Students should accustom themselves to associating sin 2 x and cos 2 x with cos 2x.
Since cos 2x = 2 cos 2 X I
= 1 - 2 sin 2 x
I
it follows that cos 2 x = - (l + cos 2x)
2
and sin 2 x = .!2 (1 - cos 2x)

Thus J cos 2 X dx = ~ J(l + cos 2x)(lx

= 21x + 4Ism
. 2
x +c

and JSin 2 x dx ~ ((1 - cos 2x)dx


"
1 - -I sm
-x . 2x + c
2 4

(c) jf(U) :~ dx = jflU) du where U = sin x or cos x

e.g. J cos x sin 2 x dx

J u dx
~ 2 du d h .. d du
= x w ere u = sm x an dx = cos x

= J u 2 du

1
= -u 3 + C
3
1 . 3
=3"smx+c

EXAMPLE 15

Evaluate (i) fo"/22 sin2 x dx (Ii) J,,::2 sin x cos 2 x dx


f
f
,,/2 ,,/2
(i) o 2 sin 2 x dx = 0 ( I cos 2x)dx

= [ X - 21 sin 2x J
,,12
0

= (~ - ~ sin 1r ) (0 ~ sin 0 )
1r
= -
2 ,
1f/2
,,/2 du du
(ij)
r
.; .,./3
sin x cos 2 x dx
J ill
- u 2 -d dx where u
x
= cos x and -dx sin x
24/CALCULUS 91

= -J 0
112
u 2 du since u o when x 7r I
= 2 and u = 2 when x = :3
7r

(d) ff(x) dx = f f(x) :; du

So for we have used the change of variable rulejf(u) ~~dX = j f(u) duo A second form
of the rule may be obtained by interchanging u and x. On doing this we obtain:

j f(x) dx = jf(X) ~~ du

= g(x), the
The first form of the rule is convenient when, by means of a substitution u
expression whose antiderivative is sought may be expressed as the product of ~~ and a
function of u.
= 15x2 (x 3 - x3 du
For example, iff(x) 1)4 and we put u 1 so that dx = 3X2 thenf(x)
may be written as
f(x) = 5(X3 - 1)4(3x2)
= 5u 4 du
dx
Whenf(x) cannot be expressed as such a product, the second form of the rule involving
a substitution x g(u) is required.
The rule is most commonly used in the form

j f(x) dx = jf(X) ~~. de


involves a substitution x = a sin eor x a cos ewhile the
{-;c---;;
The occurrence of
occurrence of a 2 + X2 involves a substitution x = a tan e. However, in most cases, an
appropriate substitution will be given.

EXAMPLE 16 ___ ~-------- :, J,.

Evaluatefo 2 dx using the substitution x = 2 sin e,

Since x = 2 sin 0,
dx
dO = 2 cos 0
and ..J 4 - X2 = r;--~,..--;:-""", ..J 4 cos 2 0 = 12 cos 01
= 2 cos 0 since - i .; ; 0 .;;; i
92 NEW SENIOR MATHS: THREE UNIT COURSE

Terminals: When x 2, 2 2 sin 0, sin 0 = 1, 0 = i


When x 0, 0 = 2 sin 0, sin 0 0, 0 : : : 0

dx = i"/2 r-;---;;: ~~ • dO

= j 0
"l2
2 cos 0.2 cos 0 dO

J
"12
= 2 0 2 cos 2 0 dO

=2 j 0
"/2
(1 + cos 20) dO

Geometrically, the graph of y ~2


between x 0 and x : : : 2 is the quarter of
the circle of equation X2 + y2 = 4 in the first
o 2 x
quadrant.
Area i. 7rr2 = 7r, since r = 2.

Note: We put -../4 -X2 = : : : 2 cos O. From our knowledge of square root,
= 12 cos 01 = 2 cos 0 if cos 0 > O. The restriction we placed on 0, namely
7r 7r 7r 3 7r ;--,:----::--;
- 2 < e < 2' ensured that -../4 cos 2 e 2 cos O. If 2 < e < ~ 4 cos 2 e = -2 cos 0
since cos 0 < O.

EXAMPLE 17

1
Evaluate 50l dx using the substitution x = tan e.
Since x tan e'dO
dx -- sec 2 0

1 1
and
sec 2 0
7r
Terminals: When x : : : 1, 1 : : : tan 0 and so 0
4
When x = 0, 0 : : : tan 0 and so e o
1 1
:.
J 0
I
1 + X2 dx J o
,,/4

1 + X2
dx
dO dO
24/CALCULUS 93

1
fr
"./4
~O . sec 2 0 dO
osee
"./4
= J0 1 dO

1r
4

EXERCISES 24(g)
Find: (1. to 10.)

1. 12 cos 2
X dx 2. J2 sin x dx 2

3. I sin 2 x cos x dx 4.Jtan x sec 2 x dx

5. I sin x cos 3 x dx 6. rcos x sin4 x dx


.t

7.1(1 + cos 2x) sin x dx 8. Jsin x cos x dx


)\'9. Jsin I dx 2
10. 1 2 cos 2 I dx

Evaluate: (11. to 18.)

I
"./4 ~ 11"/3

11.
J 0 sin 2 x dx 12.
oJ "./6
cos 2 X dx

r.
13.,
~ ;r/4
/2
sin x cos 3 x dx 14. 1-: sin3 x cos x dx
"./4
-j" ".
J
3lT12
15. 0 tan x sec 2 x dx 16. sin x cos x dx

J
"./2 (
17. JolT 2 sin 0 cos 2
0 dO 18.
-"./2
cos 2 x

19. The region under the curve y = sin x between x = 0 and x 1r is rotated about the
X-axis. Find the volume of the solid of revolution.

2O~ The region under the curve y = cos x betw!en x i and x = f is rotated about the
X-axis. Find the volume of the solid of revolution.
94 NEW SENIOR MATHS: THREE UNIT COURSE

Evaluate: (21. to 30.)

21. J~ dx using the substitution x = 4 sin O.

22. Jx 0
1
dx using the substitution x = sin 0

23. J ~x
2
0 using the substitution x = 2 tan 0

24. Jo
O'5 (I _d~2)3/2 using the substitution x = sin 0

25. J ,,/4 sin 2


,,/8
x cos 2 x dx using the fact that sin x cos x = 2! sin 2x

,j'",26. f-1/2
1/2
..J 1 X- X
2 dx using the substitution x cos 0

27 .
fv
1/2
I I - x2 d . t h e su b'
x usmg . x = cos 0
stltutlon

28 •
f +
0
-1/2
1
dx
4
X
2
.
usmg t he su b"
stltutlOn x = -21 tan 0

29. Jo\ ..J16 - x 2 dx using the substitution x 4 sin 0

J 1
.,f3

30. 0 9 x2 dx using the substitution x = 3 tan 0


CHAPTER 25

Applications of
Calculus to the
Physical World

25.1 Related rates _ _ _ _ _ _ _ _ _ _ _ __

Related rate problems arise when a function is given which relates two variables, say, x
and y, where both x and yare functions of, say, time. We may be asked to determine
CZ when ~~ is known. In such cases it is necessary to use the chain rule.
dy dy dx
dt = dx' dt
EXAMPLE 1
A spherical balloon is being inflated and its radius is increasing at the constant rate of
3 em/min. At what rate is its volume increasing when the radius of the balloon is 5 em?

It is required to find ':: when r = 5, given that r;;; 3.

dV dV dr
dt = dr' dt
But v
Thus ~~ = 4'u2 • 3 = 300?r cm 3 /min when r = 5.
96 NEW SENIOR MATHS: THREE UNIT COURSE

EXAMPLE 2
A vessel containing water has the shape of an
inverted right circular cone Of base radius 2 I
m and height 5 m. The water flows from the 1
1
1
apex of the cone at the constant rate of
0·2 m 3 /min. Find the rate at which the water
level is dropping when the depth of the water
+1
I
1

5
I I
is4 m. I I
I
h I
I
J

Fig. 25-1

At time t min, let the depth of the water be h m, the radius of the cone at water level be
r m and the volume of the water V m 3 •

It is required to find ~~ when h = 4, given that ~~ = -0,2.

The volume of the water at any time t is given by V


Both rand h are variables and so it is necessary to express r in terms of h and thus
eliminate r.
From similar triangles,
r 2. 2h
Iz = :5 I.e. r
Thus V -- 411"h J d dV _ 411"h2
75 an so dh - 25
dV dV
-'-
dh
dh dt
-0,2 = 411" h 2 • dh
25 dt
. dh 5 5
.. dt 411"h2 --whenh 4.
6411"
"" -0-0247
The negative sign indicates that the water level is decreasing at the rate of 0'0247 m/min.
Alternatively, since Vand h are both dependent on time, we can differentiate both sides
of the equation
V ~; h 3 with respect to t
dV = 411" !L(h 3 ) 411" 3h 2 dh
dt 75 dt _ . dt
411" dh
i.e. -0-2 = 25' h 2 . as before.

EXAMPLE 3

The volume of water in a hemispherical bowl of radius 10 cm is given by V


where x cm is the depth of the water at any time t.
The bowl is being filled at a constant rate of 211" cm 3 /min. At what rate is the depth
increasing when the depth is 2 cm?
25/APPLICATIONS OF CALCULUS TO THE PHYSICAl.: WORLD 97

We are given that ~ :: 271" and are asked to find


dx
dt when x 2. The related variables are V, x and t.
71"
V:: 3X2 (30 x)

:: 107I"x2 - ~X3
3 Fig 25-2
dV
- = 207l"x 7I"X2 7rX(20 x)
dx
dV dV dx .dx 2
.. dt :: x-(=
20::O-----x-::-)
dt dx' dt
1
271" = 7I"x(20 - x) : = 18(cm/s) when x = 2

Question: At what depth is the depth increasing at a minimum rate i.e. for what value of x
. dx .. ?
IS dt a mmlmum.

Note: You should be able to find the formula V :: fa 2 (30 - a) for the volume of water
of depth a cm in a hemispherical bowl of radius 10 cm by finding the volume generated by
rotating a circle X2 + y2 = 100 between y :: 10 and y = 10 - a about the Y-axis. Try it.

EXAMPLE 4
A ladder 10 m long has its upper end against a vertical wall and its lower end on the
horizontal ground. The lower end is slipping away from the wall at a constant speed of
4 m/s. Find the rate at which the upper end of the ladder is slipping down the wall when
the lower end is 6 m from the wall. What is this rate when the upper end draws close to the
ground?
At any time t, the lower end of the ladder is x m from the wall and the upper end is y m
above the ground (Fig. 25-3).
It i~ necessary to find 1; when x 6, given that ~~ 4.
By Theorem of Pythagoras,
x 2 + y2 = 100 (i) y
y :: -Jr.-IOO::C-;:-_-x-=-z, (0 ~ x ~ 10)
:: u l12 where u = 100 - X Z (0, yl
dy dy du
dx duo dx t
-x
"100 _ x2' (0 ~ X < 10)
dy dy du ~k o (X,O) X
dt = dx' dt --too-
dx "" 4(m/s)
Fig. 25-3 dt

3 when x = 6
The top of the ladder is descending at the rate of 3 mls when x = 6.
As y 0 x - + 10 and dy
-+ -+ - (X)
• . dt
How do you interpret this?

..
98 NEW SENIOR MATHS: THREE UNIT COURSE

EXAMPLE 5
A radar tracking station is located at ground
level vertically below the path of an - 900km/h A
approaching aircraft flying at 900 kmlh at a
constant height of 10000 m. Find the rate in I
I
degreels at which the radar beam to the :10000
aircraft is turning at the instant when the I
aircraft is at a horizontal distance of 3 km I
from the station. S x
S represents the tracking station and x the
horizontal displacement of the aircraft at Fig. 25-4
time t (Fig. 25-4).
1000 dx
Now 900 kmlh = 900 X 3600 mls = 250 mls and so dt = 250
dx
x = 10000 cot e and so dO = 10000 cosec 2 0
de sin 2 0
Thus dx = -10000

I,t is required to find ~: when x = 3000 (m)

e,
We are dealing with three related variables x and t.
dO = de . dx = sin2 0 250 sin2 e
dt dx dt 10000 x 40
When x 3000 , tan e
10 000
3000
10
3 so sm
. e
Hence, at this instant,
dO 100 5 5 x 180
dt 109 x 40 (rad/s) = 218 (rad/s) = 218 x 7r (0 Is) "" 1'3(° Is)

EXERCISES 25(a)
t. A spherical balloon is being filled with air at the rate of 100 cm 3 /min. At what rate is

o the radius of the sphere increasing when the radius is 5 cm?


A lamp is 6 m directly ab~ve a straight path. A man 2 m tall walks along the path
away from the light at a constant speed of 1 m/s. At what speed is the end of his
shadow moving along the path? At what speed is the length of his shadow increasing?
3. Water is being poured into an inverted right conical vessel whose apex angle is 90° at a
constant rate of 3 cm 3 Is. At what rate is the water level rising when the depth is 7r cm?
4. The perimeter of a circular sector is 20 cm. The radius is increasing at the rate of
5 cm/s. At what rate is the angle of the sector changing when the radius length is
10 cm? Also at what rate is the area changing when the radius length is 10 cm?
5. A ladder 5 metres long has its upper end against a vertical wall and its lower end on
the horizontal floor. The lower end moves away from the wall at a constant speed of
1 m/s. Find the speed at which the upper end moves down the wall 4 s after the lower
end has left the wall.
6. A vessel has such a shape that when the depth of water in it is x cm, the volume of
water is V cm 3 where V = 108x + x 3 • Water is poured into the vessel at a constant
251 APPLICATIONS OF CALCULUS TO THE PHYSICAL WORLD 99

rate of 30 cm 3 Is. At what rate is the level of water rising when the depth is 8 cm?
fr'\
:\!J The cross-section of a trough I metre long is in the shape of an isosceles triangle of
base measurement 2a metre and height b metre. Water leaks from the trough at a
constant rate of c metre 3 Imin. Find the rate at which the water level is falling when
the depth of the water is ~ metre.
8. A particle moves on the circumference of the semicircle y -J 4 - x 2 • Find 1t when

. . dx
x = 1, If dt 4.

9. Sand is poured into a heap in the shape of a right circular cone whose semi-vertex
angle is a, where tan a = ~. When the height of the cone is 16 em, the height is
increasing at the rate of 2 cm/min. At what rate is the volume increasing at that
instant?
@ When a gas expands without change of temperature, the pressure p and volume v are
given by the relation
pV!'4 = k, a constant.
At a certain instant, the pressure is 25 gm/cm2 and the volume is 32 cm 3 • If the
volume is increasing at the rate of 5 cm 3/s, at what rate is the pressure changing at
this instant?
11. A kite, 50 metres high, is being carried horizontally by the wind at a rate of 4 m/sec.
How fast is the string being let out, when the length of the string is 100 metres?
12. Rain is falling and is collected in an inverted cone so that the volume collected
increases at a constant rate of 4?r cm 3 /hour. If the radius r of the cone is half its
height h, find the rate, in cm/hour, at which the height is increasing when h = 3.
13. Sand is poured into a heap in the shape of a right circular cone whose height is always
equal to the radius of the base, at a constant rate of 4 cm 3 /min. When the heap is
10 cm high, how fast is
(a) the height increasing,
(b) the area of the base increasing?
14. A melting snowball which is always spherical in shape is decreasiI)g in volume at a
constant rate of 8 cm 3 /min. Find the rate at which the radiu~Changing when the
radius is 4 cm. ~.

15. A boat is being pulled into a wharf by a rope at a speed of 20 m/min. If the rope is
attached to a point on the boat, 7 m vertically below the wharf, at what rate is the
rope being drawn in, when the boat is 24 m from the wharf?
16. A particle is moving along a curve with equation y = x 2• If dxl dt 2 when x 3,
find dy I dt at that instant.
17. A straight railway track and a straight road in~rsect at right angles. At a given instant
a motor car, at 40 km/h, and a train, at 50 km/h, are moving away from the
intersection and are 40 and 30 km respectively from the intersection. At what rate is
the distance between them changing one hour later? At what rate would the distance
between them be changing at that instant if they were both travelling towards the
intersection?
100 NEW SENIOR MATHS: THREE UNIT COURSE

,~

:18) Two straight roads meet 8,t an angle of 60°. A starts from the intersection and travels
J along one road at 40 km/hr. One hour later B starts from the intersection and travels
along the other road at 50 km/hr. At what rate is the distance between them changing
three hours after A starts?
19. A circular oil slick lies on the surface of calm water. Its area is increasing at the rate of
10 m 2 /min. At what rate is the radius increasing at the time at which this is r metre?
20. A conical tank, whose axis is vertical, has ~emi-vertical angle 45°. Water, initially at a
depth of 5 metres, leaks out through a hole at the bottom at the rate of
0·2 -J h metre 3 /min when the depth is h metres.
Find the rate at which the depth is decreasing when the depth is 4 metres.
21. The altitude of a right-angled triangle is 6 cm and the base is increasing at a constant
rate of 2 cm/s. At what rate is the hypotenuse increasing when its length is 10 cm?
22. A loading chute is in the shape of a right square pyramid of base length 10 m and
depth 8 m. Liquid is poured in the top at a rate of 4 m 3/min. At what rate is the level
rising when the depth is4 m?
23. A metal sphere is dissolving in acid. It remains spherical and the rate at which its
volume decreases is proportional to its surface area. Show that the radius of the
sphere decreases at a constant rate.
I
;
.r;;;
~
A circular cylinder of height 6 cm and base radius 4 cm sits on a table with its axis
vertical. A point source of light moves vertically upwards at a speed of 3 cm/s above
the central axis of the cylinder, thus casting a circular shadow on the table. Find the
rate at which the radius of the shadow is decreasing when the light is at a distance
4 cm above the top of the cylinder.
25. A point P moves on the curve y x 3 and the x co-ordinate increases at a constant rate
of 5 units per second. At what rate is:
(a) the y coordinate of P increasing when x = I?
(b) the gradient of the curve increasing when x l ?
26. Grain is ejected from a chute at the rate of 0·1 m 3 /min and is forming a heap on a flat
horizontal floor. The heap is in the form of a circular cone of semi-vertical angle 45°.
Find the rate, in metres per. minute, at which the height of the cone is increasing, at
the instant 3 minutes after the opening of the chute.
27. In triangle ABC, AB = 10 cm, AC = 12 cm and angle A is increasing at the rate of
0'1 radian/second. At what rate is:
(a) the area of .6.ABC increasing,
(b) the length of BC increasing, when angle A is i radians?
28. A spherical mothball evaporates at a rate proportional to its surface area so that its
volume V cm 3 and radius r cm after t weeks are related by the equation
dV = -4k7rr 2
dt '
where k is a positive constant.
dr
(a) Show that dt = -k.

(b) If the initial radius of the mothball is 1 cm and the radius is ! cm after 10 weeks,
express r in terms of t.
25/APPLICATIONS OF CALCULUS TO THE PHYSICAL WORLD 101

29. The volume (V) and the surface area (8) of a sphere of radius r are given by V = ~1I"r3
and 8 = 411"r2 respectively.
dV dV dr
(a) Show that = 8 and dt = 8 dt'
(b) A spherical ball, of radius 24 mm, is immersed in an acid bath and its volume
decreases at a rate equal to three times its surface area at that time. Find the time
taken for (i) the radius (ii) the volume to be reduced to one-eighth of its original
size.
30. The radius of a cylinder increases at a constant rate of 0·1 cm per minute and its
height remains constant. At what rate is the volume of the cylinder, whose height is
10 cm, increasing when the radius is 2 cm?
'\
~. Show that the formula for the volume V of a right circular cone of base radius rand

height h can be expressed as V = t1l" tan 2 a h 3 where a is th~ semi-vertex angle.


Water is flowing out through a hole at the vertex of an inverted cone whose vertex
angle is 60° at a rate equal to 11" times the square root of the depth of the water at any
time. At what rate, in cm/s, would the water be flowing out when the depth is 9 cm?
25.2 Velocity and acceleration as
functions of x ______________

The expressions x(t), v(t) x(t) and a(t) = x(t), which denote displacement, velocity
and acceleration, clearly define functions of t. This is obvious on practical grounds since a
particle cannot be in two places at the same time. Thus to each t there corresponds just
one value of x(t) which satisfies the function requirement. Similarly (I, v(t» and (t, a(t»
define functions, for a particle cannot have more than one velocity or more than one
acceleration at any given time. Sometimes, however, instead of expressing acceleration in
terms of t, it is useful to express its magnitude in terms of distance x or in terms of speed v.
In this case some care is needed since an equation relating v with x need not define a
function.
Consider a particle which moves in such a way that its displacement at time t "" 0 is
given by x(t) = sin t. Obviously, the particle moves back and forth between the limits
x = ± 1. This is an example of an important kind of motion discussed later in the chapter.
A p B

-1

o 1
...x
2:

. I .
The partIc e IS at x = 2I whenever sm
.
t 2'I I.e.
.
when t = 6'11" 11" -
11"
6,211"
11"
+ 6' ... so that
= cos t defines a many
x(t) defines a many to one function of time. Similarly v(t) = x(t)
to one function of time. .
Now v 2 = cos 2 t = 1-
sin 2 t 1
x 2 • Hence v 2 = 1-
X2 expresses v in terms of x.
However, this equation does not express v as a function of x for when, for example,
1
x = 2' we may have v = ±
-J3 (1
T' For .mstance 2' -J3)
T corresponds to t = 611" whIle

2' (1 -J3)
T
corresponds to t = 5 11". Since the particle moves to and fro between A and B its velocity
6
has two possible values at each point P in between, depending on whether the particle is
moving to the right or the left. What happens at A and B? Recall that the calculus makes
102 NEW SENIOR MATHS: THREE UNIT COURSE

sense only when applied. to functions. In the present context ~; can only have meaning if
v == v(x) defines a function. Hence in the equation v! = 1 - x! we must restrict v to be
either positive or negative if we intend to differentiate. This simply means that in the
equation v! 1 - X2 we consider v to apply to velocities of the particle as it moves from
A to B, i.e. v > 0, or from B to A, i.e. v < O.
If v(x) specifies a velocity junction of x, then we may use the chain rule of differentiation
to write
dv dv dx
dt dx' dt
dv . dx
= vdx since v = dt
II d(tV2) dv
\ = --;.]V. dx
= d(iv 2 )
dx
Hence acceleration may be expressed in anyone of the forms

The form to use in a particular problem will depend on the form of the equation defining
acceleration.
dv d 2x
If a = j(t), use dt or dt 2 '

If a = g(x), use Jx .
dCV2)

!; if initial conditions are on (t, v).


If a h(v), use
{ d;
v
d
if initial conditions are on (x, v).

It is customary to denote derivatives with respect to time by dots placed above the
dependent variable, ~~-~--. ~,~ ~~"~"".-".-

dx . dv . d 2x .
e.g. dt x, dt = v, dt 2 X

EXAMPLE 6
A particle moves in a straight line so that at time t its displacement from a fixed origin is x
and its velocity is v.
If the acceleration is 3 - 2x, find v in terms of x, given that v = 2 when x = 1.
Since the rule defining acceleration is in terms of x and we are concerned with
expressing v in terms of x, the most suitable form for acceleration is d~;2).
2
d(tv ) = 3 - 2x
dx

:. ~V! = J (3 - 2x)dx
3x - X2 + C
25/ APPLICATIONS OF CALCULUS TO THE PHYSICAL WORLD 103

When x = l,v 2 and so c = 0


.'. ~V2 = 3x - X2
v2 = 6x - 2X2
V = -J6x - 2x 2,
taking the positive square root only since the given condition specified a positive value of
v, namely v = 2, when x = 1. If we were given that v = -2 when x = 1, then we would
require the negative square root. In this case v = --J6x - 2X2.
Students should note the restrictions on x for v to be real. We need
6x - 2X2 ;a. 0 and so 0 ~ x ~ 3

EXAMPLE 7
A particle moves in a straight line so that at any time t its displacement from a fixed origin
is x and its velocity is v. If the acceleration is 3x2and v = -.J2, x = 1 when t = 0, find x as
a function of t.
Since the rule defining acceleration is in terms of x and we are concerned with finding v,
the only suitable form for acceleration is d~;2).

d(!V2) = 3x 2
dx

:. ~V2 = J3x 2 dx
= x3 + C
When x = 1, v -.J2 and so c = O.

... lV = x3
2
2
v 2 = 2x 3
V = -.J2X3
= -.J2x 312
taking the negative square root only since v < 0 when x = 1.

v = dx
- = - .J2x312
dt
. dt
.. dx - ~X-312 (~~ = 1I~~)
t = - ~JX-3/2dX
= .J2x- l12 + c
When t = 0, x = 1 and so c = -.J2.
:. t = .J2x- 1I2 - .J2
- I12 _ t + .J2
X - .J2
1l2 .J2
x = t + ..J2
x = -----,=--
2
(t + .J2)2
104 NEW SENIOR MATHS: THREE UNIT COURSE

EXERCISES 25(b)
1. A particle moves in a straight line so that at time t its displacement from a fixed origin
is x and its velocity is v. If its acceleration is 4 + x and v = 1 when x 0, find v when
x = 1.

2. The acceleration of a particle moving in a straight line is given by ~~ = 7 2x and


the particle starts from rest at the point where x O. Find the speed in terms of x.
What values can x take?

3. At time t, the displacement of a particle moving in a straight line is x. If the


. IS
acce IeratlOn . given
. b y cf2x
dt 2 = 3 . Ie starts f rom rest at x
4 x and t h e partlc f' d
1,m

its velocity in any position. At what other point, if any, does the particle come to rest?

4. A particle leaves 0 with velocity 2 mls and its acceleration thereafter is - i rx m/s 2

when its displacement from 0 is x metres. Find its displacement when it first comes to
rest.

5. A particle moves in a straight line and its acceleration at any time t is cos x. If v =0
and x 0 when t 0; express v in terms of x.

6. A particle moves in a straight line and its acceleration at any time is given by
d 2x _ . 2 F' d dx. h dx_
dt 2 - sm x. m dt given t at dt - 1 when x = O.

7. If d~;2) 2x 3x 2 and v 2 when x = 0 find v in terms of x.


8. A particle moves in a straight line and, at time t, its displacement from a fixed origin
is x. If x = x - 3 and x = -2 when x = 1, find x when x = O.
dx
9. If
dt
= x + and x 0 when t = 0, find t when x = 2.

10 . If dx
dt x + 4 and x = -3 when t 0, express x in terms of t.

11. The velocity of a particle at any time t is 2(4 - x)2. Find its position x(t) at any time t,
given that x(O) = O.

12. If c::; =, (3 x)2 and x = 2 when t = 0, find


d 2x
(a) x a~ a..function of t, (b) dt 2 as a function of x.

13. A particle moves in a straight line and, at time t, its displacement from a fixed origin
is x. If x = x + 3,
(a) express x in terms of x,
(b) find x when t = 1, given that x = -2 when t O.
14. The acceleration of a body moving towards the earth under gravitational attraction
varies inversely as the square of its distance from the centre of the earth. Express this
25/ APPLICATIONS OF CALCULUS TO THE PHYSICAL WORLD 105

as a differential equation and prove that if the b6dy starts from rest at a distance a
from the centre of the earth, its speed atx from the centre of the earth iSJ2k(a - x),
ax
where k is a constant.
15. A particle is brought to top speed with an acceleration which varies linearly as the
distance travelled. It starts from rest with an acceleration of 3 m/s2 and reaches top
speed in a distance of 160 metres. Find (a) the top speed, (b) the speed when the
particle has moved 80 metres.
16. A rocket is fired vertically from the earth's surface with an initial speed V. Assuming
negligible air resistance (a) find an expression for v(x) which gives the velocity at a
distance x from the centre of the earth, (b) find how high the rocket will rise, (c) find
the magnitude of V in order that the rocket should escape from the gravitational
attraction and never return. Evaluate this particular value of V, given that the radius
R of the earth is 6400 km.
(Since the acceleration due to gravity is inversely proportional to the square of the
distance from 0, its magnitude at any point P is equal to ~.)
17. A particle moves in a straight line and its acceleration at any time t is -e-2x • If v =
and x 0 when t 0,
(a) express its velocity v in terms of x,
(b) express its displacement x in terms of time, t.

25.3 Simple harmonic motion


(S.H.M.) _ _ _ _ _ _ _ _ _ _ __

Of particular importance is a certain kind of rectilinear motion known as Simple


Harmonic Motion.

EXAMPLE 8
A particle moves in a straight line so that its displacement x cm from a fixed point 0 in the
line at time t seconds is defined by:
x = 4 sin 2t
Find expressions for (a) the velocity (b) the acceleration.
Let us consider some of the properties of the sine curve to help us determine the nature
of the movement of the particle.
When t = 0, x = 0 x
7r
When t = 4' x = 4
x = 4sin 2t
When t = !! x
2' =0
37r
When t = 4' x = -4 t
When t = 7r, X =0
-4 B
Fig. 25-5
106 NEW SENIOR MATHS: THREE UNIT COURSE

Remember that the particle moves in a straight line along the X-axis. It starts at x =0
~ ~
and takes 4 seconds (approx. 0·8 s) to move to A, another 4 seconds to return to 0,

another i seconds. to move to B and another i seconds to return to 0 again. It then


repeats this pattern every ~ seconds. We say that it oscillates from A to B about the point
0, the centre of its motion, with a period of ~. The distance between 0 and the extreme
positions A and B (in this case 4 cm) is called the amplitude.
dx v
(a) v = dt = 8 cos 2t (1)
8
When t = 0, v 8 at x = 0 v == 8 cos 2t
~
When t = 4' v 0 at x = 4
~
When t = 2' v -8 at x = 0 0 t

3~
When t = v o at x = -4
-8
When t ~, v 8 at x = 0
Fig. 25-6
Note that at A and B, the particle is at rest.
At 0, v = 8 if the particle is travelling in the positive direction and v -8 if it is
travelling in the negative direction.
Squaring (1) we get v 2 = 64 cos 2 2t
= 64(1 - sin 2 2/)
= 64(1 - ~~)
Le. v2 = 4(16 X2) (2)
(2) gives us the velocity in terms of x.
,1,;)
'

(b) acceleration = ~; -liM sin 21 (3)

!x (4)
(3) gives the acceleration in terms of 1 and (4) gives the acceleration in terms of x.
Le. x= -4x
This tells us that when x 0, x = 0 and when x 4 (at A), x =-16 and when x -4
(at B), x = -16. dv
x and x are always opposite in sign. dt
dv 16 sin 2t
16 dt
When 1 = 0, x = 0 at x 0
When 1 = r, x = -16 at x = 4

When 1 = f, x = 0 at x = 0 o t

3~ ..
When 1 = -
4'
x 16 at x = -4 -16 Fig. 25-7 .
When 1 = ~,x 0 at x 0
In this type of motion a particle moves in a straight line so that its acceleration is always
directed towards a fixed point in the line and the magnitude of this acceleration is
proportional to its distance from the fixed point.
Taking 0 as the fixed point, the above description tells us that when the particle is at
251APPLICATIONS OF CALCULUS TO THE PHYSICAL WORLD 107

B 0 A
I
x =-4
i
x=o x=4
I ..
)(=0 IXI=8 )(=0
x=16 x=o X =-16

A (x > 0) its acceleration is towards 0 (x < 0) while when the particle is at B (x < 0) the
acceleration is again towards 0 (x > 0 in this case).
Hence x and x are always opposite in sign and since the magnitude of the acceleration is
proportional to the distance from 0 we may take the differential equation x -kx as
defining the motion.
Since k is a positive constant it is frequently denoted by n 2 and we have

x= -n 2x I
as the basic equation of S.H.M.
Note: When x 0 (at A and B), the acceleration is greatest.
When x = 0 (at 0), the speed is greatest. x
Consider the more general equation a
A
x = a sin (nt + a)
If when t 0, x = 0, the particle is initially
at 0, the centre of motion, 21(" t
n
then sin a 0 B -a
:. a = 0 and so
x = a sin nt (Fig. 25-8) Fig. 25-8

If when t 0, x = a, the particle is initially x


at A, the extreme point,
then sin a 1 A a x = a cosnt
.. a=2"an d so
• 1T"
o 21(" t
x = a sin (nt + ~) B -a n

= a cos nt (Fig. 25-9)


In either case Fig. 25-9
x = asin nt (1) x = a cos nt (I)

v x = nacosnt (2) v X = - na sin nt (2)


v2 = n 2a2 cos 2 nt v2 n 2a 2 sin 2 nt
n 2a 2 (1 sin 2 nt) n 2a 2 (1 - cos 2 nt)
= n 2a 2 (1 - ;~ ) = n 2a 2 (1 - ;:)
= n 2 (a 2 - X2) (3) = n2(a 2 - X2) (3)
dv ..
dv = X -n2asinnt dt = x -n 2acosnt
dt
(4) (4)
The period, T, is the time for one complete oscillation and so
T = 21T"
n
108 NEW SENIOR MATHS: THREE UNIT COURSE

The frequency, f, is the number of oscillations per unit time and so


1
f= - = -
n
T 2?r
The amplitude; a,is the distance from the centre of motion, 0, to either of the extreme
points A or B.
Summary x= n 2x
.P = n 2 (a 2 - x 2 ), - a ".; x ".; a
.x = a sin (nt + a) a sin nt if x(O) = 0
= a cos nt ifx(O) a
1
T
f
The following diagrams illustrate the nature of the velocity and acceleration of a
particle executing S.H.M.; the extreme values are included.
v= na v= -n~2
v=O ~
.....--.- v 0
~--------------------~I------------~I--------~I
-a 0 x a
----.. ----..
v = na v = n..Ja 2 - X2

Fig. 25-10 Velocity


x= n 2a x= n2x
~ ~
x 0
I
a o x a
Fig. 25-11 Acceleration
Note that to the left of 0, x is negative and so -n 2x is positive.
The equation x = -n 2x describes the motion of a particle under the influence of a force
°
which is directed towards the origin and is proportional to the distance of the particle
from 0. This force (and hence the acceleration) is zero at 0, where the speed is greatest.
The force (and hence the acceleration) is greatest at the extreme points, where the speed is
zero.
This type of motion occurs in practice where a particle oscillates about an equilibrium
position e.g.
(a) the to and fro motion of a pendulum bob (approx. S.H.M.)
(b) the up and down motion of a mass attached to a spring
(c) the bobbing motion of a buoy.

EXAMPLE 9
The equation of motion of a particle moving along a straight line is given by the equation
d 2x -16x
dt2
If x 0 and v 4 when t = 0, find its displacement at any time t and state the period
and amplitude.
d 2x
Since 4, the motion is simple harmonic
dt 2
:. x = a sin (nt + a)
= a sin (4t + a) since n 4 (1)
and v = x.= 4a cos (4t + a) (2)
25/APPLICATIONS OF CALCULUS TO THE PHYSICAL WORLD 109

Since x ° when t = 0,
a sin a == °from (1)
:. a = °
Since v = 4 when t = 0,
°
4a cos = 4
4a = 4
a 1
:. x = sin 4t
. 211" 211" 11"
Penod = - =-
n 4 2
Amplitude = a 1
Alternatively, we could use
x a cos (41 + a)in place of (1)
and so v x -4a sin (41 + a)
Since x ° when 1 0,
=

a cos a == °
(2)

2
Since v = 4 when t == ° . 11"
4
-as10 =4
2
a = -I
:. x = -cos (41 + i)
Using the fact that cos 8 = cos (11" - 8)
-cos (4t + i)= cos (11" 41 - i)
= cos (i - 4t)
= cos( 4t - i)
:. x cos (41 i) to keep a > °
Check that cos (41 i) == sin 41
EXAMPLE 10
A particle moves in a straight line so that its acceleration at any time is given by x = -4x.
Find its period, amplitude and displacement at time t given that when t = 0, x = 3 and
v = -6.../3.
Since x= -4x = -n 2x where n 2, the motion is S.H.
x a sin (nt + a)
= a sin (2t + a) (1)
and v = x = 2a cos (21 + a) (2)
When 1 0, x = 3 and hence from (1),
a sin a =3 (3)
When t '= 0, v = -6.../3 and hence from (2),
2a cos a - 6.../3
a cos a = 3..../3 (4)
110 NEW SENIOR MATHS: THREE UNIT COURSE

Divide (3) by (4) and hence


1
tan 0: =
-Jj
57f
(since sin 0: > 0 and cos 0: < 0, 0: is
6
in the 2nd quadrant).
a sin 5 7f = 3 from (3)
6
a = 6
x = 6 sin (2t + 567f)
. 27f 27f
Period == = -
n 2
Amplitude a 6
Try this using x = a cos (nt + 0:) and the result is
x = 6 cos (21 + ~)
X

6
x 6 sin 12t + 5 71")
6
V= 6"10/'3
C ""'-A B
I I I I ... X
57f t
-6 0 3 6 71"
6
-6
Fig. 25-12(a) Fig. 25-12 (b)
Figure 25-12(a) shows the state of motion of the particle at t 0 and Fig. 25-12(b) shows
the displacement x for any time t in the domain 0 ~ t ~ 7f.
We now consider an example where the centre of motion is not at the origin O.

EXAMPLE 11
The speed v (cm/s) of a particle moving in a straight line is given by
v2 = 6 + 4x - 2x2 ,
where the magnitude of its displac~ment from a fixed point 0 is x (cm).
Show that the motion is simple harmonic and find:
(a) the centre of motion (b) the period (c) the amplitude.
Since v2 6 + 4x - 2X2 then
1
ZV 2 == 3 + 2x - X2

Thus 1..
dx 2
(!v2) 2 - 2x
-2(x - 1)
Hence x - 2(x 1) and if we put y = x -
then y = 2y since Y = x
(a) This is the S.H.M. equation about y = 0 as the centre of motion and so the particle
executes simple harmonic motion about the centre x == 1.

(b) Since n2 = 2, the period of the motion is T == ~; 7f.J2


...... - -..~-----------------------------~~

25/ APPLICATIONS OF CALCULUS TO THE PHYSICAL WORLD 111

(c) The extreme positions of the particle are given by the requirement v = O.
i.e. 6 + 4x - 2X2 = 0
- 2 (Xl - 2x - 3) = 0
(x - 3)(x + 1) = 0
x = 3 or

c
o• •
1

3

The particle oscillates between x = 3 and x = -1 about the point x 1 as centre.


Thus the amplitude is 2 (cm).

EXAMPLE 12
A particle moves in a straight line and its position at any time t is given by
x = 3 cos 2t + 4 sin 2t
Show that the motion is simple harmonic and find its greatest speed.
x = 3 cos 2t + 4 sin 2t
11"
= 5 cos (2t a) where 0 < a < 2 and tan a = 43
Thus X = -10 sin (2t - a)
X -20 cos (2t a)
::::: -4x
Since X = - n 2x, the motion is S.H.
or x::::: -6 sin 2t + 8 cos 2t
x::::: -12 cos 2t 16 sin 2t
-4(3 cos 2t + 4 sin 2t)
-4x
The speed is greatest when x :::
0 and so when
cos (2t a) = 0
11" 311"
2t a = 2' T' ...
The smallest positive value of t for which the speed is a maximum is given by
2t - a = f' in which case
x=
-10 sin!!
2
= -10
The maximum speed has magnitude 10.

EXAMPLE 13
A particle moving with S.H.M. has a velocity of 15 metres/second when passing through
its mean position. Find the amplitude of the motion and the acceleration in the extreme
positions, given that the period of the motion is 2 seconds.
211"
T ::::: - ::::: 2 and so n ::::: 11"
n
v2 = n2(a 2 - x 2)
112 NEW SENIOR MATHS: THREE UNIT COURSE

When x == 0 v2 n 2 a2 and hence


225 7r2a 2 so a 15 smce
. a >
0
7r
x == -n 2x so
15 X = - 7r2 x 15 = - 15 7r
when x == -
7r 7r
when x =
15
7r
(_1;)
Thus the amplitude of the motion is ~ metres and the acceleration in the extreme positions
7r
is of magnitude 157r m/s2.
EXAMPLE 14
A particle is moving in a straight line with S.H.M. The velocity of the particle is
respectively .J5 mls and 2 mls at distances of 1 metre and 2 metres from the centre of
motion. Find:
(a) the length of the path;
(b) the period of the motion;
(a) v 2 = n2(a 2 Xl)
When x = 1, v .J5 and so 5 n2(a 2 - 1) (1)
When x 2, v = 2 and so 4 n 2 (a 2 - 4) (2)
By division we obtain
5 a2
4 = -:;---:
Le. a2 = 16
:. a 4 since a > 0
Hence the amplitude of the motion is 4 metres and so the length of the path, 2a, is 8
metres.
(b) Substituting for a in (1) gives
5 = 15n 2

... n
1.
= ..j3 smce n > 0
Thus the periodic time is given by
T = 27r
-n = 27r.J3 (s)

EXERCISES 25(c)
1. The equation of motion of a particle moving with simple harmonic motion is
x = -9x. Find its period, amplitude and greatest speed given that
(i) x 0, x = 2 when t 0 (ii) x = 2, x = 2 when t = O.
2. A particle is moving in a straight line. If x is its displacement at time t and
(~~y = 5(4 X2) find the acceleration in terms of x only. Show that the motion is.
simple harmonic and find its period and amplitude.
3. A particle moves in a straight line. At time t seconds, its displacement x cm from a
fixed point 0 in the line is given by x = 5 sin (~t + i} Express the acceleration in
251 APPLlCA TIONS OF CALCULUS TO THE PHYSICAL WORLD 113

terms of x only and hence show that the motion is simple harmonic. Find:
(i) the period; (ii) amplitude;
(iii) the speed whenx = (iv) the acceleration when x = -21.
4. The speed v mls of a point moving along the X-axis is given by
(i) v 2 = 300 + lOOx - 25x 2
(ii) v 2 = 128 - 32x - 16x2
(iii) v 2 = 6 + 4x 2X2
where x metres is the displacement of the point from the origin. Prove in each case
that the motion is simple harmonic and find:
(a) the centre of motion; (b) the period and amplitude.
S. The displacement x at time t of a point moving in a straight line is given by
x = a sin (nt + 8). Find the form which this expression takes if initially
(a) x = 0 and x = -5, (b) x = 0 and the velocity is negative.
6. The velocity of a particle moving in a straight line is 3;/16 - X2 mls where x metres is
the displacement of the particle from a fixed point in the line. Is the motion simple
harmonic? Find the acceleration when x = 4. What is the maximum speed?

7. Solve the differential equation ~~~ + 16x = 0 subject to the conditions x = 3 and
dx
16 when t = O. Find the maximum displacement and the maximum speed if
dt
x metres is the displacement of a particle moving in a straight line at time t seconds.
8. A particle executes simple harmonic motion. If the particle starts from the equilibrium
position with velocity of 4 mls and the period is ~ seconds, find:
(a) the displacement at time t (b) the amplitude.
9. A particle executes simple harmonic motion. When it is 2 cm from its equilibrium
position, its velocity is 6 cm/s; when it is 3 cm, its velocity is 4 cm/s. Find the period
of its motion and the amplitude.
10. If x = a sin nt +
b cos nt find the velocity and acceleration of a particle whose
displacement from a fixed point 0 is x at time t. Show that x
-n 2x and find the
amplitude and maximum speed.
f,

11. The position x cm of a particle relative to a fixed point 0 at any time t is


x = 5 - 2 cos 2 t. Show, by finding its acceleration in terms of x, that the motion is
simple harmonic. Find:
(i) the centre of motion;
(ii) the period;
(iii) the amplitude.

12. A particle executes simple harmonic motion of period 8 seconds and amplitude 10 cm.
Calculate the velocity and acceleration when its displacement is 6 cm from the centre
of motion. Find also its maximum acceleration.
13. A particle moves in a straight line so that its position x from a fixed point 0 at time t
is given by x = 10 + 8 sin 2t + 6 cos 2t. Prove that the motion is S.H. Find the
period and amplitude ..
114 NEW SENIOR MATHS: THREE UNIT COURSE

14. A particle moves in a straight line. At time t seconds its distance from a point 0 on the
line is x metres. The following is an incomplete table of observations:
t o 18
x o o
Complete the table on two different assumptions, namely:
(i) that the particle moves with uniform acceleration;
(ij) that the particle performs a simple harmonic motion of period 12 seconds.
15. A floating buoy oscillates up and down with the waves, rising and falling 2 metres
about its mean position. Find its greatest velocity and acceleration if the periodic time
is 3 seconds.
16. A particle moving with S.H.M. starts from rest at x == 5 and after 2 seconds reaches
x 2·5. Find:
(i) an expression for the displacement at time t ~ 0,
(ij) the speed at x = 0,
(iii) the amplitude, frequency, and period,
(iv) the maximum speed,
(v) the maximum acceleration.
17. The amplitude of a particle moving with S.H.M. is 5 metres and the acceleration
when 2 metres from the mean position is 4 m/s 2 • Find the speed of the particle at the
mean position and when it is 4 metres from the mean position.
18. A particle is moving with S.H.M. of period 'if seconds and the maximum velocity is
8 metres/second. Find the amplitude and the velocity at a distance of 3 metres from
the central position.
19. A particle executes S.H.M. of period ~; seconds. If it starts from rest at x = to
find: (i) the frequency, (ii) the amplitude.

20. A point moving with S.H .M. has a speed of 5 metres/second when passing through its
mean position O. Find the speed and acceleration when 1'5 metres from 0, given that
the period is 'if seconds.
21. A particle is moving with S.H.M. of amplitude to metres. Find how long it takes to
travel 6 metres from its mean position if the period is to seconds.
22. A point moves with S.H.M. in such a way that its speed is 8 and 6 metres/second
respectively at distances 3 and 4 metres from the mean position. Calculate the period
of the motion and the magnitude of the greatest acceleration.
23. Assume that the tides rise and fall in S.H.M. A ship needs to metres of water to pass
down a channel safely. At low tide the channel is 9 m deep and at high tide 12 m deep.
Low tide is at 9 a.m. and high tide at 4 p.m. At what time can the ship safely proceed?
24. Assume that over several days of constant weather the cycle of temperatures each day
is S.H. between 13° at 4 a.m. and 23° at 4 p.m. At what times of the day would the
temperature be (a) 18° (b) 15° (c) 21°?
-,-_._•... --------- -------------------------

251 APPLICA nONS OF CALCULUS TO THE PHYSICAL WORLD 115

25.4 Projectile motion _ _ _ _ _ _ _ _ _ _ __


In this section we investigate the motion of a particle when projected in any direction and
subject only to gravitational acceleration, i.e. we neglect the retardation associated with
air resistance.
The motion lies entirely in the vertical plane containing the direction of the initial
velocity. The angle of projection (or angle of elevation) is the angle at which the particle is
projected, measured from the horizontal.
The range is the horizontal distance between the point of projection and the point where
the particle meets any plane through the point of projection. The trajectory is the path
described by the particle.
To obtain the equations of motion we consider a typical projectile motion as illustrated
in Fig. 25-13.
y

x r-----
I
Fig. 25-13
The origin is chosen at the point of projection I
and the initial vdocity of projection is Vsina +
denoted by V. I
I
Since the angle of projection is a, V has
I
components: I
V cos a horizontally Fig. 25-14
V sin a vertically v cos a
Now the moving particle is subject only to the acceleration due to gravity of magnitude
g and directed vertically downwards.
Hence x = 0 and Y= -g define the motion in the horizontal and vertical directions
respectively.

Horizontal motion
Since x = 0 we have x = c I and since initially x = V cos a we have

for 0 EO; t EO; T (1)

where T is the time of flight.

Hence x = f V cos a dt V cos at + Cz


When t = 0, x = 0 and so C z = o.
Thus . I x = V cos at I for 0 EO; t EO; T (2)
116 NEW SENIOR MATHS: THREE UNIT COURSE

Vertical motion
Since Y -g we have
} f- g dt = - gt + kl
When t = O,} = V sin a and hence kl V sin a.

Thus for 0 '" t '" T (3)

Hence y = J( V sin a - gt) dt

V sin at - 19t2
2 + k2
When t = 0, y = 0 and so k2
Thus for 0 '" t '" T (4)

Equations (1) and (3) are the familiar v = u + at formula and (2) and (4) the familiar
1
s = ut + iat2 formula.
Equations (2) and (4) are the parametric equations defining a point on the trajectory at
time t E [0, T] after projection.

Equation of the path


The cartesian equation to the trajectory is obtained by eliminating t from equations (2)
and (4). Substituting t x
cos a
into y = V sin at - 21gt2 gives

y = x tan a - =-=-~---:c-­
a
But sec 2 a = 1 + tan 2 a

gx2
Hence Y = x tan a - 2 V2 (1 + tan 2 a) (5)

This is the cartesian equation to the trajectory. This equation is particularly useful when
the angle of projection is sought.
Note: The parametric equations of the trajectory may be readily obtained using the
equations for uniformly accelerated motion. If x and} denote the velocity components in
the x and y directions respectively, we obtain
x = V cos a (constant velocity)
} = V sin a - gt (using v u + at)
and x V cos at

y V sin ~~_-=_~t2 (using s = ut + ~a(2)


In working mo~t~~a~~~~-,-we shall ~rit~dow~ions mi~the particular
context of the question and shall not usually derive them from first principles. This is to
conserve space; However, it is strongly recommended that students work at least some
examples from first principles.
25/APPLICATIONS OF CALCULUS TO THE PHYSICAL WORLD ii7

Maximum height

The particle has reached its highest point when y = 0 and hence from (3) when t = V sin ex .
g
Substitution in (4) then yields
1 V1 sin 2 ex
V1 sin2 ex V1 sin 2 ex
Ymax
g 2 g
Alternatively we may work from the equation to the trajectory (5). In this case y is a
function of x and hence we require: = O. Check that the same answer is obtained.
Hence, maximum height is given by

Y max

Range on a horizontal plane through point of projection


When the projectile strikes the plane we must have Y = O.
1
Thus from (4) V sin cd 'igt2 =0
1
t( V sin ex 'igt) = 0

t 2•.. . .V:......;,.,si_n_ex
= 0 or- g
Now t .
= O refers to t h e mstant f"
0 prOjectIOn an
d h ence t = 2 V sin ex.IS t he reqUlr
. ed time,
.
g
the time of flight T. Then from (2) we obtain
2 V1 sin ex cos ex
x=-----
g

i.e. where R denotes the range.

This method gives information about the time of flight. If only the range is required we
may set Y 0 in (5) to obtain

o = x ( tan ex 2 V1 ~S2 ex )
V1 sin 2ex
x = 0 or x where the latter value defines the range.
g

Maximum range
For a given speed of projection V the equation R V1 sin 2 ex d e f'mes a functIOn
. 0 f ex. R IS
.
g
extreme when sin 2ex = i, i.e. ex = If. This means that to achieve maximum range the
particle should be projected at an angle of elevation of 45°.

V1
R max = -g when ex = 45°

These formulae and results should not be memorized but calculated when necessary in a
particular problem.
118 NEW SENIOR MATHS: THREE UNIT COURSE

EXAMPLE 15
A particle is projected from a horizontal plane ~t an angle of elevation of 30° with a speed
of 100 metres/second. Find:
(a) the equation of the trajectory,
(b) the range of the projectile and the time of flight,
(c) the maximum height~
y

I
I
:125m
I
r
)(
Fig. 25-15
(a) At any time t,
x V cos OIt 50...)31
y = V sin OIt

100 sin 30 0 t - !. IOt 2 , using g = 10


= SOt - 5t 2
Eliminating t from these parametric equations gives
x x2
Y = ...)3 - 1500

(b) When y =0, we have SOt 5t 2 = 0


t = 0 or 1 10
When t 10, x = 50--/3 x 10 = 500--/3 and this defines the range in metres.
(c) By symmetry, the maximum height occurs when t = 5 and hence
Y max = 50 x 5 - 5 X 25 125
The maximum height attained is 125 metres.

EXAMPLE 16
A particle is projected from a window 9 metres above the horizontal ground at an angle 01
to the horizontal where tan 01 = ~ with an initial velocity of 20 metres/second. Neglecting

Fig. 25-16
w

o Ground level x
251APPLICATIONS OF CALCULUS TO THE PHYSICAL WORLD 119

air resistance, find:


(a) the cartesian equation of the path,
(b) the maximum height above the ground,
(c) the time taken for the particle to strike the ground,
(d) the horizontal distance from the window to the point where it strikes the ground.
Consider the origin, 0, as the point on the horizontal ground 9 metres below the window.
· 3. 3 d 4
(a ) Smce tan a = 4' sm a = .5 an cos a = s'·' Fig. 25-17
:. V cos a = 16 and V sin a = 12.

Horizontally: x=o
:. x = Cl Vcos a = 16 4 (I)
x = 16t + C 2 and since x o when t = 0, then C 2 = 0
:. x 16t (2)
..
Vertically: y = -g
:.j = -gt + k) and since j V sin a 12 when t = 0,
then kl = 12
:. j = -lOt + 12, usingg = 10 (3)
Y 512 + 12t + k2 and since y 9 when t = 0,
then k2 9
:. y 5t 2 + 12t + 9 (4)
x
From (2) t = 16 and hence from (4)
2
y= 5x 3 + 9 (cartesIan
+ 4x . equatIon
. 0 f path) (5)

(b) At maximum height j 0


:. -lOt + 12 = 0 from (3)
t 6
5
Substitute t = ~ into (4)
36 6
:. y = -5 x 25 + 12 x "5 + 9
= 16!
Maximum height above the ground is 16! metres.
(c) At the point where the particle strikes the ground, y = 0
:. -5t 2 + 12t + 9 = 0
(5t + 3){ - t + 3) = 0
:. t 3 since t ~ 0
It takes 3 seconds to reach the ground.
(d) When t = 3, x 48 from (2)
It strikes the ground at a horizontal distance of 48 metres.
Note: We could have taken the origin, 0, at the point of projection i.e. at the window. In
this case, at the point where the particle strikes the ground, y = -9. This would alter only
(4) and (5) but none of the answers to (b), (c) or (d). Try it.
120 NEW SENIOR MATHS: THREE UNIT COURSE

EXAMPLE 17
A body is projected horizontally from the top of a cliff 20 metres above the water with a
velocity of 30 m/s. Find:
(a) the time taken to reach the water,
(b) the horizontal distance travelled.
y

0 30 m/s X

I
20m l
I
I
Fig. 25-18 i
Water level
lx, -20)

Consider the origin,a, at the point of projection at the top of the cliff.
Horizontally: X- = 0
x = C 1 = 30 (1)
x ::: 301 + C z and since x 0 when I = 0, C z = 0
:. x = 301 (2)
..
Vertically: y = -g
y = - gl + k I and since y = 0 when t = 0, k I =0
:. y = -gt (3)
y =- &gtZ + k z and since y 0 when t = 0, k z = 0
1
:. y = -2,gIZ (4)

(a) At water level, Y = -20


1
- 20 = - 2,gt Z from (4)
t = 2
It takes 2 seconds to reach the water.
(b) From (2) x = 301
=
60 when t = 2
The horizontal distance travelled is 60 metres.
NOles: (i) Why is Y = 0 when I = O?
(ii) We could have taken the origin, a, at water level.
(iii) You will observe that the velocity of projection does not influence the time
taken to reach the water. Why? Drop an object and at the same time throw
another object horizontally. You will notice that they will reach the floor at
the same time.
EXAMPLE 18
A particle is projected at ground level and I! seconds later it just clears a wall 3i metres
high at a horizontal distance of 36 metres. Calculate
(a) the velocity and angle of projection,
(b) the range. ~
25/APPLlCATIONS OF CALCULUS TO THE PHYSICAL WORLD 121

y
v

----------1 x
Fig. 25-19
(a) When I = I!, x = 36 andy 31
Horizontally: x V cos 011
3
:. 36 V cos a X 1;
V cos a = 24 (1)
. 1
Vertically: Y = V sin at - 1;gt2
. 3 9
4 V smax1; 5X 4
15 = 6 V sin a - 45

~5
V sin a = 10 (2)
V sin a 10
(2) + (1)
cos a = 24
5
i.e. tan a 12
a = 22° 37' 12
Fig. 25-20
·
Smce tan a = 5 cos a = TI
12 an d'
sm a =
5
TI
From (1) = 24
V cos a
12
VXTI=24
V =26
Velocity of projection = 26 m/s.
. 1
(b) y = V sm al - -gt 2
2
o lOt 5/ 2 , putting y = 0
:. I 0 or 2
x V cos at
= 24 X 2
= 48
:. Range is 48 m
V2 sin 2a
or Range =- -g
-
V2 2 sin a cos a
X
g
26 X 26 X 2 X 5 X 12
X
= 48 (metres)
122 NEW SENIOR MATHS: THREE UNIT COURSE

EXAMPLE 19
A stone is thrown to hit a small object on the top of a wall which is 20 metres horizontally
from the point of projection and 5 metres high; If the stone is thrown from the horizontal
ground with a speed of 40 mls show that there are two angles of elevation which will
enable the object to be hit.
y

x
V 40 and y = 5 when x = 20. Fig. 25-21
Horizontally: x = V cos ext
20 = 40 cos ext (1)

Vertically: y = V sin ext - !gt 2


2
5 = 40 sin ext - 5t 2 (2)

Substituting t 1 from (1) into (2) we get


cos ex
5 = 20 tan ex ~(1 + tan 2 ex)
4
20 = 80 tan ex 5 5 tan 2 ex
5 tan 2 ex 80 tan ex + 25 = 0 ' (3)
tan 2 ex - 16 tan ex + 5 = 0
16 ± .J236 ,
tan ex = 2 ,-.
=8±.J59
15'68 or 0'32
:. ex = 86° 21' or 17° 44'
Alternatively, we could quote the cartesian equation of the path
gx2
y = x tan ex - 2 VI (1 + tan 2 ex)
substituting y 5, x 20, g = 10, V = 40 and simplifying, this yields equation (3)
above.

EXERCISES 25(dl
Take acceleration due to gravity as 10 m/s2 and ignore air resistance.
1. A body is projected horizontally from the top of a bQilding 125 metres high with a
speed of 10 metres/second. Find:
(a) the time taken to reach the ground,
(b) the horizontal distance travelled.
Explain why the time taken to reach the ground is independent of the velocity of
projection.
25/APPLICATIONS OF CALCULUS TO THE PHYSICAL WORLD 123

2. A cricket ball is hit with a velocity of 12! mls at an angle with the horizontal whose
, 3"'
tangent IS 4 F'In:
d

(a) the greatest height reached,


(b) the time of flight,
(c) the horizontal distance travelled.

6 At what angle should a particle be projected with a velocity of 50 mls to pass through
a, point on its path whose coordinates are (200, 25), distances being measured in
" 'metres?

4.. A ball is~pr(jjected so that its horizontal range is 45 metres and it passes through a
j point 22! metres horizontally from and II!, metres vertically above tqe point of
projection. Find the angle of projection and the speed of projection.

5. A stone is projected from the edge of a cliff with a speed of 30 metres per secondJto
hit an object 120 metres horizontally from the edge and 35 metres vertically below it. I
Find the angle of projection. ;

6. A ball thrown horizontally with, speed V from a point H metres above the ground
lands at at horizontal distance D metres from the point of release. Find
(a) V if D 3 and H = Ii,
(b) H if V 10 and D = 20.

7. A particle is projeCted from a point whose coordinates are (0, 0) with velocity of
20 mls horizontally and 100 mls vertically. Find when the projectile passes through
the point whose horizontal distance from 0 is 120 metres and find also its vertical
height then. '

8. A particle is projected so that at any time I, its position (x, y) is given by x = 36t,
Y = 1St - !g12 where the components are measured in metres, and time in seconds.
Find:
(a) the velocity of projection, (b) the angle of projection,
(c) the greatest height reached, (d) the range.

9. A particle is projected so that at any time t the equation of the trajectory is x = 151,
Y = 20t ~ !g12 where the components are measured in metres. Find:
(a) the point of projection;
(b) the speed of projection;
(c) the coordinates of the highest point of the path.

10. A shell is fired at 200 metreslsecond to strike a target 2 km away. At what angle
should it be fired?

11. A plane travelling at 630 kmlh releases a bomb from a height of 1000 metres.
(a) How long does it take to reach the ground?
5
(b) What horizontal distance does the bomb travel? (1 kmlh = 18 m/s)
124 NEW SENIOR MATHS: THREE UNIT COURSE

12. A particle is projected to just clear two walls of height 7 metres and distant 7 metres
and 14 metres from the point of projection. Prove that if €:X is the angle of projection,
3
then tan €:X = 2'
Prove that if the walls are h metres high and distant b metres and e metres from the
point of projection, then
tan €:X = h(b + e)
be
13. A particle is projected with a speed of 25 ml s in the direction of a point P distant 24 m
horizontally and 7 m vertically above the point of projection.
(a) Find the coordinates of the point on the trajectory directly below P.
(b) At the instant of projection of the particle, a second particle is dropped from P.
Prove that the two particles will collide.
14. A particle projected at an angle of 15° has a horizontal range of 80 metres.
(a) What is the velocity of projection?
(b) What would the range be if the angle were increased to 45° and the same velocity
of projection?
15. A stone is projected horizontally from the top of a building 30 metres high with a
velocity of 15 m/s. At the same instant another stone is projected from the foot of the
building with a velocity of 30 mls at an angle of 60° to the horizontal. The two stones
move in the same vertical plane. Prove that they collide and, if the coordinates of the
foot of the building are (0, 0), find the coordinates of the point of collision.
16. A golfer hits a ball from a point on a level fairway and 2 seconds later it strikes the
fairway 50 m away. Find:
(a) the velocity and angle of projection
(b) the maximum height of the ball above the fairway.
'1l~ ~ stone is thrown so that it will hit a bird at the top of a pole. However, at the instant
the stone is thrown, the bird flies away in a horizontal straight line at a speed of
10 ms-I. The stone reaches a height double that of the pole and, in its descent, touches
the bird. Find the horizontal component of the velocity of the stone.
18. A particle is projected with velocity 20 mls to hit a target at a horizontal distance
20 m from the point of projection and at a vertical height of 10m. Find two possible
angles of projection.
19. A particle is projected so that at any time t the co-ordinates of its position relative to a
fixed point 0 are given by the parametric equations x = 30t and y 401 - ~gt2
where the components are measured in metres and time in seconds. Find:
(a) the velocity of projection
(b) the angle of projection
(c) the cartesian coordinates of the highest point of the path.
,~

\20. A particle projected from a point meets the horizontal plane through the point of
projection after travelling a horizontal distance a, and in the course of its trajectory
attains a greatest height b above the point of projection. Find the horizontal and
vertical components of the velocity in terms of a and b. Show that when it has
described a horizontal distance x, it has attained a height of 4bx(a 2 x)
a
251APPLICATIONS OF CALCULUS TO THE PHYSICAL WORLD 125

21. A body is projected with a velocity whose horizontal and vertical components are
.
6 mls and 4 mls respectively. Find the horizontal range.
rii. A batsman hits a cricket ball 'off his toes' towards a fieldsman who is 65 m away. The
ball reaches a maximum height of 4·9 m and the horizontal component of its velocity
is 28 m/s. Find the constant speed with which the fieldsman must run forward;
starting at the instant the ball is hit, in order to catch the ball at a height of 1·3 m
above the ground. (Use g = 9'8)
23. A football, kicked at 16 mis, just passes over a cross-bar 4 m high and 16 m away.
Show that, if a is the angle of projection,
5 tan 2 a 16 tan a + 9 0
124~ Find the speed and direction of a particle which, when projected from a point 15 m
/ above the horizontal ground, just clears the top of a wall 26k m high and 30 m away.

25.5 The differential equation: ~~ = k(N - P)


where k and P are constants _______

This is an extension of the work on exponential growth and decay in Chapter 19.
dN
If dt k(N - P}, (1)

dt 1
then dN k(N _ P)' N *' P
1 j~ 1
t =k N _ pdN
1
k 10ge(N - P} + c, N > P
k(t - c} = loge(N - P}
N - P = ek(l-c)
= ekl • e-kc
= Aekl where A e- kc
N P + Aekl (2)

dN
In words, dt = k(N P) states that the rate of change of N is proportional to the excess
of N over a fixed quantity P. If k is negative, Aekl -+ 0 as t -+ 00 and hence from (2)
N - + P from above.
Some examples of this type of growth and decay include:

(a) Newton's law of cooling


The cooling rate of a body is proportional to the difference between the temperature of a
body and that of the surrounding medium.
dT = -k(T - M)
dt
where T is the temperature at any time t and M is the temperature of the surrounding
medium (constant).
126 NEW SENIOR MATHS: THREE UNIT COURSE

EXAMPLE 20
If the original temperature of a body is 100°C, the temperature of the surroundings is
20°C and the body cools to 70°C in 10 minutes, find, assuming Newton's law of cooling,
(a) the temperature after 20 minutes
(b) the time taken to reach 60°C
Let TO C be the temperature of the body at any time t.
~; = -k(T - 20)
dt 1
dT k(T _ 20), T ::1= 20

t =

= -l(1 10ge(T 20) + c, T> 20


- k(t c) = 10ge(T - 20)
T 20 = e-k(f-C)
= e-kf • e- kc
== Ae-kf where A e-kc (constant)
T = 20 + Ae-kt
When t 0, T = 100 and so A 80
When t 10, T 70 and so
70 = 20 + 80e- 10k
e-lOk 50 = 0.625
80
10k = logeO'625
= -0,47
k 0·047
:. T 20 + 80e-o'047t
T
(a) When 1 = 20, T 20 + BOe-o' 94
= 51'25 100
After 20 minutes, the temperature is 51·25°C.
T = 20 + 80e-0-0471
(b) When T = 60,60 = 20 + 80e-o'047t 70
e-o-047t = 40 0.5
80
-0,0471 == logeO'5
= -0,6931 20 - - - 1 - - - - - - - - - - - - - - -
I
1 0·6931 I

0'047 o 10 t
= 14·7 Fig. 25-22
After 14·7 minutes, the temperature is 60°.
Note from the graph (Fig. 25-22) that as
t - 00, T - 20 from above.
(b) Wilhelmy's law
Many chemical reactions follow the law which states that the velocity of the reaction is
proportional to the difference between the initial concentration of the reagent and the
amount transformed at any time.
25/ APPLICATIONS OF CALCULUS TO THE PHYSICAL WORLD 127

dx
dt == k(a - x), 0 .s;; x .s;; a
where a is the original concentration and x is the amount transformed at time t.

EXAMPLE 21
Ifa= 10 and x == 4 after 2 minutes, find the amount left after 5 minutes.
dx
dt = k(1O - x) for 0 .s;; x .s;; 10
dt 1
i.e. dx k(1O _ x) for 0 .s;; x < 10

t-.!.f~
-k lO-x
kt = -logeOO - x) +c
When t = 0, x = 0 and so c = 10gelO.
:, kt = 10ge1O - 10g.(10 - x)
10
= 10geWX

Now _10_ = ekt


10 - x
10 - x _ -kt
-10- - e
x
10 - x = lOe- kt
x = 10(1 - e- kt ) for t "'" 0
10

o 2 5 t Fig. 25-23

Note how the graph asymptotes to x = 10 from below. This is consistent with the
restricted interval 0 .s;; x < 10 for t "'" O.
When t = 2, x = 4 and so
4 = 10(1 e-2k )
0,4 = 1 - e-2k
e-2k 0'6
1
_e2k = 0,6
- or -2k = logeO'6
5
2k 10gej = -0'51
1 5
k = 210gej k = 0-255
When t = 5, x = 10(1 e-Sk ) or x = 10(1 - e- 1' 275)
But e-2k = 0'6 = 10(1 - 0'279)
e-Sk= (e-2k )5/2 (D5!2 = isGY2 = 0'279 = 10 - 2,79
x = 10(1 0'279) = 10 2'79
and so there are 2.79 units left after 5 minutes.
128 NEW SENIOR MATHS: THREE UNIT COURSE

EXAMPLE 22
Brine containing 2 g of salt per litre flows
into a tank initially filled with 50 litres of
water containing 10 g of salt. If the brine
enters the tank at 5 litres/min, the con- 5litres/min
centration is kept uniform by stirring, and ~ 10 g/min
the mixture flows out at the same rate, find
the amount of salt in the tank after 10
minutes.
Suppose there are Q g of salt in the tank
after t minutes. Since 5 litres of brine enter
and leave the tank each minute, the tank will
contain 50 litres of solution at any time.
Hence each litre of solution in the tank will
contain ~ g of salt and since it flows out at 5litres/min
I .
Q gmln
=10 ""
5 litres/min, the rate of out-flow of salt is
Fig. 25-24
Q g'/ mm.
10 .

The rate of increase of Q, i.e. ~~, is given by

~~ = rate of inflow rate of outflow


dQ Q Q
i.e. dt = 10 - 10
100 - Q
10
:. :~ = l001~ Q' 10 E; Q < 100
Hence t = - 10 10ge(l00 - Q} + c
When t = 0, Q 10 and so c 10 loge 90.
= ......--ot------:1~0------t....t
90
Hence t = 10 loge 100 _ Q
e illO 90 Fig. 25-25
Thus
100 Q
so 100 - Q = 90e- 1/10
Hence Q = 100 9Oe- "1o
When t = 10, Q = 100 90e- 1 "'" 66'9(g)
As t increases without bound, e- 1/IO -+ 0 and so lim Q
1-00
= 100. What is the physical
meaning of this result?

EXERCISES 25(e)
In questions 1. to 4. assume Newton's Law of Cooling in which the temperature TO of a
body at any time t is given by the formula
T = P + Ae- kl
1. A metal bar has a temperature of 1230°C and cools to 1030°C in 10 minutes when the
surrounding temperature is 30°C, how long will it take to cool to 80°C?
25/APPLICATIONS OF CALCULUS TO THE PHYSICAL WORLD 129

2. Water at 20°C is placed in a refrigerator at a constant temperature of L 10°C. If the


temperature of the water falls to 15°C in5 minutes, fin9 the temperature of the water
in another 5 minutes.
3. A body whose temperature is 180° is cooled by immersing it in a liquid at 60°C. In
one minute, the temperature of the body has fallen to 120°C. How long will it take
for the temperature of the body to fall to 90°C?
4. A body, initially at room temperature 20°C, is heated so that its temperature would
rise by 5°C/min if no cooling took place. Cooling does occur in accordance with
Newton's Law of Cooling and the maximum temperatur€ the body could attain is
120°C. How long would it take to reach a temperature of 100°C?
A current of i ampere flows through a coil of inductance L henries and resistance
R ohm and the current at any time t is given by
§. (1 - e- RIIL )
R
where E is the electromotive force in volts.
Show that L~: + Ri =E
6. A vessel is being filled at a variable rate and the volume of liquid in the vessel at any
time t is given by
V A (1 - e~lct)
dV
(a) Show that dt k(A - V)
(b) If one quarter of the vessel is filled in 5 minutes, what fraction is filled in the next
5 minutes?
lim V = A.
(c) Show that t- 00

7. A tank contains 100 litres of brine whose concentration is 3 grams/litre. Three litres
of brine whose concentration is 2 grams/litre flow into the tank each minute and at
the same time 3 Htres of mixture flow out each minute.
(a) Show that the quantity of salt, Q grams, in the tank at any time t is given by:
Q = 200 + 100e-O-03t
(b) Find ~~ when (i) t = 10 (ii) Q = 250
(c) Show that for any value of t, 200 < Q .;;; 300.
8. Brine containing 2 grams of salt per litre runs into a vessel initially containing
100 litres of water and 25 grams of salt. If the brine enters at 5 litres/minute, the
contents of the vessel are kept uniform by stirring and the mixture flows out at the
same rate, show that if Q grams of salt are in the vessel at any time t:
(a) dQ = 0'05 (200 Q)
dt
(b) Q = 200 - 175e-o-05t
lim Q 200
(c) t-00

9. A rectangular vessel is divided into two equal compartments by a v~rtical porous


membrane. Liquid in one compartment, initially at a depth of 20 cm, ~asses into the
other compartment, initially empty, at a rate proportional to the differ:ence in levels.
(a) If the depth of liquid in one of the vessels at any time t minutes is x ~m, show that
dx
dt = k(20 - 2x)
,

130 NEW SENIOR MATHS: THREE UNIT COURSE

(b) Show that x = 10(1 - e-Zkl )


(c) If the level in the second compartment rises 2 cm in the first 5 minutes, after what
time will the difference in levels be 2 cm?
10. A cylindrical vessel with its axis vertical is divided into two equal compartments by a
vertical permeable membrane. Initially, one compartment is filled with a liquid to a
depth of 12 cm and the other is empty. The liquid passes to the other compartment at
a rate which is proportional to the difference in level of the liquid in the two
compartments. If there is 1 cm of liquid in the second compartment after 1 hour, find
how long it will take one-quarter of the liquid to penetrate the membrane.
11. In a certain chemical process, the amount y grams of a certain substance at time
t hours is given by the formulay = 3 + e-kl •
(a) Show that CZ = k(y - 3).
(b) If initially y decreases at the rate of 0'08 grams/hour, find the value of k.
(c) Find the rate of change when y = 3'5.
(d) What values can y take?
12. A body falls from rest in a medium and its velocity, v metres/second, after t seconds
h~~ .
v = 80(1 e-Q'41)
(a) Show that the acceleration is proportional to 80 v.
(b) Calculate the distance fallen in the first 5 seconds.
(c) Calculate the distance fallen when v = 60.
I II~Ullometric
Functions
Before considering inverse trigonometric functions, it would be useful to discuss some of
the properties of inverse functions.

26.1 One-one functions _ _ _ _ _ _ _ _ _ __

y
y

x
x x

fix) x, .t g(x) = x', x ~ 0

Fig. 26-1 Fig. 26-2 Fig. 26-3


g and h are examples of one-one functions but j is not.
A line drawn parallel to the X-axis intersects the graphs of g and h only once. There are
no ordered pairs with the same second element.
On the other hand, a straight line parallel to the X-axis intersects the graph of j in two
places e.g. (-a, a2 ) and (a, a2 ) both lie on the graph of jwhich is a two-one function. An
example of a many-one function isj(x) = sin x whereas g(x) = sin x, 0 ~ x ~ ~ is a one-
one restriction of j.
The main significance of a one-one function arises in the property which it has when
reflected in the line y ~ x. This property plays a key role in the development of inverse
functions.
The reflection of the graph of j in the line y x is the parabola y2 = x which is the
graph of a relation, not a function (Fig. 26-4).
132 NEW SENIOR MATHS: THREE UNIT COURSE

The reflection of the graph of g in the line y = x is the positive branch only of the
parabola y2 x i.e. y = + ~x. This is the graph of a function (Fig. 26-5).
Y Y Y X2

y
x

x x
Fig. 26-4 Fig. 26-5

The necessary and sufficient condition for the reflection of the graph
of fin y = x to be the graph of a function is that f be one-one.

EXAMPLE 1
Define a restriction offwheref(x) = X2 - 4x + 5 which is one-one.
f(x) (x - 2)2 + 1 and the graph of fhas a minimum point (2, 1).
f is thus one-one whenever x ;;;.. 2 or whenever x ~ 2.
Thusfl' wherefl(x) X2 - 4x + 5,x;;;" 2
and f2' wheref2(x) = X2 - 4x + 5, x ~ 2
are both one-one restrictions of f. Further, any function with the same rule defined on a
subset of the domain of f1 or f2 is also a one-one restriction of f.

26.2 Increasing and decreasing


functions ________________

A functionf defined on the interval a ~ x ~ b is said to be strictly monotone increasing


if, when XI and X 2 are in the interval and X 2 > XI' then
f(x 2) > f(x 1 )
In other words, the curve slopes upwards to the right (Fig. 26-6).

Y Y

f(x.)
f(x,)

f(x,' f(x 2 l

0 a x, x. b X 0 a x, x. b X

Fig. 26-6 Fig. 26-7


26/INVERSE TRIGONOMETRIC RATIOS 133

Similarly a functionjis said to be strictly monotone decreasing if, when X2 > Xl' then
j(x2) < j(x l )
In other words, the curve slopes downwards to the right (Fig. 26-7).
Such functions are ·obviously one-one.

26.3 Inverse functions (f- 1 ) _ _ _ _ _ _ _ __

Consider two operations such that each is the "reverse" of the other. For example, the
cube of 2 is 8 and the cube root of 8 is 2.
The cube function defined by j(x) =. x 3 , X E R and the cube root function defined by
g(x) = :vx, X E R are said to be a pair of inverse functions.
j(2) 8 and g(8) = 2
If we cube any real number X and then take the cube root of the answer, the result is x.
Likewise, if we take the cube root of any number X and then cube the answer, the result
is x.
j(x) = x3 g(x) = :vx
j(g(x» (g(x»)3 g(j(x» = 4j(x)
= ($)3 {JXi
=X X
Conclusion: For any pair of inverse functions,

I j(g(x» = g(j(x» = xl

If we denote g(x) by j-I(X), then

The square of 2 is 4, but in the reverse operation 4 has two square roots, namely ± 2. If
we wish to return to 2 in the square root operation, we must take the positive square root.
The square function defined by j(x) = X2. X ~ 0 and the square root function defined
by g(x) = .JX, x ~ 0 are a pair of inverse functions. This emphasises the necessity for
functions to be one-one for the inverse to define a function;
j(x) = x 2, x ~ 0 g(x) .JX, x ~ 0
j(g(x» = (g(x»)2 g(j(x» = ..Jj(x)
= (.JX)2 = .JX2
=x = x if x ~ 0
Thusj(g(x» = g(j(x» =x
Le.j(j-l(x» = j-I(j(X» =x where g(x) = J-I(X)

EXAMPLE 2
Find the inverse functionj-l for the functionjwherej(x) 2x + 4.
We requirej-J such that
j(j-l(X» = x = j-I(j(X»
j(j-l(X» = x
:. 2j-l(x) + 4 = x
1
Le. J-I(X) = -(x - 4)
2
134 NEW SENIOR MATHS: THREE UNIT COURSE

It is now easily shown that fl(j(x» = x.


I
j-I(j(X» = :2H2x + 4) - 4]
I
= :2(2x)
=X
I
Hence j-I (x) = :2(x 4) is the rule defining the inverse function. It now remains to
discuss the domain and range of j-I .
Let df and rf denote the domain and range respectively of j
and drl and rrl denote the domain and range respectively of j-I.
Consider the following typical elements of df : 0, 1, -2, 3
under the rule, j(x) 2x + 4,

Fig. 26-8 Fig. 26-9

The correspondence may be represented in ordered pair form by (0, 4), (l, 6), (-2, 0),
(3, to) Fig. 26-8.
Every real number in df has a corresponding real number image in rf •
The inverse mapping is obtained by operating withj-I on the elements of rf' This means
that these elements are now regarded as belonging to drl.
i.e. rf = drl

We have j-I(4) = !(4


2
4) = 0 j-I(6) ~(6 - 4) = I·
fl(O) = -2 fl(lO) =3
so that the inverse correspondence may be represented by (4, 0), (6, I), (0, -2), (to, 3).
The elements 0, I, -2, 3 which belong to the range of j-I are identified as the elements of
the domain of j and so

Figs. 26-8 and 26-9, when considered together, demonstrate that an element x € df is
mapped back to itself by the composite function j(j-I(X». This is as required by the
general defining property j(j-I(X» = x.
Example 2 illustrates several important general properties of inverse functions.
1. df = rr1 and rf = drl so that domain and rapge are interchanged between a function
and its inverse. If (a, b) is a point on the graph ofjthen (b, a) is a point on the graph
of j-I. These are called corresponding points on the graphs.
2. Representing P (a, b) and Q (b, a)as corresponding points on the graphs ofjandj-I,
26/INVERSE TRIGONOMETRIC RATIOS 135

the co-ordinates of the midpoint of PQ are seen to be U<a + b), i(b + a)] which lie
on the graph of y x for each a and b. Hence y x bisects the line segment PQ so
that P and Q are mirror images of each other in this line. The graph of I-I is thus seen
to be a reflection of the graph of I in the line y = x. This is of great value in graph
sketching. It also follows that any intersection between the graphs of I and I-I must
take place on y = x.
Fig. 26-10 shows the graphs of I and I-I of Example 2
y y=X
/
/
/

",
/

/ /
~ ~ 4)
,/
~
' 1<.,
/
/ 'Ie
,
x

Fig. 26-10

The property that (x, y) and (y, x) lie on the graphs ofI and I-I respectively provides an
alternative means of finding I-I .
The function of example 2 is defined by
y = f(x) for x E df and y E rf
i.e. y = 2x + 4 for x E Rand y ER
Hence I-I is defined by
x = 2y + 4 for y E R and x E R (interchanging x and y)
. i.e. 1
Y = 2(x - 4) for x € R, y € R

Thus I-I (x) = i(X - 4) as before.


26.4 Existence of inverse functions _______
Inverse functions do not always exist although they do for linear functions.
We have noticed that when an inverse function exists, its graph is the reflection of the
graph of the original function in the line y = x. When the graph of a one-one function is
reflected in y = x, the reflection is itself the graph of a function - the inverse function.
The reflection of the graph of a many-one function is the graph of a relation not a
function. This reflection cannot, therefore, be designated I-I.
Hence it lollows that:

A lunction I will posses an in verse lunctionJ-l if and only ifI is a one-one


lunction
136 NEW SENIOR MATHS: THREE UNIT COURSE

EXAMPLE 3
Investigate the existence of an inverse function whenf is defined by f(x) = X2.
f is not a one-one function and its graph, the parabola with equation y = X2, when
reflected in the line y x is the parabola with equation y2 x. This latter parabola
consists of an upper branch y = .Jx and a lower branch y = -.Jx and is thus the graph of a
relation having two values of y for each value of x in the domain. See Fig. 26-4
Consequently f-I does not exist.

EXAMPLE 4
Investigate the existence of an inverse function when f is defined by f(x) = X2, X ~ O.
This is a restriction of the function defined in Example 3.
In this case, we have only the right-hand branch of the parabola y X2 which is the
I
graph of a one-one function. Hencef- exists and may be determined as follows.
Method 1 (Using algebraic definition)
Look for f-I such thatf(f-I (x» = x.
f(f-I (x» =x
:. (f-l(X»2 =x
Thus f-I (x) = .Jx, considering the positive square ,
root since the range off-I consists of non-negative numbers. )

Method 2 (Using reflection property)


f is defined by y X2 forx ~ Oandy ~ 0
Interchange x and y and hence
FI is defined by x = y2 for y ~ 0 and x ~ 0
i.e. by y =.Jx for x ~ 0 and y ~ 0
HenceFI(x) = .Jx as before.

EXAMPLE 5
1
Find FI, if it exists, when f is defined by f(x) =. x>O
f is a one-one function and hence f-I exists.
dJ = R+ and so rr' = R+
rJ = R- and so dr' = R-
y
1
Let y = - .Jx' x > 0, y < O.
y/ x

J
Interchanging x and y we get
/
1 /
x = .Jy' x < 0, y > 0 /
/
/

and so y
Y
1
= x 2' X < 0, y > 0
// /('0 f
x
/
/
/
/

Fig. 26-11
26/INVERSE TRIGONOMETRIC RATIOS 137

EXAMPLE 6
Given thatf(x) = X2 2x + 3, find the largest possible domain consisting of positive real
numbers such thatfis one-one. Findf-I and sketch its graph on the same diagram as the
graph off.
y
Let y = f(x) = x 2 2x +3 y=x
= (x - 1)2 + 2 /
/
and so 0, 2) is the turning point of the graph

(1,2/r
/
/
of f (Fig. 26-12). Now f is one-one when /
x ;;;.. 1 or when x ~ 1 and it follows that
f is defined by
y = X2 - 2x + 3, x;;;.. 1, y ;;;.. 2.
// (2,1)
Hence, f- I is defined by
x y2 - 2y + 3, y ;;;.. I, x ;;;.. 2. ,,/0 x
y2 2y + 1 x 2 /
(y - 1)2 = x - 2
/ Fig, 26-12
y 1= ±~
Hence y = 1 + .../x - 2, choosing positive square root, since y ;;;..

26.5 Exponential and logarithmic


functions _ _ _ _ _ _ _ _ _ _ _ _ _ __

These two functions, which you have met in previous work, provide a very good example
of a pair of inverse functions.
The functionf(x) = aX is called an exponential function. The base a is any positive real
number except 1, the domain is R and the range R+, It is a strictly monotone increasing
function, i.e. a one-one function without having to restrict its domain.
Let y f(x) aX
Interchanging x and y we get
x = aY
i.e. y = logax, by definition of a logarithm.
The function g where g(x) = logax is called a logarithmic function. Its domain is R+
(the range of f) and its range is R (the domain of f). It is also a strictly monotone
increasing function. It should be noted that for any pair of inverse functions f and g, f is
the inverse of g and g is the inverse of f. Neither has priority over the other.
y
fIx) = a'

g(x) 109a x

x
Fig. 26-13
,

138 NEW SENIOR MATHS: THREE UNIT COURSE

Other important pairs of inverse functions are the trigonometric functions, defined on a
suitably restricted domain, and their inverses. These will be studied in depth in the next
section.

EXERCISES 26(a)
Which of the following are one-one functions (1. to 10.)?
1. f(x) =x - 2 2. f(x) = X2 - 2x + 1, x ;;;. 1
3. f(x) := -J(4 - x 2 ), -2 "'" x "'" 2 4. f(x) := 9 - x
5. f(x) = 9--'
1
-x x 9 * 6. f(x) := 19 - xl
11"
7. f(x) 9 X2 8. f(x) = cos x, 0 "'" x "'" 2"
9. f(x) cos x 10. f(x) := x3 - 4x, -2 "'" x "'" 2
Find the largest possible domain for which the following are one-one increasing functions
(11. to 15.).
11. f(x) = -J(4 - X2) 12. f(x) = -J(x 2 - 4)
13.f(x) 1 14.f(x) = 3x - X2

15. f(x)
Find functions which are the inverse of each of the following. State the domain and range
of the inverse in each case (16. to 27.)
16. f(x) = 2x 4 17. f(x) = X2 - 1, x ;;;. 0
18. g(x) -J(x - 3), x ;;;. 3 19. f(x) = -J(9 - x 2 ), -3 "'" x "'" 0
20. h(x) x3 21. f(x) = (x + 2)2, X "'" -2
22. f(x) = X2 + 2x, x;;;. 0 23. f(x) = loge(x + 1), x > -1
24. f(x) = 2 - -J(xr;;- 2j, x ;;;. 2 25. g(x) = -J(5 - x) - 1, x "'" 5
1 _
26. f(x) := 2-x , x >0 27. h(x) = x-
--1' x > 1
1
28. (a) Sketch the graph of y
1 + X2
(b) State two domains for which the rule f(x) = defines one-one functions.
(e) Find and graph the two functions which are the inverse of the functions defined
in (b).
29. Explain why the following functions do not have an inverse function. Suggest suitable
restrictions to their domain so that the restricted function may have an inverse.
(i) f(x) = -J(a 2 x 2 ), -a "'" x "'" a
(ii) f(x) = 4 X2

(iii) f(x) = 1 x * <>


30. Show that the following pairs of functions are inverses by showing that
f(g(x» = g(f(x» = x
(a) (j) f(x) := 2x 1 (ii) g(x) = ~(X + 1)

1
(b) (i) f(x) 2".ex (ii) g(x) = 10ge2x
26IINVERSE TRIGONOMETRIC RATIOS 139

(c) (i) f(x) = ..J(16 - X2), -4 ~ x ~ 0 (ii) g(x) = -..J(16 x 2), 0 ~ X ~ 4


(d) (i) f(x) = 2x - x 2, X ;;:. 1 (ii) g(x) 1 + ..J(1 - x), x ~ 1
(e) (i) f(x) = 1 x > ~ (ii) g(x) x + 1 x > 0

26.6 Inverse trigonometric functions _ _ _ _ __


1. The inverse sine function
Fig. 26-14 shows part of the graph of y sin x, (1)
I ts reflection in the line with equation y x has equation x sin y, and is the graph of
a relation that is not a function. Hence the function given by (1) does not have an inverse,
The equation x = sin y has infinitely many solutions for y corresponding to x in the
interval -1 ~ x ~ 1
y
/
x /y=x
/
/
/
/
/
/
/

/
/
/
/
/
/
/
/
/

Fig, 26-14
1
x :1

For example, the intersection of the graph of x sin y with the line x ~ (Fig, 26-14)
h
sows h
tree ' f h , , 12; namely - 77f
0 f t h e so Iubons 0 t e equatIon sm y
7f 57f
6' 6' 6' Them
' f'Illite
, Iy

many solutions are often expressed as arsin ~ which means 'every number or angle whose
, ,1,
sme IS 2 '

Principal value
By restricting the domain of the sine function to ~ ~ x ~ ~, we obtain a one-one

function defined by y = sin x, - ~ ~ x ~ ~, -1 ~ Y ~ 1 (see Fig, 26-15). (2)


The reflection is given by

X = sin y, -1 ~ x ~ I, -27f ~ y ~ 27f


140 NEW SENIOR MATHS: THREE UNIT COURSE

(see fig. 26-16), and may be written in this equivalent form:

(3)

(3) defines the inverse sine function, sin-I x.


Observe how explicit mention is made of the range in defining the inverse sine function
to emphasize that it is formed from a restricted sine function.
y
- 11' 'If Y 11'
y = sin x, "2 ~ x ~ 2" 2"
1 y = sin- 1 x

'If X -1 x
2"

Fig. 26-15 Fig. 26-16

From fig. 26-16 it is evident that y = sin -I x has only one intersection with x = ~, and
that the ordinate of this intersection is sin-I ~= i. This is the only value of arsin ~ that lies
in the interval - ~ < y < ~. For this reason, sin-I x is sometimes denoted by Arsin x, i.e.
the principal value of arsin x, to distinguish it from the many-valued arsin.
Notes
1. The restricted sine function defined by y = sin x, - ~ < x < ~, is often denoted by

Sin. Thus Sin x means - ~ < x < ~ while sin x implies x E R.


2. Other choices of domain will produce one-one functions. However, in order to maintain
consistency among mathematicians, it is agreed that the restricted domain for the pur-
poses of defining Sin (and hence for defining the range of sin-I) shall be

EXAMPLE 7
Without the aid of tables, find
(a) sin-I ~ (b) sin l (-~) (c) sin -I 1·2

(d) sin-I (sin 1·2) (e) arsin ~ (f) sin-I (sin 1r)

(a) Since -1 < ~ < 1, sin-I f exists. It is a number, y, or an, angle of ye, such that

21r < y < 2'


1r h ....J3
w ose sme IS
H
ence sm- I T =
...J3 1r
3'

(b) Similarly, sin- 1 (- ~) =


26/INVERSE TRIGONOMETRIC RATIOS 141

(c) Since 1·2 , [-1, 1], sin -I 1·2 does not exist.
(d) Since sin 1·2 E [-1,1], sin- I (sin 1·2) = 1·2.

(e) In contrast with (a), arsin f is a number, y, or an angle of ye, whose unrestricted sin

. .J3
IS ..J3
H ence arSIn 2 = "37r + 2k 7r or 27r
3 + 2k h
. 7r, were k = 0, ± 1, ± 2, ...

(0 sin-I (sin 7r) sin-I 0 0, and not 7r, in contrast with (d).

Note
(a) (f) can also be expressed in degree measure. Do this.

EXAMPLE 8
With the aid of tables or a calculator, evaluate
(a) sin- I 0·6 (b) arsin 0'6 (c) sin- I (-0'6)

Using a calculator, we proceed as follows:


If we desire the angle in radians or degrees, ensure that the display indicates rad. or deg.
Operation sequence Display
(a) 6 0·6435 ...
or 6 36·869° . .. "'" 36° 52 f

(b) arsin 0-6 0-6435 + 2k7r or (7r 0-6435) + 2k7r, k = 0, ± 1, ± 2 ...


or arsin 0'6 36° 52' + k. 360° or (180 36°52') + k. 360°, k = 0, ± 1,
0
- ± 2 ...

(c) 6 -0-6435 ...


6 -36,869° ... "'" -36° 52 J

Note: sin-I ~ i and sin- 1


( -~) 7r
6' so that
sin-I (- ~) sin- I ~
In general, as may be seen from the graph sin-I (-x) -sin- I x, so that sin- I is an odd
junction.

2. The inverse cosine function


y = cos x for x E R defines the many-one cosine function. The reflection of the graph of
this function in the line y = x is the graph of the arcos relation defined by x = cos y for
-1 ~ x ~ 1. As an exercise, sketch both these graphs.
As with sine, it is straight forward to define a restriction of cos that is one-one and for
which, consequently, an inverse function is defined. We define the function Cos by
y = cos x for 0 ~ x ~ 7r.
The graphs of this function and its inverse are shown in figs. 26-17 and 27-18
respecti vely .
domain of cos- I = range of Cos n, 1] (figs; 26-17 and 26-18)
range of cos- I = domain of Cos [0, 7r]
so that we may define cos- I by
142 NEW SENIOR MATHS: THREE UNIT COURSE

y
y

y cos x. 0 ",; x ",; 11"

o 11" X
-1
-1 0 x
Fig. 26-17 Fig. 26-18

y = COS-I x, -1 EO; x EO; 1,0 EO; Y EO; 'II" I

y = arcos x means all solutions of the equation cos y = x.


y = COS-I X means (a) cos y = x and (b) 0 EO; Y EO; '11".
Since Cos and COS-I are inverse functions,
COS (COS-I x) x for -1 EO; x EO; I
COS-I (Cos x) x for 0 EO; x EO; 'II"
Again, we may stilI evaluate expressions like cos (COS-I x) and COS-I (cos x) by careful
application of the above definition. For example,
COS-I (cos 3 '11")
2
cos- 1 0 = ~ (not 32'11")
The graph of cos- 1 is not symmetrical about the Y-axis, and hence COS-I is not an even
function, even though Cos is an even function. It is easily shown that, in general,
COS-I (-x) "*
COS-I x. For example,
COS-I (-~) = 2'11" butcos-'(D ;

Hence COS-I ( -~) "* COS-I (i)


Show that COS-I <-x) = 'II" - COS-I X

EXAMPLE 9
Without the aid of tables, evaluate
(a) COS-I (- Jz) (b) sin (COS-I i) (c) cos(sin-I~)insurdform.
(a) Let y = COS-I ( - Jz)
1
Then cos y .J2 and 0 ~ y EO; 'II"

3'11"
:.y = 4
1
(b) Let COS-I 2" = y
1 .
Then cos y 2" and 0 EO; Y EO; 'II"

'II"
:. y =3
26/INVERSE TRIGONOMETRIC RATIOS 143

..• sm
• (
cos -I 211") = • 11"
sm:3 = ,J3
T

(c) Let • 1 1
sm-:3 y
1
Then sin y :3 and - 211" ~ y ~ 211"
And so y may be represented as an acute angle as in fig. 26-19. By Pythagoras,
OQ Pi
2,,2, an d so cosy = 2-./2
3' y
p

Thus cos (sin- I


i) = 2'f
o Q X
Fig. 26-19
3. The inverse tangent' function

y = tan x, x *' odd multiple of i, defines the familiar tangent function.


The graph of artan is the reflection of the graph of tan in the line y = x. As an exercise,
sketch both these graphs, so obtaining the graph of the artan relation.
Following the pattern established for sine and co'sine, we now define a one-one
restriction of the tangent function by restricting x to the open interval - i < x < ;. Why
must the interval be open? This function (Tan) is defined by
11" 11"
Y = tan x for -2 < x < 2
The inverse function, tan -I, is now defined and we have
domain of tan -I = range of Tan = (- 00, 00)

range of tan- = domain of Tan


I
= (- i, i)
so that we may define tan -I as

The graphs of Tan and tan- I are shown in figs. 26-20 and 26-21 respectively.

Y1 y

y •~ tanx,o<x<l - - - - - - -... - - - - - - - -
j
2'
I
I
x
y tan-' x

-------'lI'
-'2

Fig. 26-20 Fig. 26-21


144 NEW SENIOR MATHS: THREE UNIT COURSE

y = artan x means all solutions of the equation tan y x


1f 1f
Y = tan-I x means (a) tany xand (b) -2 <y < 2

Since Tan and tan- I are inverse functions,


tan (tan- I x) x for all x E R, and
1f 1f
tan- 1 (tan x) = x for -<x<-
2 2
As we have seen for the sine and cosine functions, we may also obtain values for
expressions like tan (tan- 1 x) and tan- 1 (tan x). For example,
1f
tan (tan-II) = tan 4
but tan -I (tan 23
1f
) tan -I ( .J3) = - j (not 23 1f
)

The form of the graph of tan- I x shows that tan- 1 is an odd function, so
tan- I (-x) = -tan- 1 x
For example, _:E = -tan -1 1
tan -1 (-I) =
4
Summary oj the inverse trigonometric junctions
sin- 1 cos- 1
domain = [-I, II domain = [-I, II domain = R

range = [ -i· ; J
y = sin- 1 x means
range = [0,

y = COS-I X
7r]

means
range =

y = tan- I x means
(-i, n
7r 7r
x = sin y and x = cos y and 0 < y < 7r X == tan y and - 2" < y < 2
sin-I is an odd function cos- 1 is neither an even tan- 1 is an odd function
so sin-I (-x) = sin-I x. function nor an odd function. so tan-ICx) = -tan- 1 x.
sin (sin -I x) = x for -I < x < I cos (cos- 1x) = xfor -1 < x < 1 tan (tan- 1 x) == x for x ER
7r 11' 11' 7r
sin-I (sinx) = x for 2 < x < 2" cos- I (cos x) = x for 0 < x < 11' tan- 1 (tanx) = x for -2 < x < 2"

EXAMPLE 10
y
Find tan( sin-I (-~)) K Q
X
Let

Then -2
3
Thus x can be represented as a 4th quadrant angle.
OQ = -15, by Pythagoras's Theorem.
P
2
Hence tan x

:. tan (sin-I (- D) 2 Fig. 26-22


26/INVERSE TRIGONOMETRIC RATIOS 145

EXAMPLE 11

Find sin (2 tan-l~)


1
Let tan- 1 -
2
=x y

1 7r 7r
Thus tan x = 2"and -2" < x < 2".
Hence x can be represented as a first quadrant angle.
Then sin (2 tan- 1 ~) = sin 2x x
= 2 sinxcosx Fig. 26-23

= 2 (Js)(3s)
4
5
EXAMPLE 12

Find sin [cos- 1 ~ + tan- 1


(- ~) J
sin [cos- 1 ~ + tan- 1 (-~)J = sin [cos- 1 ~ - tan- 1 ~J (-----
= sin (x - y) where x = cos- 1 ~ and y = tan- 1 ~

= sin (cos -1 ~ ) cos (tan -1 ~ ) - cos (cos -1 ~) sin (tan ~)-1

[Using expansion of sin (x - y)]


·
Smce 4
cos- 1 "5 = x, t h en cos x 4
= "5 and 0 ~ x ~ 7r.

. 4 4 7r 7r
Smce tan- 1 :3 = y, then tany = :3 and -2" < y < 2".
Hence both x and y may be represented as first quadrant angles (as shown in fio- ')"- ,)LI.)

. [14
sm 4J
cos- "5 + tan- -:3 1 Y

~= sin x cos y - bos x sin y


=(~)G)-(~)(~)= -;5
3

o (3, 0) (4, 0) ~.

Fig. 26-24

Note
Let Cos y = x, then y = cos- 1 x.
However, Cos y = 'sin (~ - y) = x. Hence ~ - y = sin- x; i.e., cos- x = ~ - 1 1
sin- 1 x.

:. sin- 1 x + cos- 1 X = ~
146 NEW SENIOR MATHS: THREE UNIT COURSE

EXERCISES 26(b)
1. Without the use of tables or a calculator, evaluate the following (if they exist):
(a) sin -I 1 (b) sin -I 0
(c) COS-I! (d) sin-I _ ,[3
2 2
(e) COS-I 2 (f) s~n-l (sin -0'5)
(g) COS-I (cos 0'123) .\( (h) tan-I (tan 3:)

(i) COS-I (sin ~) (j) COS-I Jz


2. With the aid of tables or a calculator, evaluate the following:
(a) sin-l 0·8 (b) sin-l 0-42
(c) cos- 1 0.4384 (d) tan- 1 0'5
(e) tan-I 2·4 (f) COS-I 0-6
(g) tan-I 1·76 (h) COS-I (-0'3)
(i) sin-I 0'77 (j) cos- 1 0-8
3. Evaluate the following:
(a) cos ( sin -I ~), (b) tan (tan-I ( 153))
\ / (c) tan -I (tan 245°) (d) COS-I (cos 540°)
(e) cos (tan -I ( -,[3» (f) cos ( 2 sin-I f)
.. (g) cos (2 COS-I 153) ' / (h) sec (sin-I (-D)

4. Show that:
. 3
(b)Sin-I~+sin-I(-~)
~
(a) tan-I 4 - tan- 1 5 = .4 0

(c) tan- I -
12
5 + cos- I -5 = -~
13 2
(d) COS-I ~5 tan-I (-~)
4
= 2:!
e
()2 sm- 13-
' = . 124
sm- - (f) sin- I ~ + tan-I .l = COS-I ~
5 25 5 24 5

5. Evaluate the following without the aid of tables:


(a) sin (Sin-I G))
+ sin (Sin- I (- ~)) (b) sin [Sin-I ~+ sin- I (- ~ )J
5
(c) cos [sin-I 13 + sin-I; J (d) sin [2 tan-I ~J
(e) cos [tan-I ~ cos- I (3J (f) sin [cos- I ~+ tan- I (- ~~

(g) tan [tan -I ~


3
+ tan -I 12
13
J
6. Prove that
(a) sin-Ix = COS-I~. 0 ~ x ~ 1
(b) COS-IX = sin- I
~,O ~ x ~ 1

7. Sketch the graphs of each of the following, stating their largest possible domain and
range.
(a) y = 2 tan-I x
-------------------~

26/INVERSE TRIGONOMETRIC RATIOS 147

(b) y 2 COS-I x - I
(c) Y = sin-I ~
2
(d) y = 4 COS-I (2x - 1)
8. Sketch, on the same set of axes, the graphs of y = cos- 1 x and y = sin- I x. Hence
sketch the graph of y == COS-I X + sin- I x, and deduce the value of cos- I x + sin- I x
when (a) -1 ~ x ~ 0, and (b) 0 ~ x ~ 1.
9. Sketch the graph of y = cot x, 0 < x < 11'", and hence obtain the graph of the inverse
function cot-I. Write down a definition of cot-I and use it to evaluate the following:
1
(a) cot- I ~
(b) cot-I (-~)
(c) cos (cot-I <-1»

(d) sin [COt-I! + cot-I (-i) ]


10. Givenf(x) = sin (2x - 3) for all x, write down the range off. Show that a r~striction
of f, namely F, defined on [1, 2], has an inverse P-I. Find the rule for p-I, and sketch
its graph stating the domain and range.
11. Find the largest possible domain for whichf-I(x) = sin- 1 (2x - 1) defines a fu'nction.
Hence find and sketch f. stating its domain and range: .

26.7 Derivatives of inverse trigonometric


functions ________________

Derivative of sin-I x y
sin- I x is defined for -I ~ x ~ 1
Let y = sin-I x
and so
~ X
Thus dy = cos y

and so dy 1 f 1(' 11'"


dx = cosy or -2 < y < 2 Fig. 26-25
dy 1 _
i.e. dx = ..JT=X2 for 1 < x < 1
Note that cos y = +..;-r:... sin 2 y = +...rt=X'i because of the restriction placed on y.

Note that the gradient is positive for all x in


the domain and that the domain off' is not the
same as the domain of f. Why?
Hint: Examine the extremities of the graph off. Fig. 26-26
148 NEW SENIOR MATHS: THREE UNIT COURSE

Derivative of sin- 1 ~ y
a
sin-I ~ is defined for -a .;:;; x .;:;; a
a

Let y = sin- 1 ~a
1f 1f
x
and so x a sin y, -2';:;; y.;:;; 2
dx a cos y
Thus
dy
dy 1f 1f Fig. 26-27
and so dx = for -<y<-
a cosy 2 2
dy 1
for -a < x < a
i.e. x
dx
x
Figure 26-28
y

Derivative of COS-I x
COS-I X is defined' for -1 .;:;; x .;:;;
Let y COS-I X
and so x= cos y and 0 .;:;; y .;:;; 1f

Thus
dx sin y
dy =
dy 1
- - , - for 0 <y <
and so 1f
dx -smy
-1 o x
---;::~1~=;:- for -1 < x <
Fig. 26-29
y

Note that the gradient is negative for all x in the o x x


domain and the domain of the derived function is not Fig. 26-30
the same as that of COS-I x. Why?
y

Derivative of cos- 1 ~
a
cos- 1 ~ is defined for -a .;:;; x .;:;; a cos-' ~
a a

Let y COS-I X
a
and so x a cos y and 0 .;:;; y .;:;; 1f
a o a X
Fig. 26-31
26/INVERSE TRIGONOMETRIC RATIOS 149

y
dx
Thus -a sin y
dy
1
and so dy
a y.
for 0 < y < 7f
dx
1
..Ja2 _ X2 for -a <x <a x

Fig. 26-32

Derivative of tan-I x --------11"


tan-I x is defined for all x. 2
Let y = tan-I x
7f 7f
and so X = tan y and - -
2
< y < -2
x
dx
Thus -
dy
= sec2 y y = tan- x
1

1 + tan2 y
7f 7f
and so dy - 1 f -<y<-
dx - 1 + tan 2 y or 2 2
1
-1- - 2 for all x Fig. 26-33
+x y

d 1
dx (tan-I x) = 1 + X2 for all x

X
Fig. 26-34

Derivative of tan-I ~ y
a

Let y x
7f 7f
and so x = a tan y and -<y<-
2 2
dx o a x
Thus - = a sec2 y
dy Fig. 26-35
= a(1 + tan 2 y)
dy 1 7f 7f
and so a(l + tan 2 y) for -2 < y < 2
dx
a
2 2 for all x
a + x

Ii (tan-t ~)
dx a
a I'or all x
a2 + x 2J '
150 NEW SENIOR MATHS: THREE UNIT COURSE

EXAMPLE 13
--,~

--
Find the derivative of cos- 1 (2x + 1) stating the values of x for which it is defined.
y
cos- 1 (2x + 1) is defined for
~ 1
i.e. for ' ~O
y = cos- ' (2x + 1)
-1 ~ x ~O
Hence cos- 1 (2x + 1) is defined for -1 ~ x ~ O.
Let y = cos- 1 (2x + 1)'
= cos- 1 u where u = 2x + 1
dy dy du
dx duo dx
-1 -'5 0 X
1 x 2
~ Figure 26-36
.Jl _ (ix + 1)2 provided -1 < x < 0
2
.J -4x (x + 1)

EXAMPLE 14
Find the derivative of x cos- 1 (2x + 1) stating the values of x for which the derivative is
defined.
By the previous example and the product rule,
d -2
- (x cos- 1 (2x
dx
+ 1)) = x'.J
-
4 (
xx+
1
)
+ cos- 1 (2x + 1)

-2x 1(2 1)
.J -4x(x + 1) + cos- x +

Now the first term on the R.H.S. is defined for -1 < x < 0 and the second term on the
R.H.S. is defined for -1 ~ x ~ O. Thus the R.H.S. is defined for -1 < x < 0 and these are
the values of x for which the derivative is defined.

EXAMPLE 15
Differentiate sin- 1 (cos x) and hence sketch the graph of y = sin- 1 (cos x), -7f ~ X ~ 7f
Let y = sin- 1 (cos x) y
= sin- 1 u where u = cos x .,.-.---.,

dy dy du
dx duo dx
1 .
.Jl_u 2 · - smx
-sin x
.Jl - cos 2 x
-sin x -1("
o 1(" X
.J sin x
2

'- sin x
isin xi
isin xi = sin x when sin x ~ 0 i.e. 0 ~ x ~ 7f
= - sin x when sin x ~ 0 i.e.-7f ~ x ~ 0 Fig. 26-37
26/INVERSE TRIGONOMETRIC RATIOS 151

. dy sin x
.. dx = - sin x = - 1 when 0 < x < 1£"
-sin x
1 when -1£" < x < 0
-sin x
and : is not defined when x = -1£",0, 1£".
If there were no restrictions on the domain, the graph would repeat itself i.e. it would be
periodic with period 21£". The range is - ~ E; Y E; ~ and the derivative is not defined for
any values of x = n 1£", n = 0, ± 1, ± 2, .... These sharp peaks are not turning points.

EXERCISES 26(c)
State the values of x for which the following functions are defined. For each given f(x)
find I' (x) stating the maximal domain of the function it defines. (1. to 22.)
~-""~~~'-""'"

1. f(x) = sin-I ~x \ . 2. f(x) COS-I -Jx


3. f(x) = tan-I 3x 4. f(x) sin-I (1 - x)
5. f(x) = COS-I (~ - 1) 6. f(x) tan-I (X2)
7. f(x) = COS-I (X2) B. f(x) sin- 1 2lxl
'IP,~:., 9. f(x) = sin-I (2x + Or ~ '-'"C('40. f(x) COS-I (1 - 2x)
'C"~l1.
, J(x) = sin- I ~
x -f.
. 12. f(x) tan-I px

13. f(x) sin-I 2x - 1


3
14. f(x) = x tan-I
1 X2
15. f(x) = sin-I x. COS-I ~ 16. f(x) = cos- I +
17. f(x) tan-I(~) lB. f(x) = tan-I x + tan- I (!)
19. f(x) = tan -I ~ 20. f(x) = tan-I (sin x)
21. f(x) = X2 sin -I x 22. f(x) = sin- I (tan x)
23. Iff(x) COS-I (cos x), -1£" .,.; X .,.; 1£", find I' (x) and sketch the graph of y = f(x). State
the range of f(x) and the domain of I' (x).

24. If sin y x and ~ < y < 1£", find : in terms of x.


25. Write down the equation of the tangent to the curves:
(a) y sin-I x at the point where x = .0.
(b) Y COS-I X at the point where x = o.
(c) y tan-I x at the point where x = 1.

26. If f(x) 3 cos~, 0 E; X E;- 21£" find the rule for f- I (x) stating its domain and range.
Sketch the graph of f- 1 (x) and find its gradient at the point where the curve crosses
the Y-axis.
27. If y COS-I X + cos- 1 (-x), find: and show that for all x in the domain, y 1£".

28.Ify sin-I x + sin-I (-x), find ddY and show that for all x in the domain, y O.
x '
152 NEW SENIOR MATHS: THREE UNIT COURSE

29. If f(x) = sin- I x + cos- I x, findf'(x) and sketch the graph of y = f(x).
30. If f{x) = cos -I (sin x), find l' (x) and sketch the graph of y = f(x) for -71" ~ X ~ 71".

26.8 Antiderivatives involving inverse


trigonometric functions _________
In 26.7 we established the derivatives of the inverse trigonometric functions from which it
follows that

f .Ja2
1
- x2
dx = sin-I x + c, -a < x < a
a .

f -1
.Ja 2 - x 2
dx = COS-I x+ c, -a < x < a
a

f a
2 a
+x
2 dx = tan-I x
a
+ c for all x.

EXAMPLE 16

(a) -;-;-~'--= =

sm - 1 X2 + C

smce a =2

(b) ~=---=- dx = COS-I 1+ c, since a = 3


(c) dx = tan-I ~ + c, since a =2

(d) = sin-I JJ + c, since a = .J3


Sometimes, however, a slight adjustment is necessary to make the coefficient of X2 equal
to 1.

EXAMPLE 17

Find (a) (b). 1 r+ dx


4X2

(a) Since .J9a - X2) = 3.J~ - X2, then

--r;==;;:=;; dx -;=;==~ dx = i sin-I 3x + c, since a


1
3"

1" 1 1 1
r+
., 1
dx
4X2
2J ~ .:.. X2 dx = 2: tan-I 2x + c, since a 2
26/INVERSE TRIGONOMETRIC RATIOS 153

(c) Since -./4 - 25x2 -./25 (245 - X2) = 5 -./ 245 - x 2, then
I' 2
J -./4 - 25x2 dx --r=;r====;; dx = 2.
"5 SIll -
15x
+ C, .
SIllce a = "52

EXAMPLE 18
2
~ ,,3 . '4 -1 r 114 dx
Evaluate (a) (0 )5 --r;;:-=;;===;;
J(
(b)
J"3 V- X
2 dx (c) (d) J-1I4 {7==;=':;

[Sin-I ~T5 (b) .C~3 f7=-=I==;; dx = [cos- ~J~~3 1

sin- 1 ~ sin-l 0
- cos- I ~3 - cos- C~3) 1

E_ 0 = E _ 51r
6 6 6
1r -21r
(; =3

1r
=6

EXAMPLE 19

Evaluate {1/2 sin-I x dx


Since there is no standard derivative which yields sin- I x, finding its primitive is beyond
the scope of your course. (The primitive, however, can be found by the method of
integration by parts).
However, since the inverse sine function is associated with the principal valued sine
function, we can find the required integral by an indirect method.
Fig. 26-38 shows the graph off(x) sin-I x and the shaded area A is the area bounded
by the graph of sin-1 x, the X-axis and the lines x 0 and x = ~, the measure of which is

f
112
o sin-I x dx.
Fig. 26-39 shows the graph of g(x) Sin x and shaded area B is the area the measure of
which is
\
154 NEW SENIOR MATHS: THREE UNIT COURSE

y
fIx) sin- 1 x
y
g(x) = sinx

Fig. 26-38 Fig. 26-39

f ~/6
0 sin x dx [ cos X I/6 =. 1 _ ~3
Now the area (A + B) is simply the rectangular area in Fig. 26~39 with dimensions ~ x i.
Thus area measure of A + B = 1; and so since A = (A + B) B, the measure of area

A
'If
12 (1 - f)
~ + .J3 -1
12 2

EXERCISES 26(d)
Find: (1. to 16.)

1.
.,!
2.
"
J91 X2dx 3. J;j 1dx_ X2
4. 5. J;j2 dx
_ X2 6. J"25 +5 X2dx
r
8. ,,2
dx
+ X2 9.
f -1 i
.J6 _ X2dx

10. 11. JI :9X2 I 12. f9+ dx


16x2

13. JX2 ~16 14. J.Jl dx


_ 3x2 15. f.J16 -1
_ 9x2dx

16. J dx
.J5 _ 4X2
/
26/lNVERSE TRIGONOMETRIC RATIOS 155

Evaluate: (17. to 36.)


17

110.../4
dx
X2 18. J~
o X2
dx
+ 9
19.
f/4 dx
o .../1 - 4X2

20. f~ dx 21. 14315 1 dx+ X2 22.


JI/2
-112
dx
.../1 - X2

24. 12
23 (I dx -1
dx J.J3 dx
25.
. Jo ..}2 - X2 .../16 - X2
o 1I.J3 1 + x 2
(1/3 dx
26.
Jo 1 + 9x 2
27. JOI
1
+ 1 + X2dx
x
28. L.J3
+
29.
J -4
4

X2
dx
+ 16 30.
i.J3/6 -1 )
'J}
(
31.
f0
1/3
1
1
+ 4x2dx

32.
l
.J2 dx
OX2 '+ 2
33. f .../2 -
-I
dx
X2 34.
J5
-5
dx
.../100 - X2

'\(, 35.
(', Jor112COS- I
x dx ,36. r
.,0
I tan-I x dx

37. Given that :t = ' 1 and y = 1 when ,x = 0, express y as a function of x.

38. Find the area bounded by the curve y = -0---:- the X-axis and the ordinates x
and x -1.

39. Sketch the graph of y 2 sin 1fX for 0 ... x ... 2 and x = 2 sin :: for 0 ... y ... 2 and,
find the area of the region enclosed by the, curves.

40. The curve y = k+x is rotated about the X-axis. Find the volume of the solid
,
enclosed between x = 1 and x .J3.
41. Sketch the curve y = tan- I x and find the area between this curve, the X-axis and the
line x = .J3.

42. Without evaluating the integral explain why f -I


I tan -I x dx is equal to zero.

43. Without using tables or a calculator, prove that

(a) f3:5 ~ ; (b) 1_:1 !Xx2


44. Find the volume of the solid of revolution formed by rotating the curve y = sin x
between x = 0 and x ~ about the X-axis. Hence, find the volume of the solid

formed by rotating the curve y = sin-I x between y == 0 and y == ~ about the Y-axis.

45. Differentiate x COS-I x - ",/1 - x 2 and use the result to evaluate fol COS-I X dx
156 NEW SENIOR MATHS: THREE UNIT COURSE

46. Sketch the graph of I(x) = 1 - cos x, 0 ... x ... 1r.

7r12
EV,aluate
f0 I(x) dx and indicate the area represented by this integral on the sketch.
Find the inverse function I-I (x), stating its domain and range.
, t'1

Evaluate J I-I (x) dx and indicate the area represented by this integral on the graph
0

of I.

47. Prove that ! (x sin-I x) sin -I x + .\j 1 ~ x 2 ' Hence find the primitive of sin -I x and

(1/2
show that J 0 sin -I x dx 1.
CHAPTER 27
Polynomials

Both arithmetic and algebra are concerned with the six operations - addition,
subtraction, multiplication, division, involution and evolution (the last two being
operations for taking the square root, cube root, etc. of a number, and squaring, cubing,
etc.). In arithmetic, these operations are generally performed on numbers, while in
algebra they are performed on algebraic expressions as well as on numbers.
The set of real numbers has several subsets. Similarly, there are subsets of the set of
. algebraic expressions. Consider the positive integer 2341. Remembering that our numbers
are written in a system of place values, we know that this number can be represented as
2 x 10 3 + 3 X 10 2 + 4 x 10 I + 1 X 10°
If we now replace each 10 with x, we have
2x3 + 3x2 + 4x + 1
x becomes the base for our number.
If we put x 5, then
2 x 53 + 3 X 52 + 4 x 5 + 1 346
The algebraic expression2x3 + 3x2 +4x + 1 is called a polynomial of the 3rd degree in x.

27.1 Definition of a polynomial _ _ _ _ _ _ __


A real polynomial P(x) is an algebraic expression of the form

where n, n - 1, ... are positive integers and an' an_I"" are the coefficients and for
convenience will usually be chosen to be integers.
The term anx n is called the leading term, and the polynomial is said to be of the nth
degree if an '*
O.
Thus 2x3 + 4X2 + 3x 5 is, a polynomial of the 3rd degree in x.
X4 - 2x + 7 is a polynomial of the 4th degree in x.
!x is a polynomial of the 3rd degree in x.
3
158 NEW SENIOR MA THS: THREE UNIT COURSE

But x l12 + 2 + x and!x + 2x- 3 are not polynomials since they include terms that contain
powers of x that are not positive integers.
We also describe a polynomial by the subset of the reals of which the coefficients an,
an -I" .• , ao are elements.
2x 2 - 7x + 1 is a 2nd degree polynomial over the integers.
i
~X5 + 2x + is a 5th degree polynomial over the field of rational numbers.
-Jix - -J6 is a 1st degree polynomial over the irrational numbers.
4X2 + 7rX - ~ is a 2nd degree polynomial over the field of real numbers.

27.2 Operations on polynomials _ _ _ _ _ _ __


One of the functions of algebra is to take an algebraic expression, such as a polynomial,
and change it into a form that is more useful under the conditions of the problem. This
may, or may not, mean simplification.
You are familiar with the operations of addition and multiplication of polynomials and
factorization of 2nd degree polynomials. We consider here the operation of division.

27.3 Division of polynomials _ _ _ _ _ _ _ __


The symbol P(x) stands for a polynomial in the variable x. For example, if
P(x) = 3x2 + 7x + 4
then P(2) = 3 (2)2 + 7 (2) + 4 30
P(-I) = 3c-tF + 7(-0 + 4 0
Pea) = 3a 2 + 7a + 4
Just as in the set of integers we perform long divisions such as 532119 28, so, in the set
o f poIynomla IVlSlOns 0 f t he type X2 +x 7+x + 12 x =1= - 3•
. Is, we can per form Iong d'"

x + 4
x + 31x2 + 7x + 12
x2 + 3x
4x +
12
4x +
12
0+ 0
In this case, there is no remainder, since x + 3 is a factor of Xl + 7x + 12.
In the integer division 3g = 3 + ~ = 3~, the quotient is 3, the remainder is 5, and the
divisor is 9. The statement could be expressed as
32 9 x 3 + 5
= divisor x quotient + remainder,
where remainder < divisor.
27/POLYNOMIALS 159

EXAMPLE 1
If P(x) 3x3 7x2 + 2x + 4, divide P(x) by x - 3.
3x2 + 2x + 8
x - 3 3x 3 - 7x 2 + 2x + 4
3x3 - 9x2
2X2 + 2x
2X2 - 6x
8x + 4
8x - 24
28
Note the steps followed in the division process:
(a) 3x 2 is obtained by dividing 3x3 - 7X2 by x 3; x 3 is then multiplied by 3x 2 to yield
3
3x - 9x • 2

(b) 3x3 9x2 is then subtracted from 3x3 7x 2 to yield 2X2.


(c) 2x is obtained by dividing 2X2 + 2x by x - 3; x - 3 is then multiplied by 2x to yield
2X2 - 6x.
(d) 2X2 - 6x is subtracted from 2X2 + 2x to yield 8x.
(e) 8 is obtained by dividing 8x + 4 by x 3; x 3 is then multiplied by 8 to yield
8x 24.
(f) 8x - 24 is then subtracted from 8x + 4 to yield a remainder of 28.
This result can be expressed in the form
3
7X2 + 2x + 4 ~
3x
x-
(3
x 2 + 2x + 8) + x- 3' or
3x 3 - 7x 2 +2x + 4 (x 3)(3x2 + 2x + 8) + 28
i.e., P(x) (x 3)Q(x) + R, (1)
where Q(x) denotes quotient, and R denotes remainder.
Just as, for integers, the remainder < divisor, so here the remainder (constant) has a
lower degree than the divisor (linear).
Since both sides of (I) are identically equal, they are equal no matter what value of x we
substitute. Putting x 3, we obtain
P(3) = (3 3) Q(3) + 28
= 0 + 28 28
This result suggests that, to find the remainder on division by x 3, we need not actually
do the division, but simply calculate P(3). Divide P(x) by x - 2 and check that the
remainder obtained is directly given by calculating the value of P(2).

EXAMPLE 2
Divide 4x3 - 19x + 9 by 2x - 3.
2X2 + 3x 5
2x - 3
4x3 6x2
6x2 19x
6x2 9x
-lOx + 9
-lOx + 15
6
160 NEW SENIOR MATHS: THREE UNIT COURSE

2x 2 + 3x - 5 is the quotient, Q(x), and -6 is the remainder.


3
4x 19x + 9 ::::: 2X2 + 3x _ 5 6
2x - 3
i.e., 4x 3 - 19x + 9 = (2x 3)(2x 2 + 3x - 5) 6
i.e., P(x) = (2x - 3)Q(x) - 6 (1)

(1) is true for all x. If we put 2x - 3 ::::: 0, i.e., x ::::: ~,


then p(~ ):: : OQ(x) - 6
::::: -6
This suggests that the remainder can be found by simply calculating pG)'
EXAMPLE 3
Divide X4 + x3 - 7x 2 X + 6 by X2 - 1

X2 +x 6
X2 1 IX4 + x3 - 7x 2 - X + 6
X4 - X2
x3 - 6x 2 - X
x3 - X

-6x 2
+6
-6x2 +6
The remainder is zero and so x4 + x 3 7X2 - X + 6 is exactly divisible by X2 - 1. This
means that (x - I) and (x + 1) are factors of the given polynomial.

27.4 Remainder theorem _ _ _ _ _ _ _ _ _ __

If a polynomial P(x) is divided by x - a until the remainder R does not


contain x, then R P(a).

We shall prove this theorem for the case of a cubic polynomial.


Let P(x) pX3 + qx2 + rx + s
so P(a) pa 3 + qa 2 + ra + s
Subtracting, P(x) P(a) p(X3 - a 3) + q(X2 - a 2 ) + r(x - a)
p(x - a)(x2 + xa + a 2) + q(x - a)(x + a) +
r(x a)
= (x a)[p(x2 + xa + a 2 ) + q(x + a) + r]
= (x a)[px2 + (pa + q)x + (paz + qa + r)]
(x - a)Q(x),
where Q(x) is a quadratic polynomial.
Hence P(x) (x - a)Q(x) + P(a),
and, since P(x) = (x - a)Q(x) + R, it follows that
R = P(a).
Notes
1. If P(x) is divided by x + a x - (-a), then R P(-a).
27!POLYNOMIALS 161

2. If P(x) is divided by ax - b = a (X ~). then R = p(~).


EXAMPLE 4
Find the remainders when P(x) = 2x3 6x2 + 4x + 3 is divided by
(a) x 2 (b) x + 3 (c) 2x - 1 (d) 3x + 2
(a) R = P(2) 2(2)3 - 6(2)2 + 4(2) + 3 = 16 - 24 + 8 + 3 = 3
(b) R P( - 3) = 2( 3)3 6( 3)2 + 4( 3) + 3 = -54 - 54 12 + 3 = 117

(c) R = pO) 20Y - GY + G) + ! 6 4 3 ~ + 2 + 3 = 3~


(d) R = P 0j) (-jY -6(-jY + (-j) +
= 2 4 3 =

EXERCISES 27(a)
1. If P(x) = 2x 3 - 4X2 + 2x 1, find P(1), P<-3), P(a), P(-2).
2. If P(x) x 4 ' 2x 3 + X2 - 4, find P(2), PO), P(a).
3. If P(x) = x 3 - 3x 2 + kx 7, find k if P(2) = O.
4. If P(x) := x4 - x 3 + pX2 - 4x + q, findp and q if P(O) = 3 and P(-l) = 11.
5. If P(x) ax3 - 2X2 + bx + c, find a, band c if P(O) = 12, P(-l) = 3 and
P(2) := 36.
Find the quotient and remainder in the following (6. to 12.):
6. 3x2 2x + 5 is divided by x 2.
7. 4x 3 - 8x 2 + 8x 1 is divided by 2x + 1.
8. x 3 - X2 + x - I is divided by x-I.
9. 2X4 3x 3 + 6x 2 - 5x - 4 is divided by 2x - 3.
10. x - 27 is divided by x - 3.
3

11. X4 + 3x 3 + 5x2 - 6x - 8 is divided by x + 2.


12. -2x 3 + 7x 2 - 7x + 1 is divided by 2x + 3.
Without actual division, find the remainders of the following divisions (13. to 17.):
13. x 3 + 3x2 + 2x - 7 is divided by x + 2.
14. 3x 2 + 7x 2 is divided by x - 4.
15. 2y3 + y2 - 2y + 3 is divided by y + 1.
16. X4 - 3x 3 + 5x 2 - 2 is divided by x 4.
17. X" + 2X2 13x 60 is divided by 2x + 3.
18. When x 3 + 3x2 + 7x b is divided by x - 2, the remainder is 20. Find b.
19. When X4 + 5x 3 - ax + band ax2 + bx 1 are each divided by x + 1, the
remainders are 7 and -6 respectively. Find a and b.
20. When the polynomial x 3 + 3x2 - mx + n is divided by x + 2, the remainder is 9;
when divided by x - 3, the remainder is 49. Find m and n.
21. If x 3 + 2X2 + 3x + p and x 3 + X2 + 9 leave the same remainder when divided by
x + 2, findp.
22. When 3x 3 - ax 2 bx + 1 is divided by x 2, the remainder is 15, and, when
divided by x - I , there is no remainder. Find a and b.
23. When 2x3 + tx l + ax + b is divided by x - 3, the remainder is 120, and, when
divided by x + 1, the remainder is -8. Find the values of a and b.
162 NEW SENIOR MATHS: THREE UNIT COURSE

27.5 The factor theorem _ _ _ _ _ _ _ _ _ __


When the remainder is zero, the divisor is a factor of the expression it divides.
For example, 2 is a factor of 14, i.e. 14 = 2 X 7 + 0; x + 3 is a factor of X2 + 4x + 3, or
X2 + 4x + 3 = (x + 3)(x + 1) + O. Under these conditions we have a special case of the
remainder theorem. The factor theorem states:

I. If, for a polynomial P(x), it is true that Pta)


of P(x).
= 0, then (x - a) is afactor
.

By finding the zeros of a polynomial (values of a such that Pta) = 0), we achieve a
factorization of the polynomial. We usually find the first factor by trial. Then, if the
polynomial is a cubic, long division yields a quadratic polynomial, which can be further
factorized if real factors exist. For higher degree polynomials, we usually have to find
more than one factor by trial.

EXAMPLE 5
Find one zero, and hence express P(x) = Xl - x 2 14x + 24 as a product of linear
factors.
The zeros, i.e., the values of x that make the value of the polynomial equal to zero,
should be sought systematically. Since the coefficient of Xl is unity, the only possible zeros
involve factors of 24, i.e., I, 2, 3, 4, 6, 8, 12 and 24.
Try x - I as a factor: P(l) = 1 - 1 - 14 + 24 = 10 *- O.
Try x + 1 as a factor: P( - 1) = -1 - 1 + 14 + 24 = 36 *- O.
Try x - 2 as a factor: P(2) 8 4 - 28 + 24 O.
Hence x 2 is a factor of P(x).
By long division, we find that P(x) + (x - 2) X2 + X - 12.
Hence P(x) = (x 2)(X2 + x 12)
= (x - 2)(x - 3)(x + 4)
The zeros of P(x) are thus 2, 3 and -4, and the factorization could have been done by
separately finding these zeros.
Notes
1. When attempting to find linear factors of a 3rd degree polynomial, as in example 5, it
is sufficient to use the factor theorem once only, in order to find the first linear
factor. It is unnecessary, and not advisable, to try to find the three factors (if they
exist) by using the factor theorem. Why?
2. When attempting to find linear factors of a 4th degree polynomial, it is necessary to
find two linear factors using the factor theorem.

EXAMPLE 6
+ Xl 7X2 - X + 6.
Find linear factors of X4
P(x) X4 + Xl - 7Xl - X + 6.
P(l) = 1 + 1 - 7 1 + 6 = O. Hence (x - 1) is a factor.
P( 1) = 1 - 1 7 + 1 + 6 = O. Hence (x + 1) is a factor.
Since both (x - 1) and (x + 1) are factors, the polynomial is divisible without
remainder by X2 1. By actual division, we find that
P(x) = (x 2 1)(x2 + X - 6) (See example 3)
• = (x l)(x + l)(x - 2)(x + 3)
27 /POL YNOMIALS 163

EXERCISES 27(b)
Use the factor theorem to find linear factors, over the rational number field, of the
following (1. to 18.):
1. x 3 + 4X2 - 7 x - 10
2. x 3 + 2:x2 41x 42
3. 2x 3 + llx2 - 43x + 26
4. 6x 3 - 5X2 - 12x - 4
5. 3X3 + IOx 2 + 9x + 2
6. 2x 3 + 7x 2 - lOx - 24
7. 5 - 6x 29x2 + 6x 3
8. 4x + 12x2 - X - 3
3
9. 6X3 + 35x2 + 34x 40
10. 6x 3 29x2 - 40x - 12
11. x 3 5x2 + 6x
12. X3 + 2X2 4x - 8
13. 6x3 - 5x2 - 2x + 1
14. 3x + 7x 2 - 6x 3
15. 2x 3 - 7X2 + 7 x 2
16. 2x 3 + 3x 2 - 9x
17. 6 + 5x - 2x 2 - x 3
18. x 3 - X2 lOx - 8
19. Show that x 3 - 8 has only one linear factor <?ver_!~ real n_umber field.
20. Find the values of a and b that make X4 + 4x3 + ax 2 - b divisible by (x 1) and
(x + 2).
21. Given that (x - 3) and (x + 2) are factors of x 3 - 6x 2 + px + q, find the values of p
and q.
22. If 5x 2 - 6x - 56 and 3x 2 - 14x + a have a common factor (x + b), find the values
of a and b.
23. Given that (x - 1) and (x + 2) are two factors of 6x 4 + ax 3 --, 17X2 + bx - 4. find a
and b and the other two factors.
24. Find the values of a and b that make 2x 3 + ax2 13x + b exactly divisible by
X2 - x 6.
25. Find the linear factors over the real number field of x 3 - 3x 2 8x + 4.

27.6 Polynomial functions _ _ _ _ _ _ _ _ __


The function f where
f(x) = anx n + an IX n- 1 + ... + a1x + ao• (an 0) '*
is called a general polynomial function. The function is defined for all real values of x and
is a continuous and differentiable function. In accordance with earlier notation P(x) often
replaces f(x) in defining rules for polynomial functions. Thus provided a 0,__________'*
f(x) ax + b (general linear f u n c t i o n ) '
f(x) = ax2 + bx + c (general quadratic function)
f(x) = ax3 + bx 2 + cx + d (general cubic function) '\
If the coefficient of the highest power of x is unity, the polynomial function is said to be
monic.
, 164 NEW SENIOR MATHS: THREE UNIT COURSE

The simplest polynomial function of degree n is given by the rule J(x) xn.
Graphs of these basic polynomials for n = 1,2,3,4 and 5 are shown in figs. 27-1 and
27-2. Fig. 27-1 shows the case where n = I, 3 and 5, and fig. 27-2 shows the case where
n 2 and 4.
y
n 3 n=2

flxl = xn

x x

Fig. 27-1 Fig. 27-2


n 5

Notes odd functions even functions


1. The X-axis is tangent to each graph at the origin (n 1). '*
2. J(x) = xn for n even, defines even functions. For example, if J(x) x4,
J( - x) = (-X)4 = X4 = J(x).
3. J(x) xn for n odd, defines odd functions. For example, if J(x) = x3,
J( - x) = Cxp = -x3 f(x).
4. Even functions are characterized by the property thatJ( -x) ::::; J(x), and their ~raphs
are symmetrical about the Y-axis. See fig. 27-2.
5. Odd functions are characterized by the property that J( - x) = -J(x). Since J( - x)
and J(x) are opposite in sign, the portion of the graph of J for x ~ 0 is a mirror-
image, in the origin, of the portion for which x ~ O. See fig, 27-1.
6. Recognizing a function as odd or even means that you need only draw one-half of a
graph in detail. The other half may be drawn using symmetry properties.
7. Odd functions are one-one functions, and even functions are many-one functions.
8. Most functions e.g., the function given by J(x) ::::; x 2 + x,J(x) ::::; eX are neither even
nor odd.

EXAMPLE 7
Sketch the graph of
(a) J(x) = x3 (b) J(x) (x - 2)3 (c) J(x) = (x 2)3 + 1
(a) J(x) = x 3 0 when x = O.
For all x < 0, J(x) < O.
For all x > 0, J(x) > 0 (fig. 27-3).
(b) The graph of J(x) = (x 2P can be produced from the graph of J(x) = x 3 by a
translation, 2 units to the right, parallel to the X-axis.
J(x) = 0 when x = 2.
For all x < 2, J(x) < O.
For all x > 2, J(x) > 0 (fig. 27-4).
27/POLYNOMIALS 165

(c) The graph off(x) = (x 2)3 + I can be produced from the graph off(x) = (x - 2)3
by a translation, I unit upwards, parallel to the Y-axis (fig. 27-5).

x x x

Fig. 27-3 Fig. 27-4 Fig. 27-5


f(x) = x3 fIx) (x - 2)3 fIx) = Ix 2)3 + 1

Similarly, we can sketch the graphs of


(a) f(x) (x 2)2 andf(x) = (x - 2)2 + 1 by translatingf(x) = x 2 , and
(b) f(x) = (x - 2)4 andf(x) = (x - 2)4 + 1 by translatingf(x) = X4.
In general, the graph off(x) = (x + b)n + e will have the same general shape as the graph
of f(x) = xn.

Cubic functions
A polynomial function, f, of the 3rd degree, defined by the rule
f(x) = ax 3 + bx2 + ex + d,
where a, b, e and d are constants, a =1= 0, is a cubic function. We shall consider only those
cubic polynomials that have at least one linear factor of the form (x + 0:), where 0: is an
integer. Such a factor can be found by the use of the factor theorem.

EXAMPLE 8
Sketch the graph of f where f(x) = (x + l)(x 2)(x - 3).
At the points where the graph of f crosses the X-axis,
f(x) 0
Le., (x + l)(x 2)(x 3) = 0
x = -1, 2, or 3
The function value will change sign from positive to negative or from negative to positive
at x = -1, x = 2 or x = 3.
For all x < -1, each of the three factors is negative, and so f(x)· O.
1f -1 < x < 2, (x + 1) is positive and (x - 2) and (x 3) are both negative, and so
f(x) > O.
If 2 < x < 3, (x + 1) and (x 2) are positive and (x - 3) is negative, and sof(x) < O.
For all x > 3, each of the factors is positive, and so f(x) > O.
This information is summarized in the following sign diagram.

+
o x
Fig. 27-6 '----'
166 NEW SENIOR MATHS: THREE UNIT COURSE

At the point where the graph off crosses the Y-axis, x = 0, and so f(O) 1 x -2 x -3 6.
The graph crosses the Y-axis at (0, 6).
y

2)(x -3)

x
EXAMPLE 9 Fig. 27-7
Sketch the graph off, where f(x) = 2x 3 - X2 - 13x - 6, and find the values of x for
which 2x3 - X2 - 13x - 6 > 0
It is useful to find real linear factors of 2x3 X2 13x 6 if they exist.
. f(1) = 2 - 1 13 6 =1= 0 and so (x 1) is not a factor.
f( 1) = -2 - 1 + 13 6 =1= 0 and so (x + 1) is not a factor.
f(2) = 16 4 26 6 =1= 0 and so (x - 2) is not a factor.
f( -2) -16 - 4 + 26 - 6 = 0 and so (x + 2) is a factor.
To get the other linear factors (if any) divide 2x 3 - X2 - 13x - 6 by x + 2, and then
factorize the quotient.
2X2 - 5x - 3
x + 2 rX3 X2 - 13x - 6
2x3 + 4X2
5x2 - 13x
-5x 2 lOx
-3x - 6
3x - 6

The factors of 2X2 - 5x - 3 are 2x + 1 and x - 3.


f(x) = (x + 2)(2x + l)(x 3)
_ 1
= 0 when x 2, - 2' 3
For all x < -2, f(x) < O.
1
If -2 < x < - 2' f(x) > O.
1
If - 2< x < 3, f(x) < O.
For all x > 3, f(x) > O.
This information is summarized in the following sign diagram.

+
-2/ -1-,/ I
0
I
2 /3
..
X

Fig. 27-8
27 IfOL YNOMIALS 167

1
Hence/(x) > 0 when -2 < x < -2" or x > 3
Also, when x 0, /(0) = 6.
The graph crosses the Y-axis at (0, -6).
y

Fig. 27-9

EXAMPLE 10
Without plotting great detail, sketch graphs of the polynomial functions defined by the
following rules:
(a) y = (x + 2)(x - 1)2(x + 1)
(b) y = X2(X - 3)(x + 1)3
(a) y = (x + 2)(x - 1)2(X + 1). When Ixl is large, y behaves like X4.
Since (x - 1)2 ~ 0, the sign of y is determined by (x + 2)(x + 1). Hence y > 0
when x < -2 or x > -1 (except x = 1), and y < 0 when -2 < x < -1.
Near x -2, y behaves like a multiple of x + 2, and the X-axis is cut in a linear
manner. (Similarly, the X-axis is cut linearly at x = -1.)
Near x = 1, y behaves like a multiple of (x 1)2, and so the graph is tangent to the
X-axis at x = 1. (See fig. 27-10)
y

Fig. 27-10

(b) y = X2(X -3)(x + 1)3. When Ixl is large, y behaves like x 6•


The sign of y is determined by (x - 3)(x + 1)3, and hence by (x 3)(x + 1). Why?
Thus y > 0 if x < -lor x > 3, and y < 0 if -1 < x < 3 (except for x = 0).
Near x = 3, y behaves like a multiple of x - 3, and the graph cuts the X-axis
linearly at x 3.
168 NEW SENIOR MATHS: THREE UNIT COURSE

Near x = O,y behaves like a multiple of X2, and the graph touches the X-axis atx O.
Near x = -1, y behaves like a multiple of (x + 1)3, and the graph cuts the X-axis at
x -1 in the form of a basic cubic curve. (See fig. 27-11)

Fig. 27-11
The graphs are not drawn to scale.

EXERCISES 27(c)
1. Sketch the graph of
(a) f(x) = x(x - 2)(x + 3),
(b) f(x) 2(X - ~}x + 1)(2x + 3),
(c) f(x) x(x 2)2, and, in each case, find the values of x for which
0) f(x) =0 (ii) f(x) > 0
2. In (a)-(d), sketch the graph of f where f(x) is the polynomial listed. In each case,
draw a sign diagram and label your sketch showing the points of intersection with the
axes.
(a)x 3 - 4x
(b)x 3 + 4X2 + 4x
(c) 2X2 - x 3
(d)x 3 - X2 - lOx - 8
Find the values of x for which f(x) ;;;. 0 for each.
3. (a) Find the linear factors of 6 + 5x - 2X2 - x 3 •
(b) Find the values of x for which
(i) 6 + 5x - 2X2 - x 3 0
(ii) 6 + 5x 2X2 - x 3 > 0
(c) Sketchthe graph offwheref(x) = 6 + 5x - 2X2 - x 3•
4. (a) Find the linear factors of x 3 5x 2 + 8x - 4.
(b) Find the values of x for which
(i) x 3 5x 2 + 8x - 4 0
(ii) x - 5x 2 + 8x - 4 > 0
3

(c) Sketch the graph off where f(x) x 3 - 5x 2 + 8x - 4.


5. (a) Find the linear factors of -Xl + 2X2 + x 2.
(b) Find, by drawing a sign diagram, the values of x for which -Xl + 2X2 + x 2>0
(c) Sketch the graph off where f(x) = -Xl + 2x 2 + X - 2.
27/POLYNOMIALS 169

6"l Describe how the graph of f(x) = x 3 can be transformed into the graph of
) (a) f(x) = (x + 3)3,
(b) f(x) (x + 3)3 - 2.
7. Show that the graph off, where f(x) = x 3 - 8, cuts the X-axis at one point only.
8. Show that the graph of f, where f(x) X2 8x + 12, cuts the X-axis at one
x3
point, and touches it at another. Find the values of x at these points.
9. Show that the graph of y = x 3 - 4X2 + 8x - 8 cuts the X-axis at one point only.
Draw sketch graphs that show the general form of the curves defined in questions
(10. to 23.).
10. y (x + 1)(x - 2)(x + 3) 11. y = x(x - l)(x 2)(x - 3)
12. y = x(x 1)2 13. y = X2(X - 1)
14. y = x(x + 3)2(2 - x) 15. y = (x - 1)(2x - 3)(x + 4)2
16. y x(2x - 1)(x - 1)3 17. y = x(x + 2)(x - 1)4
18. y = (3x 5)3(x - 1)2 19. y x(x - 1)2(2 xp
20. y = x(x - 2)3(x + 1)4 21. y = X(X2 - l)(x - 1)2
22. y (3 - 2x)(1 X)4(X + 3) 23. y = (2 - X)3(1 + X)2(X - 3)

27.7 Relationship between roots and


coefficients _ _ _ _ _ _ _ _ _ _ _ _ __

(a) Quadratic equations


The general quadratic equation is
ax 2 + bx +c 0, a *0
Dividing both sides by a, we get
b
X2 + -x
a
+ Ca = 0 (i)

Let the roots of this equation be a and (3.


Then (x - a)(x - (3) = 0
i.e. X2 - (a + (3)x + a(3 0 (ii)

i.e. X2 - (sum of roots) x + product of roots = O.


Equating coefficients of like powers of x in (I) and Oi) we get
a+(3= b (iii)
a
C
and a(3 = (iv)
a
(iii) and (iv) give us the relationship existing between the roots a and (3 and the coefficie~ts
a, band c of a quadratic equation.

EXAMPLE 11
Write down the quadratic equation whose roots are the squares oftherootsof2x2 + 3x + 5= O.
Using the relationships established above between the roots and coefficients, this type of
problem can be solved without actually finding the roots of the equation 2X2 + 3x + 5 = O.
170 NEW SENIOR MATHS: THREE UNIT COURSE

Let the roots of 2X2 + 3x + 5 0 be a and {3.


3 5
Then a + {3 = - 2: and a{3 2:
The required equation is of the form
X2 - (a 2 + (32)X + a 2{32 = 0
. 9 11
But a2 + {32 = (a + (3)2 - 2oo{3 = 4: - 5 = -4
and
Hence, the required equation is
x2 + !!.x
4
+ 25 = 0
4
i.e. 4X2 + llx + 25 = 0
The following identities will be found useful in this work:

1k
.
-0) a 2
. (ij) a l
+
+
(32 = (a
/3l = (a

(b) Cubic equations


+
+
(3)2 2oo{3
(3)l - 3 oo{3( a + (3)

The general cubic equation is


axl + bx2 + ex + d = 0, a *0
Dividing both sides by a, we get
Xl + !!.x 2 + ~x + d = 0 (i)
a a a
Let the roots of this equation be a, {3 and "(.
Then (x - a)(x - (3)(x - "() = 0
i.e. 'x l - (a + {3 + "()X2 + (a{3 + a"( + {3"()x oo{3"( = 0 (ii)

Xl - (sum of roots)x 2 + (sum of the product of the roots two at a


time)x - product of roots O.

Equating coefficients of like powers of x in 0) and (ii), we get


b (iii)
oo+{3+,,(= -
a
c
a{3 + a"( + {3,,( = (iv)
a
d
and a{3,,( = - - (v)
a
(Ui), (iv) and (v) give us relationships existing between the roots a, {3, "( and the
coefficients a, b, c, d of a cubic equation .
. Note: These relationships between roots and coefficients do not of themselves provide a
means of finding the roots of an equation without some additional information.

EXAMPLE 12
If a, {3 and "( are the roots of the equation Xl + 2X2 + 3x +4 = 0 find the value of

(a) (a 1)({3 - 1)("( - 1) (b) ~+~+~


(c) a 2 + {32 + "(2 (d) ~ + ~ +~
oo{3 a"( {3,,(
27 /POL YNOMIALS 171

Sum of roots = 0: + (J + )' = -2


Sum of roots two at a time = 0:{J + 0:)' + (J)' 3
Product of roots = 0:{J)' = -4
(a) (0: - 1)({J - 1)(), - 1) = 0:{J)' - (0:{J + 0:)' + (J)') + (0: + (J + )') - 1
:: -4 - 3 - 2 - 1
= -10
(b) ! + ! + ! :: 0:{J + (J)' + 0:)'
0: (J)' 0:{J)'
3
= -4
(c) 0: 2 + {J2 + )'2 = (0: + (J + )')2 - 2(0:{J + (J)' + 0:)')
= (-2)2 2 x 3
= -2

(d) ~+~+~=0:+{J+)'
0:{J 0:)' (J)' 0:{J)'
-2
= -4
1
2
EXAMPLE 13
Solve the equation 2x 3 - 7 X2 - 12x + 45 = 0 given that two of its roots are equal.
Let the roots be 0:, 0:, {J.
7
Then 20: + (J = 2" (i)

0: 2 + 20:{J = -6 (ii)
45
0:2{J = 2
(iii)

In place of {J in (ii) put {J = ~ - 20: from (i)


Hence (ii) becomes

0: 2 + 20:(~ - 20: )= -6
0: 2 + 70: - 40: 2 = -6
30:2 - 70: - 6 = 0,
(30: + 2)(0: - 3) = 0
2
0: = 3 or -
3
From (i) {J = -~ or 29
2 6
Now these solutions have been obtained without using (iii), In fact the solutions
0: = -~, {J = 2: do not satisfy (iii) and must thus be rejected.
5
The required roots of the equation are thus 0: = 3, {J = - 2'
EXAMPLE 14
If the roots of the equation x 3 + pX2 + qx + r = 0 are consecutive terms of an arithmetic
sequence, find an equation relating p, q and r.
172 NEW SENIOR MATHS: THREE UNIT COURSE

Let the roots be a p, a and a + P.


Then 3a -p (i)
a(a p) + a( a + p) + (a - p)( a (:J) + =q
i.e. 3a2 p2 =q (ii)
a( a - p)( a + p) = -r
i.e. a(a 2 - (2) = -r (iii)
p
From (i) a= -3
Hence (ii) becomes (:J2= q

and (iii) becomes _lj(p; _~2 + q) = -r


i.e. p(9q - 2p2) = 27r
(c) Fourth degree equations
The general fourth degree equation is
ax4 + bx 3 + cx 2 + dx + e = 0, a '* °
Dividing both sides by a we get
b
X4 + -x 3
a
+
c
-x 2
a
+
d
-x
a
+
e
a
° (i)

Let the roots of this equation be a, p, 'Y, 0.-


Then
Le. X4 (a +p+ 'Y + 0)X3
(x -' a)(x - p)(x
+
'Y)(x - 0) =
(ap + a'Y + ao + P'Y + po + 'YO)x2
°
- (ap'Y + apo + a'Yo + P'Yo)x + ap'Yo ° (ii)

X4 - (sum oj roots)x 3 + (sum oj product oj roots two at a time)x2 -


(sum oj product oj roots three at a time)x + product oj roots 0. r'"''

Equating coefficients of like powers of x in (i) and (ii), we get


b
a + p + 'Y + 0 = -- (iii)
a

(iv)

(v)

e (vi)
a
EXERCISES 27(d)
1. If a and p are roots of the equation x 2 + 8x 5 = 0, find the quadratic equation
whose roots are ~ and ~.
2. If a and p are roots of the equation X2 + 4x + 1 = 0, what is the value of
(a) a + p (b) ap (c) a 2 + p2 (d) a 3 + p3
3. If a and p are roots of the equation X2 + mx -+ n = 0, find the roots of
°
nx 2 + (2n - m2)x + n = in terms of a and p.
4. If a and pare rootsctf the equation pX2
1 1
+ qx +
1
r =
1
° find in terms of p, q and r

(a) -; + 73 (b) a2 + p2
27!POL YNOMIALS 173

5. If a and fJ are roots of X2 + 5x + 7 = 0 form the quadratic equation whose roots are
(a - fJ)2 and (a + fJ)2
6. Solve the equation x 3 3x2 - 4x + 12 = 0 given that the sum of two of its roots is
zero.
7. Solve the equation 2x 3 - 21x2 + 42x 16 0, given that the roots form
consecutive terms of a geometric sequence.
Hint: Let the roots be~, a, afJ or a, afJ, afJ2.

8. Solve the equation 4x 3 + 32x2 + 79x + 60 = 0 given that one root is equal to the
sum of the other two.
9. Solve the equation 3X3 - 17x2 - 8x + 12 0 given that the prod~ct of two of the
roots is 4.
10. Find the cubic equation whose roots are twice those of the equation 3x 3 - 2X2 +1 O.
11. If the roots of the equation x 3 + pX2 + qx + r = 0 form consecutive terms of a
geometric sequence, prove that q3 = p 3r.
Showthatthiscondition is satisfied for the equation 8x3 - 100x2 + 250x - 125 oand
'solve this equation.
®
.
Find two values of m such that the roots of the equation x 3
fJ, afJ and using these values of m find a and fJ.
+ 2X2 + mx - 16
. .
0 are a,

~- . • 1
@Ifthe roots of the equation24x4 - 52x 3 + 18x2 + 13x 6 = 0 are a, -a, fJ, ~ find a
and fJ.
14. If two ofthe roots of the equation Xl + qx + r = 0 are equal, show that 4 q 3 + 27 r2 = O.
15. If the roots of the equation x 3 + 3x2 - 2x + 1 = 0 ar~ a, fJ, "I, find the value of
(a) a 2(fJ + "I) + fJ2("f + a) + "I2(a + fJ) (b) a 2 fJ2 + 13 2 "12 + "I2a 2
16. Solve the equation 4x3 - 12x2 + 9x 2 = 0 given that two of its roots are equal.
® Solve the equation 6x4 - llx 3 - 26x2 + 22x + 24 "= 0 given that the pr~duct of two
of the roofs is equal to the product of the other two.
@ Solve the equation 6x4 29x 3 + 4Ox2 7X - 12 = 0, the product of two of the
roots being 2.
19. Solve the equation 32x1 - 48x2 + 22x 3= 0, the roots forming consecutive terms
of an arithmetic sequence.
20. Solve the equation 8x4 - 2x1 27 X2 + 6x + 9 = 0, given that the sum of two of its
roots is zero.
, . '.
21. Find a cubic equation whose roots are. the squares of those of x 3 + px + q O.
. 22. If a, fJ, "I are the roots of 3x 3 + 8x 2 - 1 = 0 find the value of
(fJ + ~)("I + ~)(a + ~)
23. Solve the equation x4 - 2xJ - 3x2 + 4x + 2 = 0 given that the sum of two of its
roots is zero.
24. If tlie roots of the equation X4 - 6X3 + 3x2 + 12x + 4 = 0 are a, 2a, fJ, 2fJ, find the
value of a and fJ.
174 NEW SENIOR MATHS: THREE UNIT COURSE

25. Show that the cubic equation 8x 3 -'- 6x + 1 = 0 can be reduced to the form cos 3 (f =
- ~ by substituting x = co~ 'fl. Deduce that
271" 471" 71" 271" 471" 71"
(a) cos 9 + cos 9 = cos 9 (b) sec 9 + sec 9 = 6 + sec 9
71" 271" 471" 71" 271" 471"
9 sec 9 9 9 + tan 9 + tan 9 = 33 (d) tan 2
2 2
(c) sec sec = 8

26. If tan a, tan (3, tan 'Yare the roots of the equation x 3 - (a + l)x 2 + (c - a)x - c = 0
. 71"
show that a + (3 + 'Y = n 71" + 4.

27. Expand cos (2A + B) and hence prove that 41 cos 38 =, cos 3 8 - 43 cos 8.-
Putting x = k cos 8 and giving k a suitable value use the preceding formula to find
the three roots of the equation 27 x 3 - 9x = 1. Hence write down the value of the
71" 371" 571" 771"
product cos 9 cos 9 cos 9 cos 9·

27.8 Approximate solutions of equations _ _ __


We should associate the roots of an equation with the x-values of the points of
intersection 9f the graph of the appropriate function with the X-axis e.g. the roots of the
quadratic equation X2 - 5x + 6 = 0 are the x-values of the points of intersection of the
parabolay = X2 - 5x + 6 with the X-axis. The problem we are confronted with is finding
where the curve crosses (or touches) the X-axis.
If, for a given function y = f(x),j(a) > 0 andf(b) < 0, there is at least one root of the
equationf(x) = 0 between x = a and x = b, provided that the function is continuous in
the interval a ~ x ~ b. This seems obvious but it needs a proof. Such a proof is beyond
the scope of this course.
We cannot always be certain, however, that when f(a) and f(b) have the same sign,
there is no root between a and b.
y y y

y = fIx)

y = fIx) y = fIx)

o a b X o a b )( 0 a c b

Fig. 27-12 Fig. 27-13 . Fig. 27-14


Fig. 27-12 shows there is no root between a and b.
Fig. 27-13 shows there are tw() distinct roots between a and b.
Fig. 27-14 shows there is one root of mUltiplicity 2 i.e. there are 2 coincident roots
between a and b.
In each of these casesf(a) andf(b) have the same sign.

If f(a) and f(b) have opposite signs, there must be at least one root
between a and b, provided the curve is continuous for a ~ x ~ b.
27 /POL YNOMIALS 175

y y y

y = fIx)

x x x

Fig. 27-15 Fig. 27-16 Fig. 27-17

In Fig. 27-15, f(a) andf(b) have opposite signs and there is one root between a and b.
In Fig. 27-16,j(a) andf(b) have opposite signs and there are several roots between a and b.
In Fig. 27-17 ,j(a) andf(b) have opposite signs and there is a multiple root between a and
b. For example, f(x) = x 3 - 6x2 + 12x 8 (x 2)3 = 0 has a triple root at x = 2.
Of course, if the function is not y
continuous between x = a and x = b, then

K
the fact that f(a) and f(b) have opposite
IX)=.!.
signs does not necessarily imply the presence I x
of a root. See Fig. 27-18 I
,
a I

~Ob x

\ Fig. 27-18

Approximate methods of solving equations usually involve starting with an estimate of


the solution and then improving on the estimate by an iterative process. An iterative
process is one in which a sequence of steps is performed repeatedly and at the end of each
cycle an improved estimate is obtained. Such a process, of course, may never yield the
exact solution. We will consider two methods.

(a) Halving the interval (Bisection method)


The first step is to obtain two estimates of the root, one below and the other above the
root.lff(x l ) andf(x z) have opposite signs, there is a root ofj(x) in the interval Xl < x < x 2 •
The second step is to compute the mid-point X3 = ~(Xl + xz) and the va\ue off(x3). If
f(x 3) = 0, X3 is the desired root.

X,
o X2 X

Fig. 27-19
176 NEW SENIOR MATHS: THREE UNIT COURSE

The third step is to decide whether the root is in the half-interval XI to X3 or the half-
interval X3 to xz. This is done by considering the sign of f(x 3).
When we know which is the appropriate interval, the process starts again.
Fig. 27-19 shows what the bisection method looks like on a graph.
Summary:
Step 1 - obtain two estimates of the root XL (below the root) and Xu (above the root).
1
Step 2 - calculate xm = 2(x L + xu)
Step 3 - if Xm is a good enough estimate of the root, stop the process; otherwise proceed
to step 4.
Step 4 -.determine whether the root is between XL and Xm or between Xm and Xu by
examining the sign of f(x m ).
Step 5 - if the root is between XL and X m' abandon the old Xu and call xm the new Xu and
return to step 2; if the root is between Xm and Xu, abandon the old XL> call xm the
new XL and return to step 2.

EXAMPLE 15
Show that x 3 + XZ + X - 8 = 0 has a root betweelJ X = 1 and X = 2 and use the bisection
method twice to determine an estimate of the root.
f(x) = x3 + XZ + X - 8
f( 1) = 1 + 1 + 1 - 8 = -5
f(2) = 8 + 4 + 2 - 8 = 6
Since f(l) and f(2) have opposite signs, there is a root between 1 and 2.
XL = 1 and Xu = 2 Y
. 1
xm = 2(1 + 2) = 1·5
f(1·5) = (1·5)3 + (1·5)2 + 1·5 - 8
(2,6)
= -0·875
<0
Since f(1·5) < 0, the root lies between 1·5 and 2.
XL = 1·5 and Xu = 2 o x
:. xm = ~(1.5 + 2) = 1·75
f(1·75) = (1·75)3 + (1·75)2 + 1·75 - 8 (1, -5)
= 2·17
Fig. 27-20
>0
Since f(1·75) > 0, the root lies between 1·5 and 1·75 and probably closer to 1·5. Why?
Continue this iterative process and you can see how you can get a better approximation.
~ctually, the given equation has only one real root. Why? (Hint: Consider the sign of the
derivative) .
(b) Newton's method
Suppose XI is close to a root of the equation f(x) = O. The equation of the tangent at

y - y I = m (x - X I)
i.e. y - f(x l ) = l' (xI)(x - XI)

This tangent crosses the X-axis at a point where X = X z and y = O.


Hence 0 - f(x l ) := 1'(xl )(XZ - XI)
27/POLYNOMIALS 177

i.e.

This equation provides the basis for an iterative process. If XI is close to the desired root,
then under certain conditions, X 2 is cloSer. If we now replace Xl by X 2 and call the new
result X3 instead of X 2 • then X3 may be closer still. Newton's method usually requires fewer
steps for a given accuracy than the bisection method but more care must be used in
applying it.
Fig. 27-21 shows that if Xl is our first approximation, then x 2 is closer to the root than Xl
and X3 is closer than x 2 • If we wish to continue the iterative process we can get an x 4 closer
to the root than x 3 and so on.
Let us consider the previous example.
y

y = fIx)

o x, x
EXAMPLE 16 Fig. 27-21
The + X2 + X - 8 = 0 has a root near X
equation x 3 = 2. Use Newton's method with two
applications to obtain a better approximation.
f(x) = x 3 + X2 + X 8
!' (x) = 3x2 + 2x + 1
Put Xl = 2 and so
f(x l ) = 8 + 4 + 2 8 6
!'(x 1) = 12 + 4 + 1 17
X = X - f(x 1)
2 I !'(x 1)
6
= 2 - 17
= 1·647
f(x 2 ) = (1'647)3 + (1'647)2 + 1·647 - 8
= 0·828 (using a calculator)
!' (x 2) = 3 (1'647)2 + 2 (1'647) + 1
12·43
_ _ f(x 2 )
X3 - x2 !' (x z)
= 1.647 _ 0-828
12-43
1·58
Since f(I'58) = 0·020 :=:: 0, x 1'58 is a close approximation to the root of the given
equation. Newton's method is useful in finding square roots, cube roots etc.
178 NEW SENIOR MATHS: THREE UNIT COURSE

EXAMPLE 17
Use Newton's method with two applications to find -J2 to two decimal places.
The solutions of the quadratic equation X2 2 = 0 provide the square roots of 2. From
the graph of f(x) = X2 - 2 we see that the parabola crosses the X-axis between x = I and
x = 2 and probably closer to x = I.
We take x I :: I as our first approximation y
f(x) :: X2 - 2 :. f(l) = -1 (2,2)
I'(x) = 2x :.1'(1) = 2
f(x ,)
Xl = XI - I'(x )
l

1 + !2
x
1'5
f(x 2 ) =
(1'5)2 - 2 = 0'25 1)
l'(x2 ) = 2(1'5) = 3
_ _ f(x 2 )
x] - Xl l' (Xl)
= 1'5 _ 0'25
3 Fig. 27-22
= 1·417
= 1'42 (to 2 decimal places)

Newton's method is applicable not only to polynomial equations but also to other
standard type equations as well as transcendental equations.

EXAMPLE 18
Find the root of 2 sin X x 0 near x = 2.
f(x) = 2 sin x x:. f(2) = 2 sin 2 - 2 = -0'1814
l' (x) = 2 cos x - I :. l' (2) = 2 cos 2 - 1 = -1,8323
Putting XI = 2 f(x , )
Xz XI - I'(x l )
2 _ -0'1814

1-9010
=
f(x z) fO-901O) -0-0090
l' (x 2 )f(1-901O) 1-6485
f(x 2 )
x] x 2 l'(x 2 )
-0,0090
= 1·9010
-1-6485
= 1·8955
f(1'8955) = 0·0000 so x = 1·8955 is a good approximation
Our initial estimate of the root, namely x = 2, was close to the actual root. Let us see
what could happen if our initial estimate is too far off, say x = 6.
27/POL YNOMIALS 179

Thenx j =6
!(X j )
x2 = Xl - !'(x )
l

6 - -6-5588 = 13-1268
0-9203
!(x2 ) = !(13-1268) = 12-0637 which is further from zero than!(x t )
!(x2 )
X3 = x2 !' (x 2 )
13·1268 - 12·0637 = 30.5071
0·6941
!(X 3) -32,0847
The function values are increasing and we are getting further away from the root.
EXERCISES 27(e)
Use the method of bisection to solve the following equations (1. to 5.) to 2 decimal places
given that:
1. X3 - 3x 20 = 0 has a root between 3 and 3-1
2. X4 + X2 80 = 0 has a root between 2·9 and 3-0
3. lOx 2 sin X + 5 has a root between 0'6 and 0'7
4. logex = cos X has a root between 1·2 and 1·4
5. log.(x + 2) ex - 2 has a root between 1'1 and 1·2
Use Newton's method to solve the following equations (6. to 10.) using the method twice
given that:
6. x 3 - 3x - 20 = 0 has a root near x 3.
7. X4 + X2 - 80 0 has a root near x = 3.
8. lOx = 2 sin x + 5 has a root near x 0'5
9. log.x = cos x has a root near x = 1
10. loge(x + 2) = ex - 2 has a root near x = 1
11. Use Newton's method to find approximately the value of
(a) lftO -J5(b) (e) .J15 (d) <t28 (e) ~

12. The equation x 3 ..:... X2 + X


- 2 0 has a root which is approximately 1. Find a better
approximation to 2 decimal using each of the two methods.
13. The equation x 3 - 8x + 1 = 0 has a root which is approximately -3. Find a better
approximation to 2 decimal places using each of the two methods.
CHAPTER 28
Permutations
and
Combinations
In this chapter, consideration will be given to certain counting techniques which will
enable us to answer such questions as:
(i) In how many ways can 10 people arrange themselves in a row, taking them 4 at a
time?
(ii) How many committees of 4 can be selected from 10 people?
In each case we are dealing with a set of 10 elements. In (i) we are concerned with
finding the number of ordered 4-subsets which can be formed from the IO-set. Each of
these 4-subsets is called an arrangement or permutation.
In (ii) we are concerned with finding only the number of 4-subsets which can be
formed without regard to the order in which the elements appear in the subset. Each of
these 4-subsets is called a selection or combination.

28.1 Definitions _ _ _ _ _ _ _ _ _ _ _ _ __

I (a) iach of the ordered subsets :hich can be formed by selecting some
or all of the elements of a set is called an arrangement or
permutation. .
(b) Each of the subsets which can be formed by selecting some or all of
· the elements of a set without regard to the order in which the
I elements app~ar in the subset is called a se~ection or combination.

The distinction between a permutation and a combination can be illustrated by a simple


example.
Consider a set of four people whom. we shall denote by A, B, C and D. In how many
ways can they be arranged, taking them two at a time? How many selections of two can be
made?
To solve this problem, we can list the possible arrangements and selections and count
them.
The possible arrangements are:
AB, Ac, AD, BC, BD, CD.
BA, CA, DA, CB, DB, DC.
28/PERMUTATIONS AND COMBINATIONS 181

Thus there are 12 arrangements.


The number of selections is only 6, namely
AB, AC, AD, BC, BD, CD.
Note that AB and BA are two different arrangements but only one selection of two
people. Order must be taken into account in permutations; in combinations, however, we
are concerned only with the number of elements in each subset.

28.2 Permutations _ _ _ _ _ _ _ _ _ _ _ __

The above method of listing and counting the arrangements would be somewhat laborious
if the number of people at our disposal were more than four and the number in each
arrangement more than two. A more systematic approach is therefore necessary.

28.3 The multiplication principle _ _ _ _ _ __

If one operation can be performed in m different ways and, when it has


been performed in anyone of these ways, a second operation can then be
performed in n different ways, the number of ways of performing the
two operations is m x n.

EXAMPLE 1
There are 4 roads from town A to town B, and 3 from town B to town C. How many
different ways are there of travelling from A to B to C?

Fig. 28-1
Consider one particular road from A to B; by travelling along this particular road, there
are 3 ways of travelling from A to B to C. Similarly, by taking another road from A to B,
there ar~ another 3 ways of going from A to B to C. Hence there are 4 x 3 ways of
travelling from A to B to C.
This multiplication principle can now be applied to solve the initial problem: 'How
many arrangements can be made of the four people A, B, C, D taken two at a time?'
This is the same as filling two places when we have four people at our disposal.
14 I3I
The first place can be filled in 4 ways because anyone of the 4 people can occupy this
place. When the first place is filled in anyone of these four ways, there are three people
left, anyone of whom may occupy the second place. Hence, applying the multiplication
principle, the number of ways of filling the two places is 12.
The symbol 4P2 denotes the number of arrangements of four different objects taken
two at a time.
:.4P2 4.3
= 12
182 NEW SENIOR MATHS: THREE UNIT COURSE

EXAMPLE 2
In how many ways can the six people A, B, C, D, E, F be arranged in a row taken
(a) 3 at a time, (b) 6 at a time?
(a) This is the same as filling three places, with six objects at our disposal
1 15IlJ
6
The first place can be filled in 6 ways, because anyone of the six people can occupy
this place. When the first place has been filled in anyone of these six ways, there
remain 5 people, anyone of whom can occupy the second place. Applying the
mUltiplication principle, the first two places can be filled in 6 x 5 ways. When the first
two places have been filled in anyone of these 30 ways, there remain four people, any
one of whom may occupy the third place. Hence, extending the multiplication
principle, the number of ways of filling the three places is 6 x 5 x 4 i.e. 120 ways.
The symbol 6P 3 denotes the number of arrangements of six objects taken three at a
time.
:.6P 3 =6.5.4
= 120.
(b) We are required to fill 6 places, with six people at our disposal
16 1 5 1 ,4 1 3 1 2 1 1I
From the extension of the multiplication principle in (a), the number of
arrangements is 6.5.4.3.2.1, the symbol for which is 6P6
:. 6P6 = 6.5.4.3.2. 1
= 720.
28.4 Factorial n _ _ _ _ _ _ _ _ _ _ _ _ __

The result of Example 2(b) can be written in a concise form; it is the product of six
consecutive integers from 6 to 1 and is written as 6! (factorial 6). The product of n
consecutive positive integers from n to 1 is n! and is called factorial n or n factorial.

, n! = n(n - 1)(n - 2) ... 3.2. 1. I

The answer to example 2(a) can also be expressed in factorial form.


6 . 5 . 4 = 6.5.4.3!
3!
6!
3!
28.5 The symbol nP _ _ _ _ _ _ _ _ _ __
r
The symbol n P r denotes the number of arrangements of n different objects taken r at a
time i.e. the number of ordered r-subsets which can be formed from an n-set. It denotes
the number of ways of filling r places with n different objects at our disposal. (r ~ n.)

The first place can be filled in n ways, because anyone of the n objects can occupy this
place. When the' first place has been filled in anyone of these ways, there remain (n - 1)
objects, anyone of which can occupy the second place. Since, by the mUltiplication
28/PERMUTATIONS AND COMBINATIONS 183

principle, each way of filling the first place can be associated with each way of filling the
second place, the number of ways of filling the first two places is n(n - 1). Similarly when
the first two places have been filled in anyone of these ways, there remain (n - 2) objects,
anyone of which can occupy the third place. Hence the first three places can be filled in
n(n -1)(n - 2) ways and so on. Hence, the number of ways of filling r places is given by:

nPr = n(n - 1)(n - 2) .. . (n - r + 1) (i)


n(n - 1)(n - 2) .. . (n - r + 1)(n - r)!
(n - r)!
n! (ii)
(n - r)!

The symbol nP no then,' would denote the number of ways of filling n places with n
objects at our disposal.
:. nP n = n(n - 1)(n - 2) ... 3.2.1
= n! from (i)
But n P n = ( _ n! n! f (..)
n )' = -0'
n. . rom II
n!
H ence n., -- O!
For this to be true, we define O! to be 1.

28.6 Arrangements with restrictions


EXAMPLE 3
In how many ways can six girls and two boys be arranged in a row if
(a) the two boys are together;
(b) the two boys are not together;
(c) there are at least three girls separating the boys?
(a) Since the boys are to be together, they may be regarded as one unit. Hence there are
seven objects (the six girls and the unit of two boys) to be arranged in a row. This can
be done in 7! ways. However, the two boys, whom we shall denote by A and B can be
arranged amongst themselves in 2! i.e. 2 ways, namely AB or BA.
:. Number of arrangements = 2! 7!
= 2.7.6.5.4.3.2.1
= 10,080.

(b) Number of arrangements without restriction = 8!


Number of arrangements with the boys together = 2. 7!
:. Number of arrangements with the boys not together = 8! - 2. 7!
= 8. 7! - 2. 7!
= 6.7!
= 30,240.
(c) There is a restriction on the places to be occupied by the boys. It is simplest then to
arrange the boys first. If A is in the first position, B can occupy the fifth, sixth,
seventh or eighth position as shown in Fig 28-2 .
:. 4 arrangements of A and B with A in first position.
3 arrangements of A and B with A in second position.
2 arrangements of A and B with A in third position.
184 NEW SENIOR MATHS: THREE UNIT COURSE

Fig. 28-2
1 arrangement of A and B with A in fourth position.
1 arrangement of A and B with A in fifth position.
2 arrangements of A and B with A in sixth position.
3 arrangements of A and B with A in seventh position.
4 arrangements of A and B with A in eighth position.
There are 20 different ways of arranging A and B, namely the sum of the above
arrangements. Why were they added? The reason will be given in the next main
paragraph-under 'addition principle'.
For each of these 20 arrangements, there are six places left which can be filled by the
girls in 6! ways.
:. Number of arrangements = 20. 6!
= 20.6.5.4.3.2.1
= 14,400.

28.7 Mutually exclusive operations-


Addition principle - - - - - - - - - - -
In example 3(c) A can occupy the first position and B the fifth or A the first and B the
sixth or A the first and B the seventh and so on. Such arrangements are said to be
mutually exclusive, i.e. A occupying the first and B the fifth position excludes the
possibility of A occupying the first and B the sixth at the same time.

EXAMPLE 4
How many numbers greater than 6000 can be formed with the digits 3, 4, 6, 8, 9 if a digit
cannot occur more than once in a number?
Numbers containing either four digits or five digits can be formed.
I 3 I 4 I 3 I 2 I
For four digit numbers, the first place can be filled in 3 ways by either 6, 8 or 9. When
the first place is filled in anyone of these 3 ways, there remain 4 digits anyone of which
can occupy second place. Hence the second place can be filled in 4 ways, the third in 3 and
the fourth in 2 by application of mUltiplication principle.
:. Number of arrangements = 3.4.3.2
= 72.

For five digit numbers, the first place can be filled in 5 ways, the second in 4 ways
and S0 on.
.'. Number of arrangements = 5.4. 3.2. 1
= 120.
If we form a four digit number we cannot at the same time form a five digit
number. These two operations are mutually exclusive.
:. Number of arrangements = 72 + 120
192.
28/PERMUTATIONS AND COMBINATIONS 185

28.8 Arrangements with repetitions


EXAMPLE 5
(a) How many different number-plates for cars can be made if each number-plate
contains three letters of the alphabet followed by three of the digits 0 to 9?
(b) What proportion of these plates contains the same letter in each position and the
same digit?
(a) 1 26 1 26 26 1 10 1 10 10 I I
There are six places to fill. The first place can be filled in 26 ways because anyone
of the 261etters of the alphabet can occupy first place. When the first place has been
filled in anyone of these ways, the second place can be filled in 26 ways also because a
letter occupying th~ first position can also occupy the second place. Similarly the third
place can be filled in 26 ways. The fourth place can be filled in 10 ways, the fifth in 10
ways and the sixth in 10 ways. Hence, by the multiplication principle, the number of
plates
= 26.26.26.10.10.10
263 • 1()3.
. (b) 126111111011111
The first place can be filled in 26 ways. When the first place is filled in anyone of
these 26 ways, the second place can be filled in 1 way only. Similarly for the third
place. the fourth place can be filled in 10 ways, the fifth in 1 way and the sixth in 1
way.
..' proportlOn
. fl 26;10
0 pates = 263. 10 3
1
67,600'
28.9 Arrangements in a circle _ _ _ _ _ _ _ __
The number of ways of arranging n different objects in a circle, ~<!-.!diJ?.K£19-£k-\Vi~~"'~.Ilc!
anticlockwise arrangements as different is
~.~ __.___ ....
,~ _~._ -~

rp-e-.
.____._........v---.¥. . -L--.. ..
--"~"<0,,--.-.

(-n---1-)!----,'

EXAMPLE 6
In how many ways can the four women A, B, C, D be arranged in a circle?
There are 2 ways of reasoning. (a) Since there is no
first place to fill we can take one of the women and put A
her in a fixed position and arrange the other three
around her. This can be done in 3! ways (b) The
diagram Fig 28-3 shows one arrangement of A, B, C, D.
D B
If we keep the four women in the same position relative
to each other and move them one place clockwise, we
still have the same arrangement. If they were arranged
in a straight line and we moved them one position to the
right so that the woman in the fourth position then
c
occupied the first position, this is a different
arrangement. Fig. 28-3


186 NEW SENIOR MATHS: THREE UNIT COURSE

[AIBlcln/ InlAIBlcl i c lnlA IBI iB!clnlAI


For everyone arrangement in a circle, there would be four different arrangements in a
straight line.
:. Number of circular arrangements =~
= 3!
= 6.
28.10 Number of arrangements of n objects in a row I

when they are not all different _ _ _ __


The number of ways of arranging n objects in a row, when there are p alike of one kind, q
alike of another etc. is

~
EXAMPLE 7
~
In how many ways can the six letters of the word 'mammal' be arranged in a row?
Since there are three letters 'm' and two letters 'a', the number of arrangements
61
;::: 3!2!
6.5.4:31
3121
60.
Reason: One of the 60 arrangements is m m m a a I. If the letters m were different say m I'
m 2, m3 this arrangement would be
mlm2m3aalmmlm3m2aal
or m 2 m3 m l a a lor m 2 m l in3 a a I
or m3 m l m 2 a a 1 or m J m 2 m l a a I.
Thus if the letters m are different there are 6 different arrangements. Remove the
subscripts and these arrangements are the same. This accounts for the division of 61 by 3!
Reasoning similarly with the two letters a, the number of arrangements is
61
3!2!

EXERCISES 28(a)
1. How many different arrangements can be made by taking 3 of the letters of the word
Sunday?
2. In how many ways can 5 boys be arranged in a row?
3. In how many ways can a first, second and third prize be awarded in a class of 8
students?
4. How many 5-digit numbers can be formed from the digits 2, 3, 5, 6. 8, 9 if no digit
can be used more than once in a number? How many even numbers can be formed?
5. Find the value of (i) 6P J • (ii) 12P4 • (iii) sP s•

6. Find the value of (i) 6!, (ii) ~:.


28/PERMUTATIONS AND COMBINATIONS 187

7. In how many ways can 4 consonants and 3 vowels be arranged in a row (a) so that the
3 vowels are always together, (b) so that the first and the last places are occupied by
consonants?
8. In how many ways can four girls and three boys be arranged in a row so that (a) the
boys are always together, (b) the ~ and boys occupy alternate places?
~1Y'h
9. In how many ways can 6 people lJe seated in a motor car if only 2 can occupy the
driver's position?
10. In how many ways can 6 people be arranged in a circle if 2 particular people are
always (a) together,flQ,i separated?
11. Father, mother and 6 children stand in a ring. In how many ways can they be
arranged if father and mother are not to stand together?
",,12. In how many ways can 5 boys and 3 women be arranged (a) in a row, (b) in a circle if
in both cases the women are always to stand together?
13. Four men and four women are to be seated alternately (a) in a row (b) at a round
table. In how many ways can this be done?
14. How many even numbers of 4 digits can be formed with the figures 3, 4, 7, 8 (a) if no
figure is repeated, (b) if repetitions are allowed?
15. How many numbers greater than 4000 can be formed from the figures 3, 5, 7, 8, 9?
(Repetitions not allowed.)
16. In how many ways can the letters of the word permute be arranged if (i) the first and
last places are occupied by consonants, (ii) the vowels and consonants occupy
alternate places?
17. If 6Pr = 120, find the value of r. '

18. The number of arrangements of 2n + 2 different objects taken n at a time is to the


number of arrangements of 2n different objects taken n at a time as 14:5. Find the
value of n.
19. How many numbers of 7 digits can be formed from the digits 1, 2, 3, 4, 5, 6, 7 if
(0 the number begins with 1 and ends with 2, (ii) there are not more than 2 digits
between 1 and 2?
20. If 2nPn =' 8. 2n-1Pn _ 1, find the value of n.
21. In how many ways can 5 different Mathematics books, 4 different Physics books and
2 different Chemistry books be arranged on a shelf if the books in each subject are to
be together?
22. In how many ways can 3 men, 3 women and 3 boys be arranged in a row if the three
boys are to remain together?
23. Find the number of arrangements of the letters in the word Pencils if (i) e is next to I,
Oi) e precedes i, (iii) there are three letters between e and i.
24. In how many ways can the letters of the word principle be arranged? In what
proportion of these arrangements do the letters 'p' come together?

,i 25. In how many ways can the letters in precision be arranged? In how many of these ~
arrangements do the vowels occupy even places? ~.,~"
188 NEW SENIOR MATHS: THREE UNIT COURSE

26. How many arrangements can be made by the letters of the word definition (i) if the
letters i do not occupy the first or last ,place, (ii) if the letters i are together?
~',

,27. How many arrangements of the letters in tomato are there, if the letters 0 are to be
separated?
28. A car can hold 3 people in the front seat and 4 in the back seat. In how many ways can
7 people be seated in the car if 2 particular people must sit in the back seat and 1
particular person is the driver?
29. In h9w many ways can 4 people be accommodated if there are 4 rooms available?
30. In how many ways can 8 oarsmen be seated in an eight-oared boat if 3 can row only
on the stroke side and 3 can row only on the bow side?
31. Prove (i) from the definition of np"
(ii) the formula for np, that n+lp, np, + r. np,_l'
32. Show that np, = n-2p, + 2r. n-2p,_1 + r(r 1) n-2p,_2'

33. In how many ways can 5 men and 5 women b~rranged in a circle so that the men are
separated? In how many ways can this be done if two particular women must not be
next to a particular man?
34. How many arrangements of the letters of the word PARRAMATTA are possible?

28.11 Combinations
In 28.1 a definition of a combination was given and its distinction from a permutation was
illustrated by a simple example. In combinations no regard is paid to the order in which
the elements appear in a subset. There are 6 selections of the four people A, B, C, D taken
two at a time, namely
AB, AC, AD, BC, BD, CD.
Each of these groups contains 2 people, who can be arranged among themselves in 2! ways.
Hence, number of combinations x.2! = number of permutations. If we denote the
number of combinations of 4 people taken 2 at a time by the symbol 4C z or (i), then
4C 2 x 2! 4PZ
. 4C _ 4P2
•• 2 - 2f
4.3
2.1
= 6.

EXAMPLE 8
How many groups of 3 can be selected from the four people A, B, C, D?
The symbol 4C3 or (~ ) denotes the number of 3 subsets which can be formed from a
4-set.
The subsets in this example can be listed by inspection:
ABC, ABD, ACD, BCD.
28/PERMUTATIONS AND COMBINATIONS 189

Consider the subset ABC. These three people can be arranged amongst themselves in 3!
ways to give 6 different arrangements: ABC, ACB, BCA, BAC, CBA, CAB. Similarly for
the other 3 subsets as shown in the followinR table:
Combinations Permutations
ABC ABC, ACB, BCA, BAC, CBA, CAB.
ABD ABD, ADB, BDA, BAD, DBA, DAB.
ACD ACD, ADC, CDA, CAD, DCA, DAC.
BCD BCD, BDC, CDB, CBD, DCB, DBC.
Hence, number of combinations X 3! = number of permutations.
i.e. 4C3 x 3! = 4P3
. 4C _ 4P3
•• 3 - 3!
4.3.2
= 3.2.1
= 4.

28.12 The symbol nCr or(~) _ _ _ _ _ _ _ __

The symbol nCr or ( ;) denotes the number of combinations of n different objects taken r
at a time, i.e. the number of r-subsets which can be formed from an n-set. Each of these
combinations consists of a group of r different elements which can be arranged amongst
themselves in r! ways.
Hence nCr X r! number of arrangements of n different objects taken r at a time.
nPr

"C = nPr
r r!
n(n - l)(n -2) .. . (n - r + 1)
r!

This formula for nCr may also be written in factorial form; if we multiply numerator and
denominator by (n - r)!, we get ...

"C = n(n -l)(n - 2) .. . (n r + l)(n - r)!


r r!(n r)!
n!
r!(n - r)!

7C _ 7.6.5 ~
e.g. 3 - or 3!4!
= 35.

EXAMPLE 9
In how many ways. can a set of 4 people be selected from 10 people, (a) if the eldest person
is included in each set, (b) if the eldest person is excluded from each set?
What proportion of all possible sets of 4 contains the eldest person.
190 NEW SENIOR MATHS: THREE UNIT COURSE

(a) Since the eldest person is to be included, we have to select 3 out of 9 people.
:. Number of selections = 9C3
9.8.7
3.2.1
84.
(b) Since the eldest person is to be excluded, we have to select 4 out of 9 people.
:. Number of selections = 9C4
9.8.7.6
4.3.2.1
= 126.
The number of possible sets of 4 which can be selected without restriction
IOC
4
10.9.8.7
4.3.2.1
= 210.

Of these 210 sets, 84 of them contain the eldest person.


84
:. Proportion
210
2
= 50r40OJo.

The number of combinations of n objects taken r at a time is equal to the number of


combinations of n objects taken (n - r) at a time; that is

For each set of r objects selected, there is left behind a set containing (n - r) objects
and thus there are as many sets containing (n - r) objects as there are containing
r objects. Alternatively,
n!
nCn _ r = ------~~-===~-
(n - r)! (n - n - r)!
n!
(n - r)!r!
= nCr
If, from a set of 9 people we select a set of 6, a set containing 3 people is left behind.
:. 9C6 = 9C3
9.8.7
3.2.1
84.

EXAMPLE 10
A class consists of 15 students of whom 5 are prefects. How many committees of 8 can be
formed, each containing (a) exactly 2 prefects, (b) at least 2 prefects?
(a) The class contains 5 prefects and 10 non-prefects from whom we wish to select
2 prefects and 6 non-prefects.
2 prefects can be selected from 5 in 5C2 ways.
6 non-prefects can be selected from 10 in IOC6 ways.
28/PERMUTATIONS AND COMBINATIONS 191

:. Number of committees =sCz • IOC6


5.4 10.9.8.7
2.1'4.3.2.1
2100.
5CZ is multiplied by IOC6 because the multiplication principle is involved in this case.
(b) The committees may consist of:
2 prefects and 6 non-prefects
or 3 prefects and 5 non-prefects
or 4 prefects and 4 non-prefects
or 5 prefects and 3 non-prefects
:. Number of possible committees
= sCz • IOC6 + 5C). IOCs + 5C4 • IOC4 + 5CS ' IOC) (addition because operations are
mutually exclusive).
_ 5.4 10.9.8.7 + 5.4 lQ.9.8.7.6 + 5.10.9.8.7 + 10.9.8
- 2. 1 . 4. 3. 2 . 1 2. 1 . 5.4. 3 .2. 1 4. 3 . 2. 1 3 .2 . 1
= 2100 + 2520 + 1050 + 120
= 5790.

EXAMPLE 11
In how many ways can 4 men and 3 boys be arranged in a row, if there are 8 men and
5 boys to select from?
4 men and 3 boys can be selected from 8 men and 5 boys in 8C4 • 5C3 ways. Each set
consists of 7 people who can be arranged amongst themselves in 7! ways.
:. Number of arrangements 8C4 • 5C3 • 7!
= 8.7.6.5 5.4 7'
4.3.2.1'2.1' .
= 3,528,000.

EXAMPLE 12
Prove that
m+nCr mCr + mCr_ 1 nC I + mCr_Z ncz + ... + mC2 nCr_Z + mCI nCr_ 1 + nCr
The symbol m+nCr denotes the number of selections, each containing r objects, when we
have two sets containing m and n objects respectively to select from.
The r objects can be selected as follows (assuming r is not greater than either m or n):
r objects from m and none from n in mCr ways
or (r 1) objects from Tn and 1 from n in mCr _ 1 nC I ways
or (r - 2) objects from m and 2 from n in mCr _ 2 nCz ways
and so on.
Since these selections are mutually exclusive and in this case exhaustive, the result
follows.
(See example 9 in the chapter on the Binomial Theorem.)

EXAMPLE 13
Prove that HICk nCk + nCk _ 1
The symbol n+ IC k denotes the number of selections, each containing k objects, when we
have two sets containing nand 1 objects respectively to select from.
The k objects can be selected as follows, assuming k ~ n:
192 NEW SENIOR MATHS: THREE UNIT COURSE

k objects from n and not the one in nCklCo = nC k ways


(k - 1) objects from n and the other one object in nCk_IIC I = nC k _ 1 ways
since ICo and IC I both equal 1.
These selections are mutually exclusive and exhaustive and hence the result follows.
Alternatively,
n! + n!
k!(n - k)! (k - 1)!(n - k + 1)!
(n - k + l)n! + kn!
k!(n - k + I)!
(n + l)n!
k!(n - k + 1)!
(n + I)!
k!(n - k + I)!
= n+ICk ·
The symbol nCr denotes the number of r-subsets in a given n-set, where 0 ~ r ~ n. If no
specific value of r is stated, r can assume any value between 0 and n inclusive. Hence the
number of subsets, each containing at least one element, i.e. the number of non-empty
subsets in an n-set is
nC I + nC2 + nC3 + ... + nCr
From a set of n objects, each object can be dealt with in 2 ways, for it may be included
or rejected. Hence the number of ways of dealing with the n objects is
2 x 2 x 2 ... to n factors
= 2n
But this includes the case when all n objects are rejected. Hence the number of
selections of at least one object is 2n - 1.
:. nC I + nC2 + ... + nCn = 2n - 1
(See the relation between binomial coefficients in the chapter on the Binomial Theorem.)

EXAMPLE 14
How many selections can be made from 5 different books, taking any number at a time?
Reasoning as above, the number of selections is 2S - 1, Le. 31; or the books may be
selected one at a time, two at a time, ... five at a time.
:. Number of selections = SCI + sC2 + sC3 + sC4 + sCs
5 + 10 + 10 + 5 + 1
= 31.

EXAMPLE 15
In how many ways can 6 men be partitioned into 3 sets containing (i) 3, 2 and 1 men
respectively, (ii) 2 in each set?
(i) 3 men can be selected from 6 in 6C3 ways. From the remaining 3 men, 2 can be selected
in 3C2 ways. This leaves one man for the remaining set.
:. Number of ways = 6C3 . 3C2
6.5.4 3
3.2.1·
60.
28/PERMUTATIONS AND COMBINATIONS 193

(ii) 2 men can be selected from 6 in 6Cz ways. From the remaining 4 men, 2 can be selected
in 4C2 ways. This leaves 2 men for the remaining set. The number of selections is
6C2 • 4C2 if the three sets of 2 are distinguishable i.e. if they were put into 3 sets labelled
A, Band C. If however no regard is paid to the order of the sets, this answer is divided
by 3! Why?

EXERCISES 28(b)
1. In how many ways can 3 books be selected from 8 different books?
2. In how many ways can a set of 2 boys and 3 girls be selected from 5 boys and 4 girls?
3. How many different selections can be made by taking 3 of the digits 4, 5,6, 7, 8, 9?
4. In how many ways can a committee of 4 men and 5 boys be formed from 8 men and
7 boys?
5. From 8 soldiers, 7 sailors and 5 airmen how many sets can be formed each containing
5 soldiers, 4 sailors and 3 airmen?
6. In how many ways can 8 boys be divided into two sets containing 5 and 3 respectively?
7. A committee of 6 is to be selected from 10 people of whom A and B are two. How
many committees can be formed (i) containing both A and B, (ii) excluding A if B is
included?
8. In how many ways can 3 cards be selected from a pack of 52 playing cards if (i) at
least one of them is an ace, (ii) not more than one is an ace?
9. In how many ways can a committee of 3 men and 4 women be chosen from 6 men and
7 women? What proI?ortion of these committees contain a particular man and
2 particular women?
10. A committee of 7 politicians is chosen from 10 liberal members, 8 labor members and
5 independents. In how many ways can this be done so as to include exactly
1 independent and at least 3 liberal members and at least 1 labor member?
11. An eleven is to be chosen from 15 cricketers of whom 5 are bowlers only, 2 others are
wicketkeepers only and the rest are batsmen only. How many possible elevens can be
chosen which contain (i) 4 bowlers, 1 wicket keeper and 6 batsmen, (ii) at least
4 bowlers and at· least 1 wicketkeeper?
12. From 7 teachers and 5 pupils a committee of7 is to be formed. How many committees
can be selected if both teachers and pupils are represented and the teachers are in a
majority?
;. L )
13. From 4 oranges, 3 bananas and 2 apples, how many selections of 5 pieces of fruit can
be made, taking at least one of each kind?
14. In Lotto, a frame contains the numbers 1 to 40 and we can select any 6 of these
numbers. How many such combinations are possible?
15. In TAB betting, the 'trifecta' pays on the first three horses in correct order, the
'Quinella' pays on the first two horses in either order. In a 10 horse race what is the
possible number of (a) trifecta combinations (b) Qui nella combinations?
t
16. In how many ways can a jury of 12 be chosen from 10 mtm and 7 women so that there
are at least 6 men and not more than 4 women on each jury?
,
194 NEW SENIORMATHS: THREE UNIT COURSE

17. In how many ways can a set of 3 or more be selected from 9 people?
@)In how many ways can n things be shared between 2 people.
19. In how many ways can a committee of 3 men and 4 boys be chosen from 7 men and
6 boys so as not to include the youngest boy if the eldest man is serving?
20. How many (i) selections, (ii) arrangements consisting of 3 consonants and 2 vowels
can be made from 8 consonants and 4 vowels?
21. In how many ways can 4 Physics books and 3 Mathematics books be arranged on a
shelf if a' selection is made from 6 different Physics books and 5 different
Mathematics books? In how many of these arrangements are the Physics books
together?
22. In how many ways can 3 boys and 2 girls be arranged in a row if a selection is made
from 5 boys and 4 girls? In how many of these arrangements does a boy occupy the
middle position?
23. How many words (arrangement of letters), containing 3 consonants and 2 vowels, can
be formed from the letters of the word promise?
24. From the definition of nCr prove each of the following:
(a) nCr + nCr+1 = n+ICr+1
(b) nCk + 2nC k _ 1 + nCk _ 2 = n+2Ck
(e) m+nC3 = mC3 + mC2 nC I + mCI nC2 + nC3
(d) n+JCr = nCr + 3 nCr_1 + 3 nC r_2 + nCr_3

(e) nCr =. n r + I . nCr_I


r
25. If nC6 nC4 , find the value of n.
26. The ratio of the number of combinations of (2n + 2) different objects taken n at a
time to the number of combinations of (2n - 2) different objects taken n at a time is
99: 7. Find the value of n.
27. In how many ways can 9 books be distributed amongst a man, a woman and a child,
if the man receives 4, the woman 3 and the child 2?
28. In how many ways can 8 boys be divided into two unequal sets?
29. In how many ways can 8 girls be divided into 4 sets of 2?

28.13 Probability associated with permutations and


combinations ______________

(a) Permutations

EXAMPLE 16
Eight people of whom A and B are two, arrange themselves at random in a straight line.
What is the probability that
(a) A and B are next to each other,
(b) A and B occupy the end positions,
(e) there are at least 3 people between A and B?
(a) A sample space of the experiment of arranging 8 people in a row may be considered as
consisting of 8! points, each equally weighted. '
i..,.
28/PERMUTATlONS AND COMBINATIONS 195

Since A and B are to be next to each other, consider them as one unit. This unit and
the other six people can be arranged in a row in 7! ways. However, A and B can be
arranged amongst themselves in 2 ways, either AB or BA. Hence 'the number of
favourable arrangements is 2. 7!
number of favourable arrangements
Hence P (A and B are next to each other)
number of possible arrangements
2.7!
8!
2.7!
8.7!
1
- 4

(b) A and B can be arranged in 2 ways, A at one end and B at the other end and vice versa.
When A and B are arranged in anyone of these two ways, the remaining 6 places can
be filled in 6! ways.
Hence the number of favourable arrangements is 2. 6!

Hence P (A and B occupy end positions) = 2. 6!


2.6!
8.7.61
I
28'
(c) With A in the first place, B can occupy 5th, 6th, 7th or 8th position: 4 arrangements of
A and B.
With A in the second place, B can occupy 6th, 7th or 8th place: another
3 arrangements of A and B; and so on. Hence there are 4 + 3 + 2 + 1 .+ 1 + 2 + 3 + 4
i.e. 20 different arrangements of A and B. Addition in this case because the
arrangements are mutually exclusive. For each of these 20 arrangements there are
6 places left which can be filled in 6! ways. Hence the number of favourable arrange-
ments is 20. 6!
20.6!
Hence P (at least 3 people between A and B)
8!
20.6!
8.7.6!
5
= 14 .

EXAMPLE 17
Mother, father and four children arrange themselves at random in a circle. What is the
probability that mother and father are not together?
The sample space of the experiment of arranging 6'people in a circle may be considered
as consisting of ~!. points, each equally weighted.
The number o(arrangemtmts of the six people in a circle so that mother and father are
together is 2. 4! Thus the number of arrangements with mother and father not together is
5! - 2.4! i.e. 3 . 4!
196 NEW SENIOR MA THS: THREE UNIT COURSE

Hence P(mother and father not together) = 3 5~!


3
5'
(b) Combinations (Sampling without replacement)

EXAMPLE 18
An urn contains 9 distinguishable cubes of which 3 are white and 6 black. Two cubes are
drawn at random without replacement. Calculate the probability that both cubes are
black.
A sample space of the experiment of drawing 2 cubes without replacement from an urn
containing 9 cubes may be considered as consisting of 9C2 points, each equally weighted,
6CZ points corresponding to the event 'both cubes are black'.
6C2
Hence P(both cubes are black) 9C
2
6.5 2.1
. '9.8
5
12'
Alternatively
Let A denote the event 'black cube in the first draw' and let B denote'the event 'black
cube in the second draw'.
Pr(A) ~ assuming that each cube is equally likely to be drawn. The outcome of the
second draw is dependent on the outcome of the first draw. If event A has happened,
there are 8 cubes left of which 5 are black.
Hence P(BIA) ~
Now P(AB) = P(A).P(BIA)
6 5
= 9'S
5
12'
EXAMPLE 19
A group of 9 people contains 3 males and 6 females. A random sample of 5 is selected.
What is the probability that it contains (a) exactly 2 males, (b) not more than 2 males.
(a) The statement 'exactly 2 males' implies that the sample of 5 contains 2 males and
3 females. A sample space of the experiment of selecting 5 people from 9 may be
considered as consisting of 9Cs equally weighted points; 3CZ • 6C3 points are favourable
to the event '2 males and 3 females'.
Hence P(2 males and 3 females)

3 6.5.4 4.3.2.1
. . . '9,8.7.6
10
28/PERMUTATIONS AND COMBINATIONS 197

Alternative method.
3
P(first person selected is male)
9
2
P(second person is male I first is male)
8
6
P(third person is female I first two are male) =7
5
P(fourth person is female I male, male, female)
6
P(fifth person is female I male, male, female, female) = ~
3 2 6 5 4
Hence P(selecting 2 males and 3 females in this order) 9' 8' 7 . 6' 5
1
21 .
The problem, however, does not restrict the selection to this order. The two males
may be selected in any two of the five selections in 5CZ ways. 1
Hence P(2 males and 3 females) 5CZ ' 21
10
21 .

(b) The statement 'not more than 2 males' implies 0 males and 5 females, or 1 male and
4 females or 2 males and 3 females.
o males and 5 females can be selected in 6C5 ways, 1 male and 4 females in 3Cl .6C4
ways and 2 males and 3 females in 3C2 • 6C3 ways. These are mutually exclusive events.
Hence P(not more than 2 males) =

6 + 45 + 60

37
42'
EXERCISES 28(c}
fl 1. Five cards are drawn at random from a pack of 52 playing cards. What is the
probability that they are all from the same suit?
r 2. A bag contains 5 red balls and 4 white balls. Three balls are withdrawn without
replacement.· Find the probability of drawing at least 2 red balls.
3. An urn contains 3 white balls, 4 red balls and 5 black balls. Three balls are drawn at
random. What is the probability that they are
(a) different colours (b) the same colour?
4. From 7 teachers and 5 pupils, a random selection of 7 is made. What is the probability
that it contains at least 4 teachers?
I 5. A committee of 3 men and 4 women is to be chosen from 6 men and 7 women. What
't, is the probability that it contains a particular man and a particular woman?
6. Five cards are drawn from a pack of 52 playing cards. What is the probability of
drawing at least 3 aces?
198 NEW SENIOR MATHS: THREE UNIT COURSE

7. Eight people are to be divided into two groups. What is the probability that there will
be 4 in each group?
8. The letters of the word 'promise' are arranged in a row. What is the probability that
there are 3 letters between p and r?
9. Six people arrange themselves at random in a circle. What is the probability that the
tallest and the shortest are together?
10. Four men and three boys are arranged in a straight line. What is the probability that
the men and the boys occupy alternate positions?
11. The number plates of a motor car contain 3 letters of the alphabet followed by
3 numerals. How many such number plates can be made? What proportion of these
would contain 3 letters the same and 3 numerals the same?
12. The letters of the word 'independence' are arranged in a row. What is the probability
of the letter~ 'e' being together?
13. A committee of 6 is to be selected from 10 people. What is the probability of the
youngest and oldest being on the same committee?
14. A party of twelve, of whom A and B are two, are arranged at random in a straight
line. What is the probability that A and B are not next to one another?
15. A j~ contains 5 red cubes and 4 white cubes. Three cubes are drawn in succession
without replacement. What is the probability that
(a) the first two cubes are red and the third one white,
(b) any two cubes are red, and one is white?
16. A carton contains 15 transistors of which 5 are defective. If a random sample of
6 transistors is drawn from the carton (without replacement), determine the
probability of 0, 1, 2, 3, 4; 5 defective valves in the sample.
17. An urn contains 12 distinguishable cubes of which 5 are red and the remainder black.
If a random sample of 6 cubes is drawn without replacement, calculate the
probabilities of 0, 1, 2, 3,4, 5 red cubes in the sample.
18. Two boxes each contain eight balls. In box A there are 3 black and 5 white balls, in
box B there are 1 black and 7 white balls. For each box find the probability that two
balls chosen at random without replacement will both be white.
19. A sample of 3 coins is selected without replacement from 8 coins, consisting of 4 five-
cent coins and 4 ten-cent coins. What is the probability that the sample contains at
least 2 five-cent coins?
20. A hand of 5 cards is dealt from a pack of 52 playing cards. What is the probability
that it will contain at least one ace?
21. From a group of 12 people of whom 8 are males and 4 are females, a sample of 4
is selected at random. What is the probability that the sample contains at least
2 females?
22. Urn A contains 6 white and 4 black balls. Urn B contains 2 white and 2 black balls.
From urn A two balls are selected at random and placed in urn B. From urn B two
balls are then selected at random. What is the probability that exactly one of these
two balls is white? (/
,.
28/PERMUTATIONS AND COMBINATIONS 199

23. From a set of 10 cards numbered 1 to 10, two cards are drawn without replacement.
What is the probability that (a) both numbers are even, (b) one is even and the other is
odd, (c) the sum of the two numbers is 12, (d) both numbers are even and the sum of
Jhe two numbers is 12?
24. The letters of the word tomato are arranged in a row. What is the probability that
(a) the two letters 0 are together, (b) the two letters 0 are not together?
25. The letters of the word Tuesday are arranged at random in a row. What is the
probability that (a) the vowels and consonants occupy alternate positions (b) the
vowels are together (c) the vowels are together and the letter T occupies the first
place?
26. Four girls and four boys arrange themselves at random in (a) a row, (b) a circle. What
is the probability in each case that the girls and the boys occupy alternate positions?
27. Six people, of whom A and B are two, arrange themselves at random in a row. What
is the probability that
(a) A and B occupy the end positions,
(b) A and B are not next to each other,
(c) there are at least three people between A and B?
28. _The digits 1, 2, 3, 4, 5, 6 are used to form numbers which contain two or more digits.
(The same digit cannot be used more than once in a number). What proportion of
such numbers are even numbers?
29. Five cards are selected without replacement from a pack of 52 playing cards. What is
the probability of (a) exactly 3 hearts, (b) 4 aces, (c) no hearts?

/
CHAPTER 29
Binomial
Theorem
and
Binomial
Probability
29.1 Expansion of (1 + x)n _ _ _ _ _ _ _ _ __
It can be shown by actual multiplication that:
n = 0, (1 + x)O = I
n = 1, (1 + X)I = 1 + x
n 2, (1 + X)2 = 1 + 2x + X2
n = 3, (1 + X)3 = 1 + 3x + 3X2 + x3
n = 4, (1 + X)4 1 + 4x + 6x 2 + 4x3 + x4
n = 5, (1 + X)5 1 + 5x + IOx 2 + IOx 3 +
+ x5 5x4
n = 6, (1 + X)6 = 1 + 6x + 15x2 + 20x3 ++ 6x 5 + x 6
15x4
We observe that, in each case, the expansion of (1 + x)n for
n = 1, 2, 3, ... is a
polynomial of degree n in the variable x.
The coefficients of the successive powers of 1 + x can be arranged in a triangular
pattern, called Pascal's triangle.
(1 + x)O
(l + X)I 1
(1 + X)2 1 2
(1 + X)3 3 3 1
(1 + X)4 4,,@-,,"6 4
(1 + X)5 5~ 10 10 5 1
(1 + X)6 6 @~ ~ 15 6
29/BINOMIAL THEOREM AND PROBABILITY 201

Pascal's triangle has the following interesting properties:


(a) The first and last number in each row is 1.
(b) Every other number is the sum of the two numbers to its left and right in the row
above it e.g.
10 = 4 + 6, 15 = 5 + 10, 20 = 10 + 10.

These properties enable us now to write down the expansion of (1 + x)n for n = 7,8,
9, ...
n = 7, (1 + X)7 = 1 + 7x + 21x2 + 35x3 + 35x4 + 2lx 5 + 7x 6 + x 7
n = 8, (1 + X)B = 1 + 8x + 28x 2 + 56x 3 + 70x 4 + 56x 5 + 28x6 + 8x 7 + x B
The disadvantage, of course, in using Pascal's triangle is that we cannot write down
the expansion of, say, (1 + X)B without the expansion of (1 + X)7 which in turn
requires the expansion of (1 + X)6 and so on. Hence, we shall now establish a
relationship for the co-efficients of the general case (1 + x)n. Writing 'C' for 'co-
efficient', the power of x as a right subscript r and the power n as a left superscript,
nCr is, by definition, the coefficient of xr in (1 + x)n.
(1 + x)n = nCo + nC1x + nC2X 2 + ... + nCrxr + ... + nCnxn

If, in place of x, we write ~ and multiply both sides by an, we get:


a

n .
= EnCran-rb r
r=O

29.2 Proof of the Pascal triangle relations _ __


There are two separate relations:
(a) for the outer coefficients
(b) for the other coefficients

(3) In the expansion of (a + b)n, put a = 1 and b = 0; hence nCo = 1.


Put a = 0 and b ,; 1 and hence nCn = 1.
(b) (1 + x)n-l n-lCo + n-lC1x + ... + n-lCrxr + ... + n-lCn_1xn - 1
=
x(1 + x)n-l
= n-lCox + ... + n-lCr_1xr + ... + n-lCn_2x n- 1 + n-lCn_1xn
By addition (1 + x)n-l + x(1 + x)n-l = (1 + x)(1 + x)n-l = (1 + x)n
:. (1 + x)n = n-lCo + (n-lCo + n-lC1)x + ... + (n-lCr_1 + n-lCr)xr +
... + (n-lCn_2 + n-lCn_1)xn - 1 + n-lCn_1xn
In the previous expansion of (1 + x)n the co-efficient of xr was nCr and in this expansion
of (1 + x)n it is n-lCr_ 1 + n-lCr. Hence, equating co-efficients of like powers of x, we get:

I nCr = n-lCr_ 1 + n-lCr!or 1 :os;; r :os;; n - 1 I

We shall see when we establish a formula for nCr that this is the relationship we saw in
Pascal's triangle
e.g. 10 = 4 + 6, 15 = 5 + 10, 20 = 10 + 10

._.- -.----.~~-
202 NEW SENIOR MATHS: THREE UNIT COURSE

29.3 Form ula for nC,


(Proof by induction)

When n = 4, (1 + X)4 = 1 + 4x + 6x l + 4x 3 + X4.


The ratios of successive coefficients are:
4 n6 3 n-14 2 n 2 1 n 3
1 = .; 4 1: = -2-; 6" = 3" and 4
Thus the co-efficients in this expansion are from left to right:
1 '!. '!.. n - 1 '!.. n - 1 . ~ and'!. . n 1. n - 2 . n - 3
, l' 1 2' 1 3 1 3
This leads us to guess that perhaps
"C = n(n - 1)(n 2) ... (n - , + 1), I .,. r .,. n
r 1.2.3 .... r
Call this statement Sen).
The coefficient nCo has already been shown to equal 1.
Step 1 (1 + X)2 = 1 + 2x + Xl = 2CO + 2C I X + 2C2 X2.
2eo = I, 2C ~
2C _ 2. 1
l' 2 -
I •

The statement Sen) is true for n = 2.


Step 2 Assume sen - 1) is true i.e. assume
n-IC = (n - l)(n 2)(n 3) ... (n r) is true
r 1.2.3, ... r
and deduce from it that Sen) is true. 1
n-
Put r = 1 and assume then that II-ICI = --1-'

We know from 29.2 that n ICO 1. Adding these two and using the Pascal triangle
relation established in 29.2.
nCI = II-ICO + n-ICI, we obtain
nCI = 1+n 1

7in agreement with Sen)


For r?J:. 2, we-have r - 1 ?J:. 1 and assume the truth of the relation:
n-IC (n-l)(n 2) ... (n-r+l) h'h' t fS( 1)
I-I = 1. 2.3 .... (r _ 1) w IC IS par 0 n
From the Pascal triangle relation, we get:
nC,. = n-ICr_ 1 + n-ICr
(n l)(n - 2) ... (n r + 1) + (n - l)(n 2) ... (n - r)
1.2.3 .... (r 1) 1.2.3 .... r
(n - l)(n - 2) ... (n - r + I) [1 + (n - r)]
1.2.3 .... (r-l) r
n(n - t)(n 2) ... (n - r + 1). 1 n r n
I . 2 . 3. ... r - Slllce + r -,.
This is the statement Sen) which is now proved from r ?J:. 2 and has been proved for r = 1
above.
Note: This formula for nCr agrees with the formula established in Chapter 28 and with
this knowledge we can consider an alternative approach .
29/BINOMIAL THEOREM AND PROBABILITY 203

29.4 Binomial theorem


(Alternative approach) _ _ _ _ _ _ _ __
The expansion of (1 + x)n is the product of n factors, each of which is (1 + x). The term
containing xr is obtained by selecting x out of any r of the n factors and lout of the
remaining (n - r) factors. The co-efficient of xr then is equal to the number of ways of
selecting r x's when we have n x's to select from. Hence the co-efficient of xr is nCr and by
putting r = 0, 1, 2, 3 ... n in succession we obtain the coefficient of each term. Thus:

n
= E nC,xr
r=O

The co-efficients nCo and nCn are both equal to one.


Putting x = ~ and multiplying both sides by an we get the expansion of (a + b )n:

Note (a) The expansion of (a + b)n contains (n + 1) terms of which nCran-rb r is the
(r + l)th term and is called a general term.
(b) If we write -b in place of b we get
(a - b)n = an - nC1an-lb + nC2 a n- 2b 2 + ... + nCran-r(-b), + ... + (-b)n
n
= E "Cran-r(-b),
r=O
The terms are alternately positive and negative.
(c) Since (a + b)n = (b + a)n and nCr nCn_" any term from one end of the
expansion of (a + b)n is equal to the same term counting from the opposite end
of the expansion of (b + a)n,

EXAMPLE 1
Write down the expansion of (2x - y)5.
(2x - y)5 (2x)5 SCI (2X)4y + 5C2 (2x)3y2 5C 3 (2x)2y3 + 5C4 (2X)y4
= 32x 5 80x4y + 80X 3y 2 40X2y 3 + lOxy4 y5

EXAMPLE 2
Write down the 6th term in the expansion of (3x 2y)9
6th term 9CS (3X)4( _2y)5
9 . 8 . 7 ,6 34 25 •..4 5
4.3.2.1' . . AY
=- 326 592x4y5.
204 NEW SENIOR MATHS: THREE UNIT COURSE

EXAMPLE 3
Calculate (a) the term independent of x (constant term); (b) the coefficient of x 8 in the
expansion of

(2X 3
1t
( 2x3- ~y2 = (2X 3)12 - 12CI (2X3)1 {1)+ 12C2(2X 3)IOGY - ...
First term contains x J.6.
Second term contains x 3 2.
Third term contains X 28 •
Hence, following this pattern, the tenth term contains XO and the eighth term contains x8 •
:. (a) Constant term = tenth term
I2C9 (2x 3)3 (-1J
12 . II . 10 .23

-1760.
(b) Coefficient of x 8 coefficient of eighth term
12C7 ·25
25344.
Alternative method:
The(r + l)thterm = 12Cr(2x3)12-r. (-~)'
Power of x :::: 36 3r - r
= 36 4r
:. (a) 36 4r = 0
r = 9

Constant term I2C9 (2x 3)3 ( -1Y


1760.
(b) 36 - 4r 8
r 7.
Coefficient of x = 12C7 .25
8

-25344.
EXAMPLE 4

Find the coefficient of x in the expansion of. (1


x + x)'¥(x X!.- )3.

(x +D4(X -1Y [X4 +4CIX 3(1) + C2X 2(1Y + C3X (1Y 4 4 +(1JJ x
. [x 3C X2G ) + C2X GY -
3
- I 3 GJl
The first term in the first expansion multiplied by the fourth term in the second
expansion yields a term in x; similarly the second term in the first expansion multiplied by
the third term in the second expansion yields a term in x and so on .
.'. Coefficient of x 1 + 4CI ;3C2 - 4C2 • 3CI + 4C3
- 1 + 12 ...:. 18 + 4
-3.
29/BINOMIAL THEOREM AND PROBABILITY 205

Alternative method:
(x + ~y(X ~y (x + ~)~2 - ;iy
= (x +~)~X2)3 3CI(XZ)z(;Z) + 3C Z(X Z)(:zY - (;zYJ
Only! multiplied by - 3CI (X Z )2G2) yields x.
:. Coefficient of x = _JC I
3

EXAMPLE 5
Three consecutive coefficients in the expansion of (1 + x)n are in the ratio 6 : 3 : 1. Find
the value of n and state which terms possess these coefficients.
Let the three consecutive coefficients be nCro nCr+l • nCr+2
nC 6 '1C
Hence __ r - and ---L±.l = 3
nCr+1 3 nCr+Z
n! x (r + 1)!(n r I)!
2
r! (n r)! n!
n! (r + 2)! (n - r - 2)!
and (r + 1)! (n - r - 1)! x n! = 3
i.e. r + 1 = 2 and r + 2 = 3
n-r n-r-
Hence 2n - 3r = 1 (i)
and 3n - 4r 5 (ii)
Solve (i) and (ii) for nand r.
:. n 11
r 7
Theterms possessing these coefficients are II.C7 X7, IICSXS and IIC9X9.

EXAMPLE 6
Find the ratio of the (r + l)th term to the rth term in the expansion of (a + b)n.
Denote the (r + l)th term by [r+1 and the rth term by Tr •
Tr+1 _ nCron-rb r
Tr - nCr-l on-r+lbr- 1
n! x (r 1)!(n - r + I)! x b
r!(n-r)!n! 0
n-r+l b
r 0
This provides us with the ratio of successive terms in the expansion of (0 + b)n and
enables us to find the greatest term or the greatest coefficient in the binomial expansion.

n-r+l b
r 0

EXAMPLE 7
Find the greatest coefficient in the expansion of (5 + 2X)IZ.
Tr+1 = 12Cr 512 - r (2x)' and Tr 12Cr _ 1 513 - r (2xy-1
206 NEW SENIOR MATHS: THREE UNIT COURSE

Let C r+ 1 and Cr be the coefficients of Tr+ 1 and Tr respectively. Then


C r+ 1 _ 12Cr5 12 - r 2 r
C -
r
12Cr _ 1 513 - r 2r - 1
12! x (r - I)! (13 - r)! x 2
r! (12 - r)! 12! 5
26 - 2r
5r
Cr + 1 ;::. C r if 26 - 2r ;::. 5r
26 ;::. 7r
r ~ 3~.
If r = 3, C 4 > C 3 If r = 4, C 5 < C 4
If r = 2, C 3 > C2 and so on If r = 5, C6 < C 5 and so on
:. C 4 > C 3 > C2 :. C 4 > C 5 > C6
Hence greatest co-efficient is C4 when r = 3.
C4 = 12C3 59 . 23
= 5 1°.25.11

EXAMPLE 8
1
Find the numerically greatest terms in the expansion of (1 - 2X)9 when x = 3.
Since we are interested only in the numerically greatest term, we can' consider the
expansion of (1 + 2X)9
Using the result of example 6,
Tr+ 1 _ n - r + 1 b
Tr - r a
20 - 2r . 2
---:--- SInce n = 9, a = 1, b = -3.
3r

Tr + l ;::. Tr 1·f 20 3r
- 2r ;;;. 1
"..
i.e. 20 - 2r ;::. 3r
20 ;::. 5r
r~4

Hence TI < T2 < T3 < T4 = T5 > T6 > T7 .••

The terms increase in magnitude until the 4th term. The 4th term is equal to the 5th term
and thereafter they decrease in magnitude:

T = 9C (~)4 = 224
5 4 3 9

T4 = 9C 3 GY = 2~4
The 4th term and the 5th term are the numerically greatest terms.
29/BINOMIAL THEOREM AND PROBABILITY 207

29.5 Relationship between binomial


coefficients _______________

+ x)n = (~)+(7)x
(1 +(~)X2 + ... +(~)xn
(i) In this identity, put x = 1.
:.2n (~)+(7)+(~)+ ... +(~)
= the sum of the binomial coefficients.
= ~(~) where(~)= nCr
2n - 1 = G)+(~) +... +(~ ) since ( ~)1.
total number of combinations of n things taking at least one object at a time.

= ~(;)
(ii) Put x = -1

i.e. the sum of the coefficients of the odd terms


=;= the sum of the coefficients of the even terms
2n - l •

EXAMPLE 9
By expanding both sides of the identity
(1 +x)m n {1 +x)m(1 +x)n,
+
show that
(m : n) (~) +(r m1}(7) +(r ~ 2) {~)+ ...
=

(;).(r ~ 2) + (7).(r ~ 1) +(;)


+
(l +x)m n= 1 +(m 7n)x +... +(m : n)xr +
+

xr =. (m : n).
Coefficient of
.,
(1 +x)m(l +x)n [1 +(7)x +.. , +(~)xr +... J

x [I +(~)x +... +(;)xr +... J

Coefficient of x' = (~) +(r ~ 1) (7) +(r ~ 2) (~) +... +(7) ~ ~ 1) +(~).
208 NEW SENIOR MATHS: THREE UNIT COURSE

Since the coefficient of xr in both expansions is the same, the result follows.
See example 12 in Permutations and Combinations. (Chapter 28)

EXERCISES 29(a)
Write down the expansions of: (1. to 9.)
2. (2x + 3y)4

a 3
5. e3 - 2 bX
8. (~2 - 3b J
Write down and simplify: (10. to 14.)

10. The 4th term of ( ; + 3n)8 11. The 7th term of (23 -
x
ixY I

12. The 5th term Of( a _ 2:)7 13. The 4th term of (x - ~J
14. The (k + 2)th term of (a + b)2n.
Find the coefficient of: (15. to 22.)

15~ X2 in (x + ~y 16. Xl7 in (2x 2 - 3X)12

17. a 5b 4 in (3a - ~J 18. a9 in 2a - ( a3)13


19 .. x- 5in (X3 -' ;2),° 20 . . (XI/2
X- 3/ 2 III -
2 - -
4
X 114
)9
21. x r + 1 in (1 + x)n-I 22. x- 4 in (ax5/2 - bX- 3/ 2)8
Find the constant term, i.e. the term independent of x in the expansion of: (23. to 26.)

23. (2X - ;2 y 24. (X2 - ;4 y2


25. e;2 + 3:Y 26',(2X + ~r
Find the middle term or terms in the expansio~ ~27. to 28.)
27. (X2 - 2~Yo 28. (3a ~ ~7
29. Simplify (--./3 + 1)6 + (--./3 - 1)6
30. Simplify (~ + 1)5 - (~ - 1)5
Express each of the f?Uowing as a binomial and evaluate: (31. to 33.)
;... \

31. 1·018 t05 significant figures. . 32. 2.2 9 to the neareslinteger.


33. (i) 0.99 10 to 5 decimal places. (ii) 0.998 20 to 6 decimal places.

34. In the expansion of (x - ~y(x + ~)8 find the coefficient of:


(i) x lO (ii) x7 (iii) x-8

35. Find the coefficient of a-2 in the expansion of (a J J.


+ ~ (a - ~

- - - - - - - - - - - - --_._-----
29/BINOMIAL THEOREM AND PROBABILITY 209

36. By means of the factors of X2 - 3x + 2, expand (x 2 - 3x + 2)6 in ascending powers


of x as far as the term in Xl.
37. Write down the expansion of (1 + 2x)(l X)8 as far as the term in X4 and use it to
evaluate 1·4 x 0-88 approximately.
38. In the expansion of (2 + 3x)n the coefficients of Xl and X4 are in the ratio 8: IS'.
Find n.
39. In the expansion of (x + a)3(x + b)5, the coefficients of x 7 and x 6 are 9 and 12
respectively. Find the values of a and b.
40. Find the value of r if the coefficients of the rth term from the beginning and the rth
term from the end of (2x + 3)15 are in the ratio 8:27.
41. Find n if the coefficients of the 2nd, 3rd and 4th terms in the expansion of (1 + x)n
are successive terms of an arithmetic sequence.
42. In the expansion of (1 + x)n, the ratios of three consecutive coefficients are 6 : 14: 21.
Find the value of n. \
43. T>he 2nd, 3rd and 4th terms in the expansion of (a + b)n are 12, 60 and 160
respectively. Find the values of a, band n.
44. Find the ratio of the (r +l)th term to the rth term in the expansions of:
(i)(2x + 3y)1O (ii) (3a b)8
45. Find which are the numerically greatest terms in the expansion of the following:
(i) (3 + 4X)12 if x ~ (ii) (1 2X)9 if x = 3
(iii) (2x + y)lO when x = 4, y = -S (iv) (2x + y)12 when x 4, y -S
1
(v) (1 3x)1 when x = 4 (vi) (2x + Sy)1O when x = 3, y = ~
46. By giving x appropriate numerical values in the expansion of (1 + x)n, prove that:

(a) t
k=1
(n)
k
= 2n -(2)-\'"\b)
~
t
k=1
(-2 y (n)
k
= r1)n
.
(c) t
k=O
3k (n)
k
= 22n

47. By expanding both sides of the identity (1 + x)m+n = (1 + x)m(l + x)n,

provethat(m; n) = (~) + (~)(~) + (7)(;) +(~}


48. By expanding both sides of the identity (1 + x)m +3 = (1 + x)m(1 + x)3,

prove that (m ; 3) ( ~) + 3 (r ~ 1) + 3(r m2) + (r ~ 3)-


49. By expanding both sides of the identity (1 + x)n + 2 = (1 + x)n(1 + x)2,
prove that (n ; 2) = (;) + 2e
n1) + (, ~ 2)-
50. By considering the coefficient of xr in the geometric progression
1 + (1 + x) + (1 + X)2 + ... + (1 + x)n,
provethat(;)+(n r I)+(n~2)+ ... +(~)=(;::)_

n
Similarly by consideringx + xn-1(i + x) + x n- 2(1 + X)2 + ... + (1 + x)n,
n

find the sum (; ) +( ; _ + (; _ ~) + ... + (n ~+ 1).


210 NEW SENIOR MATHS: THREE UNIT COURSE

51. Prove that r(; ) n(; =:)-


If P r (;)xr(l - x)n-r find the value of PI +P +... +Pn. Also show that
2

PI +2P +3P +... +nPn


2 3 nx.
52. By considering the coefficients of x2r (2r < n), in the expansions of (l + x)n(l - x)n
and (I x2)n, show that

~ n l)(r:I)-(r~2)~~2)+ ... +nY(7)(2r~ I) -nY(~)


~(;)[(;) IJ
53. If ai' a z , a 3 , a 4 are the coefficients of any four consecutive terms in the expansion of
(l + x)n, prove that

54. If X is the sum of the first, third, fifth, ... terms in the expansion of (a + b)n and Y
is the sum of the second, fourth, sixth, ... terms, show that X2 - yz (a 2 - b 2 )n.
55. By considering the value of (l + x)2n when x = 1, prove that:
t
r=O
2nC = 22n-1
r
+ (2n)!
2(n!)2

56. Write down the expansion of (l + x)n


(a) By differentiating, show that f k (Z) = n. 2 n - 1

(b) By antidifferentiating, show that


(~)+~(7)+~(~)+ ... +n~1(~)=2:+~-1
29.6 Binomial probabilities _ _ _ _ _ _ _ _ __
In the study of probability, we meet some situations in which the same trial is repeated
several times and there are two possible outcomes in each trial, either one of which must
occur.
Games of chance: If a die is tossed three times (or three dice are tossed) there is a
probability, p = ~, of a six turning up in each trial and a probability, q = ~, that a six
does not turn up in each trial. Thus the two possible outcomes in each trial are the
occurrence of a six or its non-occurrence-hence the name 'binomial' trials. The three
trials are independent, Le. the outcome of one trial has absolutely no influence on the
outcome of any other similar trial and the probability of a six in each of the three trials is
the same, ~. We are concerned, then, with calculating the probability of 0, 1,2 or 3 sixes.
If a coin is tossed 5 times, the probability of a 'head' turning up in each toss is p = !.
The 5 tosses are independent and there are two possible outcomes, a 'head' or a 'tail' in
each trial.
Target shooting: A marksman, who keeps a record of his performances, finds that he
29/BINOMIAL THEOREM AND PROBABILITY 211

scores a bull's eye on the average 4 times out of 5. Thus the probability that he will score a
bull's eye in anyone trial is p ~ and the probability of failure in each trial is q == ~. He
fires, say, 8 rounds at a target. Assuming that the 8 trials are independent and that his
chance of a bull's eye remains the same in each trial, we can calculate the probability of
0,1,2, ... 8 bull's eyes.
Sampling lots of manufactured articles: A manufacturer finds that in the long run, 10
per cent of his manufactured articles are defective. If a sample of 20 articles is randomly
selected, each article has a probability, p = 1~' of being defective and a probability,

q = 1~' of not being defective. W~ can calculate the probability of 0,1,2, ... 20 defectives
in the sample.
Clinical trials: It is found that in the long run 80 per cent of patients suffering from
complaint X are cured by drug Y. Ten patients, not specially selected, suffering from this
complaint are treated with the drug. Each has a probability, p == ~, of being cured and a

probability, q ~, of not being cured and each case is independent of the others.
The properties common to each of these examples are:
(i) there are n independent trials,
(ii) each trial has two possible outcomes, which we will call a 'success' or a 'failure',
e.g. the occurrence of a six, a head, a bull's eye, a defective article, a cure with drug
Y could be called a 'success',
(iii) the probability of a success is the same in each of the n trials.

EXAMPLE 10
A die is tossed 3 times. What is the probability of 0, 1,2, or 3 sixes turning up?
Probability of a six in a single trial == ~ p

Probabili~y of not a six in a single trial == ~ == q


Call the occurrence of a six a 'success' (S) and its non-occurrence a 'failure' (F).
The eight possible outcomes and their corresponding probabilities can be represented
with the aid of a probability tree diagram with three sets of branches representing in order
from left to right the probability of 'success' or 'failure' in each of the three tosses.

1st toss 2nd toss 3rd toss Outcomes Probability

<:
1
'6 S SSS (iP p3
1 S~
SSF liPlti = p2q
S f F 1
SFS (iI21~1 == p2q
5
"6 F -C::::;::S
~ F SFF lil(~12 == pq2
0

<= '
1
6 S FSS IW(~1 p2q
1 S~
F 6
FSF li)(~)2 = pq2
F
1

"65 F < S
FFS (i)(W = pif
§. . F FFF (~)3 = q3
6
212 NEW SENIOR MATHS: THREE UNIT COURSE

Table 29-1 lists the possible outcomes of the three independent trials, with their
respective probabilities.
TABLE 29-1

Number of Possible
Probability
sixes Outcomes

o FFF (~y q3
f' ~

SFF (~t(~)
FSF .(~Y. (~) 3q2p

FFS (~r(~)
; .,;

SSF (~)·(~r ~
2 SFS (~) ·(~r ~
3qp2

FSS G)·(~r
3
" SSS (~y ~ p3

If X is used to denote the number of successes in the three trials, X is a random variable
which can assume the values r 0, 1, 2, 3. We may consider the sample space as
consisting of 23, i.e. 8 sample points, but they are not equally weighted.
P(X 0) P (a six in none of the 3 trials)

(~y = q3
125
216
P(X 1) P (a sixjn anyone of the 3 trials and not a six in the other two)

C) (~y. (~) (D q 2p
75
216
P(X 2) = P (a six in any two of the 3 trials and not a six in the other one)

= G)a)(~y = (;)qp2
15
216
P(X 3) = P (a six in all of the 3 trials)

= (~y = p3
1
216
From this example, the pattern of binomial trials emerges. Since the above table lists all
possible outcomes of the 3 trials, all of which are mutually exclusive and one of which is
certain to happen, the sum of their probabilities is unity.
-.------.. ----------------------------~~~~=-

29/BINOMIAL THEOREM AND PROBABILITY 213

i.e. (1)
q3 + G)q2p + (;)qp2 + p3 = 1
125 75 15 1 216
Check: 216 + 216 + 216 + 216 = 216
But the left hand side of (1) is the binomial expansion of (q + p)3
i.e. (q + p)3 = + (Dq2p +
q3 G)
pq2 + p3
= P(O sixes) + P(1 six) + P(2 sixes) + P(3 sixes).

EXAMPLE 11
A marksman, who keeps a record of his performances, finds that in the long run, he
scores a bull's eye on 3 out of 4 occasions. He fires 5 rounds at a target. Assuming that
each trial is independent of any other similar trial, find the probability of (a) exactly 3
bull's eyes, (b) at least 4 bull's eyes, (c) a bull's eye in the second round only.
P (a bull's eye in a single trial) = ~ p

P (not a bull's eye in a single trial) = i = q

If X denotes the number of successes in the 5 trials, X is a binomial variable which can
assume the values 0, 1, 2, 3, 4 or 5.
(a) P(X 3) = P (a bull's eye in any three trials and a failure in the other two)

= G)(~Y(iY
270
1024
135
=512
(b) The statement 'at least 4 bull's eyes' means 4 bull's eyes Qr 5 bull's eyes. These events
are mutually exclusive.
Hence P(X ;;a. 4) = P(X 4) + P(X = 5)

= (~)(~Y(i) + (~y
405 243
1024 + 1024
648
1024
81
= 128

(c) If w~ are concerned with the probability of a bull's eye in the second round only, then
the pattern of successes and failures is
FSFFF with probability of
13111. 3
4' 4' 4' 4' 4 l.e. 1024

Note ~he absence of the coefficient (i). Why? This is an event of small probability and
hence it is most unlikely. If it did actually happen, what would we conclude? How many
successes would we expect?
214 NEW SENIOR MATHS: THREE UNIT COURSE

EXAMPLE 12
For a certain species of bird, there is a chance of 3 in 5 that a fledgling will survive the first
month after birth. From a brood of 10 chicks, what is the probability that:
(a) none will survive, (b) more than one will survive?
3 2
p q S·
If X denotes the number which survive, then X is a binomial variable which can assume
the values 0, 1, 2, 3, ... 10.

(a) P(X 0) (~)'O "'" 0.0001


This is an event of very small probability. If there were actually no survivors, what
might we conclude? How many survivors would we expect?
(b) P(X > 1) = P(X 2) + P(X 3) + ... + P(X = 10).
However it is certain that there will be 0, 1, 2, ... 10 survivors and hence:
P(X > I) = 1 - [P(X = 0) + P(X = 1)]

1 - [(~yo + (\O)U)ar]
"'" 1 - [0-0001 + 0,0015]
0·9984
From these examples, we can generalize for n independent trials.
(q + p)n = qn+ (7)qn-l p + (~)qn-2p2 + ... + (;)qn-rpr + ' .. + pn
= P(X = 0) + P(X I) + P(X = 2) + ... + P(X = r) + ... + P(X n)
Thus if X denotes the number of successes in n independent trials, the probability that
there will be exactly r successes and (n r) failures is given by the (r + l)th term in the
expansion of (q + p)n,

i.e, P(X r) (~)qn-rpr where 0 .;;; r .;;; n

= (~)(l - p)n-rpr since q +p =

Sampling with replacement from a small population

EXAMPLE 13
A carton contains 15 transistors of which 5 are defective. One transistor is withdrawn
from the carton, examined and then put back into the carton. Another drawing is made,
examined and replaced. This process is continued four times. What is the. probability of
(a) at least one defective being withdrawn, (b) not more than two defectives?
Since each transistor is replaced after withdrawal and inspection, the probability, p, of
a success, namely the drawing of a defective transistor, remains the same, } for each of the
4 trials.
Hence the binomial probability applies in this case. The binomial variable X can assume
the values 0, 1, 2, 3; 4.
1 2
p =y q = 3' n = 4
29/BINOMIAL THEOREM AND PROBABILITY 215

(a) Pr(X ~ 1) = P (X = 1) + P (X = 2) + P (X 3) +P (X = 4)
1 - P (X = 0)

1 - (~y
65
81
(b) P(X ~ 2) = P(X = 0) + P(X = 1) + P(X = 2)

(~y + (i)(~Y(D + (i)(~JGY


16 32 24
= 81 + 81 + 8T
8
= 9

Sampling without replacement from a large population


In Example 13 we considered sampling from a small population of 15 transistors. If the
sampling were done without replacement, the probability that the first transistor
withdrawn was defective would be 15' The probability that the sec~nd transistor with-
5

drawn was defective would be 1~' given that the first transistor was defective and so on.
The probability, p, of a defective has altered considerably, in which case we are not
dealing with a binomial variable.
If, however, we take a s,ample without replacement from a large population of mass-
produced articles, say 1000 or more, of which 20 per cent are defective, then the pro-
bability that the first article selected is defective is 12:. The probability that the second

article selected is defective, given that the first is defective, is then ~~~ which is approxi-
mately 0·2. Hence the pro ability, p, of any article being defective remains practically
unaltered by the sampling. In this case the binomial distribution can be used as an
approximation.
EXAMPLE 14
A manufacturer finds that in the long run X per cent of his articles are defective, If a
sample of 5 articles is randomly selected, calculate the probability that the sample
contains (a) 0 defectives, (b) exactly 2 defectives, (c) more than 1 defective.
' 'I approxImatIon
The b momm . . can be usedWIt
' hp x q = 100100
= 100' - x an d n =.
5 Th e

binomial variable X can assume values 0, 1, 2, ... 5

(a) P(X = 0) q 5=(100-X)5


100
= 10-1 °(100 - X)5

(b) P(X 2) = C20) (I~ye~oo xy


= 10-10 x 45x2(100 - X)3

(c) P(X> 1) P(X 2) + P(X = 3) + P(X = 4) + P(X = 5)


= 1 - [P(X = 0) + P(X = I)]
216 NEW SENIOR MATHS: THREE UNIT COURSE

_[ (100100- X)5 + (5)I (100100- X)4~J


100
10- 1°[(100 X)5 + 5x(100 - X)4)

EXAMPLE 15
Three coins are tossed 400 times. On how many occasions would we expect 0, I, 2, 3
heads?
Let X denote the number of heads in a single toss of 3 coins.
The number of occasions on which we would expect r heads =400 P(X = r), r =0, 1,2,3.
4OOP(X = 0) = 4oo(!Y 50

400 P(X = I) = 4OO(i) (!YG) = 150

400 P(X 2) = 4ooG)(!)UY 150

400 P(X = 3) = 400GY 50.

EXAMPLE 16
A die is tossed 30 times. What is the most likely number of sixes? (of examples 7 and 8).
We are asked to find the greatest term in the binomial expansion of (q + p)n
. (56 + 61)30. th'IS case.
I.e. In

Tr + 1 n - r +1 P 31 r I
-:y;= r q= r '5
, 31 - r
Tr+ 1 ~ Tr If ----sf ~ 1
31 r ~ 5r
31 ~ 6r
r .;;; 5k.
The greatest term occurs when r = 5
T6 = P(X = 5)

= e~)( ~y5(~Y.
The most likely number of sixes is 5, This is the result we would expect. The probability of
a six in a single trial is ~ and hence in 30 trials we would expect 5 successes,

EXERCISES 29(b)
1. In 12 throws of a die, what is the probability of exactly 4 sixes turning up?
2. Hospital records show that of patients suffering from a certain complaint, 75 per cent
die of it. What is the probability that, of 6 randomly selected patients, all will
recover?
3. A marksman finds that, on the average, he hits the target 4 times out of 5. If he fires
4 shots, what is the probability of (i) more than 2 hits, (ii) at least 3 misses?
4. What proportion of families with exactly 6 children should be expected to have at
29/BINOMIAL THEOREM AND PROBABILITY 217

least 3 boys? (Assume that the probability that a child is a boy i's ~ and ignore the
possibility of multiple births.)
5. A manufacturer of metal pistons finds that on the average 12 per cent of his pistons
are rejected because they are either oversize or undersize. What is the probability that
a batch of 10 pistons will contain (i) no more than 2 rejects, (ii) at least 2 rejects? '
6. It is known that, in the long run, 7 out of every 10 students from the Year 12 of a
certain school will enter the university. Assuming a binomial distribution, find the
probability that out of a group of 5 students chosen at random from this form,
(i) exactly 3 will enter the university, (ii) any number up to and including 3 will enter,
{iii) only the first 3 chosen will enter.
7. In the long run forty per cent of patients treated for a particular disease with the drug
streptomycin are cured. If a doctor were to treat 10 randomly chosen patients with the
drug, what is the probability of (i) 9 cures, (ii) more than 8 cures?
A second doctor treats 10 patients with a new treatment and obtains more than 8
cures. Comment on this result.
S. A dental inspector finds that about 20 per cent of children of a certain area have tooth
decay. What is the probability that out of 6 children examined, only the first, third
and fifth will have tooth decay? r'
9. A long series of hospital records shows that 40 per cent of cases of a certain disease
fail to recover. Calculate the probability of a sample of 10 patients yielding only one
death.
If it were known that these ten patients had been subjected to a new treatment,
what inference would you make about the efficaey of this treatment?
10. A Gallup Poll establishes that 60 per cent of people interviewed are in favour of a
certain proposal. What is the probability that if 5 people are selected at random, there
will be a majority of them against the proposal?
11. Three uniform tetrahedra (triangular pyramids) each has one face black, another
white, another red and the other ,green. When tossed onto a table, three faces of each
tetrahedron can be seen. What is the probability that (i) no black face can be seen,
(ii) exactly 2 black faces can be seen, (iii) at least 2 red faces can be seen, (iv) 3 white
faces and only 1 green face can be seen?
12. A die is rolled ten times. What is the probability of at least two sixes turning up?
13. A die is loaded such that in 10 independent trials the probability that an even number
appears 5 times is twice the probability that an even number appears 4 times. What is
the probability that an even number will appear in a single trial?
14. What is the probability that a family of four children 'will contain 4 girls?
15. The probability that persons aged 60 and 65 will survive the hext year is 0·97 and 0'96
respectively. From a group of five people of whom three are aged 60 and two aged 65,
what is the probability that (i) not more than one will die within the next year, ~)the
two aged 65 survive and not more than one dies within the next year?
16. How many times must a fair coin be tossed so that the probability of at least one head
exceeds 0'9?
17. In an examination consisting of ten true-false questions, a student guesses the
218 NEW SENIOR MATHS: THREE UNIT COURSE

answers. What is the probability that he will gain 70 per cent or better?

18. Assuming that the chance of death by accident is 3~0' estimate the probability that
out of a random selection of 150, there will be at least 1 death by accident.
19. John estimates that the probability of his winning anyone game of tennis against a
particular opponent is j. How many games should they play so that the probability
that John wins at least one game is greater than 0'9?
20. On the average, a typist has to correct one word in 800 words. Assuming that a page
contains 200 words, find the probability of more than one correction per page.
21. Steel rods are manufactured to be 4 cm in diameter but they are acceptable if they are
between 3·98 cm and 4·02 cm. The manufacturer finds that in the long run 3 per cent
are rejected as oversize and 3 per cent as undersize.
A random sample of 10 rods is selected from the population. What is the
probability that it contains not more than one which has to be rejected?
0. Four families each have four children. What is the probability that (i) at least one of
these families has two boys and two girls, (ii) each family has at least one boy?
23. A marksman finds that on the average he hits the target 9 times out of every 10 and
scores a bull's eye on the average once every 5 rounds. He fires 4 rounds. What is the
probability that (i) he hits the target each time, (ii) he scores at least 2 bull's eyes,
-'(~9 he scores at least 2 bull's eyes and he has hit the target on each of the 4 rounds?

24. In a packet of flower seeds 40070 are known to be pink flowering and the remainder
yellow. Calculate the probability of getting 0, 1, 2.. 6 pink flowers in a row of
6 plants. If 2500 rows each of 6 plants are planted, how many rows would be expected
to contain, (a) all pink flowers (b) all yellow flowers?
25. If ten coins are tossed 50 times, in how many cases would we expect the number of
heads to exceed the number of tails?
26. On the average, a marksman scores a bull's eyes ~n 2 out of every 3 shots. He fires at a
target in multiples of 5 shots. Estimate how many multiples of 5 shots with 3 bull's
eyes would occur in a thousand multiples.
27 A factory has 7 machines, 4 of model A which are in use, on the average, 80 per cent
of the time and 3 of model B which are in use, on the average, 60 per cent of the time.
If the foreman walks into the factory at a randomly selected time, what is the
probability that (i) he will find 2 machines df model A and 1 of model B in use, (ii) he
finds 2 machines in use.
28. If, over a certain period of the year, rain falls at random and on the average on
12 days in every 30, find the probability that (0 the first 3 days of a given week will be
fine and the remairtder wet, (ii) rain will fall on just 3 days of a given week, (iii) at
least 3 days in a given week will be fine?
29. A manufacturer of razor blades finds that on the average 1 blade in every 20 is faulty.
The razor blades are marketed in packets of 5. Out of a batch of 200 packets, how
many would be expected to have 0, 1, 2, 3, 4, 5 faulty blades?
29/BINOMIAL THEOREM AND PROBABILITY 219

30. During the course· of a particular morning, a machine in a factory workshop was
active or idle as specified below:
Time (a.m.) State of Machine
9.00-9.30 Idle
9.30-10.20 Active
10.20-10.30 Idle
10.30-11.30 Active
11.30-12.00 Idle
If a work study engineer had walked through the workshop at a time randomly
selected between 9.00 a.m. and 12.00 noon, determine the probability that he would
have found the machine idle.
Determine also the probability that if he had observed it at six randomly selected .
times during this period he would have found idle (i) all of them, (ii) all but one
of them.
:fi> In order to form an estimate of the quality of mass-produced articles, a customer
adopts the following double sampling scheme. A random sample of 10 is inspected; if
no defectives are found, he accepts the lot; if 4 or more defectives are found, he
rejects the lot; but if 1, 2 or 3 defectives are found, he takes a second sample of 10 and
if the total of defectives in the two samples is 4 or more, he rejects the lot, but
otherwise accepts the lot. Calculate the probability that a lot which is known to be 10
per cent defective is accepted.
32. If p is the probability of a success in each of n independent trials, show, using the
binomial expansion of (q + p)n, that the most likely number of successes is np.
33. Find the most likely number of successes in each of the following;
(a) number of heads in 20 tosses of a fair coin,
(b) number of defective articles in a sample of 25 articles drawn from a population
which is 10 per cent defective,
(c) number of people in favour of a certain candidate in a sample of 30 voters taken
from a population which is 60 per cent in favour of the candidate.
TEST PAPERS (Time: 2 hours)

TEST PAPER 1. . . . . . . . . . . . . . . . . . . . . .__


1. (i) If a, b, c are the roots of the equation x 3 -: X2 + 4x - 1 = 0, find the value of
(a + l)(b + 1)(c + 1).
(ii) If students are required to study four subjects out of ten which are classified into
two groups each containing five subjects, how many subject combinations are
possible if a student must take
(a) two subjects from each group,
(b) at least one subject from each group?

2. (i) Evaluate (a) Jt it


0 1 + X2dx
x
(b) J 0 1 + X2dx
(ii) Find, by integration, the volume of the solid generated by rotating the curve

y cos x between x = 0 and x = ~ about the X-axis.


3. (i) Prove, by induction, that an - bn is divisible by a - b for all positive integer values
of n.
(ii) ABCD is a parallelogram. X and Yare two points on the diagonal AC such that
AX = CY. Prove that DXBY is a parallelogram.
X2 - 4
4. (i) For what values of x is > O?
x
(ii) Find all angles 0 for which
(a) cos 20 = cos 0
(b) sin 0 + cos () = I

5. (i) Find the tangent of the acute angle at which the parabolas y = X2 and y (x - 2)2
intersect. .
(ii) Prove that the tangents at the extremities of a focal chord of a parabola intersect at
right angles. '

6. (i) Sketch the graph of y = x 2 : 4 after finding:


(a) the co-ordinates of the turning points,
(b) the equation of the asymptote,
(c) the range.
(ii) A spherical balloon is being inflated and its volume increases at the constant rate
of 50 mm 3 per second. At what rate is its surface area increasing when the radius is
20 mm?
7. (i) A projectile is fired with initial speed .J2ga to hit a target at a horizontal distance
a from the point of projection and at a: vertical distance ia above it. Show
that there are two possible angles of projection and that the ratio of the times of
flight is 1 : .J5. .
(ii) A particle moves along the X-axis so that its velocity is proportional to x 2 • Prove
that its acceleration is proportional to x 3•
TEST PAPERS 221

TEST PAPER 2

1. (0 If y X cos- J 2x 0t1 - 4x2 , find :t.


(ii) If f(x) 2 ..JX" x ~ 0 and g(x) = (x - 2)2 for all x, find
(a) the values of x for whichflg(x)] x = g[f(x)]
(b) f-1(x) giving its domain, range and rule.

2. 0) The positive square root of 10 is approximately 3. Using two applications of


Newton's method, find a better approximation.
(Ii) Write down the binomial expansion of (2 + x)". By giving x an appropriate
numerical value, show that

3. 0) Find the ratio of the (r + l)th term to the rth term in the expansion Of( 2x + ~ y 2

and hence find the greatest co-efficient in the expansion.


(iO It is estimated that the probability that a torpedo will hit its target is ~ .
.<a) If 5 torpedoes are fired, what is the probability of 3 successes?
(b) How many torpedoes must be fired so that the probability of at least one
success should be greater than 0'9?

4. (i) (a) Use the factor theorem to find linear factors of 2x 3 + x2 - 13x + 6.
(b) For what values of x is 2x 3 + X2 13x + 6 > O?
(ii) Find the term independent of x in the binomial expansion of (2X - ;2Y2.
5. (0 Find all values of k for which y ekx satisfies the equation 2 ~ - :t - 3y = O.
(ii) (a) Find ddY if Y = tan-I(sin x)
. x
dx
I
I
(b) Evaluate o.J2 _ x 2'
6. (i) Express sin 2x + .J3 cos 2x in the form r sin (2x + ex) and hence write down the
maximum and minimum values of sin 2x + .J2 cos 2x and the smallest values of x
for which they occur.
(ii) Iff(x) = 1 - X2, a 0;;;; ~ 0;;;; 2, find the smallest value of a for whichjJ(x) exists
and findjl(x).

, (,,/3
7. 0) EvaluateJo sin3x cos xdx, using a substitution u = sin x.

.. ) F'Ind I'Im-
(II sin 5x.
x_a 2-x
(iii) Sketch the graphs of y = sin x and y = cos x for 0 0;;;; x 0;;;; 211" and find:
(a) the values of x at which they intersect,
(b) the area between the two curves.
222 NEW SENIOR MATHS: THREE UNIT COURSE

TEST PAPER 3 _ _ _ _ _ _ _ _ _ _ __

1. P(2at, a( 2 ) is a variable point on the parabola X2 = 4ay and C(O, c) is a fixed point on
the Y-axis (c =1= 0).·
(a) Express the co-ordinates of Q, the midpoint of PC, in terms of t.
(b) Find· the cartesian equation of the locus of Q and show that the locus is a
parabola.
(c) Find the vertex and focus of this parabola.
2. (i) Three fair dice with faces numbered one to six are rolled. Find the probability that
the three numbers showing uppermost are (a) the same (b) aU different.
(ii) A chain is made by joining successive links. Each link has a probability ~ of being
'strong' and probability ~ of being 'weak', independently of other links.
(a) What is the probability that the first n consecutive links will contain one weak
link?
(b) What is the probability that there will be n strong links followed by the first
weak link?
(c) What is the largest value n can have so that a chain of n links consists of all
strong links with probability at least i?
3. (i) Water flows from a hole in the base of a cylindrical vessel at a rate given by the

equation ~~ = -k-Jii, where k is constant and h is the depth of the water at any
time t.
If the depth of the water falls from 100 mm to 25 mm in one minute, find how
much longer it will take for the vessel to empty?
(ii) A projectile is fired from a point 0 with speed 14 m/s. T seconds later, a second
projectile is fired with the same speed from the same point, at a different angle of
elevation.
The two projectiles collide at a point 4 metres horizontally from 0 and 6 metres
vertically above the level of O.
Find the value of T to the nearest one-tenth of a second. (Use g 9·8)
4. (i) A particle moving in a straight line travels a distance x in time t. Prove that its
acceleration ca? be expressed in the form i! (V2).

d 2x 4 - 3x
If the acceleration obeys the law dt 2 = x3 and the particle starts from
rest at the point x l , find the velocity in terms of x and show that the particle is
restricted to moving between two fixed points on the line.
(ii) Five cubes are identical except for colour. Two are coloured red, one white, one
blue, one yellow. They are then placed at random in a row. What is the probability
that:
(a) the end positions are occupied by red cubes,
(b) the red cubes are together?
5. (i) AB is a diameter and AC a chord of a circle. The bisector of angle BAC cuts the
circle at D. Prove that the tangent at D is perpendicular to AC.
TEST PAPERS 223

(ii) ABCD is a quadrilateral and each of its sides is a tangent to a circle centre O.
Prove that

6. (0 Prove, by induction, that


N 1 N
E(2n + 1)(2n 1) 2N + l'
Oi) Sketch the graph of y = -:-x_-:-.

(a) State the largest possible domain.


(b) Show that Yx < 0 for all x in the domain.
(c) Write down equation of the asymptotes.
(d) The area bounded by the curve, the X-axis and the ordinates x = 3 and x 4
is equal to ! log., a. Find a.
7. (i) Write down the derivative of (a) 1 ~ x (b) loge (sin x + cos x).
x

Oi) Evaluate exactly )1 rol(e-x + 1 +


+x ~
1 )dX.
(iii) If f(x) sin-I x + COS-I x, -1 ~ x ~ 1,
(a) show thatj'(x) = 0 for all x,
(b) show thatf(x) 211" for all x,

(c) evaluate r>(X)dx.

TEST PAPER 4
1. (i) The angle of elevation of the top of a tower from a point A due east of it is 40°
and at a point B due south of A the angle of elevation is 20°. If the distance from
A to B is 100 metres, find the height of the tower.
(ii) (a) cos (0 ¢) = cos 0 cos ¢ + sin 0 sin ¢
In place of ¢ write 90° ¢ and hence show that
sin,(O + ¢) = sin 0 cos ¢ + cos 0 sin ¢.
(b) Find all values of 0 for which 2 sin2 0 3 cos O.
2. (0 A(I, 5), B(-4, 2) and C(3, -1) are the vertices of a triangle ABC. If D is the mid-
point of BC, find the co-ordinates of a point M which divides AD in the ratio 2 : I.
(ii) Solve for x: (x + kJ - 5(X + k)+ 6 = O.
3. (i) The parabolas y2 x and X2 8y intersect at the origin and the point (a, b)
(a) Find the values of a and b.
(b) Prove that the curves divide the rectangle whose vertices are (0, 0), (a, 0),
(a, b) and (0, b) into three regions of equal area.

(ii) Differentiate (X2 + 2x + 2) e- and hence evaluate


X
I 2
X2 e-x dx to 3 decimal places.
.. -",

224 NEW SENIOR MATHS: THREE UNIT COURSE

X2
4. For the curve y = -1--2 ' find:
-x
(a) the co-ordinates of the points of intersection with the axes
(b) the equation of the asymptotes,
(c) the co-ordinates and nature of the turning points,
(d) the range i.e. the values that y can take.

S. (i) A curve is represented by the parametric equations x = sin t, y = sec t, 0 ~t~ f.


. equatIOn
(a) Show t h at t he cartesIan . 0 fh . y = -..)(1 1_ x )' 0
t e curve IS 2 ~ X ~
-J3

(b) Find the area of the region below the curve and above the X-axis.

(ii) Evaluate r x-..)(I + x2)dx using the substitution u = X2 + l.

6. (i) An inverted cone of radius 20 cm and depth 30 cm is filled with water. The water
flows out through the apex of the cone at a constant rate of 10 7r cm3/min. Find the
rate at which the water level falls when the depth of the water is 20 cm.
(ii) A particle moves in a straight line with Simple Harmonic Motion. At time t s, its

displacement x m from a fixed point 0 in the line is given by x = 5 sin ~(t + ~).
(a) Show that x = - i 2
x.
(b) State the period and amplitude.
(c) Find the magnitude of the velocity and acceleration when x = -2~

7. (i) A body whose temperature is 180°C is immersed in a liquid kept at a constant tem-
perature of 60°C. In one minute, the temperature of the immersed body falls to
l20°e. How long will it take for the temperature of the body to fall to 90°C,
assuming Newton's law of cooling?
(ii) A projectile is fired at an angle of 30° above the horizontal with speed 20 mls from
a stationary platform at ground level.
(a) Find the horizontal range of the projectile.
(b) By how much is the range increased if the platform is moving with speed
10 mls in the direction of the target? Use g = 10 m/s2.
TEST PAPER 5 _ _ _ _ _ _ _ _ _ _ __

1. (i) How many numbers greater than 4000 can be formed with the digits 2, 3, 4, 5, 6 if
no digit is used more than once in a number?
(ii) From a group of 7 people consisting of 5 females and 2 males, a group of 3 people
is to be chosen at random. What is the probability that it contains at least
2 females?
2. (i) On the average, batsmen in a certain cricket team make a scoring shot on every
third ball.
(a) What is the probability that in a particular six ball over, scoring shots will be
made on at least 3 balls?
(b) In 200 overs, how many overs would we expect two scoring shots to occur?
(ii) Prove that nCI + 2 nC2 + 3 nC3 + ... + n nCn = n. 2n- 1 by differentiating both
sides of the expansion of (1 + x)n with respect to x.
TEST PAPERS 225

3. (i) Express cos (J - sin (J in the form r cos «(J + 0:) and hence, or otherwise, solve the
equation cos (J - sin (J = 1, -'IT .;;; (J .;;; 'IT
(ii) Simplify
(a) tan 2x tan x
+ tan tan x (b) cos 22x - sin 2x.
2

4. (i) If sin 0: = ~, ~ < 0: < 'IT and sin (1 i,o < (1 < ~ write down the exact value of:
(a) sin 20: (b) cos (0: - (j) (c) tan (0: + (1)
(ii) If tan (J ~ express b sin (J cos (J + a sin 2 (J in' terms of a and b.
5. (i) Prove, by induction, that the sum of the first n odd numbers is equal to n 2
i.e. 1 + 3 + 5 + ... + (2n - 1) n 2•
(ii) The equation X2 1 - x has a solution x = 0'5 approximately. Use one
application of Newton's method to obtain a better approximation.

6. (i) The coefficient of xin the expansion of (x + a~2 Yis j. Find all possible values
of a.
(ii) Write down the primitive (indefinite integral) of sin4 x cos x.
2
(•..
III
) If dy
dx = 5x and dx
dt
-2 fi d dy
,In dt an
ddy
dt 2 •
,,/6
(iv) Find c if 1 ~os .x dx loge c.
/ o SIn X

7. 0) Find the value of x in each of the following circles centre 0:

(a) (b)

(ii) Two chords AB and CD of a circle meet when produced at a point P outside the
circle. Prove that 6ADP and 6CBP are similar.
TEST PAPER 6 ________________________
1. From a point A the bearings of two points Band Care 065°T and 105°T respectively.
From a point D 5 km due east of A, the bearings are 030 0 T and 117°T respectively. If
the distance between Band Cis d km, show that:
d2 25{(Sin 60°)2 + (Sin 27°)2
sin 35° sin 12°
2 sin 60° sin 27° cos
sin 35° sin 12°
400}
!
226 NEW SENIOR MATHS: THREE UNIT COURSE

2. (i) Find all values of 0 in the range 0° ~ O~ 360° for which 4 cos 0 + 3 sin 0 = 1.
(ii) Find all values of 0 for· which tan 0 = cot o.
2
(iii) Show that, if sec () = a + 1, then sec () + tan 0 = a or l.
2a a

3. (i) Show that lim 2 3+sin () 0 = ().


6-0 cos
(ii) Find, without the aid of tables or a calculator, the acute angle between two lines
with gradients -3 and 2.
(iii) Write down the equation of the tangent and normal to the parabola x = 2/, Y = /2
at the point where t = 2.
4. (i) M is the foot of the perpendicular from a point P(2at, at 2) on the parabola
X2 = 4ay onto the Y-axis. The tangent at P meets the Y-axis at T. Prove that the
vertex of the parabola is the midpoint of TM.
(ij) A particle moves in a straight line with an acceleration given by x
velocity v =·1 and x = 0 when t = 0 find
(a) its velocity in terms of x at any time t,
(b) its position in terms of / at any time t.
S. (i) Solve the equation:
+ x 3 6x 2 + X + 2 0
2X4
(ii) Find an equation whose roots are one more than the roots of the equation
x3 5x 2 + 6x 3 = O.
(iii) In four throws with a pair of dice, what is the probability that the sum of the
numbers uppermost is 7 at least twice?

6. (i) A missile is projected with a speed of 50 mis, at an angle of elevation of 45°,


aimed at a tall building which is at a horizontal distance of 200 metres from the
point of projection.
(a) Find the time of flight until the missile hits the building.
(b) Find how high on the building the missile hits.
(ii) The speed v mls of a particle moving along the X-axis is given by
V2 = 18 + 32x - 8x 2 ,
where x is the distance of the point from the origin. Prove that the motion is
simple harmonic and find
(a) the centre of motion (b) the period (c) the amplitude.
7. (i) Find the greatest coefficient in the binomial expansion of (3 + 5X)20.
(ii) By considering the coefficient of xn on both sides of the identity
(1 + x)n(l + x)n (1 + x)2n
show that

t e: )
(iii) By differentiating both sides of the identity

(I + x)2n = xk
k=O
and then, giving x an appropriate numerical value, show that

1: k (2n)
2n
= n. 22n
k I k
TEST PAPERS 227

TEST PAPER 7 _ _ _ _ _ _ _ _ _ _ __

1. (i) Prove that


(a) sin-I x + COS-I X
2
(b) COS-I (-x) = 11" COS-I x
(ii) Differentiate with respect to x:
(a) sin- 1 2x (b) tan-1 (cos x) (e) cos- 1 G)
(iii) Evaluate:
2.J3
(a) j 0 x : 1 dx (b) j "/3
,,/6 (1 cos2 x)dx

2. (i) The curve y -=-::-=----c:o-:- is rotated about the X-axis. Find the volume of the
solid of revolution between x = 0 and x 3";3.
(ii) Using the substitution x = sin (), find, by integration, the area of the circle
X2 + y2 1.
3. (i) Find the largest possible domain for functions with the following rules:
(a) I(x) ";[4 ";(x 1)]
(b) I(x) = sin-I (loge x)
(e) I(x) log" (2x - x 2)
(ii) Show, by a sketch, that the curves y = X2 and y -i1 Ioge x meet at one pomt
.

only. Prove that, at this point, the tangents to the two curves intersect at right
angles.
4. (i) Solve the equation tan x + cot x = 2, 0° ~ x ~ 360 0 •
(ii) Prove that
(a) tan (i + () ) - tan (i - ()) = 2 tan 2()
(b) cosec () 2 cot 2() cos () = 2 sin ().
11" •
(iii) If cos () 0·8 and -i ~ () ~ 0 find the value of
()
(a) cos 2() (b) cosio

n
5. (i) Prove, by induction, thatL k 3 = '-'--'~':-'-':'-<-.
k=1

(ii) For what values of x is x + 41 < 6.


x-
(iii) A, Band C are three points on a circle, centre O. The tangent at A meets BC
produced at T. If D is the midpoint of BC, prove that
(a) A ODT is a cyclic quadrilateral
(b) LAOT = LADT.
6. (i) A bag contains 6 white cubes and 4 black cubes. A cube is drawn out, its colour
noted and then replaced. This process is repeated 6 times.
(a) What is the probability that at least 4 white cubes are withdrawn?
(b) Find the number of white cubes that is most likely to be withdrawn.
(ii) If A is the sum of the odd terms and B the sum of the even terms in the expansion
of (x + a)n, prove that A 2 - B2 (x 2 a 2)n.
228 NEW SENIOR MATHS: THREE UNIT COURSE

7. (i) Eight points are marked at random on a circle. How many triangles can be
formed?
(ii) A carton contains ten items, four of which are defeetive. A random sample of
three items is selected without replacement, what is the probability that there is at
most one defective item in the sample?
(iii) How many different arrangements of the letters of the word SERIES are possible if
(a) the two letters S are to be together
(b) no two vowels are together?
TEST PAPER 8 _______________________
1. (i) For what values of 1 is 21 ~ 1< 1
(ii) Two circles intersect at M and N. MP and MQ are diameters in the respective circles.
Prove that the points P, N, Q are collinear.
2. (i) (a) Show that cos 2 (I - cos 2 3(1 = sin 4(1 sin 2Ofor all values oro.
(b) Find all values of (I in the range 0° ~ (I ~ 360° for which 3 cos (I + .J3 sin (I .J3
Oi) Prove, by induction, that k~ (2k + 1)1(2k l) 2n n+ 1

3. Sketch the graph of y = X2 : 4 after finding


(a) the equation of the asymptote
(b) the coordinates of the turning points
(c) the range of the function i.e. the values that y can take.
Find also the area of the region enclosed by the curve, the X-axis and the lines x
and x 2.
4. (i) Evaluate

i
~/4
(a) sin x cos 3 xdx
-~/4

(b) J "'(1 \
0
3 x) dx, using the substitution x u2 - 1

(ii) Find, by integration, the volume of the solid of revolution formed by the rotation
about the X-axis of that part of the curve y = cos 2x between the lines x = 0 and
1f
x = S'
5. (i) The equation sin x = I - 2x has a root near x 0·3. Use one application of
Newton's method to obtain a better approximation.
(ii) pet + 3, t 2 - 5) is a variable point on a parabola. A(l, 3) is a fixed point not on
the parabola. Q is a variable point on the line AP and divides AP in the ratio 2 : I.
Find the cartesian equation of the locus of Q.
6. (i) A point P moves on the curve y = x 3 in such a way that its x-coordinate is
changing at a constant rate of 4 units/so At what rate is
(a) the y coordinate (b) the gradient changing when x = I?
(ii) The rise and fall of the tide at a certain harbour may be considered as simple
harmonic motion. The interval of time between successive high tides is
TEST PAPERS 229

approximately 12 h 30 min. The depth of the water at a certain point in the


harbour is 6 m at low tide and 14 m at high tide. If low tide occurs at noon, at
what time is the depth 12 m? .
7. A ball is projected from a point on the ground distant a from the foot of a vertical wall
of height b. The ball is projected at an elevation a with speed V. If the ball just clears
the wall prove that the greatest height reached is 41 [aa
2
tantan
2
a _a b J
SUMMARY
Plane geometry (circle) _ _ _ _ _ _ _ _ _ __
1. Equal angles at the centre of a circle stand on equal chords and conversely equal
chords subtend equal angles at the centre.
2. Equal arcs on circles of equal radii (or on the same circle) sub tend equal angles at the
centre and conversely, equal angles at the centre stand on equal arcs.
3. The perpendicular from the centre of a circle to a chord bisects the chord and
conversely, the line from the centre of a circle to the mid-point of a chord is
perpendicular to the chord.
4. Equal chords in equal circles (or the same circle) are equidistant from the centre and
conversely, chords which are equidistant from the centre are equal.
5. The angle at the centre is twice the angle at the circumference subtended by the same
arc.
6. Angles in the same segment are equal.
7. An angle in a semicircle is a right angle.
8. Opposite angles of a cyclic quadrilateral are supplementary and conversely, if the
opposite angles of a quadrilateral are supplementary, the quadrilateral is cyclic.
9. If one side of a cyclic quadrilateral is produced, the exterior angle so formed is equal
to the interior opposite angle.
10. A tangent to a circle is perpendicular to the radius at the point of contact and
conversely a line drawn perpendicular to the radius at the point of contact is a
tangent.
11. The tangents to a circle from an external point are equal in length.
12. If two circles touch, the line joining their centres passes through their point of
contact.
13. If two circles intersect, the line joining their centres is the perpendicular bisector of
the common chord.
14. The angles between a tangent and a chord through the point of contact are equal to
the angles in the alternate segment.

Trigonometry _ _ _ _ _ _ _ _ _ _ _ _ _ __

1. Addition theorems
sin (0 ± ¢) = sin 0 cos ¢ ± cos 0 sin ¢
cos (0 ± ¢) = cos 0 cos ¢ =1= sin 0 sin ¢
tan 0 ±
tan (0 ± ¢) = ~"'--'--=-:~-'-:-
SUMMARY 231

2. Double angle formulae


sin 20 2 sin 0 cos 0 cos 20 = cos2 8 sin 2 8
2 tan 8 = 2 cos2 8 I
1 + tan2 0 1 2 sin2 0
2 tan 0 1 - tan 2 8
tan 28 =
1 - tan2 0 +
If tan £ t, then tan 0 =
2t
cos 8 = 1
1 - t2 ,
+ t 2 ' sm 0 1 +
2t
t2'
3. Products as sums or differences
2 sin 0 cos ¢ = sin (0 + ¢) + sin (0 ¢) = sin (sum) + sin (difference)
2 cos 0 sin ¢ = sin (0 + ¢) sin (0 - ¢) = sin (sum) sin (difference)
2 cos 0 cos ¢ cos(O + ¢) + cos (0 - ¢) = cos (sum) + cos (difference)
2 sin 0 sin ¢ = cos (0 - ¢) cos (0 + ¢) = cos (difference) - cos (sum)
4. Sums or differences as products
' 0 + sm
sm ',I,
'+' =
2,0+
sm O-¢
cos - 2-

i.e. sin + sin = 2 sin ( ~ sum) cos (~ difference)


'8
sm ',1,
- sm '+' =
2
cos 0+ ,O-¢
sm -2-

i.e, sin - sin = 2 cos ( ! sum) sin (! difference)


0+ O-¢
cos 8 + cos ¢ = 2 cos cos - 2 -

i.e. cos + cos = 2 cos (~ sum) cos (~difference )

cos 0 cos ¢ = -2 sin 8 + sin 0 ; ¢

i.e, cos - cos -2 sin (~ sum) sin (! differenCe)


5. Transformations
a sin x ± b cos x =r sin (x ± a)
a cos x ± b sin x = r cos (x =F a)
b ~
where r = -Ja 2 + b 2 and tan a = ii'O < a < 2'
6. General solutions of equations
sin 0 = a 0 = n~ + O)n sin-I a
sin 0° = a 80 = n. 1800 + (-I)n sin-I a
cos 0 = a 0 = 2n ~ ± COS-I a
cos 0° = a 0° = n. 360° ± COS-I a
tan 0 = a 0 = n ~ + tan-I a
tan 0° = a 0° n. 180 0 + tan-I a
.
7. Inverse sIDe: y '1
= sm - x,1- ...;; x...;; 1 means x 'd~
= sm y an - 2: ...;; y ~
...;; 2
Inverse cosine: y cos-1 x, -1 ...;; x ...;; 1 means x = cos y and 0 ...;; y ...;; ~
~ ~
Inverse tangent: y tan -I x, x E R means x = tan y and - 2 < y < 2'
,
232 NEW SENIOR MATHS: THREE UNIT COURSE

Co-ordinate geometry _ _ _ _ _ _ _ _ _ _ __

1. Angle between two intersecting lines:


tan a -;-'---''------'"'''- where ml and m 2 are the gradients of the lines.
+ m l m2
2. Division of an interval in a given ratio:
y1
( mx2 + nx , mY2 + n ) divides AB internally in ratio m : n
1
m+n m+n
(m.;;: ~ ';1, m~ ~YI) divides AB externally in the ratio m : n

3. Parabola ii 4ay:
Parametric equation: x = 2al, Y = al 2
Equation of tangent: Y = Ix - al 2 at point (2al, a( 2 )
XXI = 2a(y + YI) at point (XI' Yl)
Equation of normal: Y = -~+ at 2 + 2a at point (2al, a(2)

Equation of chord of contact of tangents from external point (x o, Yo):


XXo = 2a(y + Yo)
Calculus _ _ _ _ _ _ _ _ _ _ _ _ _ _ _ __
' sin () 1
1. IIm-()- = .
0-0

2. Integration by change of variable:

jf(U)~~dX = jf(U)dU
jf(X)dX = jf(x):.dU
3.

Y f(x) dy = f'(x) jf(X)dX


dx

sin-I x 1
a .../(a 2 - X2)
COS-I x -1
a .../(a2 - X2)
tan-1 x a
a a 2 + X2
1 . IX
sm- . +c
.../(a 2 X2) a
1 x
.../(a 2 - X2)
COS-I +c
a
a x
a 2 + X2
tan- 1 +c
a

dN
4. If dt = k(N - P), then N P + Aekl •
~~-~ .... -----------------------~~~

SUMMARY 233

5. Acceleration may be expressed in the forms:


dv
-
d- 2x..
X
dv -(-v
v- d 1 2)
dt' dt 2 ' , dx'dx 2

Projectile motion _ _ _ _ _ _ _ _ _ _ _ _ __

Horizontally Vertically
Acceleration x 0 Y= -g
Velocity x = V cos a y V sin a - gt
Position x = Vcos at y V sin at - !gt 2
2

Cartesian equation of path: y


2
= 'x tan a - f~2 (l + tan a) 2
V2 sin 2 a
Maximum height :y = when t = V sin
g
a

V2 sin 2a
Range :x
g
V2
Maximum range :x = g
when a = 45°.

Simple Harmonic motion ___________


x = a sin(nt + a) {a sin nt if x = 0 when t = 0
',= a cos nt if x = a when t = O.
v x:::: na cos(nt -li- 'a)'~
v 2 :::: n2(a 2 - x2.)
x:::: -n 2a sin(nt + a)
-n 2x
Period:
. T :::: 211'
n j! where j is the frequency.
Amplitude: a

Polynomials _ _ _ _ _ _ _ _ _ _ _ _ _ __

P(x) = a"x" + a,,_lxn - 1 + ... + a1x + ao


1. Remainder Theorem: If a polynomial P(x) is divided by x - a until the remainder R
does not contain x, then R P(a)
2. Factor Theorem: If, for a polynomial P(x), P(a) = 0, then (x - a) is a factor of P(x).
3. Relationship between roots and coefficients .
(a) Quadratic equations:
X2 - (sum of roots)x + product of roots = O.
(b) Cubic equations:
x 3 (sum of roots)x 2 + (sum of product of roots two at a time)x - product of
roots O.
(c) Fourth degree equations:·
X4 -: (sum of roots)x 3 + (sum of product of roots two at a time)x2
(sum of product of roots three at a time)x + product of roots = O.

----.-~-~-.-.-----~ ... -~--- .. -----


234 NEW SENIOR MATHS: THREE UNIT COURSE

4. Approximate solutions of equations


(a) Newton's method

(b) Halving the interval


~(XI + x~ wherej(x1) andj(x~ have oP~osite signs.
X3 =

Permutations _ _ _ _ _ _ _ _ _ _ _ _ _ __

n! = n(n - l)(n 2) ... 3.2. 1 = number of arrangements of n unlike objects in a row


np, = n(n - 1)(n. - 2) ... (n - r + 1) = (n n'')'
r.
= number of arrangements of n
unlike objects in a row taken r at a time.
Number of arrangements of n unlike objects in a circle = (n - I)!
Number of arrangements of n objects in a row when p are alike of one kind, q alike of
anot her k'Ind etc = n!...
p!q!

Combinations ________________

nC, =(n)= n(n - 1)(n - 2) ... (n - r + 1) = n!


r. r! r! (n - r)!
= number of selections of n objects taken r at a time
nCr = nCn_,·
nC, n'-ICr_ 1 + n-lC,

Binomial Expansions _ _ _ _ _ _ _ _ _ _ _ __

+ nC,xr + ... + xn
n
= Enc,x'
r=O

n
EnCran-rbr
,=0
n - r + 1 b. h
T,+ I
T, a
. f "
r ' IS t e ratio 0 any two successive terms In the expansion of
(a + b)n
Binomial Probability _ _ _ _ _ _ _ _ _ _ __
ANSWERS
CHAPTER 20

EXERCISES 20(a)
2. 51 3. 40 . 4. 84 5. 100, 50 6. 30
7. 230 8. 15, 40 9. 124, 118 10. 60, 40, 40 11. 90
12. 75, 75, 15 13. 68, 22, 22 14. 50, 50 15. 50, 25, 25 16. 28, 28, 40, 40
17. 42, 35, 42, 55 18. 40, 50, 40,40, 50, 50 19. 28, 28, 28
20. L AEC L BED, LACE = L DBE, L CAE = L BDE.
21. LABX = 90°, LABY = 90°
22. (i) angle in a semi-circle (ii) both 90° (iii) angles in same segment
(iv) L:1s are equiangular.
25. (i) L:1 ABC is isosceles (ii) angles in same segment (iii) adding same angle to both sides
(iv) angle in a semi-circle. (v) sum of angles of a triangle equals 180°
32. arc of circle passing through A, P and B. 33. circumference of circle with diameter AB.
34. 25°, 37°, 65°

EXERCISES 2O(b)
1. 82, 102 2. 65, 115 3. 54 4. 115 5. 76
6. 105, 100,55 7. 97,83 8. 122, 18 9. 74, 106,53,42 10. 109
11. 52 12. 71, 109, 109,71 13. 105, 100, 105 14. 108,72, 100 15. 123,37,33
16. (i) base angles of isosceles triangle, (ii) straight angle = 180°,
(iii) opposite angles of cyclic quadrilateral are supplementary, (iv) corresponding angles equal
20. (i) AB II DC, (ii) exterior angle equals remote interior angle.

EXERCISES 20(c)
1.62 2. 35 3. 62, 28 4. 34, 90, 56 5. 23, 67
6. 36 7. 28, 62, 56, 34 8. 36, 36, 54 9. 44, 68, 68 10. 90, 120, 30
11. 22,22,44,68 12. 25,65,25,65 13. 36, 52,~ ~ 14. 20 em
15. (x + 3)2 + 24 2 (x':' 21)2; 4 16. (a) 5 em (b) 31.2 em
17. 4.JW em 18. (a) 21 em (b) 14.../2 em 19. (a) 115 em (b) 46.J6 em
20. (a) 50° (b) 100° 24. 88°, 103°, 92°, 77° 29. (4 + 2.,.13) em
37. (a) ~ or 3; (b) 47rr 38. i(3 + 2.,)3)r

39. (a) 57rr (b) ~(57r + .,)3)r2; t(3 + 2.,)3)r 40. (a) 232.4 em (b) 46.4 cm 2

41. (a) &(2.,)3 - 7r)a2 (b) }(2.,)3 - 3)a

CHAPTER 21

EXERCISES 21 (a)
1. 20'06 em; 122-5 cm 2 2. 39-97 m; depression 0° 56' 3. 212-3 m
5. (a) 30 m, 40 m (b) 1O..J7 m 11. 10 em 12. 0'9067 km 13. 70° 32'
14. 16 m 15. 71 km/h 18. 131 m 20. 26-55 em 22. 6488 m
23. 147'4 m 24. 3'882 km 27. (a) 34'2 m (b) 18° 32' (c) 73°
236 NEW SENIOR MATHS: THREE UNIT COURSE

EXERCISES 21 (b)

1. sin A cos 2B + cos A sin 2B 2. sin 2x cos y - cos 2x sin y


3. cos 2x cos 3y + sin 2x sin 3y 4. 2!cOS 20 - .J33 sin 20 5. tan 0 + tan a
2 I-tanOtana
6tanA+I 7. sin (2A - B) 8. cos 2a 9. sin A 10. 0
·I-tanA
11. tan (0 - 20°) 12. tan 3a 13. sin A 14. cos 0 15. tan 2x
44 3 44 24· 24 56
16. tan (A + B + C) 20. (a) 125 (b) 5 (c) 117 22. (a) 25 (b) -7 (c) 65

24. (a) .J6 ~.J2 (b) 2 -.J3 (C).J6;.J2 25. (a) - 3--: (b) - ~ (c) 3.J7; 3rd quadrant

26. (a) 3 sin 0 - 4 sin 3 0 (b) 4 cos 3 0 - 3 cos 0 27 . .J2(l.; .J3)


28. (a) tan A (b) sin 2 0 29. (a) cos 40 (b) 21 30. (a) k sin 8x (b) 2 sin 2 0

31. (a) ~ sin 4x (b) sin 4x 32. (a) 1 + sin 20 (b) 1 - sin 2A 33. (a) 1 (b) 1

34. (a) k cos 2x (b) tan 2 0 35. (a) cos 6x (b) cos 0 47. sin (90° + A) = cos A
48. cos (90° + A) = -sin A 58. (a) p (b) q 59. 1 - tan 0
tan 0(1 + tan 0)
60-15+1 61 . .J2 - 1
. 4

EXERCISES 21 (e)

1. sin 60 + sin 20 2. cos 8A + cos 2A 3. k(sin 6A - sin 2A)

4. k(Sin 20 + sin 2a) 5. 1 - sin 2A 6. k[COS (30 - a) + cos (0 + 3a)]

7. sin (4a + (3)"'- sin (2a + 3(3) 8. cos 0 - cos 30 9. k(COS 60 - cos 20)

10. 2
1(.
sm 3x + sm
.)y 11. 1 + .J33 12 • 2
I( cos 2{3 - cos.)
2A 1+4
13 . - 4" .J3
2
14. sin 20 + sin 2cf> 15. k(Sin A + sin B) 16. t(~3 + cos 500) 17. sin 40 + sin 20

18. ¥sin (30 + 3cf» + sin (0 - cf»] 19. sin 6A - sin 2A 20. sin lOA + sin 4A

21. 2 cos 2x sin x 22. 2 cos (x + ~)sin ~ 23. -2 sin (x + ~)sin ~


24. 2 sin 0 cos a 25. 2 cos
ocos 2a
2 26. -2 sin (A + C) sin B

27. 2 cos
3x +
2
3y
cos
~
2 28. 2 cos
A +
2
B
cos
A - B
2 29. 2 cos 2A cos A

30.0 . (3A +
31 . -2 sm 2 3B) sm ~ B)
. (A 2 32. 2 cos A cos (B + C)

33. 2 sin ¥ i cos 34. 2 cos 2A sin 2B 35. - f


36. 2 cos (0 ; cf> + 45°) sin( 0 ~ cf> - 45°) 37. -.J3 sin 15° 38. .J3 cos 10°

39. _2 cos 2 A
2 -B .B
.sm '"
40. -'Y3sin20°
2
EXERCISES 21(d)
1 + 2( - (2 (2 + 2( - 1 4 + 6( - 4(2 2 - 2( - 2(2
1. 1 + (2 2. 1 + (2 3. 1 + (2 4. 1 + (2
ANSWERS 237

1 - (2 1 6t 2 + t 4 1 - 6t 2 + t4 1
5. -2-t- 6. 2t(1 _ t2) 7. 1 + 2t2 + t4 8. 1 + t2
1 + t2 !
9. 1 _ t2 10. t 2 11. t 12.
t
13. t 14. t 15. t = -'-=---'-=---'--''- 16. -5
c
EXERCISES 21 (e)

1. cosec 2 ex 2. siri2 1
f) 3. 1 4. 2 5. cos 2 0 6. sin (0 + <f;)

7. tan ( () - ~) 8. sin 2x
3
9. (a) - - (b)
.J34 -.J34
5
(c) 17
8

10 '(a).JIT (b) -5.JIT _5.JIT 15 240


· 6 18
(c)
7
11. (a) "8 (b) 161
4
12. (a) 5 (b) -53 -100
(c) 117 21. (a) -1 (b)
1
2
2 cos ~
22. 2 cos £±1:' 2
4Im([2 - m 2)
26. 1 or Ii1 28 21m
• [2 - m ' (P + m 2)2
2

CHAPTER 22

EXERCISES 22(a)

1 JI 21r 2 37r 77r 3 27r 47r 4 JI


· 3' 3 . 4' 4 . 3' 3 • 6' 6
77r 117r 197r 237r 77r 117r
5. 66 0 25' , 293 0 35 0 7. 6' 6
6. 12' 12' 12' 12
7r 77r 137r 197r 7r 97r 157r
9. 13 0 17', 103 0 17', 193 0 17',283" 17' 8' 8' -8-
8. 12' 12' 12' 12 10. 8
7r 37r 57r 77r 12 57r 77r 13 7r 157r 13. 71 0 34', 161 0 34',251" 34',341 0 34'
11. 4' 4' 4' 4 '8'8'8'8
27r 7r 7r 27r 77r 57r 7r 37r
14. -3' -3' 3' 3 15. -8' -8' 8' 8 16. 0, 27r 17. 0, 7r, 27r
27r 57r 7r 7r 77r 37r
18. 0, 3' 7r, 3' 27r 19. 6' 2' 6' 2
7r 7r 137r 57r 23 57r 77r 137r
22.128 0 12',171 0 48',308" 12',351" 48'
21. 12' 4'12' 4 '8'8'8'8
24.8" 26',151 0 34' 188" 26', 331" 34' 25. i. 7r,27r 26.0, 27r

27.0, S:' 7r, 1~7r, 27r 28. ~,567r, 6 1~7r 29. i, 547r 30.0, i, 7r, 547r, 27r
27r 47r 27r 47r 7r 7r 27r 47r
31. 3' 3 32. 0, 3' 3' 27r 33. 2' 2 34. 2' 3' 3
35. 68 0 12', 108° 26',248° 12',288" 26' 36. 90°,216" 52', -323 0 8'
JI JI 57r 77r 57r 117r 38 0 7r 37r 2 39 JI __ x :r;;: 57r 7r 57r
37 • 6' 4' 6' 6' 4' 6 • '2' 2' 7r • 6'" "" 6 40. :3 < x < 3
7r 117r 7r
41. 0 < x < 7r 42. 0 " x < 6" or 6 < x " 27r 43. 0 " x " 27r, except x = 2"
7r:r;;: :r;;: 57r 7r 37r 7r 7r 57r 37r
44 · 12 "" x "" 12 45. 0 " x < 2 or 2 < x " 27r 46. 4" < x < 2" or 4 < x < 2
37r 57r 7r 37r 57r 77r
47. 0 " x < 4 or 4 <x " 27r 48. 4" x '" 4 or 4 " x " 4
7r 7r 77r 47r
49. 0 " x " (; or :3 " x " 6 or 3 " x " 27r
7r 27r 47r 57r
so. 0 " x < 3or 3 < x < 3 or '3 < x " 27r
- - ...... - .~----.---

238 NEW SENIOR MATHS: THREE UNIT COURSE

EXERCISES 22(b)

2 J! 51r 3. 0, 1r, 21r


. 3' 3
1r
5. 26" 34', 63° 26' , 206" 34',243" 26' 6. 6 0, 6' 1r
1r 51r 1r 571: 771:
4' 4' 4' 27r 10. 128" 10' , 231 0 50'
8. 6' 6' 2 9. 0, 4 71:,
1r 1r 71r 41r 12. 0, 1r, 271:
11. 6' 3' 6' 3
EXERCISES 22(c)

1. 0) (x + ~ ) 2.J3 sin (x + ~ ) 13 sin (x: + 67" 23') (iv) -J5 sin (x" + 26 34')
.fi sin (ii) (iii) 0

2. 2 sin (x
(i) i )(ii) .Ji3 sin (x~' 56" 19') -J5 sin (x" - 26" 34') (iv) 3.J:i sin (x ~) (iii)

3. cos (x - ~ )Oi) 25 cos (x~


(i).fi 16° 16') 4 cos (x - i)(iV).Ji3 cos (x~ - 33° 41') (iii)

4. .fi cos (x + ~) Oi) 2 cos (x + ~)


(i) '.J65 cos (x" + 7" 7') (iv) .J34 cos + 30° 58')
(iii) (XO

5. 0) (a) 2, 6 (b) -2, I ~ 71: (ii) (a) .fi, ~ (b) -.fi, 5 71:
4
(iii) (a) 4, 6 (b) -4, 5 71: (iv) (a) 13, 22" 37' (b) -13, 202" 37'
6
8. 33° 41' 9. i, 71: 10. 1560 52' , 276" 52'
12. 511"
3 13. ~, 11" 14. -76° 43', 150" 27'

15. 95" 16', 155" 16' 16. (a)


71: 311" (b) 0
6' 2 ~ x < ~ or 3 11" <x ~ 211"
2
17. (a) -71:, -~, 11" (b) -~ ~ x ~ 11" or x = -11"

EXERCISES 22(d)

1 0 1r 211" 2 0 11" 311" 2 3 1r 11" 511" 31r 1111"


. '2' 11", 2 • '2' 1r, 2' 11" • 6' 2' 6 4. 8' 8
1r 51r 11" 211" 11" 1r 11" 11" 511" 11" 211" 311"
5. 4' 4 6. 0, 2' 3' 11" 7. -6' -2' 6 6' 2' 6 8. 4' 3' 4

EXERCISES 22(e)

1. n1r + el)n ~ 2. 2n1l" ± ~ 3. n1l" ~ 4. n1l" + Cl)n f 5. 2n1l" ± 211"


3
6. n1l" +~ 7. n1r + (-on ( -~)+ ~ 8. 4n1l" 9. n7r ± ~
10. 2n1r ± 31r or 2n1l" ±
21r
"3 11. 2n1l" 12. n1l" or n1r + (-on 2:11"
13. n1l" + n)n ~ or n1r + CW ( -~) 14.2n1r ± I and 2n1r 15. 2n1l" ± 1r 16. n1l" ± ~

17. 2n1l" or 2n1r ± 3211" 18.


n1r
2 _
(I)
n
411" 19. n1r or n1l" +4
1r
20. n1l" ± 12
1r
+
1r
6
21. n 1r
2
+ n)n 8 22. n1l" 24. n1r or 2n1l" ± 41r or 2n1r ± 4
11" J! J!
25. n1l" ± 3" 28. 2n1l"
4 3
ANSWERS 239

29. n.360° ± 101° 32' + 36° 52' 30. 2n7r ± ~ - ~ 31. n;


2 7r 2 7r 7r n7r 7r
32. :3n7r ± 6 or n7r 33. 3mr ± 6 or 2n7r ± 2" 34. "2 35. 2n7r ± :3
36. n7r 37. n 7r
2
38. n7r ± 1~ +~ 39. n7r + (-1)n I+~
27r
40. 2n7r or 2n7r ± 3 41. 2n 7r ± .37r - .37r
7r
43. n7r +4
46. n7r 47. n 7r + ~
2

3 or 2 n7r +- .37r
49 . n7r 50. 2n7r or ~n ± ~

CHAPTER 23

EXERCISES 23(al

3. all x 4. nox 5. x .,.; - t or x ~ 4


6. x.,.; -2 or x ~ 7. x < -3 or 1 < x < 2 8. -2 < x < -lor x > 5
. 1 4
9. x.,.; 1 10. x < -lor x > 3 11. 0 < x < (; 12. x < -3 or x >
3
1
13. x .,.; -3 or x >
2
14. x > .31 or x < 1
3 15. x
1
< 2 or x ~
3
4
4
16. x < 1 or x > 3 17. 2 <x <3 18. .3 .,.; x < 2 19. 1 <x <3
20. x < -4 or -3 < x < 3 or x > 4 21. x>

22. x < -3 or x > 130 23. I~ < x < 1,30 24. x < -6! or x > 3!
25. x «: -I! or -1 <x < I~ orx > I! 26. 0 .... x .,.; 2 27. x = 0
28. -4 < x < 6 29. I .,.; x < 2 30. -1 < x < 0 or 0 < x < 2

EXERCISES 23(bl
4. 10° 18' 5. 40° 36' 6. 45°
7. 2 9. (a) (-2, 3) (b) (1,0) (c) (Ii, -~) 10. (a) (4, -3) (b) (4, -3)

11. (3, 4), (5, 6) 12. (a) (l, 2) (b) (3, ~) (c) (5!, Ii) (d) (3, -~ )
14. 3: 1 15. (1,2)

CHAPTER 24

EXERCISES 24(a)
1. no turning points; asymptotes: x = O,x = -2, Y 0; range R
2. (-3, t
)min; (I, -2) max; asymptotes x = 0, x 3, y = I range: y ~ for y ",. -2.
3. no turning points; asymptotes: x = ± I,'y 0; range R
4. (-3, -~)min; asymptotes: x = 1, Y = 0; range: y ~ -k
240 NEW SENIOR MATHS: THREE UNIT COURSE

5. no turning points: asymptotes: x = ± 3, Y := 0; range R


6. (-I, ~)min; (I, I}) max; asymptotes: y := 1 range: k..;; y ..;; I!

7. (0,0) min. ( ~, -8) max; asymptotes: x = 1, x = 2, y = 1; range: y ..;; -8 or y ~ 0


8. (0,0) max; asymptotes: x = ± 2, y = 1; range: y ..;; 0 or y > 1.
9. no turning points; asymptotes: x ± 3, y 0; range R
10. no turning points; asymptotes: x = 3, y = 0; range: all R except 0 and -~.
11. (0, 4) max; asymptotes: y 0; range: 0 < y ..;; 4
12. n, -2) max;( 3, ~) min; asymptotes: x 0, x = -3, y = 1 range: y ..;; -2 or y ~~
13. (0, 1) max; asymptote: y = -1; range: -1 < y ..;; 1
14. ( ~,-4) min; (-2, 1) max; asymptote: y = ~; range -4 ..;; y ..;; 1

15. (-1, t) min; ( 2, ~) max; asymptote: y 1; range t.; y ..;; ~


16. (~, 3) min; asymptotes: y 4x2 and x 0; range R

17. (2, 2) min; n, -3) max; asymptotes: y = !x - ~ and x = -!; range: y ..;; -3 or y ~2

EXERCISES 24(b)

1. 2y x+4 3. y = x1, x "* 0


5. y = ..)4 - X2, -2 ..;; x ..;; 2 6. y := x2 - 6x + 4, x ~ 3 7. 2y = 3x + 8,0 ..;; x ..;; 4
8. y = 1 x2l3, -1 ..;; x ..;; 1 9. y = x 2 -4x +3 10. 2y2 = X + I, -1 ..;; x ..;; 1
11. y2 = 8x, x ~ 0 12. 3xl + 4y2 = 12, -2 ..;; x ..;; 2 13. y ~ '-"4 - X2, -2 ..;; x ..;; 2

EXERCISES 24(c)
1. y = x 1; x + y = 3 2. y = 2x + 1; 2y + x 12 3. y = 2x 5; 2y + x 5
4. 4y = (x + 2); 2y + 4.J2x .J2 5. y = 2x 12; 2y + x = -4 6. y ,= 1; x 2

7. x 2 := 8y; x 2 -4(y 3); (±2.J2, l) 8. (a) ~,~ (e) C~, ~~) (d) (t - ;. ~ + ~)
9. (a) y = tx at2 (b) (at, 0); (0, -at2 ) -i (e) xl y 10.f t'2 =: 2 (y -,~); (0, !); (0,1)
12. x 2 = 9y 13. (0,2t 2 + x =
4); 2 2(y - 2) ~. y- ~ 'J . fl· ~~

14. (a) (p2 2; I, q2 2~ 1) (e) a(p ~). i~2 + ;2) 15. 0, 1), (-6, 9), (8, 16)

16. X2 := 2a ~ - i} (0, ~); (0, a) 19. x 2 = 4a(y - 4a)


22. (a) (-2at, at l + 4a) (b) x 2 = 4a(y 4a)
23. (a) C2at, 2a aP) (b) x 2 = -4a(y - 2a) 26. 2; (± 6a, 9a)
27. (a) 3x - 4y + 8 = 0 (b) 2x + y :; 5 (e) 5x + y = 6 28. (3, 2) 29. (3, 1)

EXERCISES 24(d)

1. 2 4. ° 5.2 6. 1 8.
2
3' 9. 2 10. 2:3
EXERCISES 24(e)

1
~(X3 +
1. S(x2 - 1)5 +
-
C 3. 1)3/2 + C 4. l(2t + 1)4 + C
---------------------------------------------------~

ANSWERS 241

5. 2 .,/(x2 4) + c 6. k(X2 + X + 2)6 + C 7. -i(2X + 3)-2 + C

1 -1 1
8. -14<3 2x)1 + c 9. 2(3x2 + 2x + 5) + c l{). ts< x 3 - 3x
2
+ 1)5 + C

11. ~(X3 + 1)5 + C 12. -"/0 - t 2 ) + C 13. ~(3X - W13 + C

2 1 2 3
14. -3(1 - (2)312 + c I S . -3<a - x 2)312 +c . 16. g(Z2 + 1)413 +c
17. ~5(y + 1)512 ~3(y + 1)312 + c 18. - _1_ - +c
x-I 2(x

19. ~(2X 1)312 + ~(2X 1)112 + c 20. ~(1 + X3)312 + c

1 5
21. Y = 3(x2 - 4)312 +3 22. f(x) = 3 23. x .,/ t 2 - 21 +4 +8
1 19 26. x = 4 _ _4_·-
24. r = 1 - 3i7:F=~ 25. Y = 3(2x - 1)3/2 + 3"
81 + 1
EXERCISES 24(f) /~
?~
1. } 2. ~.J3 3. • 4. 0 5. 16
6. 0 7. 1 8. 69i
4 - 2..J'i 9~ - 3
9. -61 11.
3 12. :51 13. 0 14.
8

15. ¥ 16. 8i 17, 7rr 18. 211 19. 2"/3 20. -51!
21. t units 2
22. 1 unit 2 23 • 15
8 umts. 2 24. 12(4"/; - 1) units2 25. 2 units 2

EXERCISES 24(g)

1. x + ~ sin 2x + c 2. x - ~ sin 2x + c' 3. ~ sin 3 x + c


5. - ~ eos 4 x + c 1 . 5
6 . Ssm x +c 7. -3eos3 x
2
+c 8. ~ sin 2 x + c
xl.
9
'22 smx + c 10. x + sin x + c 11. g7r - 41 12. I~
1
13. 16
1 1
14. 0 15. 2 16. 2 17. 34
21. 87r 22. 31 23. i 24. 3
\
28. ~ 29 64
. 3 30. I~

CHAPTER 25

EXERCISES 25(a)

1.1 em/min
7r
2. It m/s; ~ m/s 3 em/s
3. "3
7r
4. -1 cis; -50 cm 2/s 5. It m/s
4
6. 0'1 em/s 7. ;, m/min 8.
.,/3 , em 3 /min
9. 288,7r 10. 5g gm per cm 2/s
11. 2.,/3 m/s 12. 1~ em/h 13. (a) 257r em/min (b) 0'8 em 2/rnin
I
14. 8 7r em/min
15. 19'2 m/min 16. 12 17. 45.,/2 km/h; 50 km/h 18. 43'1 km/h
19 . .2.- mlmin
7rr
20. ~7r mlmin 21. 1'6 em/s 22. 2~ mlmin 24. 4- emls
25. (a) 15 units/s (b) 30 units/s 26. 0'0732 m/min 27. (a) 3 em 2/s' (b) 3 # em/s
242 NEW SENIOR MATHS: THREE UNIT COURSE

t I
28. (b) r = 1
20
29. (b) (i) 7 units (ii) 4 units 30. 47r cm 3 /min 31. 9 cm/s

EXERCISES 25(b)

. 1
1. v'TO 2. v = -J 14x - 2X2, 0 .;;; x .;;; 7 3. v2 = -2(2x2 - 3x + 1); x =2
4. 8 m 5. v = ± -J2 sin x 6. y,---X----.1-s-;-in---c2""""x-+-;--;1
7. v = -J2x2 - 2x3 + 4 8. -3 9. 10 10. x = e l - 4
32t 3t + 2 d 2x _ 3
11. x(t) = 8t + 1 12. (a) x = t+T (b) dt 2 = 2~~ - x) 13. (a) x + 3 (b) e - 3

d 2x2 = -k
14. -d 2 15. (a) 4.J30 mls (b) 6"""
10 mls
t x
g 2
16. (a) v2 = VZ - 2 R + 2gR2 (b) x = 2 R ,,,,. (c) V > 11200 mls
g x 2gR - y-
17. (a) v = e-X (b) x = !oge(t + 1)

EXERCISES 25(c)

7r
1. (i) 23 , ~, 2 (ii) 23 ,
7r
2~1O, 2-J1O 2. x= -5x, T = ~~, a = i
2 2
3 • X·, = 7r
-4x (I') 4 S (1'1') 5 cm (... ) h-J3 I
III -4- cm s
(.IV) 8
57r cmI s2

.. 27r" 7r
4. (i) x = -25(x - 2); (a) x = 2 (b) 5' 4 (ii) x = -16(x + 1); (a) x = -1 (b) 2' 3
(iii) x = -2(x - 1); (a) x = 1 (b) -J27r, 2
5. (a) x = -5 sin ( nt + i) (b) x = -a sin nt
6. yes; -36 m/s 2; 12 mls 7. x = 4 sin 4t + 3 cos 4t; 5 m; 20 mls
8. (a) x = sin 4t (b) 1 9. 7r s, -J13 cm 10. -Ja 2 + b 2; ny'---a"2---;-+-b";2
x = -4(x - ---- 37r2 57r2
11. 4) (i) x = 4 (ii) 7r s (iii) 12. 27r cm/s; 8 cm/s 2; 8 cm/s 2

13. 7r, 10 14. (i) i, 1~4 i, (ii) 1 15. vmax


_ 47r.
-
2
3 mis, acc max -_ 87r9 mls 2
16. (i) 5 cos ~t (ii) 5 7r (iii) 5, /2' 12 (iv) 5 7r (v) 53~2 17. 5.J2 m/s; 3.J2 mls
6 6
18. 4 m; 2.J7 mls 19. ~ vibrations;'s; 10 20. 4 m/s; 6 m/s 2 21. 1'02 s
2
22. 7r s; 20 m/s 23. Between 11'44 a.m. and 8'16 p.m.
24. (a) 10 a.m. (b) 7'33 a.m. (c) 12'27 p.m.
EXERCISES 25(d)

1. (a) 5's (b) 50 m 2. (a) 5 m (b) 2 s (c) 15 m 3. 36° 52',60° 15'


4. 45°, 15-J2 mls 5. 36° 52' 6. (a) 6 mls (b) 20 m 7. 6 s; 420 m
8. (a) 39 mls (b) 22° 37' (c) lIt m (d) 108 m 9. (a) (0, 0) (b) 25 mls (c) (30, 20)
7.J2
10. 15° or 75° 11. (a) 1O.J2 s (b) 4 km 13. (a) (24, 2)
14. (a) 40 mls (b) 160 m 15. (10,.)3, 23t) 16. (a) 5.J'i9 mis, 21 ° 48' (b) 5 m
17. i2'1 mls 18.45°,71° 34' 19. (a) 50m/s (b) 53° 8' (c) (120,80)
20. ~Jfi. .J2ib 21. 4'8 m 22. 7 mls 24. 25 mis, 36° 52"

EXERCISES 25(e)

1. 2 h 54 min 2. 11 °C 3.2 min 4. 32'2 min 3


6. (b) 16

-,-~ _ _ _ _ _ _ _ _ _ _ _ _...... _~~~~._.c--"-="


-.----,---------,..---------------------
~ ..

ANSWERS 243

7, (b) (i) -2'2 grams/min (ii) -1-5 grams/min 9. (c) 51-6 min 10. 3'84 h
11. (b) 0'08 (c) 0'04 grams/hour (d) 3 < y." 4 12. (b) 227 m (c) 127 m

CHAPTEFt26

EXERCISES 26(a)
1. yes 2. yes 3. no 4. yes 5. yes
6. no 7. no 8. yes 9. no 10. no
11. -2 " x ,,0 12. x ;;;. 2 13. x < -2 14. x " I! 15. x ;;;. -3
16. I-I(X) = ~(X + 4); domain R, range R. 17. I-I (x) = ..j(x + 1), x ;;;. -1, rex) ;;;. 0
18. g-l(X) 2
= x + 3, x ;;;. 0, g-I(X) ;;;. 3 19. FI(x) = ..j(9 - X2), 0 " x .;; 3, -3 "I-I(X) .;; 0
20. h-I(x) ~x, domain R, range R. 21. FI(x) = ""2 - ..jx ,x ;;;. O,FI(x) .;; -2
22. I-I(X) = ..j(x + 1) I, x;;;. 0,F1(x) ;;;. 0 23. FI(x) = eX 1, domain R,r(x) > -1
24. I-I (x) x 2 - 4x + 6, x " 2,FI(x) ;;;. 2 25. I-I(X) = 4 2x - X2, X ;;;. -1,f-I(X) .;; 5
26. FI(x) -log2 x, 0 < x < I,I-I(X) > 0 27. h-I(x) = 1. 1, x > 0, h-I(x) > -1

28. (b) X" 0 or x;;;. 0 (c) FI(x) ==JI ~ x,O < x" 1 0:FI(x) = -Jt ~ x,o <x " 1
EXERCISES 26(b)

1. (a) ~ (b) 0 '(c) ]I (d) (e) Does not exist


3 3
(f) -0'5 (h) ]I (i) ]I
(g) 0'123 (j) ]I
4 3 4
2. (a) 0'9273 (b) 0-4334 (c) 1-117 (d) 0'4636 (e) 1'176
(f) 0'9273 (g) 1'054 (h) 1'876 (i) 0'8788 (j) 0'6435

3. (a)
.J3
2 (b) 5
(c) 65 0
(d) 180 0
(e) -
1
13 2
(f)
1 119 (h) !f1..
2 (g) -169
4-

5. (a) 0 (b) 0 16 (d) 24 63


(c) 65 (e) 65
25
7 88
(f) 25 (g)
9
7. (a) domain R, range -11" < y < 11" (b) domain -1 .;; x " 1, range -1 .;; y " 211"
(c) domain -2 " x .;; 2, range (d) domain 0 " x " 1, range 0 " y " 411"

8. (a) ~ (b) f 9. x cot y, 0 < y < 11" and x € R (a) "311" (b) 6 (c) 1 1
..fi (d) 5..fi

10. -1 .;; I(x) " 1; P-1(X) ~(Sin-l x + 3), -0'8415 " x .;; 0'8415, I " p-I (x) .;; 2

11.0.;; X" I;/(x) == ~(SinX + 1), -~" x.;;~, 0 "/(x) " 1


EXERCISES 26(c)
1
1. -2 " x .;; 2;1' (x) = -;=;===::;;:, -2 < x < 2 2. 0.;; x.;; I; I' (x) ~2'0<X< 1
2vx - x~
3. x € R; P (x) 1 +\x2' x E R 4. 0 " x " 2; I' (x) = - j;iif====;;; 0<x<2

1 1 1
5. x ;;;'2-;P(x) == ~,x > 2- 6. x€R;I'(x) =
xv2x - 1
7. -I.;; X" I; I' (x) = -r.=====;, -1 < x < 1

.. __ .. _---- ..... - .... ~---.--- .... -- . .. - --- .._--- -.--....- . -


244 NEW SENIOR MATHS: THREE UNIT COURSE

8. !~ x ~ !;!'(X) = vI ~ 4 :Z' 0
x
< x < ! !<x<o
2
9. -1 ~ x ~ O;!'(x) -:-;::'===,-1 < x < 0 10. 0 ~ x ~ I!, (x) = V~;:::X(=l=-=X=)= , 0 < x < 1
V-x(l-x)
11. x >rrorx<-rr;f'(x) = x> 71" V 271" 2,X<~7I"
x X - 71"
12. x€R;!'(x) = 1 +Pp2x2 ,XER 13. -1 ~ x ~ 2;!'(x) =.J2 + ~ _ x 2' -1 < x <2
14. x ~ I;!, (x) = - + tan-I x < 1
. 2(2 - x)
15. -I ~x~ I;!'(x) = hCOS-1 x Sin-Ix, -1 <x< 1
1- x 2
2
16. x € R; f' (~) = 1 + x 2 ' X > 0 X < 0, not defined x = 0

17. All x, except x = 0; !'(x) = x *0 18. All x except x = O;!' (x) = 0, x *0


19. x ~ I or x ~ -l;!,(x) = -::-;;:::::::;==-, x > I or x < -I

20. x € R;!'(x) = I cos,x2 ,x E R


+smx
21. -I ~ x ~ I;!'(x) = b I-x
+ 2x sin-I x, -1 < x < 1

22. - 471" ~ x ~ 4;71" f'(x), = t=i'===:==:;;= <x<1f


4
-I, -71" < X < 0
23. !' (x) = I, 0 < x < 71" ; 0 ~ f(x) ~ 71"
{
not defined, x -71", 0, 71"
til _ -I :!! _
24. dx - ~ 25. (a) y x (b) x + y 2 (c) 4y - 2x + 71" - 2

26. f-'(x) = 2 COS-I ~, -3 ~ x ~ 3, 0 ~ fl(X) ~ 27r; i 27. 0 28. 0 29. 0

I, -71" ~ < 71" 71"


X
2' 2 < x < 71"

30.!'(x) =
{
-1,; <X';;;1[

71" 71"
not defined, x
2' 2
EXERCISES 26(d)

1. sin-I ~ +c 2. tan- l ~ +c 3. sin- I x +c X


4 • cos -I .J5 +c
. -I X
5• sm .J2. + c 6. tan- 1 ~ + c 7. ~ tan- l + c i 8. J2 tan- 1 +c

9.' COS-I ;6 + c 10. ~ sin- 1 2x + c 11. ~ tan- l 3x + c 12. 12 tan-I ~x + c


13. i tan-I ~ + c 1 ' 1 13
14• .J3 sm- " x +c 15 1
. 3" cos-
1 3x
4' + c 16 1 . -I 2x
. 2sm .J5 +c
71" 371"
17. ~ 18. ~ 19. 12 20. 4 21. 471" 22. 371" 23. 471" 24.
71"
6
71" I
25'.i 26. 27. 4 + 2: loge 2 28. 2:71" 29. 2:71" 30. 9
71"

32. 7I"t2 33. f 34. f 35. ~- "f + I 36. ~ ! loge 2

37. Y = 1 + sin-I x 38. f 39. 16 - 4 40.


71"2 ')
6
U" 41.
3
-- -----------------, -~~------------------------

ANSWERS 245

CHAPTER 27

EXERCISES 27(a)

1. -1, -97, 2a 3 - 4a 2 + 2a - 1, -37 2. 0, 0, a4 - 2a 3 + a 2 - 4


3. 5! 4. 2,3
S. 3, 4, 12 6. 3x + 4 and 13 remainder
7. 2X2 - 5x + 6! and -7! remainder 8. x 2 + 1 and 0 remainder
9. x 3 + 3x + 2 and 2 remainder 10. x 2 + 3x + 9 and 0 remainder
11. x 3 + x 2 + 3x - 12 and 16 remainder 12. -x2 + 5x - 11 and 34 remainder
13. -7 14. 74 15. 4 16. 142
17. -3ol~ 18. 14 19. a = 3, b = 8 20. m = 2, n = 1
21. 11 22. 1, 3 23. 4, -9

EXERCISES 27(b)
. 1. (x + l)(x - 2)(x + 5) 2. (x + l)(x + 7)(x - 6) 3. (x - 2)(2x2 + 15x - 13)
4. (x - 2)(3x + 2){2x + 1) S. (x + 1)(3x + l)(x + 2) 6. (x + 4)(2x + 3)(x - 2)
7. (x - 5)(2x + 1)(3x - 1) 8. (x + 3)(2x + 1)(2x - 1) 9. (x + 4)(3x - 2)(2x + 5)
10. (2x + 1)(3x + 2)(x - 6) 11. x(x - 2)(x - 3) 12. (x - 2)(x + 2)(x + 2)
13. (x - 1)(3x - 1)(2x + 1) 14. x(3 - 2x)(1 + 3x) 15. (x - 1)(2x - l)(x - 2)
16. x(2x - 3)(x + 3) 17. (1 + x)(2 + x)(3 - x) 18. (x + l)(x + 2)(x - 4)
19. x - 2 20. 7, 12 21. -1, 30 .
22. a = 8, b = -4 23. -1, 16; (2x - 1)(3x - 2) 24. a = -1, b = -6
25 (x
.
+ 2) (x _ ~
2
_ .ff7)(x _
2
~2 + .ff7)2

EXERCISES 27(c)
1. (a) (i) 0, 2, -3 (ii) -3 < x < 0 or x > 2 2. (a) -2 ~ x ~ 0 or x ~ 2
(b) (i) -I!, -1, ~ (ii) -I! < x < -1 or x > ~ (b) x ~ 0 or x = -2
(c) (i) 0, 2 (ii) 0 < x < 2 or x > 2 (c) x ~ 2
(d) -2 ~ x ~ -lor x ~ 4
3. (a) -(x + l)(x + 3)(x - 2) 4. (a) (x - 1)(x - 2)(x - 2)
(b) (i) -1, -3, 2 (ii) x < -3 or -1 < x < 2 (b) (i) 1, 2 (ii) 1 ~ x < 2 or x > 2
S. (a) -(x - l)(x + l)(x - 2) 8. x = -3,2
(b) x < -1 or 1 < x < 2

EXERCISES 27(d)
ex (3
1. 5x 2 + 74x + 5 = 0 2. (a) -4 (b) 1 (c) 14 (d) -52 3. ~'~
q 2 - 2pr
4. (a) _'1 (b) "--------;;---''--
r .r2
S. x 2 - 22x - 75 = 0 ,. -2, 2, 3 7. 2'1 2, 8
2 _ ~\
9. 6, 3' 1 10. 3x 3 - 4x 2 + f4J = 0 11. ~, 5(2 ~ .J3)

1 2 1 1
12. -12, -20; 1 ± .J5, -4; -3 ± .J5, 4 13. 2' 3 15. (I) 9 (ii) -2 16. 2' 2' 2

17. ~, -~, 1 ±.J3.. 18.~, ~, 1 ± .J2 19. ~, ~, ~ 20. ± .J3, !, 2


21. x3 + 2px 2 +p2X - q2 = 0 22. -a "!.../l 23. .J2, ;j2, 1 + .J2, 1 - .J2
/ 2 1
24. 1 + .J2, 1 .~ .J2 27. k = 3; 4'
/
246 NEW SENIOR MATHS: THREE UNIT COURSE

EXERCISES 27(e)

1. 3'08 2. 2'91 3.0'62 4. 1'30 S. 1'15


6. 3'081 7. 2'912 8. 0'615 9. 1'303 10. 1'146
11. (a) 2'154 (b) 2'236 (c) 3'873 (d) 3'037 (e) 2'024
12. 1-35 13. -2'89

CHAPTER 28

EXERCISES 28(a)
1. 120 2. 120 3. 336 4. 720; 360 5. (i) 120 (ii) 11,880 (iii) 6,720
6. (i)720 (ii) 336 7. (a) 720 (b) 1440 8. (a) 720 (b) 144 9. 240
10. (a) 48 (b) 72 11. 3600 12. (a) 4320 (b) 720 13. (a) 1152 (b) 144
14. (a) 12 (b) 128 15. 216 16. 0) 720 (ij) 72 17. 3 18. 3
19. 0) 120 (ii) 3600 20. 4 21., 34,560 22. 30, 240
2
23. (i) 1440 (ii) 2520 (iii) 720 24. 90, 720; 9 25. 181, 440; 1440
26. (i) 141, 120 (ii) 20, 160 27. 120 ,28. 288 29. 256
30. 1152 33. 2880; 864 34. 37800

EXERCISES 28(b)

1. 56 . 2. 40 3. 20 4. 1470 5. 19,600 6. 56
1
7. (i) 70 (ii) 140 8. (i) 4804 (ii) 21,808 9. 700; '1 10. 73,080
11. (i) 280 (ii) 742 12. 595 13. 98 14. 3838380 15. (a) 720 (b) 45
16. 1946 17. 466 18. 2 n - 2 19. 375 20. (i) 336 (ii) 40,320
21. 756,000; 86,400 22. 7200; 4320 23. 1440 25. 10
26. 5 27. 1260 28. 92 29. 105

EXERCISES 28(c)

25 3 149
1. 16660 2. 42 3. (a) U (b) 1.-
44 4. 198 5. '21
35 2
6. 10829 7. 81 8. '11 9. 5 10. 35
1
11. 17,576,000; 67600 12. 55 13. 3'1 14. 6"
5 10 10
15. (a) 63 (b) 21

16. 0 2 3 4 I 5
tf_:. '4

0'041 0'2517 0'419~'" 0'2400 0'0450 0'0020


2.- 25 25 2.- 5 3 1
17.
44'66'66'44'132 18. 14' 4 19. 2: 20. 0'3412 21. 165 22. 225
2 5 1 2 1 1
23. (a) 9 (b) 9 (c) 45 (d) 45 24. (a) 3" (b) 3" 25. (a) 35 (b);; (c) 35

26. (a) 35 (b) 35 27. (a) 115 (b) 32 'i


A~
(c) 28. 2:1
2109
29. (a) 33320 (b) 54145 (c) 9520

CHAPTER 29

EXERCISES 29(a)

1. x lO + 5x Sa + lOx 6 a 2 + lOx 4x 3 + 5x2a 4 + a 5


2. 16x4 + %x3y + 216x 2y 2 + 216xy 3 + 81 y 4
'"
-------,,---- -----------_._.. ,

ANSWERS 247

3. a 6 + 6a 4 + 15a2 + 20 + !12 + ~4 + ~6
a a a
4. XIO - 8
15x y 2 + 90X6y 4 - 270x4y 6 + 405x2y 8 - 243 y IO
32a 5 4Oa 4 20a 3 15a2 135a 243
5. 243 - 27b + 3b 2 - ~ + 8b4 - 32b5
6 16y4 - 48 y 3x 2
.
+ 54y 2x 4 - 27yx6 +81x8
16
7. X18 - 12x i3 + 60x 8 - 160x3 + 240x-2 - 192x-7 + 64X- 12
JO
8. a _ 15 a8b + 45 a6b2 _ 135 a4b3 + 405 2b4 _ 243b5
32 16 4 2 2 a

9.( ~J -7(~y + 21(~y - 3~a + 3!b - 21(~y + 7(~J - (~J


10 . 189 5 3
4 m.n 11. 693 12 560 a3b4
. 81 13.
-n(n - l}(n - 2) n-6
6 x
x
15. 56 16. -n, 28 ,3 9 17. 378 18.13,2 12 ,3 3
(n - 1)!
19. -15360 20. 9,2 15 21. . 23. -5376 24. 495
(r + I)!(n - r - 2)!
8 26 (2n)!, 2n 27 _ 63 5 28 189 5b12, 21 a 4b 15 29. 416
25. 24ij • (n!)2 . 8x • 8 a , 16
30. IOx 2 - 8 31. 1'0829 32. 1207 33. (i) 0'90438 (ii) 0'960751
34. (i) -5 (ii) 0 (iii) -12 35. -6 36. 64 - 576x + 2352x2 - 5706x3
37. 1 - 6x + 12x2 - 42x4; 0'2128 38. 8 40. 7

41. 7 42. 9 44. (i) 3(11 - r)y (ii) (r - 9)b


43. 1,2,6
. 2rx 3ra
45. (i) T6 (ii) T9 (iii) T5 (iv) T5 and T6 (v) T4 (vi) T4
46. (a)x= 1 (b)x=-2 (c)x=3 50.C~I)-1 51.1-(l-x)n

EXERCISES 29(b)

1. 0,08883 2. 0'0002441 3. (i) 0'8192 (ii) 0'0272

5. (i) 0'8916 (ii) 0'3415 6: (i) 0'3087 (ii) 0'4718 (iii) 0'03087
~~t..
7. (i) 0'001573 (ii) 0'001678 8. 31~ ~5 9. 0,04031 10. 0'3174

11 • (I') 64
1 ('.) 27
II 64
(' .. ) 27 (.) 3
III 32 IV 32 12. 0'5155 13. 8"
5 14. ~
16
1~ (i) 0'9893 (ii) 0'9192 16. 4 18. 0'394 19. 6 20. 0'0625

21. 0,8824 22. (i) !~~~ (ii) G~Y 23. (i) 0'6561 (") 113
625 III 1250
II
("'). 177

24. 0'0041,0'0368,0'1382,0'2764,0'3109,...0'1865,0'0466 (a) 10 (b) 117


25. 19 26. 329 27. (i) 0'04422 (ii) 0'0179
28. (i) 0'0055 (ii) 0'2903 (iii) 0'9038 29. 155, 41, 4, 0, 0, 0
30. 18; (i) 0'003457 (ii) O'O3:~6
7
31. 0'868 33. (a) 10 (b) 2 (c) 18

TEST PAPERS

TEST PAPER 1

1. (i) 7 (ii) (a) 100 (b) 200 2. (i) (a) ~ (b) ~ loge 2 •• )
( II
7[2
-
4
7r
4. (i) -2 < x < 0 or x > 2 (ii) (a) 2n 7r or 2mr ± 23 (b) n 7r if n is zero or even, n7r - ~ if n is odd,

I 5. (i) ~ 6. (i) (a) ( 2, ~). (-2, -~) (b) y =0 (c) -~ "" y "" ~ (ii) 5 mm2 /s 7. (i) 45 0 ,71 0 34'
'~. -_
... __ .. "--'--'--"'. __. ... \ .... _. --- - -- ------ . . .- - - - - -
/
248 NEW SENIOR MATHS: THREE UNIT COURSE

TEST PAPER 2
1. (i) cos-12x (ii) (a) 0 ..: x ..: 2 (b) Pl(X) = (2 - X)2, x..: 2;Pl(X) ;;;. 0
2. (i) 3'162 (ii) x = 2 3. 0) 12C7 25 .3 7 OJ) (a): (b) 6

4. (i) (a) (x - 2)(x + 3)(2x - 1) (b) -3 < x < ~ or x > 2 (ii) 5.9.11. 28

5. (0 -1, I! (ii) (a) 1 cos.xz (b) 4!!


+ sm x
6. (i) 2 sin (2X + ~) max. is 2 when x 2 min. is -2 when x = ~;
(ii) a O;Pl(X) = ~,-3 ..: x..: 0

7. (i) 64 (ii) ~ (iii) (a) ~,547r (b) 2.fi

TEST PAPER 3
z
1. (a) x at, Y al 2+ (b) XZ ~) 2a (Y - (c) vertex (0, i) focus (0, c ; a)

2. (i) (a) 36 (b)


=

~
C

(ij) (a)
1
nc1or- 0) (b) (~r G) (c) n = 3

1~ ~
2
3. (i) 1 min (ii) 1'7 s 4. (i) v ,.; - 2(x ; 3x + 2),1 ..: x ..: 2 (ii) (a) (b)
6. (ii) (a) x =1= ±2 (c) x = 2, x -2, Y = 0 (d) a = 2'4
7. 0) (a) -(x: 2), x =1= 0 (b) ':::.=-='::: (iii) (c) 7r
X smx + cos x

TEST PAPER 4

1. (i)40'4m (ii)(b)2n7r± ~ 2. (0(0,2) (ii)3 ±2..,[5, 13. (i)(a) a = 4,b = 2 (ii)-x 2e-x ;0'486
4. (a) (0, 0) (b) x = ± I, Y -1 (c) (0, 0) min. (d) Y ;;;. 0 or y < -1
5. (i) (b) f· (ii) 5";5 3 2";2

6. (i) 160 cmlmin (ii) (b) 4 s; 5 (c) 5~.J3 mis, 5;2 m/s 2
7. (i) 2 min (ii) (a) 20";3 m (b) 20 m

TEST PAPER 5

1. (i) 192
.
(ii) Ii
7 2. (i) (a) ;~! (b) 66

3. (i) .J2 cos (0 + ~); 0, (ii) (a) tan x (b) cos 4x

4. (i) (a) - 4t5 (b) "'---12= (c) 3~;5- +2t (ii) a 5. (ii) 0'625

6. (i) ± 3 (II.• ) 5
1 . 5
sm x +c (iii) -lOx; 20 (iv) 1'5 7. (i) (a) 78° (b) 20°

TEST PAPER 6

2. (i) 115° 20',318° 24' (ii) n7r ± ~ 3. (ii) 45° (iii) y = 2x - 4; x + 2y = 12

4. (ii) (a) v = e-x/2 (b) x 2 loge (1 + ~ )


5. 0) I, I, -2, -k (ii) Xl - 8x3 + 19x - 15 = 0 (iii) I~
ANSWERS 249

6. (i) (a) 4.J2 s (b) 40 m (ii) (a) x == 2 (b) ~ (c) 2'5

TEST PAPER 7
2 -sin x -I
1. (ii) (a).J(l 4X2) (b) 1 + cos2 X (c) x.J(x2 _ 1) (iii) (a) ~+ loge 2 (b) l~
2. (i);- (ii) 7r 3. (0 (a) 1 ~ x ~ 17 (b) e- 1 ~ x ~ e (c) 0 < x <2
4. (I) 45°, 225 0 (IIi) (a) 0'28 (b)...r0:9 5. (Ii) x >2 or x < 1
6. (i) (a) 0'54432 (b) 4 7. (i) 56 (ii) ~ (iii) (a) 60 (b) 18

TEST PAPER 8

1. (i) t < ~ or t > 1 2. (i) (b) 90 0 ,330 0

3. (a) y == 0 (b) (2, i)max., (-2, -i)min. (c) -i ~ y ~ ~; ~ loge 1-6


4. (i) (a) 0 (b) 12 Oi)?1.7r 8-~ 5. (i) 0'335 (ii) (3x - 7)2 == 6y + 14
6. (I) (a) 12 units/s (b) 24 units/s (Ii) 4-10 p.m.
I,NDEX
Ac<;eleration 101, lOS quadratic 43 One to one function 131
Addition trigonometric 41 ff.
principle 184 Exponential functions 125 ff. Parabola
theorems 26 ff. cartesian equation of 77
Alternate segment 15 Factor theorem 162 focus and directrix of 75
Amplitude 108 Factorial notation 182 parametric equation of 72
Angle Focus 75 tangent and normal to 72
between .lines 60 Frequency lOS Parametric equations 71
of projection 115 Functions of parabola 72
Applied trigonometry 22 ff. cubic 165 Pascal's triangle 200
Arrangements derivative of inverse Permutations 181 ff.
in a circle 185 trigonometric.l47 ff. Polynomial functions 163 ff.
in a row 180, 183 exponential and logarithmic 137 Polynomials 157 ff.
with restrictions 183 increasing and decreasing 132 Probability 194, 210
Asymptote 68 ff. inverse 132 Products as sums 33
inverse trigonometric 139 ff. Projectile motion 115 ff.
Binomial many to one 131 Proof by induction 64 ff .• 202
coefficients 207 one to one 131
probability 210 ff. polynomial 163 ff. . Quadratic equations.43. 169
theorem 200 ff. rational algebraic 68 ff. Quadrilateral, cyclic 9 ff:
Fundamental limit 81
Cartesian equation Radian measure 38
of parabola 72 Gradient 75 Range 115
of projectile 116 maximum 117
Change of variable 82 ff. Half-angle formulae 36 Rates 95 ff.
Circle 1 ff. Rational functions 68 ff.
Coefficients Increasing functions 132 Remainder theorem 160
binomial 207 Independent events 210 ff: .Roots and coefficients 169 ff.
roots and 169 ff. Induction 64 ff., 202
Chord of contact 77 Inequations 57 ff. Sampli,ng
Combinations 188 Integration with replacement 214
Concentric circles 2 change of variable 82 ff., 91 without replacement 196,215
Cubic functions 165 of sin2x and cos2x 90 ff. Secant 1 . .
Cyclic quadrilateral 9 ff. Inverse Segment
functions 132 alternate 15 :
Decreasing function 132 trigonometric functions 139 ff. of a circle 2
Definite integral 87; 153 Simple harmonic motion 105
Derivative of Kinematics 101 ff. Summary 229 ff·.
sinx and cosx 82 Sums as products 33
inverse. trigonometric Limit sinO 81
functions 147 ff. 6-0 0 Tangent
Differential equations 125 ff. Lines, angle between 60 to circle 13
Division of Locus 74 to parabola 75
an interval 61 Logarithmic functions 137 Transformations 46
polynomials 158 Tree diagrams 211
Double angle formulae 28 Many to one function 131 Trigonometric equations 41 ff., 51
Mathematical induction 64 ff.
Equation(s) . Multiplication principle 181 Velocity 101, 108
approximatesolution.of 174ff. Mutually exclusive events 184
bisection method 175 Wilhelmy's law 126
differential· 125 ff. nCr 189
general solution of 51 ff. nPr 182
Newtdh's method 176· Newton's
of pathof projectile 116 law of cooling 125
parametric 71 method of solving equatjons176

You might also like